Preview only show first 10 pages with watermark. For full document please download

Organic Chemistry 8th Edition Mcmurry | Martín Castillo Guerrero ...

If Organic as an adjunct to the larger Chemistry is the guidebook to your study of organic chemistry, then the Solutions Manual is the roadmap that shows you how to find what you need. Acknowledgments I would like to thank my husband, John McMurry, for offering me the opportunity to write this book many years ago and ...

   EMBED

  • Rating

  • Date

    April 2017
  • Size

    21.7MB
  • Views

    2,074
  • Categories


Share

Transcript

Preface What Some of you may think, might include "pre-med, "pressure," "difficult," or "memorization." Although formally the study of the compounds of carbon, the discipline of organic chemistry encompasses many skills that are common to other areas of study. Organic chemistry is as much a liberal art as a science, and mastery of the concepts and techniques of organic chemistry can lead to improved competence in other fields. As you work on the problems that accompany the text, you will bring to the task many problem-solving techniques. For example, planning an organic synthesis requires the skills of a chess player; you must plan your moves while looking several steps ahead, and you must keep your plan flexible. Structure-determination problems are like detective problems, in which many clues must be assembled to yield the most likely solution. Naming organic compounds is similar to the systematic naming of biological specimens; in both cases, a set of rules must be learned and then applied to the specimen or compound under study. The problems in the text fall into two categories: drill and complex. Drill problems, which appear throughout the text and at the end of each chapter, test your knowledge of one fact or technique at a time. You may need to rely on memorization to solve these problems, which you should work on first. More complicated problems require you to recall facts from several parts of the text and then use one or more of the problem-solving techniques mentioned above. As each major type of problem— synthesis, nomenclature, or structure determination— is introduced in the text, a solution is extensively worked out in this Solutions Manual. enters your mind when you hear the words "organic chemistry?" "the chemistry of life," or "the chemistry of carbon." Other responses Here are several suggestions that may help you with problem solving: 1. The text is organized into chapters that describe individual functional groups. As you study each functional group, make sure that you understand the structure and reactivity of that group In case your memory of a specific reaction fails you, you can rely on your general knowledge of functional groups for help. . Use molecular models. It is difficult to visualize the three-dimensional structure of an organic molecule when looking at a two-dimensional drawing. Models will help you to appreciate the structural aspects of organic chemistry and are indispensable tools for understanding stereochemistry. 2. been made to make this Solutions Manual as clear, attractive, and you should use the Solutions Manual in moderation. The principal use of this book should be to check answers to problems you have already worked out. The Solutions Manual should not be used as a substitute for effort; at times, struggling with a problem is the only way to teach yourself. 3. Every effort has error-free as possible. Nevertheless, Look through the appendices at the end of the Solutions Manual. Some of these appendices contain tables that may help you in working problems; others present 4. information related to the history of organic chemistry. Although the Solutions Manual is written to accompany Organic Chemistry, it contains Each chapter of the Solutions Manual begins with an outline of the text that can be used for a concise review of the text material and can also serve as a reference. After every few chapters a Review Unit has been inserted. In most cases, the chapters covered in the Review Units are related to each other, and the units are planned to appear at approximately the place in the textbook where a test might be given. Each unit lists the vocabulary for the chapters covered, the skills needed to solve problems, and several important points that might need reinforcing or that restate material in the text from a slightly different point of view. Finally, the small self-test that has been included allows you to test yourself on the material from more than one chapter. several unique features. many types of study aids in this Solutions Manual. Nevertheless, and more complete textbook. If Organic Chemistry is the guidebook to your study of organic chemistry, then the Solutions Manual is the roadmap that shows you how to find what you need. I this have tried to include book can only serve as an adjunct to the larger I would like to thank my husband, John McMurry, for offering me the opportunity to write this book many years ago and for supporting my efforts while this edition was being prepared. Although many people at Brooks/Cole Publishing company have given me encouragement during this project, special thanks are due to Elizabeth Woods. I also would like to acknowledge the contribution of Bette Kreuz, whose comments, suggestions and incredibly thorough accuracy checks was indispensable. Acknowledgments Contents Solutions to Problems Chapter 1 Structure and Bonding 1 Chapter 2 Polar Covalent Bonds; Acids and Bases 20 Review Unit 1 38 Chapter 3 Organic Compounds: Alkanes and Their Stereochemistry 41 Chapter 4 Organic Compounds8 Cycloalkanes and Their Stereochemistry 64 Chapter 5 Stereochemistry 88 Review Unit 2 112 Chapter 6 An Overview of Organic Reactions 116 Chapter 7 Alkenes: Structure and Reactivity 132 Chapter 8 Alkenes: Reactions and Synthesis 158 Review Unit 3 186 Chapter 9 Alkynes: An Introduction to Organic Synthesis 190 Chapter 10 Organohalides 213 Chapter 1 1 Reactions of Alkyl Halides: Nucleophilic Substitutions and Eliminations 233 Review Unit 4 264 Chapter 12 Structure Determination: Mass Spectrometry and Infrared Spectroscopy 268 Chapter 13 Structure Determination: Nuclear Magnetic Resonance Spectroscopy 289 Review Unit 5 316 Chapter 14 Conjugated Dienes and Ultraviolet Spectroscopy 319 Chapter 15 Benzene and Aromaticity 342 Chapter 16 Chemistry of Benzene: Electrophilic Aromatic Substitution 361 Review Unit 6 400 Chapter 17 Alcohols and Phenols 404 Chapter 18 440 Ethers and Epoxides; Thiols and Sulfides Review Unit 7 469 Car bony I Preview 4 72 Chapter 19 Aldehydes and Ketones: Nucleophilic Addition Reactions 474 Chapter 20 Carboxylic Acids and Nitriles 518 Chapter 21 Carboxylic Acid Derivatives: Nucleophilic Acyl Substitution Reactions 544 Review Unit 8 584 Chapter 22 Carbonyl Alpha-Substitution Reactions 588 Chapter 23 Carbonyl Condensation Reactions 616 Chapter 24 Amines and Heterocycles 654 Review Unit 9 Chapter 25 Chapter 26 698 Biomolecules: Carbohydrates 701 Biomolecules: Amino Acids, Peptides, and Proteins Review Unit 10 762 Chapter 27 Biomolecules: Lipids 765 Chapter 28 Biomolecules: Nucleic Acids 790 Chapter 29 The Organic Chemistry of Metabolic Pathways 807 Review Unit 11 832 Chapter 30 Orbitals and Organic Chemistry: Pericyclic Reactions Chapter 31 Synthetic Polymers 857 Review Unit 12 874 Appendices Functional-Group Synthesis 877 Functional-Group Reactions 882 Reagents in Organic Chemistry 886 Name Reactions in Organic Chemistry 893 Abbreviations 901 Infrared Absorption Frequencies 904 Chemical Shifts Proton 907 Nobel Prize Winners in Chemistry 908 Answers to Review -Unit Questions 917 NMR 733 836 Chapter 1 - Structure and Bonding Chapter Outline I. Atomic Structure (Sections 1.1-1.3). A. Introduction to atomic structure (Section 1.1). 1 An atom consists of a dense, positively charged nucleus surrounded by negatively charged electrons. a. The nucleus is made up of positively charged protons and uncharged neutrons. b The nucleus contains most of the mass of the atom. -10 c. Electrons move about the nucleus at a distance of about 2 x 10 (200 pm). 2. The atomic number (Z) gives the number of protons in the nucleus. 3 The mass number (A) gives the total number of protons and neutrons. 4 All atoms of a given element have the same value of Z. a. Atoms of a given element can have different values of A. b Atoms of the same element with different values of A are called isotopes. B. Orbitals (Section 1.2). 1 The distribution of electrons in an atom can be described by a wave equation. a. The solution to a wave equation is an orbital, represented by W. b Vr predicts the volume of space in which an electron is likely to be found. 2 There are four different kinds of orbitals (s, p, d,f). . . m . . . . . . c. The s orbitals are spherical. The p orbitals are dumbbell-shaped. Four of the five d orbitals are cloverleaf-shaped. An atom's electrons are organized into electron shells. a. The a. b. 3 . b shells differ in the numbers and kinds of orbitals they contain. Electrons in different orbitals have different energies. Each orbital can hold up to a maximum of two electrons. The two lowest-energy electrons are in the Is orbital. a. The 2s orbital is the next higher in energy. b. The next three orbitals are 2p x 2p y and 2p z which have the same energy, c 4. . . , i. Each p , orbital has a region of zero density, called a node. c. The lobes of ap orbital have opposite algebraic signs. C. Electron Configuration (Section 1.3). 1 The ground-state electron configuration of an atom is a listing of the orbitals occupied by the electrons of the atom in the lowest energy configuration. . 2. Rules for predicting the ground-state electron configuration of an atom: a. Orbitals with the lowest energy levels are filled first, The order of filling is Is, 2s, 2p, 3s, 3p, 4s, 3d. i. b Only two electrons can occupy each orbital, and they must be of opposite spin. If two or more orbitals have the same energy, one electron occupies each until c. all are half-full (Hund's rule). Only then does a second electron occupy one of . the orbitals. i All of the electrons in half-filled shells have the same spin. Chemical Bonding Theory (Sections 1.4-1.5). A. Development of chemical bonding theory (Section 1.4). 1 Kekule and Couper proposed that carbon has four "affinity units"; carbon . II. . tetravalent. 2 . Kekule suggested that carbon can form rings and chains. is 2 Chapter Van't Hoff and Le Bel proposed that the 4 atoms to which carbon forms bonds sit at the corners of a regular tetrahedron. 4. In a drawing of a tetrahedral carbon, a wedged line represents a bond pointing toward the viewer, a dashed line points behind the plane of the page, and a solid line lies in the plane of the page.. Covalent bonds. 1 Atoms bond together because the resulting compound is more stable than the individual atoms. a. Atoms tend to achieve the electron configuration of the nearest noble gas. b. Atoms in groups 1A, 2 and 7 either lose electrons or gain electrons to form ionic compounds. c Atoms in the middle of the periodic table share electrons by forming covalent 3 B . 1 . . A A . bonds. The neutral collection of atoms held together by covalent bonds is a molecule. Covalent bonds can be represented two ways. a. In electron-dot structures, bonds are represented as pairs of dots. b. In line-bond structures, bonds are represented as lines drawn between two bonded atoms. 3 The number of covalent bonds formed by an atom depends on the number of electrons it has and on the number it needs to achieve an octet. 4. Valence electrons not used for bonding are called lone-pair (nonbonding) electrons, a. Lone-pair electrons are often represented as dots. C Valence bond theory (Section 1 .5). Covalent bonds are formed by the overlap of two atomic orbitals, each of which 1 contains one electron. The two electrons have opposite spins. 2 Bonds formed by the head-on overlap of two atomic orbitals are cylindrically symmetrical and are called o bonds. 3 Bond strength is the measure of the amount of energy needed to break a bond. 4 Bond length is the optimum distance between nuclei. 5 Every bond has a characteristic bond length and bond strength. Hybridization (Sections 1.6-1.10). 3 A. sp Orbitals (Sections 1.6, 1.7). 1 Structure of methane (Section 1 .6). a. When carbon forms 4 bonds with hydrogen, one 2s orbital and three 2p orbitals combine to form four equivalent atomic orbitals (sp 3 hybrid orbitals). b These orbitals are tetrahedrally oriented. c. Because these orbitals are unsymmetrical, they can form stronger bonds than unhybridized orbitals can. d. These bonds have a specific geometry and a bond angle of 109.5°. 2. Structure of ethane (Section 1.7). a. Ethane has the same type of hybridization as occurs in methane. 3 b The C-C bond is formed by overlap of two sp orbitals. c. Bond lengths, strengths and angles are very close to those of methane. 2 B. sp Orbitals (Section 1.8). 2 1 If one carbon 2s orbital combines with two carbon 2p orbitals, three hybrid sp orbitals are formed, and one p orbital remains unchanged. 2. The three sp~ orbitals he in a plane at angles of 120°, and the unhybridized p orbital is perpendicular to them. Two different types of bonds form between two carbons. 3 a. A a bond forms from the overlap of two sp" orbitals. b A 7i bond forms by sideways overlap of two p orbitals. c This combination is known as a carbon-carbon double bond. d. 2 . . . . . . . . III. . . . . . . . Structure and Bonding 4. 3 is composed of a carbon-carbon double bond and four a bonds formed between the remaining four sp 2 orbitals of carbon and the Is orbitals of hydrogen, a. The double bond of ethylene is both shorter and stronger than the C-C bond of Ethylene ethane. C. sp Orbitals (Section 1.10). 1 If one carbon 2s orbital combines with one carbon 2p orbital, two hybrid sp orbitals are formed, and two p orbitals are unchanged. 2. The two sp orbitals are 180° apart, and the two p orbitals are perpendicular to them and to each other. 3 Two different types of bonds form. a. A a bond forms from the overlap of two sp orbitals. b. Two jt bonds form by sideways overlap of four unhybridized p orbitals. c This combination is known as a carbon-carbon triple bond. 4 Acetylene is composed of a carbon-carbon triple bond and two a bonds formed between the remaining two sp orbitals of carbon and the Is orbitals of hydrogen, a. The triple bond of acetylene is the strongest carbon-carbon bond. D. Hybridization of nitrogen and oxygen (Section 1.10). 1 Covalent bonds between other elements can be described by using hybrid orbitals. 2 Both the nitrogen atom in ammonia and the oxygen atom in water form sp' hybrid . . . . . . orbitals. _____ ~j a The lone-pair electrons in these compounds occupy sp orbitals. The bond angles between hydrogen and the central atom is often less than 109° because the lone-pair electrons take up more room than the a bond. Because of their positions in the third row, phosphorus and sulfur can form more than the typical number of covalent bonds. a. 3 . 4. IV. Molecular orbital theory (Section 1.11). A. Molecular orbitals arise from a mathematical combination of atomic orbitals and belong to the entire molecule. 1 . Two can combine in two different ways. is a bonding and is lower in energy than the two hydrogen Is atomic orbitals. and is higher in energy than b The subtractive combination is an antibonding the two hydrogen Is atomic orbitals. Two p orbitals in ethylene can combine to form two st MOs. a. The bonding has no node; the antibonding has one node. A node is a region between nuclei where electrons aren't found. a. If a node occurs between two nuclei, the nuclei repel each other. a. Is orbitals The MO additive combination MO . 2 . 3 . MO MO V. Chemical structures (Section 1.12). A. Drawing chemical structures. 1 . Condensed CH 3 CH2 , 2 . structures don't and CH show C-H bonds and don't show units. Skeletal structures are simpler still. Carbon atoms aren't usually shown. b. Hydrogen atoms bonded to carbon aren't usually shown. c. Other atoms (O, N, CI, etc.) are shown. a. the bonds between 4 Chapter 1 Solutions to Problems 1.1 To find the ground-state electron configuration of an element, first locate its atomic number. For oxygen, the atomic number is 8; oxygen thus has 8 protons and 8 electrons. Next, assign the electrons to the proper energy levels, starting with the lowest level. Fill each level completely before assigning electrons to a higher energy level. Notice that the 2p electrons are in different orbitals. According to Hund's rule, we must place one electron into each orbital of the same energy level until all orbitals are half-filled. (a) 4- fOxygen 25 Remember that only two electrons can occupy the same orbital, and that they must be of opposite spin. A different way to represent the ground-state electron configuration is to simply write down the occupied orbitals and to indicate the number of electrons in each orbital. For A. • example, the electron configuration for oxygen (b) Nitrogen, with an atomic number of 7, is Is 2s 2p . has 7 electrons. Assigning these to energy levels: Nitrogen 2p 2s -i — -i — -| -ff Is The more concise way Is 2 (c) 2s 2 2p to represent ground-state electron configuration for nitrogen: 3 Sulfur has 16 electrons. 2 4 2 2 6 is 2s 2p 3s 3p Sulfur 3/> 4f 3s If 2p 2s 4f is if Structure and Bonding 1 . 2 The elements of the periodic table are organized into groups that are based outer-shell electrons each element has. shell electron, and an element number of outer-shell in group For example, an element 5A has in group five outer-shell electrons. 1 5 on the number of A has one outer- To find the electrons for a given element, use the periodic table to locate its group. (a) Magnesium (group 2A) has two (b) Cobalt is a transition metal, 3d subshell. Selenium (group 6A) has six electrons in (c) 1.3 electrons in its outermost which has two electrons shell. in the 45 subshell, plus seven its electrons in its outermost shell. A solid line represents a bond lying in the plane of the page, a wedged bond represents a bond pointing out of the plane of the page toward the viewer, and a dashed bond represents a bond pointing behind the plane of the page. H ^-C_ v *ci Chloroform cr ci 1.4 Ethane 1 . 5 Identify the group of the central element to predict the number of covalent bonds the element can form. Carbon (Group 4A) has four electrons in its valence shell and forms four bonds to achieve the noble-gas configuration of neon. A likely formula is CCI4. (a) Element (b) Al (c) C (d) Si (e) N Group 3A 4A 4A 5A Likely Formula AlH^ CH 2C1 2 SiF4 CH3NH2 Width: 612 Height: 792 Chapter 6 1 . 6 Start (1) 1 by drawing the electron-dot structure of the molecule. Determine the number of valence, or outer-shell electrons for each atom in the molecule. For chloroform, we know that carbon has four valence electrons, hydrogen has one valence electron, and each chlorine has seven valence electrons. • c- 4 X 1 = 4 H- 1 X 1 = 7 X :Cl- 3 1 = 21 26 (2) valence electrons total Next, use two electrons for each single bond. H Cl:C: Cl Cl (3) Finally, use the For a line-bond remaining electrons to achieve an noble gas configuration for all atoms. between two atoms with a line. structure, replace the electron dots Molecule Electron-dot structure Line-bond structure P (a) CHCl 3 : CJ C : : Cl : : Cj— C— "Cl ":a:" (b) CH3NH2 : H H H H H CH 3 Li . 7 : : : H H H— G U— H 1 H:C:Li 8 valence electrons 1 H C N H ' 14 valence electrons (d) h— ^: h:S: H2 S 8 valence electrons (c) :a: H — C— Li H Each of the two carbons has 4 valence electrons. Two electrons are used to form the carbon-carbon bond, and the 6 electrons that remain can form bonds with a maximum of 6 hydrogens. Thus, the formula C2H7 is not possible. Structure and Bonding 1 . 8 Connect the carbons and add hydrogens so that all 7 carbons are bonded to four different atoms. H H H— C— C— C— H H --C— U H 'sp 3 H III H H H Propane \ ^ The geometry around carbon atoms all C _H is tetrahedral, and all bond angles are approximately 109°. 1.9 H c W H v v c p y p v p p H— C— C— C— C— C— C— H c i\ R R i\ i\ H H HH HH P H i I H Ii i I H I i H H Ii H Hexane i I H 1.10 H H —2^C H— C— C \\ ,C-H H C /3 H | H 1C_ H H The C3-H bonds are Propene SP SP o bonds formed by overlap of an sp 3 orbital of carbon 3 with an s orbital of hydrogen. The C2-H and Cl-H bonds with an The C2-C3 bond 2 are 2 a bonds formed by overlap of an sp orbital of carbon hydrogen. s orbital of is a o bond formed by overlap of an sp orbital of carbon 3 with an carbon 2. There are two C1-C2 bonds. One is a cr bond formed by overlap of an sp orbital of carbon 1 with an sp" orbital of carbon 2. The other is a n bond formed by overlap of a p orbital of carbon 1 with a p orbital of carbon 2. All four atoms connected to the carbon-carbon double bond lie in the same plane, and all bond angles between these atoms are 120°. The bond angle between hydrogen and the s/^-hybridized carbon is 109°. sp orbital of 2 3 2 H-^P^C^P^C^ 1 4

fa - (b) CHo - CHo CH 2 -OH J CH 2 ~ NH HO- C n^/°\^° CI" .c I f HoN SP 3 SP 3 4? CH2"" CH3 3 3 HO c \ 9 > 7 HO OH 2 Vitamin Procaine C 1.41 Pyridoxal phosphate The bond angles formed by atoms having s/r The bond angles formed by atoms having sp 2 hybridization are approximately 109 hybridization are approximately 120 Skeletal Structures 1.42 (b) 1.43 OH 3H 3H (c) a (d) CI CI p Width: 612 Height: 792 16 Chapter 1 1.44 H / \ H n H Quetiapine (Seroquel) C 2 iH 2 5N30 2 S 1.45 Oseltamivir (Tamiflu) C 16 H 28 N 2°4 Clopidogrel (Plavix) C 16 H 16 CIN0 2 S General Problems 1.46 compound containing a carbon-carbon triple bond, atoms bonded to the sp-hybridized carbons must lie in a straight line. It is not possible to form a five-membered ring if four carbons must have a linear relationship. In a Structure and Bonding 17 1.47 The central carbon of allene forms two a bonds and two it bonds. The central carbon is sp-hybridized, and the two terminal carbons are sp -hybridized. The bond angle formed by the three carbons is 180°, indicating linear geometry for the carbons of allene. 1.48 7i bonds k bonds Carbon dioxide is a linear molecule. 1.49 or. II I " t ")c— H Caffeine I CH, All of the indicated atoms are sp'-hybridized. 1.50 (a) The positively charged carbon atom is surrounded by six valence electrons; carbon has three valence electrons, and each hydrogen brings three valence electrons. (b) The (c) A carbocation is planar about the positively charged carbon. positively charged carbon is s/7~-hybridized. 18 Chapter 1 1.51 A (a) carbanion is isoelectronic with (has the nitrogen compound. 1.52 same number of electrons (b) The negatively charged carbanion carbon has (c) The carbon atom (d) A carbanion is tetrahedral. is as) a trivalent eight valence electrons. sp -hybridized. to the Pauli Exclusion Principle, two electrons in the same orbital must have opposite spins. Thus, the two electrons of triplet (spin-unpaired) methylene must occupy different orbitals. In triplet methylene, sp-hybridized carbon forms one bond to each of two According hydrogens. Each of the two unpaired electrons occupies a p orbital. In singlet (spin-paired) methylene the two electrons can occupy the same orbital because they have opposite spins. Including the two C-H bonds, there are a total of three occupied orbitals. We predict sp hybridization and planar geometry for singlet methylene. vacant p orbital Singlet methylene Triplet methylene (planar) (linear) 1.53 CH 3 CH3CHCH3 CH3CH2CH2CH3 The two compounds differ in the way that the carbon atoms 1.54 H H2C — CH CH3 One compound has H H a double bond, and one has a ring. are connected. Structure and Bonding 19 1.55 CH3OCH3 CH 3 CH 2 OH The two compounds differ in the location of the oxygen atom. 1.56 CH3CH2CH — CH2 CH3CH — CHCH3 H 2 C=C CH, The compounds differ in the way that the carbon atoms are connected and in the location of the double bond. 1.57 HoC # OH Naproxen Ibuprofen O # N # * = # = .^-hybridized carbon sp 3 -hybridized carbon CH, Acetaminophen (a), (b) Compound sp -Hybridized carbons sp -Hybridized carbons Ibuprofen 7 Naproxen Acetaminophen 7 1 1 Each of the structures has a six-membered ring containing three double bonds, each has a methyl group, and each has a C=0 group. (c) Chapter 2 - Polar Covalent Bonds; Acids and Bases Chapter Outline I. Polar covalent bonds (Sections 2. 1-2.3). A. Electronegativity (Section 2.1). Although some bonds are totally ionic and some are totally covalent, most chemical bonds are polar covalent bonds. a. In these bonds, electrons are attracted to one atom more than to the other atom. 2. Bond polarity is due to differences in electronegativity (EN). a. Elements on the right side of the periodic table are more electronegative than 1 . elements on the left side. Carbon has an EN of 2.5. Elements with EN > 2.5 are more electronegative than carbon. c d. Elements with EN < 2.5 are less electronegative than carbon. The difference in EN between two elements can be used to predict the polarity of a bond. a. If AEN < 0.4, a bond is nonpolar covalent. b. If AEN is between 0.4 and 2.0, a bond is polar covalent. b . . 3 . AEN > 2.0, a bond <5+ and c. If d. The symbols e. A crossed arrow i . The tail is <5- ionic. are used to indicate partial charges. is used to indicate bond polarity. of the arrow is electron-poor, and the head of the arrow is electron- rich. maps are also used to show electron-rich (red) and electronpoor (blue) regions of molecules. 5 An inductive effect is an atom's ability to polarize a bond. B. Dipole moment (Section 2.2). Dipole moment is the measure of a molecule's overall polarity. 1 2. Dipole moment (n) = Q x r, where Q = charge and r = distance between charges, a. Dipole moment is measured in debyes (D). Dipole moment can be used to measure charge separation. 3 4 Water and ammonia have large values of D; methane and ethane have D = 0. C. Formal charge (Section 2.3). 1 Formal charge (FC) indicates electron "ownership" in a molecule. Electrostatic potential 4. . . . . . 2 ~ ' |~ ' ^ - 1_ # of valence 1 _ electrons J |~ # of bonding electrons |~# J ~ L nonbondingl | L 2 electrons J H. Resonance (Sections 2.4-2.6). A. Chemical structures and resonance (Section 2.4). 1 Some molecules (acetate ion, for example) can be drawn as two (or more) different . electron-dot structures. a. These structures are called resonance structures. b The true structure of the molecule is intermediate between the resonance . structures. c. 2 . 3 . The true structure is called a resonance hybrid. Resonance structures differ only in the placement of n and nonbonding a. All atoms occupy the same positions. Resonance is an important concept in organic chemistry. electrons, Polar Covalent Bonds; Acids and Bases B 21 Rules for resonance forms (Section 2.5). 1 Individual resonance forms are imaginary, not real. 2 Resonance forms differ only in the placement of their n or nonbonding electrons, a. A curved arrow is used to indicate the movement of electrons, not atoms. 3 Different resonance forms of a molecule don't have to be equivalent. a. If resonance forms are nonequivalent, the structure of the actual molecule resembles the more stable resonance form(s). 4 Resonance forms must obey normal rules of valency. 5 The resonance hybrid is more stable than any individual resonance form. C. A useful technique for drawing resonance forms (Section 2.6). 1 Any three-atom grouping with a multiple bond adjacent to a nonbonding p orbital has two resonance forms. 2 One atom in the grouping has a lone electron pair, a vacant orbital or a single . . . . . . . . electron. 3 III. . By recognizing these three-atom pieces, Acids and bases (Sections 2.7-2.1 1). A. Br0nsted-Lowry definition (Section 2.7). 1 A Br0nsted-Lowry acid donates an H + . 2. The product resonance forms can be generated. ion; a Br0nsted-Lowry base accepts H+ when a base gains H* is the conjugate acid of the base; when an acid loses H* is the conjugate base of the acid. that results product that results . the Water can act either as an acid or as a base. B. Acid and base strength (Section 2.8-2.10). 3 . 1 . 2 . 3 . A strong acid reacts almost completely with water (Section 2.8). The strength of an acid in water is indicated by Ka the acidity constant. , Strong acids have large acidity constants, and weaker acids have smaller acidity constants. 4. a. b . c. 5 . 6 . pKa is normally used to express acid strength. pKa = -log Ka A strong acid has a small pKa and a weak acid has The , The conjugate base of a strong weak acid is a strong base. acid is a large pKa . a weak base, and the conjugate base of a Predicting acid-base reactions from pKa (Section 2.9). a. An acid with a low pA'a (stronger acid) reacts with the conjugate base of an acid with a high pKa (stronger base). b. In other words, the products of an acid-base reaction are more stable than the reactants. Organic acids and organic bases (Section 2. 10). a. There are two main types of organic acids: Acids that contain hydrogen bonded to oxygen. i. Acids that have hydrogen bonded to the carbon next to a C=0 group. ii. b The main type of organic base contains a nitrogen atom with a lone electron . pair. C. Lewis acids and bases (Section 2.11). 1 A Lewis acid accepts an electron pair. a. A Lewis acid may have either a vacant low -energy orbital or a polar bond to hydrogen. b Examples include metal cations, halogen acids, group 3 compounds and transition-metal compounds. 2 A Lewis base has a pair of nonbonding electrons. a. Most oxygen- and nitrogen-containing organic compounds are Lewis bases. b. Many organic Lewis bases have more than one basic site. 3 A curved arrow shows the movement of electrons from a Lewis base to a Lewis . . . . acid. 22 Chapter 2 IV. Noncovalent interactions in molecules (Section 2.12). A. Dipole-dipole interactions occur between polar molecules as a result of electrostatic among dipoles. interactions These interactions may be either attractive or repulsive. 2. The attractive geometry is lower in energy and predominates. Dispersion forces result from the constantly changing electron distribution within 1 B . . molecules. These forces are transient and weak, but their cumulative effect may be important. 1 C. Hydrogen bonds. 1 Hydrogen bonds form between a hydrogen bonded to an electronegative atom and an unshared electron pair on another electronegative atom. 2. Hydrogen bonds are extremely important in living organisms. 3 Hydrophilic substances dissolve in water because they are capable of forming hydrogen bonds. 4. Hydrophobic substances don't form hydrogen bonds and usually don't dissolve in . . . water. Answers 2. 1 Problems After solving this problem, use Figure 2.2 to check your answers. the more electronegative the element. More (a)H (2.1) (b)Br (2.8) B (2.0) (c)Cl (3.0) I (2.5) (d)C H (2.1) (2.5) As in is slightly Problem 2. 1 more , The larger the number, Less electronegative li (1.0) electronegative Carbon 2.2 to electronegative than hydrogen. use Figure 2.2. The partial negative charge is placed on the more is placed on the less electronegative electronegative atom, and the partial positive charge atom. ^ ^ X X 6+6- H 3 C— CI (e) sulfur (f) x_ x 6-6+ H 3 C— MgBr H 3C— SH X X 6-6+ H 2 N— H H 3C~ NH 2 (d) Carbon and ^ M X 6+6- x_ 6+6x H3C — have identical electronegativies. 2 3 . Use Figure 2.2 to find the electronegativities of each element. Calculate AEN and rank the answers in order of increasing AEN. Carbon: Lithium: EN = 2.5 EN =1.0 AEN = 1.5 Carbon: Potassium: EN = EN = 2.5 Fluorine: 0.8 Carbon: AEN = 1.7 EN = EN = 4.0 AEN = 1.5 2.5 Polar Covalent Bonds; Acids and Bases EN = EN = Carbon: Magnesium: AEN = 2.5 Oxygen 1.2 Carbon: The most polar bond has H 3C-OH 2.4 < EN = 3.5 EN = 2.5 AEN= 1.3 the largest H3C— MgBr 23 AEN. Thus, 1.0 in order of increasing bond polarity: H3C— Li, H 3 C-F < H3C-K < In an electrostatic potential map, the color red indicates regions of a molecule that are that chlorine is the most electronegative atom in chloromethane, and the direction of polarity of the C-Cl bond is: electron-rich. The map shows 6- Chloromethane It H' vf^H 8+ H 2.5 H0 ^?\ * H s 0H Ethylene glycol H The dipole moment of ethylene glycol is zero because the bond polarities of the two carbon-oxygen bonds cancel. 2 6 . For each bond, identify the more electronegative element, and draw an arrow that points from the less electronegative element to the more electronegative element. Estimate the sum of the individual dipole moments to arrive at the dipole (a) moment for the entire molecule. (b) hL . H dipole f H IT moment t net dipole moment H (c) (d) Ck ft . ci net dipole net dipole s moment H i H moment 24 2.7 Chapter 2 To find the formal (1) charge of an atom in a molecule, follow these two steps: Draw an electron-dot structure of the molecule. (2) Use the formula in Section 2.3 (shown below) to determine formal charge for each atom. The periodic table shows the number of valence electrons of the element, and the electron-dot structure shows the number of bonding and nonbonding electrons. t- i i \# of valencel /t-v-^ Formal charge (FC) =|_ electrons J T -[ # of bonding electrons f "1 [# nonbonding! J-[_ electrons J (a) h 2 c=n=n: For carbon: For nitrogen 1 For nitrogen 2: : FC FC FC electrons are the electrons characteristic of a specific element. Bonding electrons are those electrons involved in bonding to other atoms. Nonbonding electrons are those electrons in lone pairs. Remember: Valence ( b> .. H 3 C— C=N— O: = 1.4 2 h:c:c: : : n: o: H Forcarbon 1: FC = 4- -y -0 = For carbon 2: FC = 4- -|- -0 = 5- -y- -0 = +1 2 = FC = 6--^--6 -l For nitrogen: For oxygen: FC = 2 Polar Covalent Bonds; Acids and Bases (c) i!i H 3 C— N=C: For carbon 1 For carbon 2: For nitrogen : : 25 2 = h:c:n: ::c: FC 4-f-0 FC 4-f -2 = -l FC 5- -0 = +1 -f = 2.8 [ #of valence 1 u /t-/^\ Formal charge (FC) =[_ electrons J t- i -, H I 1A . . :o: # of bonding electrons ~| J - T# nonbondingl electrons |_ J 2- 3 . II -[[ . H— C— O— P— O: Methyl phosphate I H '•?.'• 4 For oxygen 1 For oxygen 2: : For oxygen 3 : For oxygen 4: FC = 6 FC = 6 FC = 6 -1 FC = 6 -1 Oxygen atoms 3 and 4 each have a formal charge of -1, and oxygen atoms 1 and 2 have a formal charge of 0. 2.9 Try to locate the three-atom groupings that are present in resonance forms. (a) These two structures represent resonance forms. The three-atom grouping (C-C double bond and an adjacent vacant p orbital) is pictured on the right. (b) These two structures represent different compounds, not resonance structures. Width: 612 Height: 792 26 2.10 Chapter 2 Look for three-atom groupings that contain a multiple bond next to an atom with a p Exchange the positions of the bond and the electrons in the p orbital to draw the resonance form of each grouping. orbital. (a) Methyl phosphate anion has 3 three-atom groupings and thus has 3 resonance forms. :0: :0: :0: I I ~ CHoO^Q 3 - -:0: Recall from Chapter four covalent bonds 1 that phosphorus, a third-row element, CH 3 0'\^P : can form more than (b) :o: ? :o O: _ (c) :0 O: H 2 C— CH=CH 2 CH2 When an acid loses a proton, the product is the conjugate base of the acid. gains a proton, the product is the conjugate acid of the base. H— N03 + : NH 3 Base Acid 2.12 o: + CH H2C-— 2.11 :o When a base NH4 NOc Conjugate base Conjugate acid Recall from Section 2.8 that a stronger acid has a smaller pKa and a weaker acid has a larger pKa Accordingly, phenylalanine (pKa = 1.83) is a stronger acid than tryptophan . (ptfa 2.13 = 2.83). HO-H is a stronger acid than H2N-H. conjugate acid of H2N~ (HO-H). (H2N-H) is Since H2N~ is a stronger base than HO~, the a weaker acid than the conjugate acid of HO Polar Covalent Bonds; Acids and Bases 2.14 Use Table 27 A reaction takes place as written if the 2.3 to find the strength of each acid. is the reactant. stronger acid (a) H— CN pKa = CH 3 C02"Na + + + Na CN P^a = 4.7 Stronger acid 9.3 Weaker CH 3 C0 2 H + acid Remember that the lower the pKa , the stronger the acid. Thus CH3CO2H, not HCN, stronger acid, and the above reaction will not take place to a significant extent in the direction written. (b) CH 3 CH 2 0— H Na+ + CN ? _ CH 3 CH 2 Na+ is the . HCN + pKa =\6 pKa = 9.3 Stronger acid Weaker acid Using the same reasoning as we can in part (a), see that the above reaction will not occur to a significant extent. 2.15 if HqC ch 3 + Na + c. M "T CH- 2 :-Na ; :'nh2 HaC'' pKa =19 Weaker written, the difference in 2.16 above reaction will take place to virtual completion acid due to the large pKa values. Enter -9.3 1 into a calculator and use the 10 a = 4.9 x 1(T K 2.17 :NH 3 pK a = 36 Stronger acid As + INV LOG function to arrive at the answer . Locate the electron pair(s) of the Lewis base and draw a curved arrow from the electron pair to the Lewis acid. The electron pair moves from the atom at the tail of the arrow (Lewis base) to the atom at the point of the arrow (Lewis acid). (Note: electron dots have been omitted from CT to reduce clutter.) H (a) CH 3CH 2 OH + H-C3I CH 3 CH 2 OH + CI H HN(CH 3 ) 2 P(CH 3 ) 3 + + H-^CI H-^-CI hn(ch 3 ) + 2 H-P(CH 3 ) 3 cr + cr 28 Chapter 2 (b) 2.18 Hp: + HO: + HO: + + HO— CH3 CH 3 B(CH 3 ) HO-B(CH 3 3 )3 HO— MgBr2 MgBr 2 (a) The nitrogen on the left is more electron-rich and more most electron-poor (bluest) and is most acidic. more basic. The indicated hydrogen is basic (red) 2.19 IHoOH only one -OH group several Vitamin Vitamin A * -OH groups = polar group C is water-soluble (hydrophilic) because it H0 * * Vitamin 0H C has several polar -OH groups that can form hydrogen bonds with water. Vitamin A is fat-soluble (hydrophobic) because most of its atoms can't form hydrogen bonds with water. Polar Covalent Bonds; Acids and Bases 29 Visualizing Chemistry 2.20 Naphthalene has three resonance forms. 2.21 Ibuprofen 2.22 Electrostatic potential same maps show that the electron-rich regions of the cis side of the double bond, leading to a net dipole isomer he on the moment. Because the electron-rich regions of the trans isomer are symmetrical about the double bond, the individual dipole moments cancel, and the isomer has no overall dipole moment. net dipole moment zero dipole moment H VCI / C=C\ H / H H cis- 1 ,2-Dichloroethylene c=c\ CI trans- 1 ,2-Dichloroethylene 2.23 (a) (b) 99.9%) with NaOH, an acid must have a pKa at least 3 units smaller than the pKa of 2 0. Thus, all substances in the previous problem except acetone react completely with NaOH. To react completely H 2.42 stronger the acid (smaller pKa ), the weaker its conjugate base. Since + stronger acid than CH3NH3 , CH3NH2 is a stronger base than NH3. The NH4 + is a 2.43 CH 3 CH 3 H 3 C— C— O'K* + H^C— C— OH H2 CH 3 + 4" K "OH CH 3 pK a = 15.7 stronger acid a 18 weaker acid The reaction takes place as written because water is a stronger acid than terr-butyl alcohol. Thus, a solution of potassium te/t-butoxide in water can't be prepared. 2.44 Ka = 5x 2.45 (a) Acetone: 2.46 (a) Nitromethane: i-20 10" pKa = 10.30 (b) Formic acid: (b) Acrylic acid: Ka = 1.8 x 10 pKa = 4.25 Polar Covalent Bonds; Acids and Bases 35 2.47 H2 Formic acid + 0.050 [ K a Formate M] - " + H3O 4 [x] 4 1.8 x 10- = « = [x] 0.050 - x 3 0.050 - x = 0.050, then x = 3.0 x 10" and pH = 2.52. If you calculate x exactly -3 using the quadratic equation, then x = 2.9 x 10 and pH = 2.54. If 2.48 you let Only acetic acid will react with sodium bicarbonate. Acetic acid Problem 2.40 that is a stronger acid than carbonic acid. is the only substance in General Problems 2.49 moments add to produce a net dipole moment for the whole molecule. The individual dipole moments in fumaric acid cancel, resulting in a zero dipole moment. In maleic acid, the individual dipole \A%/ HO— C .0 HO— C C— OH \ / \ \ / c=c / H H H H / c=c \ C— OH // 2.50 \ Fumaric acid Maleic acid Sodium bicarbonate reacts with acetic acid to produce carbonic acid, which breaks down to form CO2. Thus, bubbles of CO2 indicate the presence of an acid stronger than carbonic acid, in this case acetic acid, as the pKa values indicate. Phenol does not react with sodium bicarbonate. 2.51 Reactions (a) and (c) are reactions between Br0nsted-Lowry acids and bases; the stronger acid and stronger base are identified. Reactions (b) and (d) occur between Lewis acids and bases. CH3OH + base H + CH3OH acid (b) II CH3CCH3 base TiCI 4 + O II + TiCI 4 acid CH3CCH3 Width: 612 Height: 792 36 Chapter 2 (c) O H + NaH base acid (d) ~BHo H BH 3 + O base 2.52 Pairs (a) acid and (d) represent resonance structures; pairs (b) and (c) do not. For two all atoms must be in the same positions in all resonance be resonance forms, structures to forms. 2.53 (b) (a) 0: H 3 C— H 3 C— :0: (c) _ The W 0=0-0: : 0-0=0 :0: ..- + H 2 c— n=n: h 2 c=n=n: 2.54 +> cation pictured can be represented by two resonance forms. Reaction with water can at either positively charged carbon, resulting in two products. occur H HoO C H 3 C^ HoC ^C^ CH2 + H+ I H H t H H I ^.C^, HoC 3 y. C I H CH 2 H2 I C H 3 C' *C CH2°H I H + H+ Polar Covalent Bonds; Acids and Bases 37 2.55 2.56 2.57 :b: When anion 2.58 :0: phenol loses a proton, the resulting anion is not stabilized by resonance. :o: is stabilized :o: :o: by resonance. The methanol Review Unit 1: Bonds and Bond Polarity Major Topics Covered (with vocabulary:) Atomic Structure: atomic number mass number wave equation orbital shell node electron configuration Chemical Bonding Theory: Lewis structure lone-pair electrons line-bond structure valence-bond theory sigma (a) bond bond strength bond length molecular orbital theory bonding covalent bond MO MO antibonding Hybridization: sp 3 hybrid orbital bond angle sp 2 hybrid orbital pi (,t) bond sp hybrid orbital Polar covalent bonds: polar covalent bond electronegativity (EN) electrostatic potential dipole moment formal charge dipolar molecule maps inductive effect Resonance: resonance form resonance hybrid Acids and Bases: Br0nsted-Lowry acid Ka pKa organic acid Chemical Structures: condensed structure Br0nsted-Lowry base conjugate acid conjugate base organic base Lewis acid Lewis base skeletal structure space-filling acidity constant models ball-and-stick models Types of Problems: After studying these chapters you should be able to: - Predict the ground state electronic configuration of atoms. Draw Lewis electron-dot structures of simple compounds. - Predict and describe the hybridization of bonds in simple compounds. - Predict the direction of polarity of a chemical bond, and predict the dipole Predict bond angles and shapes of molecules. compound. - - Calculate formal charge for atoms in a molecule. Draw resonance forms of molecules. Predict the relative acid/base strengths of Br0nsted acids and bases. Predict the direction of Br0nsted acid/base reactions. - Calculate: - Identify - pKa from Ka and vice versa. pH of a solution of a weak acid. , Lewis acids and bases. structures from molecular formulas, and vice Draw chemical versa. moment of a simple Review Unit 39 1 Remember: Points to In order for carbon, with valence shell electron configuration of 2s 2 2p 2 , to form four sp 3 hybrid orbitals, it is necessary that one electron be promoted from the 2s subshell to the 2p subshell. Although this promotion requires energy, the resulting hybrid orbitals are able to form stronger bonds, and compounds containing these bonds are more stable. is helpful in showing where the electrons in a bond are located. Even if a bond is polar covalent, in some molecules the electrons "belong" more to one of the atoms than the other. This "ownership" is useful for predicting the outcomes of chemical reactions, as we will see in later chapters. Assigning formal charge to atoms in a molecule Resonance and nonbonding electrons in a and the molecule doesn't change back and forth, from structure to structure. Rather, resonance structures are an attempt to show, by conventional line-bond drawings, the electron distribution of a molecule that can't be represented by any one structure. structures are representations of the distribution of n molecule. Electrons don't As move around in the molecule, in general chemistry, acid-base reactions are of fundamental importance in organic chemistry. Organic acids and bases, as well as inorganic acids and bases, occur frequently in reactions, and large numbers of reactions are catalyzed by Br0nsted acids and bases and Lewis acids and bases. Self-Test: ABC Oxaflozane Ricinine (a toxic component 1,3,4-Oxadiazole (an antidepressant) of castor beans) For A (ricinine) and B (oxaflozane): Add all missing electron lone pairs. Identify the hybridization of all carbons. Indicate the direction of bond polarity for all bonds with A EN In each compound, which bond is the most polar? Convert and B to molecular formulas. > 0.5. A Draw Add draw a resonance structure for B. Which atom (or atoms) of B can act as a Lewis base? missing electron lone pairs to C. Is it possible to draw resonance forms for one resonance form, and describe it. at least C? If so, Review Unit 40 1 Multiple Choice: 1 . 2. Which element has 4s2 4p2 as (a)Ca (b)C (c)Al (d)Ge Which compound (a) NO3 (b) O3 its valence shell electronic configuration? (or group of atoms) has an oxygen with a +1 formal charge? (c) acetone anion (d) acetate anion The following questions involve these acids: (iii) HY (pKa = 10); (iv) HZ (pKa = 20). 3 . 4 . . HW (pKa = 2); (ii) HX (pKa = 6); Which of the above acids react almost completely with water to form hydroxide (a) none of them (b) all of them (c)HYandHZ (d) HZ The conjugate bases of which of the above hydroxide ion? (a) none of them 5 (i) If you want (b) all of them (c) ion? acids react almost completely with water to form HZ (d)HYandHZ HX to X-, which bases can you use? to convert (a)W- (b)Y- (c)Z- (d)Y-orZ6 . If you add equimolar amounts of HW, X- and HY to a solution, what are the principal species in the resulting solution? HY (a)HW, HX, 7 . (b) W-, HX, HY (c) HW, X~ HY (d) What is the approximate pH difference between a solution of HW, HX, Y1 M HX and a solution of M 1 HY? (a) 8 . . (b)3 (d)6 you wanted to write the structure of a molecule that shows carbon and hydrogen atoms as groups, without indicating many of the carbon-hydrogen bonds, you would draw a: molecular formula (b) Kekule structure (c) skeletal structure (d) condensed structure Which of the following molecules has zero net dipole moment? (a) \/ C=C /\ H CI 10. (c)4 If (a) 9 2 In (b) \/ c=c /\ (c) \/ c=c /\ (d) CI H CI H CI H H CI H H which of the following bonds (a)C-Br (b)C-I (c) C-P is CI \/ c=c /\ H carbon the more electronegative element? (d)C-S CI H Chapter 3 - Organic Compounds: Alkanes and Their Stereochemistry Chapter Outline Functional Groups (Section 3.1). A. Functional groups are groups of atoms within a molecule that have a characteristic chemical behavior. B The chemistry of every organic molecule is determined by its functional groups. C. Functional groups described in this text can be grouped into three categories: 1 Functional groups with carbon-carbon multiple bonds. 2 Groups in which carbon forms a single bond to an electronegative atom. 3 Groups with a carbon-oxygen double bond. H. Alkanes (Sections 3.2-3.5). A. Alkanes and alkane isomers (Section 3.2). 3 1 Alkanes are formed by overlap of carbon sp orbitals. 2 Alkanes are described as saturated hydrocarbons. a. They are hydrocarbons because they contain only carbon and hydrogen. b. They are saturated because all bonds are single bonds. c. The general formula for alkanes is C n H2«+2For alkanes with four or more carbons, the carbons can be connected in more than 3 I. . . . . . . . one way. a. If the carbons are in a row, the alkane is a straight-chain alkane. b If the carbon chain has a branch, the alkane is a branched-chain alkane. Alkanes with the same molecular formula can exist in different forms known as . 4 . isomers. Isomers whose atoms are connected differently are constitutional isomers, i. Constitutional isomers are always different compounds with different properties but with the same molecular formula. b Most alkanes can be drawn in many ways. Straight-chain alkanes are named according to the number of carbons in their chain. 5 B. Alkyl groups (Section 3.3). 1 An alkyl group is the partial structure that results from the removal of a hydrogen atom from an alkane. a. Alkyl groups are named by replacing the -ane of an alkane name by -yl. b n- Alkyl groups are formed by removal of an end hydrogen atom of a straighta. . . . . chain alkane. Branched-chain alkyl groups are formed by removal of a hydrogen atom from an internal carbon. i. The prefixes sec- and ten- refer to the degree of substitution at the branching carbon atom. There are four possible degrees of alkyl substitution for carbon. a. A primary carbon is bonded to one other carbon. b A secondary carbon is bonded to two other carbons. c A tertiary carbon is bonded to three other carbons. d A quaternary carbon is bonded to four other carbons. e. The symbol R refers to the rest of the molecule. Hydrogens are also described as primary, secondary and tertiary. a. Primary hydrogens are bonded to primary carbons (RCH3). b. Secondary hydrogens are bonded to secondary carbons (R2CH2). c. Tertiary hydrogens are bonded to tertiary carbons (R3CH). c. 2 . . . . 3 . 42 Chapter 3 C. Naming alkanes (Section 3.4). 1 The system of nomenclature used in this book is the IUPAC system. In this system, a chemical name has a locant, a prefix, a parent and a suffix. i. The locant shows the location of substituents and functional groups. . The prefix indicates the type of substituent or functional group. The parent shows the number of carbons in the principal chain. iv. The suffix identifies the functional group family. Naming an alkane: ii. iii. 2 . a. Find the parent hydrocarbon. i. Find the longest continuous chain of carbons, and use its name as the parent name. If two chains have the same number of carbons, choose the one with more branch points. Number the atoms in the parent chain. Start numbering at the end nearer the first branch point. i. ii. If branching occurs an equal distance from both ends, begin numbering at the end nearer the second branch point. Identify and number the substituents. i. Give each substituent a number that corresponds to its position on the parent ii. b . c. chain. Two substituents on the same carbon receive the same number. Write the name as a single word. i. Use hyphens to separate prefixes and commas to separate numbers. ii. d. Use ii. the prefixes, di-, tri-, tetra- if necessary, but don't use them for alphabetizing. e. Name a complex substituent as if it were a compound, and set it off within parentheses. i. Some ii. The simple branched-chain alkyl groups have common names. prefix iso is used for alphabetizing, but sec- and tert- are not. D. Properties of alkanes (Section 3.5). 1 Alkanes are chemically inert to most laboratory reagents. 2 Alkanes react with O2 (combustion) and CI2 (substitution). 3 The boiling points and melting points of alkanes increase with increasing molecular . . . weight. III. a. This effect b The . is due to weak dispersion forces. strength of these forces increases with increasing molecular weight. 4. Increased branching lowers an alkane's boiling point. Conformations of straight-chain alkanes (Sections 3.6-3.7). A. Conformations of ethane (Section 3.6). Rotation about a single bond produces isomers that differ in conformation. 1 a. These isomers (conformers) have the same connections of atoms and can't be . isolated. 2. These isomers can be represented in two ways: a. Sawhorse representations view the C-C bond from an oblique angle. b Newman projections view the C-C bond end-on and represent the two carbons . as a circle. 3 . There is a barrier to rotation that makes some conformers of lower energy than others. a. b . c. The lowest energy conformer (staggered conformation) occurs when all C-H bonds are as far from each other as possible. The highest energy conformer (eclipsed conformation) occurs when all C-H bonds are as close to each other as possible. Between these two conformations lie an infinite number of other conformations. Organic Compounds: Alkanes and Their Stereochemistry 4. The staggered conformation is 43 12 kJ/mol lower in energy than the eclipsed conformation. a. This energy difference is due to torsional strain from interactions between C-H bonding orbitals on one carbon and C-H antibonding orbitals on an adjacent carbon, which stabilize the staggered conformer. b. The torsional strain resulting from a single C-H interaction is 4.0 kJ/mol. c. The barrier to rotation can be represented on a graph of potential energy vs. angle of rotation (dihedral angle). B. Conformations of other alkanes (Section 3.7). Conformations of propane. 1 a. Propane also shows a barrier to rotation that is 14 kJ/mol. b. The eclipsing interaction between a C-C bond and a C-H bond is 6.0 kJ/mol. 2 Conformations of butane. a. Not all staggered conformations of butane have the same energy; not all eclipsed conformations have the same energy. i. In the lowest energy conformation (anti) the two large methyl groups are as far from each other as possible. The eclipsed conformation that has two methyl-hydrogen interactions and a ii. H-H interaction is 16 kJ/mol higher in energy than the anti conformation. hi. The conformation with two methyl groups 60° apart (gauche conformation) is 3.8 kJ/mol higher in energy than the anti conformation, (a). This energy difference is due to steric strain - the repulsive interaction that results from forcing atoms to be closer together than their atomic . . radii allow. iv. The highest energy conformations occur when the two methyl groups are eclipsed. This conformation is 19 kJ/mol less stable than the anti conformation. The value of a methyl-methyl eclipsing interaction is 1 1 kJ/mol. The most favored conformation for any straight-chain alkane has carbon-carbon bonds in staggered arrangements and large substituents anti to each other. At room temperature, bond rotation occurs rapidly, but a majority of molecules adopt the most stable conformation. (a). b c . . Chapter 3 44 Solutions to Problems 3 . 1 Notice that certain functional groups have different designations if other functional groups are also present in a molecule. For example, a molecule containing a carbon-carbon double bond and no other functional group is an alkene; if other groups are present, the group is referred to as a carbon-carbon double bond. Similarly, a compound containing a benzene ring, and only carbon- and hydrogen-containing substituents, is an arene; if other groups are present, the ring is labeled an aromatic ring. (a) sulfide O ?°2 H (b) carboxylic carboxvlic „ acid carboxylic CHq CH 3 SCH 2 CH 2 CHCOH NH aromatic ring amine C Methionine (c) acid Ibuprofen O ether C-C amide double bond CH, alcohol 3.2 (a) CH 3 OH Methanol (e) O CH3COH CH 3 NH 2 Acetic acid Methylamine m II CH 3 CCH 2 NH 2 Aminoacetone 1,3-Butadiene 3.3 CH, amine H O' H ester C N H H O C-C AH H double bond Arecoline C 8 Hi 3 N0 2 Organic Compounds: Alkanes and Their Stereochemistry 3 4 . We know that carbon forms four bonds and hydrogen forms one bond. Thus, draw 45 all possible six-carbon skeletons and add hydrogens so that all carbons have four bonds. draw all possible skeletons in this problem: (1) Draw the six-carbon straight-chain To Draw a five-carbon chain, identify the different types of carbon atoms on the and add a -CH3 group to each of the different types of carbons, generating two skeletons. (3) Repeat the process with the four-carbon chain to give rise to the last two skeletons. Add hydrogens to the remaining carbons to complete the structures. skeleton. (2) chain, CH 3 CH 3 I CH3CH2CH2CHCH3 CHgCh^Ch^Ch^Ch^CHg CHo3 CHo3 I I CH3CHCHCH3 CH3CH2CCH3 CH 3 CHo 3.5 Nine isomeric Problem 3.4. (a) esters of formula C5H10O2 can be drawn. The procedure O o II II is described in if CH3CHCOCH3 CH3CH2CH2COCH3 CH3CH2CHCH2CH3 CH3CH2COCH2CH3 CHo O CH 3 if II CH3COCHCH3 CH3COCH2CH2CH3 O CH 3 O II if I HCOCH2CH2CH2CH3 CH 3 1 HCOCH 2 CHCH 3 HCOCHCH 2 CH 3 o II I HCOCCHo3 I CHo (b) Two isomers can be drawn. CH 3 CH 3 CH 2 CH 2 C= N (c) and CH 3 CHC= N Three isomers can be drawn. CH3CH2SSCH2CH3 CH3SSCH2CH2CH3 CH3SSCHCH3 CH 3 Width: 612 Height: 792 46 3.6 Chapter 3 Two (a) alcohols have the formula C3H 8 0. OH I CH 3 CH 2 CH 2OH CH 3 CHCH 3 and Four bromoalkanes have the formula G^HgBr. (b) CHo3 Br Br I I I CH3CH2CHCH3 CH3CH2CH2CH2Br Four thioesters have the formula (c) O O ii ii CH3CH2CSCH3 CH3CCH3 CH3CHCH2Br QHgOS. O CH 3 if r HCSCHCH3 CH3CH2SCCH3 HCSCH2CH2CH3 3.7 CH3CH2CH2CH2CH2~^— CH3CH2CH2CH CH3CH2CH — ^ CH3CH2CHCH2""^ CH2CH3 CHr CH3 CHr CH 3 ^p^3 I CH3CHCH2CH2 CH 3 CH 2 C—|- ^ CH 3 CHCH-|- \ CH 3 CH3CCH2 ^ I GH« CHr CHr. 3.8 (a) (b) p (c) P t p CH3CHCH3 CHo3 p p CH3 CH3 I CH3CHCH2CH2CH3 t s s p p p - primary; s = 3.9 The carbons and (a) secondary; t CH3CH2CHCH2CH3 CH3CHCH2 - C p p s p s t = tertiary; q = P same classification. (c) p t p CH3CHCH3 CH 3 CH 3 P quaternary the attached hydrogens have the (b) CH3 W p s t p CH 3 p 9H3 I CH3CHCH2CH2CH3 p s t s -* CH3CHCH2 C CH3 p t s p CH 3 P CH3CH2CHCH2CH3 p p s t s p I 3.10 (a) CH 3 t (c) CH3CHCH3 t CHo CH3CH2CHCH2CH3 CH 3 II I ' 2 OH 52 3.26 Chapter 3 (a) Although it is stated that biacetyl contains no rings or carbon-carbon double bonds, it is obvious from the formula for biacetyl that some sort of multiple bond must be present. The structure for biacetyl contains two carbon-oxygen double bonds. (b) Ethyleneimine contains a three-membered ring. (c) Glycerol contains no multiple bonds or rings. O CH3C CCH3 \ Q Q O n ii n HCCH2CH2CH (c) (b) also possible: (a) O N / \ HOCH 2 CHCH 2 OH CH2 HCCH2CCH3 Biacetyl Ethyleneimine Glycerol Isomers 3.27 (a) Eighteen isomers have the formula CgHis. Three are pictured. Ch^CHg CHo CHo ,3,3 CHoC CCHq3 3 T CH3 CH3 CH3CH2CH2CHCH2CH3 CH3CH2CH2CH2CH2CH2CH2CH3 1 formula C4H8O2 may represent esters, carboxylic acids or other complicated molecules. Three possibilities: (b) Structures with the many O O II CH 3 CH 2 CH 2 COH HOCH 2 CH=CHCH 2 OH CH 3 CH 2 COCH 3 3.28 CH 3 3 I CH3CH2CH2CH2CH2CH2CH3 Heptane CH 3 CH2CH2CH2CH2CHCH2 2-Methylhexane CH3CH2CH2CHCH2CH3 3-Methylhexane CH 3 CHo3 I CHgCh^Ch^CCHg CH 3 2,2-Dimethylpentane CH2CH2CHCHCH2 CH 3 2,3-Dimethylpentane CH 3 3 ,3-Dimethy lpentane CH 3 2,4-Dimethylpentane CHo ,3,3 CHo r CH3CH2CCH2CH3 CH3CHCH2CHCH3 CHo CH3CH2CHCH2CH3 CH2CHg 3-Ethylpentane CH3CH CCH3 CH 3 2,2,3-Trimethylbutane Organic Compounds: Alkanes and Their Stereochemistry 3.29 (a) CHo3 Br I I CH3CHCHCH3 CH3CHCHCH3 CH 3 Br same Br same same same same (c) different CH2CH3 CH3 CH 3 CH 2 CHCH 2CHCH 3 containing the CH 3 CH 2 CHCH 2 CHCH 2OH same same " 1" to the carbon bonded to -OH, and count to find the longest chain -OH group. The isomers may be either alcohols or ethers. OH CH3CH 2 CH 2 CH 2 OH CH3CH 2 CHCH3 CH 3 CH3CHCH 2 — OH CH3CCH3 - OH OCH3 CH3CH 2 CH 2 3.31 First, (a) draw CH3 CH 3 1 Give the number CH3 HOCH 2 CHCH 2 CHCH 3 CH 2 OH different 3.30 CH3CHCHCH3 OCH3 CH3CHCH3 all straight-chain isomers. CH3CH 2 Then proceed OCH 2 CH3 to the simplest branched structure, There are four alcohol isomers with the formula C4H10O. OH CH3CH 2 CH 2 CH 2 OH CH3CH 2 CHCH3 OH 9 H3 CH3CHCH 2OH CH3CCH3 ,CCh } l CH 3 53 Chapter 3 (b) There are 17 isomers of C5H13N. Nitrogen can be bonded to one, two or three alkyl groups. NH 2 NH 2 CH3CH2CH2CHCH3 CH3CH2CH2CH2CH2NH2 NH ? NHo CH3CH2CHCH2NH2 CH3CH2CCH3 CH3 CH3CH2CHCH2CH3 H2NCH2CH2CHCH3 CH3CHC-HCH3 CH3 ^^3 CHo3 CHo3 I I CH 3 CCH 2 NH 2 CH 3 CH 2CHNHCH 3 CH3CH2CH2CH2NHCH3 CH 3 CHo CHo I CHq I I CH 3 CHCH 2 NHCH 3 CH3CNHCH3 CH 3 CHNHCH 2 CH 3 CH3CH2CH2NHCH2CH3 CH 3 CH3 ^p^3 ^p^3 CH3CH2CH2NCH3 CH3CHNCH3 CH3CH2NCH2CH3 CH 3 (c) There are 3 ketone isomers with the formula C5H10O. if ff CH3CH2CH2CCH3 if CH3CHCCH3 CH3CH2CCH2CH3 CH 3 (d) There are 4 isomeric aldehydes with the formula C5H10O. Remember that the aldehyde functional group can occur only at the end of a chain. H if if CH3CH2CH2CH2CH if CH3CHCH2CH CH3CH2CHCH CH3 (e) There are 4 esters with the formula (f) CH3 C4 H8 O O O II II II CH 3 CH 2 COCH 3 CH 3 COCH 2CH 3 O CH 3 II HCOCH 2 CH 2 CH 3 - H CH 3 flip CI Cf-*— —*- H trans- 1 -Chloro-3-methylcyclohexane 2 (H - CH3 ) = 2 (H 7.6 kJ/mol The second conformation is more stable than the - CI) = 2.0 kJ/mol first. (b) cis- 1 -Ethyl-2-methylcyclohexane oneCH 3 ^ CH 2 CH 3 gauche (H-CH 2 CH 3 ) = = 3.8 kJ/mol 8.0 kJ/mol Total = 11.8 kJ/mol interaction 2 The second conformation is slightly more oneCH 3 ^ CH 2 CH 3 gauche 2 3.8 kJ/mol 7.6 kJ/mol = 11.4 kJ/mol Total stable than the = (H-CH 3 ) - interaction first. Chapter 4 cis- 1 -Bromo-4-ethy lcyclohexane 2 (H - CH2 CH3 = ) The second conformation 2 (H 8.0 kJ/mol is more stable than the - Br) = 2.0 kJ/mol first. C(CH3 ) 3 cis- 1 -te7t-Butyl-4-ethylcyclohexane 2 [H - C(CH3 )3] = The second conformation 2 (H 22.8 kJ/mol is more stable than the - CH2 CH3 ) = 8.0 kJ/mol first. three substituents have the orientations shown in the first structure. To decide if the conformation shown is the more stable conformation or the less stable conformation, perform a ring-flip on the illustrated conformation and do a calculation of the total strain in each structure as in the previous problem. Notice that each conformation has a CI-CH3 gauche interaction, but we don't need to know its energy cost because it is present in both conformations. The - CH 2rH~m~ 2 [H The illustrated conformation 3] is = 7.6 kJ/mol =2.0k.l/ m ol = 9.6 kJ/mol the less stable chair form. 2 [H - CH3 ] = 7.6 kJ/mol 7ra«s-decalin is more stable than cw-decalin because of three 1 ,3-diaxial interactions present in the cis isomer. You can recognize these interactions if you think of the circled parts of cw-decalin as similar to axial methyl groups. The gauche interactions that occur with axial methyl groups also occur in cw-decalin. 4.21 Both ring fusions are trans because the bridgehead groups are on opposite faces of the fused ring system. Visualizing Chemistry 4.22 (a) 4.23 H CI trans- 1 -Chloro-3-methylcyclohexane 2 (H ~ CH 3 ) = 7.6 kJ/mol The conformation shown (the left 4.24 2 (H ~ CI) = 2.0 kJ/mol structure) is the less stable conformation. 74 4.25 Chapter 4 The green substituent is axial, and the red and blue substituents are equatorial. •;'^reerO 4.26 difference between a-glucose and /3-glucose is in the orientation of the -OH carbon 1 the -OH group is axial in a-glucose, and it is equatorial in /3-glucose. would expect /3-glucose to be more stable because all of its substituents are in the The only group You at : equatorial position. Additional Problems Cycloalkane Isomers 4.27 The 4.28 last two structures are cis-trans isomers. Constitutional isomers differ in the way that atoms are connected. ds-l,2-Dibromo- d5-l,3-Dibromo- cyclopentane cyclopentane fra«s-l,3-Dibromocyclopentane constitutional isomers of c/s-l,2-dibromocyclopentane Organic Compounds: Cycloalkanes and Their Stereochemistry 4.29 Stereoisomers have different three-dimensional geometry. H H3C X/SX H cis- 1 ,3-Dimethylcyclobutane trans- 1 ,3-Dimethylcyclobutane 4.30 (a) Br constitutional > isomers Br' H cis- 1 ,3-Dibromocyclohexane (b) trans- 1 ,4-Dibromocyclohexane CH 3 H H3 3^ ^ CHgCh^C^- CHCHg CHgCH^Ch^CHCHCHg CH 3 constitutional isomers H 3C 2,3-Dimethylhexane 2,3 ,3-Trimethylpentane (c) CI 4.31 f CI CI > identical Stereoisomers H CI CI cis- 1 ,2-Dichlorocyclobutane trans- 1 ,2-Dichlorocyclobutane CI v^ci H H Constitutional isomers of ?ra«s-l,2-dichlorocyclobutane: H CI L* CI L* CI 1,1-Dichlorocyclobutane c/s-l,3-Dichlorocyclobutane j H trans- 1 ,3-Dichlorocyclobutane is also a constitutional isomer. 75 Width: 612 Height: 792 76 Chapter 4 4.32 CH 2 OH OH red cis relationship: red-green, blue-black trans relationship: red-blue, green-blue red-black, green-black OH blue 4.33 Two cis-trans isomers of 1,3,5-trimethylcyclohexane are possible. In one isomer (A), methyl groups are cis; in B, one methyl group is trans to the other two. all Cycloalkane Conformation and Stability 4.34 4.35 Make a model same all cis substituents are on the two adjacent cis substituents have an axial-equatorial Now, perform a ring-flip on the cyclohexane. of cis- 1 ,2-dichlorocyclohexane. Notice that side of the ring and that relationship. After the ring-flip, the relationship of the two substituents is still axial-equatorial. No two adjacent cis substituents can be converted to being both axial or both equatorial by a ringflip. Don't forget that there are only two chair conformations of any given cyclohexane. Organic Compounds: Cycloalkanes and Their Stereochemistry 4.36 77 For a trans- 1 ,2-disubstituted cyclohexane, two adjacent substituents must be either both axial or both equatorial. A ring flip converts two adjacent axial substituents to equatorial substituents, and vice versa. As in Problem 4.35, no two adjacent trans substituents can have an axial-equatorial relationship. 4.37 A cis-l,3-disubstituted isomer exists almost exclusively in the diequatorial conformation, which has no The trans isomer must have one group axial, leading Thus, the trans isomer is less stable than the cis isomer. When a molecule has two conformations available, the molecule exists mainly in the lower energy conformation. 1,3-diaxial interactions. to 1,3-diaxial interactions. 4.38 X trans cis The trans- 1 ,4-isomer is more stable because both substituents can be equatorial. 78 Chapter 4 4.39 more less stable stable trans cis Two types of interaction are present in ds-l,2-dimethylcyclobutane. One interaction occurs between the two methyl groups, which are almost eclipsed. The other is an across-the-ring interaction between methyl group at position 1 of the ring and a hydrogen at position 3. Because neither of these interactions are present in trans isomer, it is more stable than the cis isomer. In fra/«-l,3-dimethylcyclobutane, an across-the-ring interaction occurs between the methyl group at position 3 of the ring and a hydrogen at position 1 Because no interactions are present in the cis isomer, it is more stable than the trans isomer. . 4.40 To solve this problem: (1) Find the energy cost of a 1,3-diaxial interaction by using Table Convert this energy difference into a percent by using Figure 4.12. 4.1. (2) (a) 2 (H «* CH(CH 3 ) 2 ) = % equatorial % axial (b) H *—-»- F 9.2 kJ/mol = 97 = 3 2(H**F) = 1.0 kJ/mol % equatorial = 63 % axial = 37 (c) H - CN 2 (H — CN) = 0.8 kJ/mol % equatorial = 60 % axial = 40 Organic Compounds: Cycloalkanes and Their Stereochemistry 4.41 Make sure you know the difference between axial-equatorial and 79 cis-trans. Axial substituents are parallel to the axis of the ring; equatorial substituents lie around the equator of the ring. Cis substituents are on the same side of the ring; trans substituents are on opposite side of the ring. (a) 1,3-trans axial, equatorial equatorial, axial (c) 1,3-cis equatorial, equatorial axial, axial (d) 1,5-trans is the (e) 1,5-cis is the (f) same same as 1,3-trans as 1,3-cis 1,6-trans equatorial, equatorial axial, axial 1,6-trans is the same as 1,2-trans. 80 Chapter 4 Cyclohexane Conformational Analysis 4.42 cis- 1 -Chloro-2-methylcyclohexane Use Table 1 ,3-diaxial interactions. For the first conformation, the kJ/mol = 2.0 kJ/mol. The steric strain in the second conformation is 2 x 3.8 kJ/mol, or 7.6 kJ/mol. The first conformation is more stable than the second conformation by 5.6 kJ/mol. 4. 1 steric strain is to find the values of 2x 1.0 4.43 no 2 x 3.8 kJ/mol = 7.6 kJ/mol 2 x 1 .0 kJ/mol = 2.0 kJ/mol 1,3-diaxial interactions = 9.6 kJ/mol trans- 1 -Chloro-2-methylcyclohexane Use Table 4.1 to find the values of 1,3-diaxial interactions. stable than the second conformation by a The first conformation maximum of 9.6 kJ/mol. (A gauche is more interaction between the two substituents in the diequatorial conformation reduces the value of the energy difference, but its value can't be determined with the given data.) In this conformation, all substituents, except for one hydroxyl group, are equatorial. Organic Compounds: Cycloalkanes and Their Stereochemistry 4.45 81 drawing you can see that the methyl group and the -OH group have a and the isopropyl group has a trans relationship to both of these groups. Draw a chair cyclohexane ring and attach the groups with the correct relationship. From the flat-ring cis relationship, (CH 3) 2 CH^Z^^^/ HO H CH(CH 3 ) 2 In this conformation, all substituents are equatorial. Now, perform a ring flip. (CH 3 2 CH ) "7^ CH, Hcry^ (CH 3 2 CH ring flip ) CH 3 H The second conformation is less stable because all substituents are axial. 4.46 X X H3C H H3C H~ TV HO-- CH(CH 3 H HO ) 2 Menthol The substituents ) , 2 — CH 3l — OH H v H CH(CH 3 ) • H 2 " * CH(CH 3 -->V HO ) H 2 on the ring have the following relationships: — CH(CH 3 2 — CH 3 -CH(CH 3 • H cis-trans isomers of menthol Menthol ) HO— -OH Isomer A Isomer B H Isomer trans trans cis CIS trans cis trans cis cis trans trans cis C CH(CH 3 ) 2 82 Chapter 4 4.47 diequatorial The between conformations between the two methyl groups. large energy difference interaction 4.48 diaxial Diaxial cis- 1 ,3-dimethylcyclohexane contains three CH3 interactions of 3.8 kJ/mol each, and one CH3 is 1 ** due to the severe 1 ,3 ,3-diaxial interactions CH3 diaxial - two H ** interaction. If the diaxial conformation is 23 kJ/mol less stable than the diequatorial, 23 kJ/mol - 2(3.8 kJ/mol) kJ/mol of this strain energy must be due to the CH3 ** CH3 interaction. » 15 4.49 2 H CH3 interactions = 7.6 kJ/mol 2 1 H ** CH? CH3 interactions ~ CU2 interaction Conformation A is favored. Conformation A of c/s-l-cMoro-3-methylcyclohexane has no = 7.6 kJ/mol = 15 k.T/mol m 23 kJ/mol 4.50 1,3-diaxial interactions and due to one CH3 ** H interaction (3.8 kJ/mol), one CI ** H interaction (1.0 kJ/mol) and one CH3 ** CI interaction. Since the total-strain energy of B is 15.5 kJ/mol, 15.5 kJ/mol - 3.8 kJ/mol - 1.0 kJ/mol = 10.7 kJ/mol of strain is caused by a CH3 ** CI interaction. is the more stable conformation. Steric strain in B is Organic Compounds: Cycloalkanes and Their Stereochemistry 83 General Problems 4.51 Note: In working with decalins, it is essential to use models. Many structural features of decalins that are obvious with models are not easily visualized with drawings. H H H ds-Decalin /rans-Decalin No 1,3-diaxial interactions are present in fraws-decalin. At the ring junction of cw-decalin, one ring acts as an axial substituent of the other (see The circled part of ring has two 1,3-diaxial interactions with ring (indicated A B circled bonds). by arrows). Similarly, the circled part of ring A has two 1,3-diaxial interactions with ring B; one of these interactions is the same as an interaction of part of the B ring with ring A. These three 1,3-diaxial interactions have a total energy cost of 3 x 3.8 kJ/mol = 1 1.4 kJ/mol. C/f-decalin is therefore less stable than trans-decakn by 1 1.4 kJ/mol. 4.52 A ring-flip converts an axial substituent into an equatorial substituent and vice versa. ring junction of frarcs-decalin, each ring At the a trans-trans diequatorial substituent of the other. If a ring-flip were to occur, the two rings would become axial substituents of each other. You can see with models that a diaxial ring junction is impossibly strained. Consequently, ?ra/w-decalin does not ring-flip. The is by an axial bond and an equatorial bond. After a by an equatorial and an axial bond. No additional strain rings of ds-decalin are joined ring-flip, the rings are still linked or interaction is introduced by a ring-flip of c/s-decalin. H 84 4.53 Chapter 4 Build models to see the stability difference between the two [4. 1 .0] ring systems. In both cases, fusing a three-membered ring to a six-membered ring distorts the bond angles of both rings, causing angle strain. This strain is much more severe in the trans isomer than in the cis isomer. H H H c/s-Bicyclo[4. 1 .Olheptane rram -Bicyclo[4. 1 .0]heptane , 4.54 Simvastatin (Zocor) Pravastatin (Pravachol) Atorvastatin (Lipitor) (a) (b) The indicated bonds on simvastatin are trans. The -H bond and the -OH bond have a cis relationship. The third bond is trans to both of them. (c) The three indicated bonds on atorvastatin are attached to sp -hybridized carbons of a planar ring and he in the same plane. Organic Compounds: Cycloalkanes and Their Stereochemistry 4.55 In the flat-ring structure shown, all 85 -OH groups have a alternating relationship except for had a trans relationship, the most stable conformation would have all -OH groups in the equatorial position. We expect that the most stable conformation of this structure has one group in the axial position. Draw both rings and add -OH groups having the indicated relationships. Perform a ring-flip on the structure you have drawn to arrive at the other conformation. the starred group (below). If all of the groups 4.56 OH OH OH OH There are eight cis-trans stereoisomers of rayo-inositol. The first isomer because all hydroxyl groups can assume an equatorial conformation. is the most stable Width: 612 Height: 792 86 Chapter 4 4.57 1-Norbornene If you build a model of 1-norbornene, you will find that it is almost impossible to form the bridgehead double bond. s/^-Hybridization at the double bond requires all carbons bonded to the starred carbons to lie in a common plane in order for the p orbitals to overlap to form the n bond.The bicyclic ring system forces these atoms out of plane, and the bridgehead double bond can't form. 4.58 A steroid ring system is fused, and ring-flips don't occur. Thus, substituents such as the methyl groups shown remain axial. Substituents on the same side of the ring system as the methyl groups are in alternating axial and equatorial positions. Thus, an "up" substituent at C3 (a) is equatorial. Substituents on the bottom side of the ring system also alternate axial and equatorial substituent at C7 (b) is axial, and one at CI 1 ( c) is equatorial positions. A 4.59 4.60 The two fra«s-l,2-dimethylcyclopentanes are mirror images. Organic Compounds: Cycloalkanes and Their Stereochemistry 4.61 87 four possible isomers of 4-terr-butylcyclohexane- 1 ,3-diol. Make models of these isomers also. The bulky tert-butyl group determines the stable conformation because of its strong preference for the equatorial position. Draw the HO Only when the two hydroxyl groups are cis diaxial (structure 1) can the acetal ring form. In any other conformation, the oxygen atoms are too far apart to be incorporated into a six- membered ring. (CH 3 3 C ) 4.62 H All four conformations of the two isomers are illustrated. The second conformation of each pair has a high degree of steric strain, and thus each isomer adopts the first conformation. Since only the cis isomer has the hydroxyl group in the necessary axial position, faster than the trans isomer. it oxidizes Chapter 5 - Stereochemistry at Tetrahedral Centers Chapter Outline I. Handedness (Sections 5.1-5.4). A. Enantiomers and tetrahedral carbon (Section 5.1). 1 When four different groups are bonded to a carbon atom, two . different arrangements are possible. a. These arrangements are mirror images. b The two mirror-image molecules are enantiomers. The reason for handedness in molecules: chirality (Section 5.2). 1 Molecules that are not superimposable on their mirror-images are chiral. a. A molecule is not chiral if it contains a plane of symmetry. b A molecule with no plane of symmetry is chiral. 2 A carbon bonded to four different groups is a chirality center. 3 It is sometimes difficult to locate a chirality center in a complex molecule. 4. The groups -CH 2- -CH 3 C=0, C=C, and CaC can't be chirality centers. . B . . . . . , C. Optical 1 . 2 . 3 . activity (Section 5.3). Solutions of certain substances rotate the plane of plane-polarized light, a. These substances are said to be optically active. The angle of rotation can be measured with a polarimeter. The direction of rotation can also be measured. A compound whose solution rotates plane-polarized light to the right is termed a. dextrorotatory. b A compound whose solution rotates plane-polarized light to the left is termed . levorotatory. 4 . Specific rotation. a. The b Specific rotation c . . extent of rotation depends on concentration, path length and wavelength. is the observed rotation of a sample with concentration = 1 g/mL, sample path length of 1 dm, and light of wavelength = 589 nm. Specific rotation is a physical constant characteristic of a given optically active compound. D. Pasteur's discovery of enantiomerism (Section 5.4). Pasteur discovered two different types of crystals in a solution that he was 1 . evaporating. The crystals were mirror images. Solutions of each of the two types of crystals were optically active, and their specific rotations were equal in magnitude but opposite in sign. 4 Pasteur postulated that some molecules are handed and thus discovered the phenomenon of enantiomerism. Stereoisomers and configurations (Sections 5.5-5.8). A. Specification of configurations of stereoisomers (Section 5.5). 2 3 . . . II. 1 . Rules for assigning configurations at a chirality center: a. Assign priorities to each group bonded to the carbon by using Cahn-Ingold-Prelog rules. i Rank each atom by atomic number. (a). An atom with a higher atomic number receives a higher priority than an atom with a lower atomic number. If a decision can't be reached based on the first atom, look at the second or ii. . third iii. atom until a difference is found. Multiple-bonded atoms are equivalent to the same number of single-bonded atoms. Stereochemistry at Tetrahedral Centers 89 Orient the molecule so that the group of lowest priority is pointing to the rear. Draw a curved arrow from group 1 to group 2 to group 3. d. If the arrow rotates clockwise, the chirality center is R, and if the arrow rotates counterclockwise, the chirality center is S. 2. The sign of optical rotation is not related to R,S designation. 3 X-ray experiments have proven R,S conventions to be correct. B. Diastereomers (Section 5.6). molecule with two chirality centers can have four possible stereoisomers. 1 a. The stereoisomers group into two pairs of enantiomers. b stereoisomer from one pair is the diastereomer of a stereoisomer from the other pair. 2 Diastereomers are stereoisomers that are not mirror images. Epimers are diastereomers whose configuration differs at only one chirality center. 3 C. Meso compounds (Section 5.7). 1 meso compound occurs when a compound with two chirality centers possesses a plane of symmetry. b . c. . . A . A . . . 2. A A meso compound is achiral despite having two chirality centers. The physical properties of meso compounds, diastereomers and racemic mixtures from each other and from the properties of enantiomers. D. Racemic mixtures and the resolution of enantiomers (Section 5.8). 1 A racemic mixture (racemate) is a 50:50 mixture of two enantiomers. a. Racemic mixtures show zero optical rotation. 2 Some racemic mixtures can be resolved into their component enantiomers. 3 . differ . . a. If a racemic mixture of a carboxylic acid reacts with a chiral amine, the product ammonium b . salts are The diastereomeric diastereomers. salts differ in chemical and physical properties and can be separated. c The original enantiomers can be recovered by acidification. of isomerism (Section 5.9). A. Constitutional isomers differ in connections between atoms. 1 Skeletal isomers have different carbon skeletons. 2 Functional isomers contain different functional groups. 3 Positional isomers have functional groups in different positions. B Stereoisomers have the same connections between atoms, but different geometry. 1 Enantiomers have a mirror-image relationship. 2. Diastereomers are non-mirror-image stereoisomers. a. Configurational diastereomers. b Cis-trans isomers differ in the arrangement of substituents on a ring or a double bond. IY. Chirality at atoms other than carbon (Section 5.10). A. Other elements with tetrahedral atoms can be chirality centers. B Trivalent nitrogen can, theoretically, be chiral, but rapid inversion of the nitrogen lone pair intercon verts the enantiomers. C Chiral phosphines and trivalent sulfur compounds can be isolated because their rate of . III. A review . . . . . . . . inversion is slower. V. Prochirahty (Section 5.11). A. A molecule is prochiral step. if it can be converted from achiral to chiral in a single chemical Chapter 5 90 B . Identifying prochirality. 2 1 . For sp~ carbon, draw the plane that includes the atoms bonded to the sp carbon. a. Assign priorities to the groups bonded to the carbon. b. Draw an curved arrow from group 1 to group 2 to group 3. c. The face of the plane on which the curved arrow rotates clockwise is the Re face. 2 . The face on which the arrow rotates counterclockwise is the Si face. -hybridized may have a prochirality center if, when one of its attached groups is replaced, it becomes a chirality center. a. For -CH2X, imagine a replacement of one hydrogen with deuterium. b. Rank the groups, including the deuterium. c. If the replacement leads to R chirality, the atom is pro-R. d. An atom that is sp S chirality, the atom is pro-S. involve prochiral compounds. VI. Chirality in nature and chiral environments (Section 5.12). A. Different enantiomers of a chiral molecule have different properties in nature. 1 (+)-Limonene has the odor of oranges, and (-)-limonene has the odor of lemons. 2 Racemic fluoxetine is an antidepressant, but the S enantiomer is effective against migraine. B In nature, a molecule must fit into a chiral receptor, and only one enantiomer usually d . If the replacement leads to C Many biochemical reactions . . . . fits. Solutions to Problems 5. 1 5.2 Objects having a plane of symmetry are achiral. Chiral: screw, shoe. Achiral: soda can, screwdriver. Use the following rules to locate centers that are not chirality centers, then examine the remaining centers to find a carbon with four different groups attached. . All -CH3 and -CX3 carbons 2. All -CH2- and -CX2- carbons 1 Coniine are not chirality centers. are not chirality centers. Menthol Dextromethorphan Stereochemistry at Tetrahedral Centers 5 3 . Refer to Problem 5.2 you need if help. C0 2 H C0 2 H H2N ™* H3 C 91 / CvV-- H Alanine CH 3 5.4 (b) (a) HO \ V H H r C^C^ H H F *0 By F \ F ; ^Q? ^O' F HO H CI Enflurane Threose 5.5 F w \ I convention, a (-) rotation indicates rotation to the left, and thus cocaine is levorotatory. 5.6 Use = the formula [a] D , / [a] D = specific rotation a = observed = / C = « = In this problem: = I 5 7 . Review the sequence Rule 1 rotation path length of cell (in dm) concentration (in g/mL) 1.21° 5.00 C = [a]D u where C x cm = 1.50 g/10.0 0.500 mL dm = 0.150 g/mL = 0.500 dm = + rules presented in Section 5.5. An atom with 16.1° x 0.150 g/mL a higher atomic A summary: number has priority over an atom with a lower atomic number. Rule 2 be reached by using Rule 1 look at the second, third, or atom away from the double-bond carbon until a decision can be made. Multiple-bonded atoms are equivalent to the same number of single-bonded If a decision can't , fourth Rule 3 atoms. Higher (a) -Br (c)-CH 2 CH 3 (e) -CH 2 OH Lower -H -CH 3 -CH 3 Rule 1 2 2 (b) Higher Lower -Br -CI (d)-OH (f) -CH=0 -NH 2 -CH 2 OH Rule 1 1 3 92 5 8 . Chapter 5 Use the sequence rules in Section 5.5. By Rule (a) 1, -CH 2 CH 2 OH of lowest priority, and -OH is of highest priority. of higher priority than -CH 2 CH 3 -H is is By Rule 2, . Lowest Highest -OH, -CH 2 CH 2 OH, -CH 2 CH 3 -H , —O O — C— OH. \ / (b) By Rule 3, -CO2H is considered as Because 3 oxygens are attached carbon and only one oxygen is attached to -CH2OH, -CO2H is of higher -CH2OH. -CO2CH3 is of higher priority than -CO2H by Rule 2, and -OH of highest priority by Rule 1 to a -CO2H priority than »» Highest (b) (c) (d) 5 9 . is Lowest -OH, -C0 2 CH 3 -C0 2 H, -CH 2 OH -NH 2 -CN, -CH 2 NHCH 3 -CH 2 NH 2 — SSCH3, -SH, -CH2 SCH 3 -CH 3 . , , , All stereochemistry problems are easier if you use models. Part (a) will be solved by two methods - with models and without models. (a) With models: Build a model of (a). Orient the model so that group 4 is pointing to the rear. Note the direction of rotation of arrows that go from group 1 to group 2 to group 3. The arrows point counterclockwise, and the configuration is S. Without models: Imagine yourself looking at the molecule, with the group of lowest Your viewpoint would be at the upper right of the molecule, and you would see group 1 on the left, group 3 on the right and group 2 at the bottom. The arrow of rotation travels counterclockwise, and the configuration is S. priority pointing to the back. Stereochemistry at Tetrahedral Centers 5.10 93 Step 1. For each chirality center, rank substituents by the Cahn-Ingold-Prelog system; give the number 4 to the lowest priority substituent. For part (a): Substituent Priority -SH -CO9H -CH 3 -H 1 2 3 4 As in the previous problem, orient yourself so that you are 180° from the lowest group (indicated by the arrow in the drawing). From that viewpoint, draw the molecule as it looks when you face it. Draw the arrow that travels from group 1 to group 2 to group 3, and note its direction of rotation. The molecule in (a) has S configuration. Step 2. priority In (b), the observer is behind the page, looking out and down toward the right. In (c), the observer is behind the page looking out and up to the left. CH 3 CH 2 CH 2 CCH 3 (5)-2-Pentanol H Substituent -OH — CH2CH2CH3 -CH 3 -H Priority 1 2 3 4 HO CH 3 V CH2CH2CH3 HO J CH, CH2CH2CH3 94 5.12 Chapter 5 Fortunately, methionine 3 * CH 3 SCH 2 CH^ 4 is shown in the correct orientation. ^2 C0 2 H (^-Methionine NH 2 5.13 For (Note: the phosphate group (a): H represented as P.) 9H ° pCh>? H0A H^^OH | 9 H °nu HW^OH m is ! ] H^pOH 3CH(OH) H^f^OH CH 2 0P CH 2 OP CH 2 OP (d) 5,5 d are enantiomers and are diastereomeric with and are diastereomeric with b, c. (b) 5,/? a, (c) b, c are enantiomers Structure (a) and is CHO HoCTT2cH(CH \ Vg> R f ; j SCH^OP a, d. D-erythrose 4-phosphate, structure (d) is its enantiomer, and structures (b) (c) are its diastereomers. 5.14 N— CH 3 Morphine Morphine has five chirality centers and, in principle, can have 2 = 32 stereoisomers. Most of these stereoisomers are too strained to exist. 5.15 HoC Isoleucine H0 2 c \ CH 2 CHg g H H NH 2 Stereochemistry at Tetrahedral Centers 5.16 To decide if a structure represents a meso compound, try to locate a plane 95 of symmetry that divides the molecule into two halves that are mirror images. Molecular models are always helpful. (a) OH / / plane of symmetry OH meso *H (b) and (d) meso not (c) are structures. H H CH 3 Br^ A CH, ! plane of symmetry H3C r Br H ! ^ ch, H Br 5.17 ! meso For a molecule to exist as a meso form, it must possess a plane of symmetry. 2,3Butanediol can exist as a pair of enantiomers or as a meso compound, depending on the configurations at carbons 2 and 3. (a) H H HO k ^ CH 3 H 3 C^ ! ~ fs S n r H3 C OH A OH CH HO, plane of ff? A? ! H not meso (b) 2,3-Pentanediol has symmetry c HO r ^ CH 3 ! (c) ! ' HO H no symmetry plane and thus 2,4-Pentanediol can exist in a ! H ^ CH 3 meso can't exist in a meso form. meso form. H HO^ > CH 3 ! CH, HO r ^ ! 7 plane of symmetry CH, H 2,4-Pentanediol can also exist as a pair of enantiomers (2R,4R) and (25,45') that are not meso compounds. Width: 612 Height: 792 96 5.18 Chapter 5 The molecule meso compound. The symmetry plane passes through represents a carbon bearing the groups. the -OH group and between thr two ring carbons that are bonded to methyl HoC H plane of symmetry OH 5.19 O W C— OH + /> CH 3 HO— C a \ H3 C W catalyst / The product is the pure S-ester. + H3C (5)-2-Butanol and the configuration /> CH 3 c— o— CH 2 CH 3 Acetic acid H O acid HoO CH 2 CH 3 sec-Butyl acetate No new chirality centers are at the chirality center of (5")-2-butanol formed during the reaction, is unchanged. 5.20 C0 2 H H j CH 3 HO C0 2 CH 3 S C RC j" I H2 N H' " ? CH< H3N + HO Y An R,S salt + HO S HO H-.q^CHj S | C0 2 H H3 N + I CO, An The two product 5.21 salts S,S salt have the configurations (R,S) and (S,S) and are diastereomers. (a) CH, ^ CH 2 CH= CHCH (5)-5-Chloro-2-hexene 3 Chlorocyclohexane These two compounds are constitutional isomers (b) The two dibromopentane stereoisomers (skeletal isomers), are diastereomers. Stereochemistry at Tetrahedral Centers 5.22 97 2 For each molecule, replace the left hydrogen with H. Give priorities to the groups and assign R,S configuration to the chirality center. If the configuration is R, the replaced hydrogen is pro-R, and if the configuration is S, the replaced hydrogen is pro-S. (a) 2 1 CHO 5 HO HO H H -* H pro-S H H — pro-R CHO iJ CH(OH)CHO HO H (5)-Glyceraldehyde 0>) 2 1 pro-R H H (^-Phenylalanine 5.23 2 Draw the plane that includes the sp carbon and its substituents, and rank the substituents. For the upper face, draw the arrow that proceeds from group 1 to group 2 to group 3. If the direction of rotation is clockwise, the face counterclockwise, the face is the Si face. Hydroxyacetone 5.24 Use the 2 Re face; if rotation Crotyl alcohol Si face strategy in the previous sp carbon. the (b) /teface (a) is problem Re is face Si face to identify the faces of the plane that contains the Draw the product that results from reaction at the Re face, and configuration to the chirality center. assign 98 5.25 Chapter 5 Addition of -OH takes place on the Re face of C2 of aconitate. Addition of -H occurs on the Re face of C3 to yield (2/?,3S)-isocitrate. add from opposite sides of the and double bond. OH H "OoC m-Aconitate >C— CO? (2/?,3S)-Isocitrate o 2 cch ? — a. 3 Visualizing Chemistry 5.26 Structures (a), (b), and (d) are identical (R enantiomer), and (c) represents the enantiomer. 5.27 (a) 2 /C0 2 H H0 2 C^ A\ H CH 2 OH ^< H2NV 4 1 NH 2 C CHoOH (5)-Serine ( b> H OH 1^ 4 ^2 NHCH 3 HO. HO vH t ^ CH2NHCH 9 3 R ) HO R= ring (/?)-Adrenaline 5.28 Locate the plane of symmetry that identifies the structure as a meso compound, (a) (b) (c) | IH H H H HO H OH H meso " H 2 N H H2N H >4X i meso S Stereochemistry at Tetrahedral Centers 5.29 HO H \ r S,C^ ^ NHCH 3 " C^o H3 C H 5.30 (a) /tn (C) Q Pseudoephedrine ^—^ (b) Additional Problems Chirality 5.31 and Optical Activity Chiral: (d) golf club, (e) spiral staircase Achiral: (a) basketball, (b) fork, (c) wine glass, (f) snowflake. 5.32 (a) CHo3 U CHo3 I CH3CH2CH2CHCH2CHCH3 2,4-Dimethylheptane has one chirality center. CHo3 CH9CH0 2 3 I I CH3CH2CCH2CHCH2CH3 5-Ethyl-3,3-dimethylheptane is achiral. CH 3 (c) / •' *"*~^ \ ' c/s-l,4-Dichlorocyclohexane is achiral. Note the plane of symmetry that passes through the -CI groups. 99 100 Chapter 5 5.33 (b) (a) (c) OH CI I CH 3 I CHoCHpCHpCHCHo CHoCHpCHpCHpCHCHo * CH3CH2CHCH—— * 2-Hexanol 2-Chloropentane 3-Methyl- 1 -pentene (d) CHo I CH3CH2CH2CH2CHCH2CH3 3-Methylheptane 5.34 OH OH CH 3 CH3CH2CHCH2CH3 CH 3 CCH 2 OH I CH3CH2CH2CH2CH2OH CH3CH2CH2CHCH3 * achiral achiral chiral OH OH CH 3 CHo achiral CH " I CH 3 CH 2 CHCH 2 OH CH3CH2CCH3 * chiral HOCH 2 CH 2 CHCH 3 CH 3 C-HCHCH 3 CH 3 achiral achiral CH 3 chiral 5.35 (a) (b) OH I I CH3CH2CHCH3 (d) (c) CH 3 CH2CH2CHCO2H Br I OH ! CHgCHCHCHg O Br I // CHgCHC^ H Erythronolide B has ten chirality centers. Stereochemistry at Tetrahedral Centers 101 Assigning Configuration to Chirality Centers 5.37 Identical molecules: b (5 enantiomer), c (/? enantiomer), d (5 enantiomer). Pair of enantiomers: a 5.38 The specific rotation of (2/?,3/?)-dichloropentane is equal in magnitude and opposite in sign to the specific rotation of (25,35)-dichloropentane because the compounds are enantiomers. There is no predictable relationship between the specific rotations of the (2R,3S) and (2/?,3/?) isomers because they are diastereomers. 5.39-5.40 H H i i HCW^CHg R H H H 3 C^^^OH HCW^CHg C^^OH i i H3 IS I* \.S \R CH2 CH2 CH2 CH2 )^?^OH HO^?^(C4H9 S ' HO^?^(C4H9 ) (C 4 H 9 )^?^OH R S (C 4 H 9 H H H 25,4/? 2RAS 25,45 enantiomers ) H 2RAR enantiomers The (2/?,45) stereoisomer is the enantiomer of the (25,4/?) stereoisomer. The (25,45) and (2RAR) stereoisomers are diastereomers of the (25,4/?) stereoisomer. 5.41 (a) G) —a (b) m ^ — a 102 Chapter 5 5.42 Lowest Highest (a) -C(CH 3 ) 3 , -CH=CH 2 -CH(CH 3 , ) 2 -CH 2 CH 3 , 7 (b) (c) \ -C^CH, -C(CH 3 , // ) 3 , -CH=CH 2 -C02 CH 3 -COCH 3 -CH 2 OCH 3 -CH 2 CH 3 , (d) -Br, (a) H , — CH 2 Br, -CN, , -CH 2 CH 2 Br 5.43 OH H (°) HOCH 2 OCH 3 C0 2 H 5.44 (a) (b) OH (c) H^S^-s^CHg HOwfl/\Sy OH pH CH 3 CH 2I H3C \ / *CH 3 CI H 5.45 (a) (b) O H H H ^^ S S HO H HO H CH 2 CH 2 CH 2 CH 2 C0 2 Prostaglandin Biotin 5.46 (b) (a) ^*~~4^ Clw V I 3 3 h> CH 3 9 CH 2CH 3 2 CI'y C ^CH2 CH2 CHo ^T^l CH 3 CH 2V > CI H- CI CH=CH 2 2 CH 2 CH 3 Stereochemistry at Tetrahedral Centers 5.47 (a) (b) H H 3 C. NH 2 H H */r J<§ CH 3 S,Ts \ \ x (+)-Xylose HO H Meso Compounds 5.51 (b) (a) (c) CH 2 CH 3 H^i^CHo C * J H^9^CH3 CH 2 CH 3 CH 2 CH 3 plane rW^OH H^l^CHo J C symmetry \Z > symmetry 7 / fi CH 3 plane . p H^V^OH C-, qi_I 2 q|_| This 3 £ also CH 3 compound is a meso compound. 103 104 Chapter 5 5.52 (a) H \ C^ H3 >s CH^CH^s OH ( b) f V H3 C CH 3 ! (c) plane of CH3 OH- symmetry HqC 5.53 plane of plane of symmetry symmetry Both of the diastereomers shown below are meso compounds with three chirality centers. Each is a meso compound because it has a symmetry plane, and in each structure the central carbon is bonded to four different groups (a group with R configuration, a group with S configuration, -OH, and -H). 9H 3 9 H3 S H*-C — OH >s H»-C-OH -H^e— OH— — I H©*-€^«rTt— If? '/? H—C-OH H—C-OH i i CH 3 5.54 (aHc) H H OH H H CHO H HO CHO HO H OH H OH HO H HO H Enantiomer of ribose Ribose Ribose has three chirality centers, which give (d) Ribose has six diastereomers. H H H .H OH H rise to eight (2~ H HO CHO HO H H OH HO H _H HO H H H OH H OH H CHO CHO H OH H OH HO H HO H H H H OH H _H HO H CHO HO H H stereoisomers, _H CHO H ) OH CHO H OHHO H Stereochemistry at Tetrahedral Centers 5.55 105 is an optically inactive meso compound. Catalytic hydrogenation converts the aldehyde functional group into a hydroxyl group and makes the two halves of ribitol mirror images of each other. Ribitol ; H H H OH plane of symmetry HjOH HOHgC^^f^. CH 2OH He CHO Pt catalyst ! HO H HO H 7\, HO H;h6 H i Ribose Ribitol Prochirality 5.56 ( a) pro-S A H (b) pro-R pro-R pro-S W V HO H ~ 3-r pro-R Malic acid pro-R pro-S A H H CH 3 S H0 2 C (c) H JH' COc CO, HS H H H3 N H H3N + I + Methionine Cysteine 5.57 (a) Re face i CO' t Pyruvate 5.58 Crotonate Si face Remember that each 2 sp carbon has a Re Re face face and a Si face. OH s ~ H 3 C , -c. j CH 2 C0 2 H t Re face H3C I Si face H+ (5)-3-Hydroxybutyrate Width: 612 Height: 792 106 Chapter 5 5.59 If you perform the "replacement test" to assign pro-R/pro-S prochirality you will see that "arm" of citrate is pro-R and the product pictured on the right is formed. The proS arm is unchanged. , the right "OoC 5.60 pro-S CH 2 OH HO*~ CH 2 OH ADp ATp HO»- H0 2 C A C0 2 H C0 2 H trans cis meso B a and C are enantiomers and are optically active. Compound A is their diastereomer and is meso compound, which is not optically active. The two isomeric cyclobutane-l,3-dicarboxyUc acids are achiral and are optically inactive. H0 2 C 7^ trans H0 2 C C0 2 H crs C0 2 H Stereochemistry at Tetrahedral Centers 5.63 H rR H02 C"/ C H 2 N"y CH 2 SH C ^-CH SH 2 C0 2 H HoN 5.64 H H H 2 N" j ^*CH ? SH C0 2 H V~C0 2 H HSCHo NH, H 2 N'~y H ^"CH2S— SCHg" C~C0 2 H C0 2 H Cystine has the (S,S) configuration and is optically active. 5.65 (a) H R Br (b) Br . H (25,3/?)-2,3-Dibromopentane 5.66 me.s0-3,5-Heptanediol All chirality centers of Cephalexin have an (R) configuration. H *V" P H H Cephalexin C0 2 H 5.67 H. OH CH 2OH H OoN NHCOCHCI 2 Chloramphenicol NH 2 107 108 Chapter 5 5.68 Mycomycin contains no chiral carbon atoms, yet To is chiral. see why, make a model of mycomycin. For simplicity, call -CH=CHCH=CHCH 2C0 2 H "A" and-OCOCH "B". The carbon atoms of an allene have a linear relationship and that the jt bonds formed are perpendicular to each other. Attach substituents carbons. at the sp" Hb H a and all carbon atoms he in a plane that is perpendicular Hb and all carbon atoms. Notice that the substituents A, to the plane that contains B, , , Now, make another model identical to the first, except for an exchange of A and H a This new allene is not superimposable on the original allene. The two allenes are enantiomers . and are 5.69 chiral because they possess no plane of symmetry. 4-Methylcyclohexylideneacetic acid is chiral for the same reason that mycomycin (Problem 5.68) is chiral: It possesses no plane of symmetry and is not superimposable on its mirror image. As in the case of allenes, the two groups at one end of the molecule lie in a plane perpendicular to the plane that contains the two groups at the other end. 5.70 ^ — 9 H3 ^ i H**^ ^T. <^ % CHoCHo CH 2 CI 9H3 s Clo s V /7 3 2 \ |_|-' _y CH 2 CI , 9H3 + 9 + I Qj-'C^R CHoCHoCI 2 2 _, CH 2 CI , (5)-l-Chloro- (5)-l,4-DicMoro- (/?)-l,2-Dichloro- 2-methylbutane 2-methylbutane products on OH ru ^H 2 C H CI 3 2 2-methylbutane Y 1 C2 is not + other q (5)-l,2-Dichloro- ^* (b) Chlorination at , CHoCHq 2 3 2-methylbutane H3 s C~ 1 : 1 mixture carbon 4 yields an optically active product because the chirality center at affected. Chlorination at carbon 2 yields an optically inactive racemic product. Stereochemistry at Tetrahedral Centers 109 5.71 9 H3 •S 9H3 9H3 >S IS H»-C— Br H**C** Br I I \R I Is \R Br^C-H H»»C-Br CH 3 A I H^C— CI CI^C— \R H^C-Br iR Br—C— Br CI-C^H H^C-CI 9 H3 CH 3 B H^C-*Br CH 3 C CH 3 D There are four stereoisomers of 2,4-dibromo-3-chloropentane. C and D are enantiomers and are optically active. A and B are optically inactive meso compounds and are diastereomers. 5.72 H 3 C- H cis- 1 (a) There trans- 1 ,4-Dimethylcyclohexane only one stereoisomer of each of the 1,4-dimethylcyclohexanes. ,4-dimethylcyclohexane is chiral. The two 1,4-dimethylcyclohexanes are diastereomers. is (b) Neither (c) H ,4-Dimethylcyclohexane 1 5.73 H H3 C cis- 1 ,3-Dimethylcyclohexane (a) There is H3C trans- 1 ,3-Dimethylcyclohexane one stereoisomer of cw-l,3-dimethylcyclohexane, and there are two stereoisomers of ?ra«s-l,3-dimethylcyclohexane. (b) cis- 1 ,3-Dimethylcyclohexane is an achiral meso compound; trans-1,3dimethylcyclohexane exists as a pair of chiral enantiomers. (c) The two trans stereoisomers are enantiomers, and both are diastereomers of the cis stereoisomer. 110 Chapter 5 5.74 CH 3 cis- 1 ,2-Dimethylcyclohexane The two ds-l^-dimethylcyclohexane enantiomers leading to an optically inactive 1 rapidly interconvert by a ring flip, mixture. 1 : 5.75 CH2CH3 HS: CH2CH3 — .C-pBr rf J {j HqC HS W The product 5.76 Cs :Br: CH 3 is (/?)-2-butanethiol. The reaction proceeds by addition of acetylide anion to the carbonyl group and occurs with equal probability from either face of the planar ketone carbon. : OH o: CH2CH3 HgC k § $ HC H3C 4 CH2CH3 H3C k HC CH2CH3 :c^CH^ „ 4,0 k HC^H3 C (a) The product is an (b) The two enantiomers OH U T. CH 2 CH 3 optically inactive racemic mixture. are formed in a 50:50 ratio. k HC^HgC CH 2 CH 3 Stereochemistry at Tetrahedral Centers 111 5.77 H N CH 3 .0 C— 2. H3 OH R ac-c? ^ + (3fl,4fl)-4-Phenyl- l-pentyn-3-ol CH (3S,4/?)-4-Phenyll-pentyn-3-ol Reaction of sodium acetylide with a chiral aldehyde yields chiral products; the product mixture is optically active. (b) The two products are a mixture of the (3R,4R) and (3S, 4R) diastereomers of 4-phenyll-pentyn-3-ol. The product ratio can't be predicted, but it is not 50:50. (a) Review Unit Major Topics Covered 2: Alkanes and Stereochemistry (with Vocabulary): Functional Groups. Alkanes: straight-chain alkane branched-chain alkane isomer constitutional isomer primary, secondary, tertiary, quaternary carbon IUPAC system of nomenclature primary, secondary, tertiary hydrogen paraffin cycloalkane cis-trans isomer stereoisomer saturated aliphatic alkyl group Alkane Stereochemistry: conformer sawhorse representation Newman projection staggered conformation eclipsed conformation torsional strain dihedral angle anti conformation gauche conformation steric strain angle strain heat of combustion chair conformation axial group equatorial group ring-flip 1, 3 -diaxial interaction conformational analysis boat conformation twist-boat conformation polycyclic molecules bicycloalkane Handedness: stereoisomer enantiomer chiral plane of symmetry achiral chirality center plane-polarized light optical activity levorotatory dextrorotatory specific rotation Stereoisomers and configuration: configuration Cahn-Ingold-Prelog rules absolute configuration diastereomer meso racemate resolution prochirality Re face Si face prochirality center pro-R pro-S compound Types of Problems: After studying these chapters, you should be able to: - draw molecules containing a given functional group. isomers of a given molecular formula. Name and draw alkanes and alkyl groups. Identify carbons and hydrogens as being primary, secondary or tertiary. Draw energy vs. angle of rotation graphs for single bond conformations. Draw Newman projections of bond conformations and predict their relative stability. Identify functional groups, and Draw all Understand the geometry of, and predict the stability of, cycloalkanes having fewer than 6 carbons. - Draw and name - Calculate the specific rotation of an optically active compound. Locate chirality centers, assign priorities to substituents, and assign R,S designations to substituted cyclohexanes, indicating cis/trans geometry. Predict the stability of substituted cyclohexanes by estimating steric interactions. chiraiity centers. - Given a stereoisomer, draw its enantiomer and/or diastereomers. Locate the symmetry plane of a meso compound. Assign Pro-R and Pro-S designations to prochiral groups. Identify the face of an sp 2 -hybridized carbon as pro-R or pro-S. Review Unit 2 Points to * 113 Remember: In identifying the functional groups in a compound, some groups have different designations that depend on the number and importance of other groups in the molecule. For example, a compound containing an -OH group and few other groups is probably named as an alcohol, but when several other groups are present the -OH group is referred to as a hydroxyl , of functional groups in the Appendix of the textbook, and priority order will become more apparent as you progress through the text. group. There * It is is surprising a priority list how many errors can be made in naming compounds this as simple as alkanes. Why is this? Often the problem is a result of just not paying attention. It is very easy to undercount or overcount the -CH 2 - groups in a chain and to misnumber substituents. Let's work through CH>3 I a problem, using the rules in Section 3.4. CHpCHq I CHgCCh^CHCH^CHg CH 3 Find the longest chain. In the above compound, the longest chain is a hexane (Try all compound. Identify the substituents. The compound has two methyl groups and an ethyl group. It's a good idea to list these groups to keep track of them. Number the chain and the groups. Try both possible sets of numbers, and see which results in the lower combination of numbers. The compound might be named either as a 2,2,4-trisubstituted hexane or a 3,5,5-trisubstituted hexane, but the first name has a lower combination of numbers. Name the compound, remembering the prefix di- and remembering to list substituents in alphabetical order. The correct name for the - possibilities; there are two different six-carbon chains in the above compound is 4-ethyl-2,2-dimethylhexane. The acronym FINN (from the first letters of each * When step listed above) ) may be helpful. performing a ring-flip on a cyclohexane ring, keep track of the positions on the ring. X A helpful strategy for assigning R,S designations: Using models, build two enantiomers by adding four groups to each of two tetrahedral carbons. Number the groups to represent priorities of groups at a tetrahedral carbon, and assign a configuration to each carbon. Attach a label that indicates the configuration of each enantiomer. Keep these two enantiomers, and use them to check your answer every time that you need to assign R,S configurations to a chiral atom. When assigning pro-R or pro-S designations to a hydrogen, mentally replace the hydrogen The other hydrogen is then positioned for prochiraUty assignment without manipulating the molecule. If the designation is R, the replaced hydrogen is pro-R; if the designation is S, the replaced hydrogen is pro-S. that points out of the plane of the page. 114 Review Unit 2 Self-test CH(CH 3 ) CH, 2 CH 3 CHCH2CH2CH 2 CHNHCH(CH3)2 CH3CHCH2CH2CCH2^/HCH2CH3 A CH 3 CH 2 CH 3 B Metron S (an antihistamine) Name A, and identify carbons as primary, secondary, tertiary or quaternary. an amine with two alkyl substituents. Name these groups and identify alkyl hydrogens as primary, secondary or tertiary. B is CH 3 CH 3 -=< CHCOCHg W C CCH 2 OCH 3 CH 3 o HoC' Metalaxyl (a fungicide) Identify all functional groups of C D Br D (metalaxyl). Name and indicate the cis/trans relationship of the substituents. conformations, and calculate the energy difference between them. Ubenimex (an antitumor drug) Draw both possible chair Epiandosterone (an androgen) Assign R,S designations to the chiral carbons in E. Label the circled hydrogen as pro-R or pro-S. Indicate the chirality centers in F. How many stereoisomers of F are possible? Review Unit 2 Multiple Choice 1 . Which of the following (a) 2 . 3 . 5 . . contains primary, secondary, tertiary and quaternary carbons? (a) 2,2,4-Trimethylhexane (b) Ethylcyclohexane (c) 2-Methyl-4-ethylcyclohexane (d) 2,2-Dimethylcyclohexane How many isomers of the formula C^sB^ are there? . 4 (b)6 (c)8 (d) 9 The lowest energy conformation of 2-methylbutane occurs: (a) when all methyl groups are anti (b) when all methyl groups are gauche (c) when two methyl groups are anti (d) when two methyl groups are eclipsed The (a) 6 functional groups doesn't contain a carbonyl group? (c) ether (d) ketone (b) ester Which of the following compounds (a) 4 aldehyde In strain in a cyclopentane ring is due to: angle strain (b) torsional strain (c) steric stain which molecule do the substituents in the more (d) angle strain and torsional stable conformation have a diequatorial relationship? (a) cis- 1 ,2 disubstituted (b) cis- 1 ,3 disubstituted (c) trans- 1 ,3-disubstituted (d) cis- 1,4 disubstituted 7 . Which group is (a)-CH(CH 3 ) 2 8 A meso compound and a racemate are identical in all respects except: (a) (c) 9. -CH=CH2? -CH=C(CH 3 ) 2 (c)-C s CH (d)-C(CH 3 ) 3 of lower priority than (b) molecular formula connectivity of atoms Which of the following (b) degree of rotation of plane-polarized light (d) physical properties projections represents an (C) R enantiomer? H0 2 C H^ c ^ CH 3 ' i H 10. How many prochirality centers does (a) none (b) 1 (c) 2 (d) 3 Br 1-bromobutane have? strain (d) 115 Width: 612 Height: 792 Chapter 6 - An Overview of Organic Reactions Chapter Outline I. Organic Reactions (Sections 6.1-6.6). A. Kinds of organic reactions (Section 6.1). Addition reactions occur when two reactants add to form one product, with no 1 atoms left over. 2. Elimination reactions occur when a single reactant splits into two products. 3 Substitution reactions occur when two reactants exchange parts to yield two new . . products. Rearrangement reactions occur when a single product undergoes a rearrangement of bonds to yield an isomeric product. B Reaction mechanisms - general information (Section 6.2). 1 A reaction mechanism describes the bonds broken and formed in a chemical reaction, and accounts for all reactants and products. 2. Bond breaking and formation in chemical reactions. a. Bond breaking is symmetrical (homolytic) if one electron remains with each 4 . . . fragment. Bond breaking is unsymmetrical (heterolytic) if both electrons remain with one fragment and the other fragment has a vacant orbital. c. Bond formation is symmetrical if one electron in a covalent bond comes from each reactant. d. Bond formation is unsymmetrical if both electrons in a covalent bond come from one reactant. 3 Types of reactions. a. Radical reactions involve symmetrical bond breaking and bond formation. b Polar reactions involve unsymmetrical bond breaking and bond formation. c Pericyclic reactions will be studied later. C. Radical reactions (Section 6.3). 1 Radicals are highly reactive because they contain an atom with an unpaired electron. 2 A substitution reaction occurs when a radical abstracts an atom and a bonding b. . . . . . electron 3 . 4 . An from another molecule. addition reaction occurs when a radical adds to a double bond. Steps in a radical reaction. a. b . The initiation step produces radicals by the symmetrical cleavage of a bond. The propagation steps occur when a radical abstracts an atom to produce a new radical and a stable molecule. i. c. This sequence of steps is a chain reaction. A termination step occurs when two radicals combine. In radical reactions, all bonds are broken and formed by reactions of species with odd numbers of electrons. D. Polar reactions (Sections 6.4-6.6). 1 Characteristics of polar reactions (Section 6.4). a. Polar reactions occur as a result of differences in bond polarities within molecules. b These polarities are usually due to electronegativity differences between atoms. i. Differences may also be due to interactions of functional groups with solvents, as well as with Lewis acids or bases. Some bonds in which one atom is polarizable may also behave as polar ii. 5 . . . bonds. An Overview c. of Organic Reactions 117 one molecule react with electron-poor another molecule. The movement of an electron pair in a polar reaction is shown by a curved, fullheaded arrow. An electron pair moves from an atom at the tail of the arrow to a second i. atom at the head of the arrow. In polar reactions, electron-rich sites in sites in d e . . The reacting species: A nucleophile is a compound with an electron-rich atom. i. An electrophile is a compound with an electron-poor atom. Some compounds can behave as both nucleophiles and as electrophiles Many polar reactions can be explained in terms of acid-base reactions. f An example of a polar reaction: addition of HBr to ethylene (Section 6.5). a. This reaction is known as an electrophilic addition. ii. iii. . 2 . The 7i electrons in ethylene behave as a nucleophile. + The reaction begins by the attack of the % electrons on the electrophile H The resulting intermediate carbocation reacts with Br~ to form bromoethane. d Rules for using curved arrows in polar reaction mechanisms (Section 6.6). b . c. . . 3 . Electrons must . c . The nucleophile can be either negatively charged or neutral. The electrophile can be either positively charged or neutral. The octet rule must be followed. nucleophilic source to an electrophilic sink. d Describing a reaction (Sections 6.7-6.10). A. Equilibria, rates, and energy changes (Section 6.7). All chemical reactions are equilibria that can be expressed by an equilibrium 1 constant tq that shows the ratio of products to reactants. . II. move from a a. b . K a. b. 2. > 1, [products] K < 1, [reactants] If A"eq If eq > [reactants]. > [products]. For a reaction to proceed as written, the energy of the products must be lower than the energy of the reactants. a. The energy change that occurs during a reaction is described by AG°, the Gibbs free-energy change. b c . . d. 3 . Favorable reactions have negative AG° and are exergonic. Unfavorable reactions have positive AG° and are endergonic. AG° = -RT\n AG a. b . is A// is i. If ii. If . ii. . , and AS , which is temperature-dependent. a measure of the change in total bonding energy during a reaction. AH AH is negative, a reaction is positive, a reaction AS° (entropy) i Keq composed of two terms - A// is is is exothermic. endothermic. a measure of the freedom of motion of a reaction. A reaction that produces two product molecules from one reactant molecule has positive entropy. A reaction that produces one product molecule from two reactant molecules has negative entropy. aG° = AH°-TAS°. None of these expressions predict the rate of a reaction. B Bond dissociation energies (Section 6.8). The bond dissociation energy (D) measures the heat needed 1 c. 4 . . . 2 3 . . to break a bond to produce two radical fragments. Each bond has a characteristic strength. In exothermic reactions, the bonds formed are stronger than the bonds broken. 118 Chapter 6 C. Energy diagrams and transition states (Section 6.9). 1 Reaction energy diagrams show the energy changes that occur during a reaction, a. The vertical axis represents energy changes, and the horizontal axis (reaction . 2 . a. b . c. 3 . coordinate) represents the progress of a reaction. transition state is the highest-energy species in this reaction. It is possible for a reaction to have more than one transition state. The difference in energy between the reactants and the transition state The energy of activation AG*. Values of AG* range from After reaching the transition is the 40-150 kJ/mol. state, the reaction can go on to form products or can revert to starting material. 4 Every reaction has its own energy profile. D. Intermediates (Section 6.10). 1 In a reaction of at least two steps, an intermediate is the species that lies at the energy minimum between two transition states. 2 Even though an intermediate lies at an energy minimum between two transition states, it is a high-energy species and usually can't be isolated. 3 Each step of a reaction has its own AG* and AG°, but the total reaction has an . . . . overall AG°. 4. Biological reactions take place in several small steps, each of which has a small value of AG*. III. A Comparison of biological and laboratory reactions (Section 6. 1 1). A. Laboratory reactions are carried out in organic solvents; biological reactions occur in aqueous medium. B Laboratory reactions take place over a wide variety of temperatures; biological reactions take place at the temperature of the organism, usually within narrow limits. C. Laboratory reactions are uncatalyzed, or use simple catalysts; biological reactions are . enzyme-catalyzed. D. Laboratory reagents are usually small and simple; biological reactions involve large, complex coenzymes. E. Biological reactions have high specificity for substrate, whereas laboratory reactions are relatively nonspecific. Solutions to Problems 6.1 (a) CH 3 Br KOH + CH 3 OH + KBr substitution (b) H 2 C=CH 2 + CHgCh^Br (c) — CH2 "J - Hj CHgCHg HBr elimination addition An Overview of Organic Reactions 119 6.2 CH3CH2CH2CHCH2CI + CH 3 CH 2 CH 2 CCH 3 + CI 1 CH 3 CH3CH2CH2CHCH3 2-Chloro-2-methylpentane -Chloro-2-methylpentane CH 3 CHo3 3 I CH3CH2CHCHCH3 + hv CI 2-Methylpentane CH 3CHCH 2 CHCH 3 CI 3-Chloro-2-methylpentane 2-Chloro-4-methylpentane CH 3 CICH2CH2CH2CHCH3 1 6 3 . -Chloro-4-methylpentane Even though this molecule is complex, concentrate on the bonds formed and the bonds The tails of the arrows show the location of the bond to be broken, and the heads show where the electrons are moving. In radical reactions, the arrow is a fishhook (half- broken. headed). C0 2 H The reaction is a radical addition to a double bond and is a rearrangement. 120 Chapter 6 6 4 Keep . in mind: An electrophile is electron-poor, either because positively charged, because it has a has a vacant orbital. (2) A nucleophile is electron-rich, either because it has a negative charge, because it has a functional group containing a lone electron pair, or because it has a functional group that is negatively polarized. (3) Some molecules can act as both nucleophiles and electrophiles, depending on the (1) functional group that is it is positively polarized, or because it reaction conditions. (a) The electron-poor carbon (b) CH3S acts as an electrophile. a nucleophile because of the sulfur lone-pair electrons and because - is it is negatively charged. (c) C4H6N2 is a nucleophile because of the lone-pair electrons of nitrogen. (Only one of the nitrogens is nucleophilic, for reasons that will be explained in a later chapter.) (d) CH3CHO is both a nucleophile and an electrophile because of its polar C=0 bond. (a) ..6- 6+ H3C — CI: <— ^ .._&- &+ H3C nucleophilic — S: t nucleophilic electrophilic (d) (c) 6- N-~ \ u n CH 3 / nucleophilic 6 5 . BF3 is likely to — ^ C <*— H C :q:-« 3 be an electrophile because the 6+ : F : B : F : vacant orbital electrophilic H electrostatic potential electron-poor (blue). The electron-dot structure shows that octet and can accept an electron pair from a nucleophile. :f: nucleophilic II BF3 map indicates that it is lacks a complete electron An Overview 6 6 . of Organic Reactions 121 Reaction of cyclohexene with HC1 or HBr is an electrophilic addition reaction in which a halogen acid adds to a double bond to produce a haloaLkane. Br H— Br + Bromocyclohexane H CI H— CI + Chlorocyclohexane H 6.7 The mechanism is pictured in Figure 6.3. The steps: (1) Attack of the n electrons of the double bond on HBr, forming a carbocation; (2) Formation of a C-Br bond by electron pair donation from Br~ to form the neutral addition product. (\ H— Br Br H3C% . H I HoC^' HoC (2) (1) . CH, 2-Bromo-2-methylpropane carbocation 6 8 J HoC For curved arrow problems, follow these steps: (1) Locate the bonding changes. In (a), a bond from nitrogen to chlorine has formed, and a CI CI bond has broken. (2) Identify the nucleophile and electrophile (in (a), the nucleophile is ammonia and the electrophile is one CI in the CI2 molecule), and draw a curved arrow whose tail is near the nucleophile and whose head is near the electrophile. (3) Check to see that all bonding changes are accounted for. In (a), we must draw a second arrow to show the unsymmetrical bond-breaking of CI2 to form CI . (a) a: H— N— H + : CI — U CI N— + :Cl: I H (b) H 3C— O: + — G-r Br: H H ^ H 3 C— O— CH 3 + :Br: A bond has formed between oxygen and the carbon of bromomethane. The bond between carbon and bromine has broken. CH 3 is the nucleophile and bromomethane is the electrophile. 122 Chapter 6 (c) HgC^OCHg + HgC^OCHo : S : :Cl:^ A double bond has formed between oxygen and carbon, and a carbon-chlorine bond has broken. Electrons move from oxygen to form the double bond and from carbon to chlorine. 6 9 . This mechanism will be studied in a later chapter. T : -% -C0 2 + H2 > _ ,C— c^ CH 2 C0 2 2 C'i C0 2 C0 2 h + \ h2 H H > H— o: H 6.10 A negative value of AG° -44 kJ/mol 6.11 is indicates that a reaction is favorable. Thus, a reaction with AG° = more favorable than a reaction with AG° = +44 kJ/mol. K expression AG° = -RT In A^eq we can see that a large e0L is related to a large = 1000 is more a favorable reaction. Consequently, a reaction with exergonic than a reaction with eq = 0.00 1 From the negative , AG° and K 6.12 A reaction with AG* = 45 kJ/mol is faster than a reaction with AG* = 70 kJ/mol because a larger value for AG* indicates a slower reaction. 6.13 Intermediate Product Reaction progress An Overview of Organic Reactions 123 Visualizing Chemistry 6.14 CH 3 CH 2 CH 2 CH=CH2 + HBr CH3CH2CH2CHCH3 or CH 3 CH 2 CH=CHCH 3 + HBr 6.15 GH 2 H— Br: \Q :Br: Br .CHc H— Br: 6.16 (a) The electrostatic potential map shows that the formaldehyde oxygen is electron-rich, and the carbon-oxygen bond is polarized. The carbon atom is thus relatively electron-poor and is likely to be electrophilic. (b) The sulfur atom is more electron-rich than the other atoms of methanethiol and is likely to be nucleophilic. 6.17 Transition state 1 Transition state 2 Products AG° (positive) I d CD Reactants Reaction progress (a) AG° (b) There are two steps in the reaction. There are two transition states, as indicated on the diagram. (c) is positive. 124 Chapter 6 6.18 Reaction progress (a) The reaction involves four steps, noted above. 1 is the most exergonic because the energy difference between reactant and product (AG°) is greatest. (c) Step 2 is slowest because it has the largest value of AG*. (b) Step Additional Problems Polar Reactions 6.19 (b) (a) 6+ (c) /\&f,a 6- CH 3 CH 2 C=N ( \ ketone 6.20 6— 6— n n ester ^ N^ff jf CH 3 CCH 2 C— OCH 3 6+6- 6+ 6-0 6-.0 ^ ketone T ketone (e) carbon-carbon double bonds / CH 3 ether nitrile (d) Yi 6+ 6-P ^^6^C^6amide carbon-carbon double bond C aromatic m- aldehyde nn (a) The reaction between bromoethane and sodium cyanide is a substitution because two reagents exchange parts. (b) This reaction is an elimination because two products (cyclohexene and H2O) are produced from one reactant (c) Two reactants form one product (d) This is a substitution reaction. in this addition reaction. An Overview 6.21 e = electrophilic n= site (a) of Organic Reactions nucleophilic site (b) „ Testosterone 6.22 a: (b) + u H OH Co: :or" CI CHr. a: 6.23 (a) :OH ^C^ H 3° ' H*C -OCH HO: H/% a HoC + CHo + H2 :OCH 3 (b) C / \ H H2 CHr C I H H CHo3 H— CI 125 Width: 612 Height: 792 126 Chapter 6 Radical Reactions 6.24 Irradiation initiates the chlorination reaction chlorine radical consumed by producing chlorine in the propagation steps, a new the reaction. After irradiation stops, chlorine radicals are still For every formed to carry on carry on the radicals. CI- radical is present to propagation steps, but, as time goes on, radicals combine in termination reactions that remove them from the reaction mixture. Because the number of radicals decreases, fewer propagation cycles occur, and the reaction gradually slows down and stops. 6.25 6.26 a a b c b Pentane has three types of hydrogen atoms, CH3CH2CH2CH2CH3. Although monochlorination produces CH3CH?CH2CH2CH?C1, it is not possible to avoid producing CH3 CH2CH2CH(C1)CH3 and CH3 CH2CH(C1)CH2 CH3 as well. Since neopentane has only one type of hydrogen, monochlorination yields a single product. The following compounds yield single monohalogenation products because each has only one kind of hydrogen atom. Energy Diagrams and Reaction Mechanisms 6.27 A transition state represents a structure occurring at an energy maximum. An intermediate occurs at an energy minimum between two transition states. Even though an intermediate may be of such high energy that it cannot be isolated, it is still of lower energy than the transition states surrounding it. 6.28 Transition state Reaction progress AG° is positive because Keq < " 1. An Overview of Organic Reactions 127 6.29 Transition state 1 Intermediate ' ^ Transition state 2 9 S3 wq Products Reaction progress AG° 6.30 negative because is Keq > 1. Problem 6.29 shows a reaction energy diagram of a two-step exergonic faster than step 1 because AG*2 < AG*i. 6.31 Transition state OJQ Reactants Products Reaction progress A reaction with Keq = 6.32 (a) The (b) AG° = AH° - 7AS° reaction is 1 has AG° = 0. exothermic because the sign of Ai/° is negative, = -44 kJ/mol - (298 K) [-0.12 kJ/(K-mol)] = -44 kJ/mol + 36 kJ/mol = -8 kJ/mol The reaction is favorable because the sign of AG° is negative. reaction. Step 2 is 128 Chapter 6 6.33 ^ _ [Products] [Reactants] _ 0.70 _ 0.30 (b) Section 6.9 states that reactions that room temperature. Since kJ/mol at AG* probably close to 80 kJ/mol. is occur spontaneously have AG* of less than 80 proceeds slowly at room temperature, this reaction product Reaction progress 6.34 An Overview of Organic Reactions 129 General Problems 6.35 fH3 HoO H 3C— C— CI H 3 C— C + T.S. CHo f CH 3 1 H 3/H H2 aO— C— O + T.S. 2 Intermediate H 3 + H 3° H 3C— C— H CH 3 f T.S. 3 O— H CH 3 + CI" Intermediate 2 1 AG* for the first step is approximately 80 kJ/mol because the reaction takes place slowly at room temperature. AG* values for the second and third steps are smaller perhaps 60 kJ/mol for Step 2, and 40 kJ/mol for Step 3. AG° is approximately zero because A^eq is close to 1 (a) (b) T.S. 2 T.S. 1 T.S. 3 OJQ products reactants Reaction progress 6.36 H ? CH 3 H3 .. H 3 C— C-t CI %r H H3 r H3C- Lowest Priority -OH, -CH 3 -H , -CH 2 OH, -CH=CH 2 -CH 2 CH 3 -CH 3 -C0 2 H, -CH 2 OH, -CaN, -CH 2 NH 2 -CH 2 OCH 3 -CaN, -C^CH, -CH 2 CH 3 , , , Rule 1 1 3 138 Chapter 7 7.13 (a) Low H 3 C^ / /C High CH 2 OH Low =cx CH 3 CH 2 z High CI consider the substituents on the right side of the double bond. -CI ranks higher than 1 of the Cahn-Ingold-Prelog rules. On the left side of the double bond, -CH2CH3 ranks higher than -CH3. The isomer is Z when the two higher priority groups lie on the same side of the double bond. Otherwise, the isomer is E, First, -CH 2 OH by Rule (b) High Cl^ CH 2 CH 3 Low CH 2 CH 2 CH 3 High / C=C Low CH3O High H3C (c) \r--\ / C0 2 H High CH 2 OH Low Low Notice that the upper substituent on the left side of the double bond because of the methyl group attached to the ring. (d) Low \ / High CN H H3C / High & C=C E \ CH 2 NH 2 Low 7.14 ^C0 2 CH 3 High CH 2 OH Low is of higher priority Structure and Reactivity Alkenes: 7.15 More stable Less stable (a) H \ / H CHo6 \ C=C / \ / H 1-Butene 2-Methylpropene disubstituted double CHoCHo d 6 c=c\ H CH 3 H / bond monosubstituted double bond (b) ^Ch^Ch^CHg H^ / \ C=C\ H3C H E /C =c HgC H z CH2CH2CH3 d.s-2-Hexene trans-2-Hexene no H / steric strain steric strain same of groups on the side of the double bond (c) 1 trisubstituted 7.16 3-Methylcyclohexene -Methylcyclohexene double bond disubstituted double bond HX to an alkene. All of these reactions are electrophilic additions of Use Markovnikov's rule to predict orientation. (a) H + Chlorocyclohexane HCI (b) Br (CH 3 2 C=CHCH 2 CH3 + ) HBr — I** (CHg^CCh^Ch^CHg 2-Bromo-2-methylpentane In accordance with Markovnikov's rule, H forms a bond to the carbon with fewer and Br forms a bond to the carbon with more substituents. substituents, (c) CHo3 CHo3 I I CH3CHCH2CH — CH2 H 2 S0 4 OH I CH3CHCH2CHCH3 139 140 Chapter 7 id) aCH 2 + o HBr 1 4* -Bromo- 1 -methylcyclohexane 7.17 Think backward in choosing the alkene starting material for synthesis of the desired haloalkanes. Remember that halogen is bonded to one end of the double bond and that more than one starting material can give rise to the desired product. (a) Br + HBr Cyclopentene (b) a or CH2CH3 KI CHCH 3 CH2CH3 H3PO4 (c) r CH 3 CH 2 CH=CHCH 2 CH 3 + HBr CH3CH2CH2CHCH2CH3 3-Hexene (d) + 7.18 The more HCI stable carbocation is formed (a) CHg CHg CHo3 CHo3 I I CH 3 CH 2 C=CHCHCH 3 + HBr I CHgCH2CCH2CHCHg carbocation intermediate CHo3 CHo3 I I CH3CH2CCH2CHCH3 Br (b) CHCH 3 I KI CH2CH3 H3PO4 CH2CH3 carbocation intermediate Alkenes: Structure and Reactivity 141 7.19 Two representations of the secondary carbocation are shown on the left below. This secondary carbocation can experience hyperconjugative overlap with two hydrogens under normal circumstances. However, in the alignment shown in the drawing, only one hydrogen (circled) is in the correct position for hyperconjugative overlap with the carbocation carbon. Because there is representation on the rotation about the carbon-carbon bonds, all of the far right can be involved in hyperconjugation + \ CHCHCH 3 the U .-C- \ HoC 7.20 The second H 3C H3C HoC CH-C X H3 C H step in the electrophilic addition of Hammond postulate, hydrogens starred in the at some time. H ^-C-CH-C. H3 C HC1 to an alkene * H is exergonic. According to the transition state should resemble the carbocation intermediate. >, mi 7.21 H— Br ~ :Br: ^HCHg 1. H Step 1: Electrophilic addition Step 2: Hydride shift that of .Br 2. Ch^CHg H + to double bond. forms a more stable Step 3: Reaction of carbocation with Br~. tertiary carbocation. 142 Chapter 7 Visualizing Chemistry 7.22 2,4,5-Trimethyl-2-hexene 1 -Ethyl-3 ,3-dimethylcyclohexene 7.23 (b) C=0 Low High High Low Low OH 7.24 CHCH 3 CI HCI or CH2CH3 CH2CH3 CH2CH3 two compounds shown can form the illustrated tertiary carbocation when they HCI. In the conformation shown, the three circled hydrogens are aligned for maximum overlap with the vacant p orbital. Because of conformational mobility, the three starred hydrogens are also able to be involved in hyperconjugation. Either of the react with Alkenes: Structure and Reactivity 143 7.25 HgC ^3^ Additional Problems Calculating a Degree of Unsaturation 7.26 The purpose of this problem is to give you experience in calculating the number of double a formula. Additionally, you will learn to draw structures containing various functional groups. Remember that any formula that satisfies the rules of valency is acceptable. Try to identify functional groups in the structures that you draw. Many structures are acceptable for each part of this problem. bonds and/or rings in (a) C10H16 - 3 degrees of unsaturation. Examples: (b) CsHsO. The equivalent hydrocarbon is CsHs, which has 5 degrees of unsaturation. 144 Chapter 7 (c) C7H10CI2 has C7H12 as degrees of unsaturation. its equivalent hydrocarbon formula. C7H10CI2 has two triple double bonds bond halide '? t CH 3 CH 2 CHCHCH 2 C= CH H3 / halide \ H , C= CHCCH=C / CI f CI N I _ - CH 3 CI halides double bond double bond halide halides HoC (d) hahde CI CI halide C10H16O2 - 3 degrees of unsaturation (CioHu = equivalent hydrocarbon formula), double bonds H 2 C= CHCH ketones, ester = CHCH 2CH2 CH2 CH2 COCH 3 ketone alcohol HO alcohols o — OH HO—< > ether (e) C5H9NO2 - 2 degrees of unsaturation (CsHg = equivalent hydrocarbon formula). amide ketone V^11O II' CH 3 CH 2 CCH 2 CNH 2 double bond H2 C=CHCH 2 NHCH 2 COH amine double bond nitro O J group H 2 C= CHCH 2 CH 2 CH 2 N O carboxylic —O amide , ketone amine HO alcohol O — ether Structure and Reactivity Alkenes: (f) CgH 10 ClNO - 4 degrees of unsaturation (CgHio amide double bonds ,C= CHCH / double UUI1US UU1C bonds x formula). - = CHCH= CHCH 2 CNH2 halide ci = equivalent hydrocarbon 145 amine — Ni^^^"^^ C double N-* H halide-*- CI aromatic — / bond amine double bond " J HO CH 2 NH 2 V alcohol CI Compound 7.27 ketone s amine^ halide ^ ^_ (b) (c) 7.28 C 8H ? 2 C7H9N C9H7NO kel()ll , O h9 N Equivalent hydrocarbon formula (a) amine halide Degree of unsaturation Complete formula QH18 C 7 H 16 3 C 8 H 12 2 C9H20 4 C7H13N C9H13NO 2 Solve this problem in the same way as we solved problems 7.3 and 7.27. A C22 hydrocarbon with 12 degrees of unsaturation (four rings and eight double bonds) has a formula C22H46 - H24 = C-r^xi- Adding two hydrogens (because of the two nitrogens) and subtracting one hydrogen (because of the chlorine), gives the formula C22H23CIN2O2 for Loratadine. CH 2 CH3 Loratadine Width: 612 Height: 792 146 Chapter 7 Naming Alkenes 7.29 (a) (b) \ 23 CH3CHCH2CH2CH CHCHoCHo H / c=c / / \ H3C CH3 H h^C^— CCH2CH3 H 2-Ethyl-l-butene (Z)-4-Ethyl-3,7dimethyl-2-octene w (d) (e) /CH 3 / C=C\ \ / CH, C=C\ / H2C H 3C x H 2 C=CHCHCH (f) H H \ CH 3/C =C H CH2CH3 C=Cv H3C (£)-4-Methyl-2-hexene (c) CH2CH3 CHr — C — CHCH 3 CH 3 CH2CH2 CH 3 (2Z,4£)-4,5-Dimethyl2,4-octadiene (5£)-3,4-Dimethyl1,5-heptadiene 1,2-Butadiene 7.30 (a) (b) CH 3 H CHoC J — CHo I H \ / 2 2 c=c CHC(CH 3 )2CH^— CH2 CH 3 CH2 H3C CH2CH2CH 3 (4£)-2,4-Dimethyl- 1 ,4-hexadiene c/5-3,3-Dimethyl-4-propyl- 1 ,5-octadiene (d) (c) H CH 3 CH 3 CHCH=C=CH 2 I I H»v ^.C^ H C H I 4-Methyl- 1 ,2-pentadiene (e) H (3£,5Z)-2,6-Dimethyl-l,3,5,7-octatetraene (f) CH 3 CH2CH2CH2 H (CH 3) 3 C C=C c=c CH 3 CH2CH2CH2 3-Butyl-2-heptene H CH 3 H C(CH 3 ) 3 /ra«s-2,2,5,5-Tetramethyl-3-hexene Alkenes: Structure and Reactivity 7.31 (a) (b) 3-Methylcyclohexene (d) 1 (c) ,5-Dimethylcyclopentene Ethyl- 1 ,3-cyclobutadiene ® (e) 1, 2-Dimethy 1-1,4- 5-Methyl-l,3cyclohexadiene cyclohexadiene 1 ,5-Cyclooctadiene 7.32 Because the longest carbon chain contains 8 carbons and 3 double bonds, ocimene is an numbering at the end that will give the lower number to the first double (1,3,6 is lower than 2,5,7). Number the methyl substituents and, finally, name the octatriene. Start bond compound. Ocimene (3£)-3,7-Dimethylocta- 1 ,3,6-triene 7.33 a-Farnesene (3£,6£)-3,7, 1 1 -Trimethyl- 1 ,3,6, 10-dodecatetraene 7.34 Menthene / 7.35 CH3CH2 CH0CH0 CH3CH2CH2CH — CH2 / P=% H 1-Pentene H \ H H (Z)-2-Pentene CH 3 (£)-2-Pentene CH 3 CH0CH9 6 d \ H / \ H / s H 2-Methyl-l-butene H HoC \ p=c P=°s H 3C CHoCH 6 / H 3-Methyl- 1 -butene HoC / CHtv c=c\ H 2-Methyl-2-butene 147 148 7.36 Chapter 7 Start with 1-hexene and continue on designations have been made when until all hexenes are named, making sure that E,Z necessary. Then move on to all 1-pentenes, 2-pentenes, etc. CHgCh^CH^Ch^CH^- CH2 CH3CH2CH2 CH3CH2CH2 / H H 1-Hexene CH3CH2 / CH2CH3 c=c( H c =c s CH2CH3 (E)-3-Hexene (Z)-3-Hexene H )c=c ; H H (£)-2-Hexene CH3CH2CH2 H ; CH 3 H (Z)-2-Hexene CH3CH2 C=C\ H3 C H 2-Methyl- 1 -pentene CH 3 Lowest Priority 7.37 Highest priority -CH 3 -H -OCH3, -OH, -C0 2 H, -H -C0 2 CH 3 -C0 2 H, -CH 2 OH, -CH 3 -COCH3, -CH 2 CH 2 OH, -CH 2CH 3 -CH 3 (a) -I, (b) (c) (d) -Br, , , , (e) -CH 2 Br, -C=N, -CH 2 NH 2 -CH=CH 2 (f) -CH 2 OCH 3 -CH 2 OH, -CH=CH 2 CH 2 CH 3 , , , 7.38 (a) (b) High HOCH 2 CH 3 ... „. High Low Low H3C H Low High High NC , , (c) H0 2 C Low c=( Low ) CH 2 OH High CH 3 CH 2 Low H02 C 7.39 (a) High H 3( C0 2 H V / ^C=C^ N Low Low H / High CH 2 CH(CH 3 H 3C (correct) E (correct) E (incorrect) CH 2 CH=CH 2 High C= C High Z Low H ' ) 2 Low (r) v ' High Br^ / CH 2 NH 2 Low C= C / Low H \ CH 2 NHCH 3 CH=CH 2 High H 3 C0 2 C E i High Low OCH 3 High CI (d) CH 3 ) (b) H c= i CH 2 CH 3 High z Low 150 Chapter 7 (d) High NC^ / / Low (e) C=C\ (CH 3 ) 2 NCH 2 High Low CH 3 E CH 2 CH 3 (correct) High Br C=C This compound doesn't show E-Z isomerism. Low ® Low HOCH :H 2 / 7.40 As expected, the two cis-trans difference trans is C0 2 H High C=C\ H 3 COCH 2 High / E Low COCH 3 compounds (correct) are more stable than their cis counterparts. The much more pronounced for the tetramethyl compound, however. Build a model of c/.s-2,2,5,5-tetramethyl-3-hexene and notice the extreme crowding of the methyl groups. Steric strain makes the cis isomer much less stable than the trans isomer and causes cis A//°h ydrog to have a much larger negative value than trans A//°hydrog f° r the hexene isomers. trans isomer 7.41 A model of cyclohexene shows that a six-membered ring is too small to contain a trans double bond without causing severe strain to the ring. A ten-membered ring is flexible enough to accommodate either a cis or a trans double bond, although the cis isomer has less strain than the trans isomer. 7.42 Build models of the two cyclooctenes and notice the large amount of torsional strain in fra/w-cyclooctene relative to ds-cyclooctene. This torsional strain, in addition to angle causes the trans isomer to be of higher energy and to have a A//°h ydrog larger than the A//°hydrog of the cis isomer. strain, 7.43 Models show that the difference in strain between the two cyclononene isomers is smaller than the difference between the two cyclooctene isomers.This reduced strain is due to a combination of less angle strain and more puckering to relieve torsional strain and is two cyclononene isomers are more strained than the cis isomer. reflected in the fact that the values of A//°hydi- g f°r the relatively close. Nevertheless, the trans isomer is still A Ike ik's : Structure and Reactivity 7.44 Clomiphene Tamoxifen Carbocations and Electrophilic Addition Reactions 7.45 The third product results from rearrangement to a more H2C— CHCH2CH2CH2CH — CH2 + 2 - HCI stable tertiary carbocation. CH3CHCH2CH2CH2CHCH3 151 152 Chapter 7 7.46 (a) aCH a 2 + HBr (b> + (c) HBr CH 3 CH 3 Br CH 3 CH= CHCHCH 3 + HBr Br I I I CH 3 I CH 3 CHCH2CHCH 3 + CH 3 CH2CHCHCH 3 CH 3 + CH 3 CH2CH2^j/CH 3 Br The third product results from rearrangement to a more stable tertiary carbocation. 7.47 (a) CH 3 CH 3 I CH 3 CH2C^= CHCH 3 + H2O acid 1 * CH 3 CH2CCH2CH 3 catalyst I OH (b) OH CHr CH, acid + HoO catalyst (c) CH 3 CH 3 ' CH 3 CHCH 2 CH=CH 2 + H2 acid OH ' ' CH 3 CHCH 2 CHCH 3 catalyst 7.48 (a) CH 3 CH2CHCH 3 CH 3 CH2CHCH2 primary carbocation The primary carbocation shift. secondary carbocation rearranges to the more stable secondary carbocation by a hydride Alkenes: CHCHg CH3C CHgC 153 CH^CHg + I CHo CHo secondary carbocation This hydride Structure and Reactivity shift tertiary carbocation produces a tertiary carbocation from rearrangement of a secondary carbocation. (c) -CH 3 + JH 2 primary carbocation tertiary carbocation An alkyl shift forms a tertiary carbocation from a primary carbocation. In this example, rearrangement involves migration of the electrons from one of the cyclobutane ring bonds to form a cyclopentane ring. 7.49 CI H— ci: 3° 3° 6 carbocation ^shrff carbocation General Problems 7.50 The central carbon of allene forms hybridized, and the carbon-carbon two o bonds and two n bonds. The central carbon is spbond angle is 180°, indicating linear geometry for the carbons of allene. The hydrogen atoms on one terminal pair of hydrogen atoms on the other =CH2 group. =CH2 units are perpendicular to the 154 7.51 Chapter 7 heat of hydrogenation for a typical diene is 2 x (A//°h ydrog °f an alkene) = -252 kJ/mol. Thus, allene, with A//°hydrog = -295 kJ/mol is 43 kJ/mol higher in energy than a typical diene and is less stable. The 7.52 trans trans C0 2 H Retin A Retin A contains five carbon-carbon double bonds. Since the ring double bond can't isomerize, Retin A can have 2 4 = 16 isomers. 7.53 (a) 3 1 5 4 5 Ectocarpene Fucoserraten 6-[(Z)- 1-Butenyl]- 1 ,4-cycloheptadiene (3£, 5Z)-l,3,5-Octatriene 7.54 Treatment of the te/t-butyl ester with trifluoroacetic acid cleaves the -OC(CH3>3 group and with an -OH group, which has a lower priority than the -OCH 3 group on the upper carbon and the -OC(CH 3) 3 group that was removed. The result is a change in the E,Z designation around the double bond without breaking any of the bonds attached to the double-bond carbons. replaces it Low H C— OChhd Low \ / C=C^ z Low CF3COOH H \ / High H 3 C C-OC(CH 3 / ) 3 High High C- c=c c— OH H3C O o 7.55 CH 3 bond shift q: (alkyl shift) OCH3 High / :ci: E Low Alkenes: Structure and Reactivity 155 7.56 Attack of the n electrons of the double bond on H + yields the carbocation pictured on the far right. A bond shift (alkyl shift) produces the bracketed intermediate, which reacts with Br~ to yield 1 -bromo-2-methylcyclobutane. 7.57 (a) C27H46O (b) (c) C14H9CI5 C20H34O5 (d) C8H10N4O2 (e) C21H28O5 (f) C17H23NO3 7.58 The reaction is 5 degrees 8 degrees 4 degrees 6 degrees 8 degrees 7 degrees of unsaturation of unsaturation of unsaturation of unsaturation of unsaturation of unsaturation exergonic because it is spontaneous. According to the the transition state should resemble the isobutyl cation. 7.59 Reaction progress Hammond postulate, Width: 612 Height: 792 156 Chapter 7 7.60 Transition State #1 2-Bromopentane path 0.0 Transition State 5+ #2 H 39 H f C^'C C3 ^^f\~/P%TH CsHtVA H :Br: CoH'3 n 7 o ^y&—XL a *+vr\ 1-Bromopentane path „^c — C^d C3 H 7 first step ^ 1-1 H The h (carbocation formation) H (J is :Br: (~~~~^>CC~~~) ^i^j C H 4 9 endergonic for both reaction paths, and both transition states resemble the carbocation intermediates. Transition states for the exergonic second step also resemble the carbocation intermediate. Transition bromopentane is more bromopentane. state #1 for like the carbocation intermediate than is transition state 1- #1 for 2- 7.61 Step 2, in addition. which the double bond electrons add to the carbocation, is an alkene electrophilic Alkenes: Structure and Reactivity 157 7.62 ep/-Aristolochene Steps 1 and 2 are alkene electrophilic additions, and steps 3 and 4 involve carbocation rearrangements. 7.63 Reaction of 1-chloropropane with the Lewis acid AICI3 forms a carbocation. The less stable propyl carbocation (primary undergoes a hydride shift to produce the more stable isopropyl carbocation (secondary), which reacts with benzene to give isopropylbenzene. CH 3 CH 2 CH 2 — CI + CH 3 CH 2 CH 2 AICI3 CHgCH AICI4" CH3CHCH3 CH2 CH 3 CHCH 3 CH3CHCH3 + 7.64 HoC 3, — C^— CH2 I CH3CH CH 3 CH 3 CHo3 HBr CHgCH —* C~~ CHg " KOH CH30H H3 f CHgC CH 3 C CHg Br 2,3-Dimethyl- 1 -butene 2-Bromo-2,3- 2,3-Dimethyl-2-butene dimethylbutane The product, 2,3-dimethyl-2-butene, is formed by elimination of HBr from 2-bromo-2,3it has the more substituted double bond. dimethylbutane. This product forms because Chapter 8 - Alkenes: Reactions and Synthesis Chapter Outline I. Preparation of alkenes (Section 8.1). A. Dehydrohalogenation. Reaction of an alkyl halide with a strong base forms an alkene, with loss of HX. 1 B. Dehydration. 1 Treatment of an alcohol with a strong acid forms an alkene, with loss of H2O. Addition reactions of alkenes (Sections 8.2-8.6). A. Addition of halogens (halogenation) (Section 8.2). Bi2 and CI2 react with alkenes to yield 1,2-dihaloalkanes. 1 2. Reaction occurs with anti stereochemistry: Both halogens come from opposite sides of the molecule. 3 The reaction intermediate is a cyclic halonium intermediate that is formed in a single + + step by interaction of an alkene with Br or Cl B Addition of hypohalous acids (Section 8.3). 1 Alkenes add HO-X (X = Br or CI), forming halohydrins, when they react with halogens in the presence of H2O. 2 The added nucleophile (H2O) intercepts the halonium ion to yield a halohydrin. 3 Bromohydrin formation is usually achieved by NBS in aqueous DMSO. 4 Aromatic rings are inert to halohydrin reagents. C. Addition of water to alkenes (Section 8.4). . . II. . . . . . . . . Hydration. a. Water adds to alkenes to yield alcohols in the presence of a strong acid catalyst. b Although this reaction is important industrially, reaction conditions are too severe for most molecules. 2. Oxymercuration. a. Addition of Hg(OAc)2, followed by NaBH4, converts an alkene to an alcohol. b The mechanism of addition proceeds through a mercurinium ion. c The reaction follows Markovnikov regiochemistry D. Addition of water to alkenes: hydroboration/oxidation (Section 8.5). BH3 adds to an alkene to produce an organoborane. 1 a. Three molecules of alkene add to BH3 to produce a trialkylborane. 2. Treatment of the trialkylborane with H2O2 forms 3 molecules of an alcohol. 3 Addition occurs with syn stereochemistry. 4 Addition occurs with non-Markovnikov regiochemistry. a. Hydroboration is complementary to oxymercuration/reduction. 5 The mechanism of hydroboration involves a four-center, cyclic transition state. a. This transition state explains syn addition. b Attachment of boron to the less sterically crowded carbon atom of the alkene also explains non-Markovnikov regiochemistry. Reduction and oxidation of alkenes (Sections 8.6-8.8). A. Reduction of alkenes (Section 8.6). 1 In organic chemistry, reduction increases electron density on carbon either by forming C-H bonds or by breaking C-O, C-N, or C-X bonds. 2 Catalytic hydrogenation reduces alkenes to saturated hydrocarbons. a. The catalysts most frequently used are Pt and Pd. b Catalytic hydrogenation is a heterogeneous process that takes place on the surface of the catalyst. c. Hydrogenation occurs with syn stereochemistry. d. The reaction is sensitive to the steric environment around the double bond. 1 . . . . . . . . . III. . . . Alkenes: Reactions and Synthesis 159 3 Alkenes are much more reactive than other functional groups. Oxidation of alkenes (Sections 8.7-8.8). . B . In organic chemistry, oxidation decreases electron density on carbon either by forming C-O, C-N, or C-X bonds or by breaking C-H bonds. 2. Epoxidation (Section 8.7). a. Epoxides can be prepared by reaction of an alkene with a peroxyacid RCO3H. i. The reaction occurs in one step with syn stereochemistry. b Epoxides are also formed when halohydrins are treated with base. c. Acid-catalyzed reaction of an epoxide ring with water yields a 1,2-diol (glycol). i Ring opening takes place by back-side attack of a nucleophile on the protonated epoxide ring. ii. A trans- 1,2-diol is formed from an epoxycycloalkane. 1 . . . Hydroxylation. a. OSO4 causes the addition of two -OH groups to an alkene to form a diol. Hydroxylation occurs through a cyclic osmate intermediate. i. b A safer reaction uses a catalytic amount of OSO4 and the oxidant NMO. c. The reaction occurs with syn stereochemistry. 4 Cleavage to carbonyl compounds (Section 8.8). a. O3 (ozone)causes cleavage of an alkene to produce aldehyde and/or ketone fragments. i. The reaction proceeds through a cyclic molozonide, which rearranges to an ozonide that is reduced by Zn. b KMn04 in neutral or acidic solution cleaves alkenes to yield ketones, carboxylic acids or CO2. c. Diols can be cleaved with HI04(periodic acid)to produce carbonyl compounds. IV. Addition of carbenes (Section 8.9). carbene (R2C:) adds to an alkene to give a cyclopropane. A. B The reaction occurs in a single step, without intermediates. C Treatment of HCCI3 with forms dichlorocarbene. 1 Addition of dichlorocarbene to a double bond is stereospecific, and only cisdichlorocyclopropanes are formed. D. The Simmons-Smith reaction (CH2I2, Zn-Cu) produces a nonhalogenated cyclopropane via a carbenoid reagent. V. Radical additions to alkenes: chain-growth polymers (Section 8.10). A. Many types of polymers can be formed by radical polymerization of alkene monomers. There are 3 steps in a chain-growth polymerization reaction. 1 a. Initiation involves cleavage of a weak bond to form a radical The radical adds to an alkene to generate an alkyl radical. i. b The alkyl radical adds to another alkene molecule (propagation) to yield a second radical. i This step is repeated many, many times. c Termination occurs when two radical fragments combine. 2 Mechanisms of radical reactions are shown by using fishhook arrows. 3 . . . . A . KOH . . . . . . . 3 . As in electrophilic addition reactions, the formed more stable radical (more substituted) is in preference to the less stable radical. B. Biological additions of radicals to alkenes (Section 8. 1 1). Biochemical radical reactions are more controlled than laboratory radical reactions. VI. Stereochemistry of reactions (Sections 8.12-8.13). A. Addition of H2O to an achiral alkene (Section 8.12). 1 When H2O adds to an achiral alkene, a racemic mixture of products is formed. 2 The achiral cationic intermediate can react from either side to produce a racemic 1 . . . mixture. 3 . Alternatively, the transition states for top side reaction and enantiomers and have the same energy. bottom side reaction are 160 Chapter 8 4. Enzyme-catalyzed reactions give a single enantiomer, even when the substrate is achiral. B. Addition of H 2 1 . 2 . 3 . to a chiral alkene (Section 8.13). When H + adds The to a chiral alkene, the intermediate carbocation is chiral. original chirality center is unaffected by the reaction. Reaction of H2O with the carbocation doesn't occur with equal probability from either side, and the resulting product is an optically active mixture of diastereomeric alcohols. 4. Reaction of a chiral reactant with an achiral reactant leads to unequal amounts of diastereomeric products. Solutions to Problems 8.1 CH CH 3 C CH3CH2CCH3 KOH CH 3 CH 2 OH CH3CH CH 3 = CCH3 + CH3CH2C = CH2 Br Dehydrobromination may occur in either of two directions to yield a mixture of products. 8.2 CH3CH2 / CH3CH2 ^^^0^3 C=C )c=<( c=c\ H I CHqCHoCHoCCHoCHo 2 2 2 3 3 | HgSO^ H2 CH 3 CH3CH2CH2 / / HoC 130 + CH2CH3 n H "13 CH, (£)-3-Methyl-3-hexene (Z)-3-Methyl-3-hexene OH CH3 CH3CH2CH2 CH,1 H / c=c\ / C=C\ H3C ngo H n v^ng CHr (£)-3-Methyl-2-hexene (Z)-3-Methyl-2-hexene CH3CH2CH2 c= ^H ' ' CH3CH2 H 2-Ethyl-l-pentene Five alkene products, including E, methyl-3-hexanol. Z isomers, might be obtained by dehydration of 3- 8.3 HoC 1 ,2-Dimethylcyclohexene trans- 1 ,2-Dichloro- 1 ,2- dimethylcyclohexane The chlorines are trans to one another in the product, as are the methyl groups. Alkenes: Reactions and Synthesis 161 8.4 Addition of hydrogen halides involves formation of an open carbocation, not a cyclic halonium ion intermediate. The carbocation, which is ^"-hybridized and planar, can be attacked by chloride from either top or bottom, yielding products in which the two methyl groups can be either cis or trans to each other. 8.5 HO -Br and 8 6 . H -OH are trans in the product. Reaction of the alkene with Bi2 (formed from When this bromonium ion NBS) produces a cyclic bromonium opened by water, a partial positive charge develops carbon whose bond to bromine is being cleaved. is ion. at the + CBr: BrS vs H3 C^ \ 6+ CHc l H 2 o: H 2 b": 2 less favorable Since a secondary carbon can stabilize this charge better than a primary carbon, opening of the bromonium ion occurs at the secondary carbon to yield the Markovnikov product. 8 7 . Keep in mind that oxymercuration is equivalent to Markovnikov addition of H2O to an alkene. (a) OH CH 3 CH 2 CH 2 CH=CH 2 (b) H 2° " I : \ CH3CH 2 CH 2 CHCH3 CH 3 1 CH3CH 2 CH^= CCH3 Hg(OAc) 2 H 2 Q , 2. NaBH 4 CH3CH 2 CH 2 CCH3 OH 162 Chapter 8 8 8 Think backwards . to select the possible alkene starting materials for the alcohols pictured, (a) CHgC— CHCH2CH2CH3 CHr 1.Hg(OAc) ,H O 2 or 2. g C H 3 i C H 2 CH 2 CH 2 CH 3 > NaBH 4 CHr H 2 C~~~ CCH 2 CH 2 CH 2 CH 3 CHo (b) 1 Hg(OAc) 2 H 2 Q 2. NaBH4 , Oxymercuration occurs with Markovnikov 8.9 orientation. Hydroboration/oxidation occurs with non-Markovnikov regiochemistry to give products in which -OH is bonded to the less highly substituted carbon. (a) CH 3 1 BH 3 THF 2. H2 , CHgC^^ CHCH2CH3 2 , "OH 4 23 CHCH0CH0 CH^C I I OH H less substituted (b) less substituted t ^CHo 8.10 1 BH 3 THF 2. H2 , 2 ,"OH As described in Worked Example 8.2, the strategy in this sort of problem begins with a look backward. In more complicated syntheses this approach is essential, but even in problems in which the functional group(s) in the starting material and the reagents are known, this approach is effective. All the products in this problem result from hydroboration/oxidation of a double bond. The -OH group is bonded to the less substituted carbon of the double bond in the starting material. (a) CH 3 CH 3 CHCH — CH 2 I _ CHoC d — CHCHo 6 CH 3 1 BH 3 THF 2. H2 1 , . 2. 2 ,"OH BH 3 THF , H2 _ 2 , OH CH 3 CHCH 2 CH 2 OH I CHaCHCHCHo | OH This product can also result from oxymercuration of the starting material in (a). Alkenes: O (c) CH 2 8.11 1 BH 3 THF 2. H2 , 2 , Reactions and Synthesis "OH 163 CH 2 OH the transition states resulting from addition of BH3 to the double bond of the cycloalkene. Addition can occur on either side of the double bond. The drawings below show HoC BH, HoC CH 3 THF HoC, Reaction of the two neutral alkylborane adducts with hydrogen peroxide gives two alcohol isomers. In one isomer, the two methyl groups have a cis relationship, and in the other isomer they have a trans relationship. H HoC H HoC CH, BR 2 H OH H2 BR C 2 OH ~OH HoC HoC CH, CH, H 8.12 CH, Catalytic hydrogenation produces alkanes from alkenes. CH, CH3C H. — CHCH2 CHg Pd/C in ethanol 2-Methyl-2-pentene CH3CHCH 2 CH 2 CHg 2-Methylpentane (b) *#*\ CH, CH, H2 Pd/C in ethanol CH, 3 ,3-Dimethylcyclopentene CH, 1 , 1 -Dimethy lcyclopentane 164 Chapter 8 CHo3 I C— CHo3 \ H, Pd/C in CHo ethanol tert-Butylcyclohexane 3-terr-Butylcyclohexene 8.13 Epoxidation using m-chloroperoxybenzoic acid (RCO3H) is a syn addition of oxygen to a double bond. The original bond stereochemistry is retained. O RCO3H H.. H3C / \ CHj HoC CH, m-2,3-Epoxybutane c/s-2-Butene In the epoxide product, as in the alkene starting material, the methyl groups are 8.14 cis. Reaction of an alkene with a catalytic amount of OSO4, in the presence of N-morpholine TVoxide (NMO), yields a diol product. To pick a starting material for these products, choose an alkene that has a double bond between the diol carbons. The products in (b) and (c) can also be formed by ring opening of an epoxide formed either from a peroxyacid or from a halohydrin. (a) /\zCH u r 1 1 3 NMO catalytic OsO, -Methy lcyclohexene (b) OH CH3CH2CH _ — C(CH3)2 OsQ 4 ^ NMO 1 OH 1 CH3CH2CH CCH3 2-Methyl-2-pentene CH 3 HO OH (c) CH 2? =CHCH=CHo2 1,3-Butadiene Os0 4 **» NMO I I HOCHoCHCHCHoOH 2 2 Reactions and Synthesis Alkenes: 8.15 8.16 165 Both sets of reactants cleave double bonds. Aqueous KMn0 4 produces a carboxylic acid from a double bond carbon that is monosubstituted and a ketone from a double bond carbon that is disubstituted. Ozone produces an aldehyde from a double bond carbon that monosubstituted and a ketone from a double bond carbon that is disubstituted. If the double bond is part of a ring, both carbonyl groups occur in the same product molecule. Orient the fragments so that the oxygens point toward each other. and draw a double bond between the remaining carbons. (a) (CH 3 2 C=CH 2 1 ) Q z ° H3 Q + » (CH 3 2 ) C=0 + Remove the is oxygens, 0=CH2 (b) CH 3 CH 2 CH=CHCH 2 CH 3 8.17 1. Oo 2 Zn H 3 + * CH 3 CH 2CH=0 + 0=CHCH 2 CH 3 1 Reaction of a double bond with chloroform under basic conditions gives a product with a cyclopropane ring in which one of the carbons has two chlorine atoms bonded to it. Reaction of a double bond with CH 2 l2 yields a product with a cyclopropane ring that has a -CH 2- group. (a) CH 2 ^ CH 3 I + CHCI 3 _ CH 3 CHCH 2 CH^— CHCH 3 KOH Zn(Cu)^ + CH 2 I 2 CH 3 I CHo2 / \ CH 3 CHCH2 CH Depending on the stereochemistry of the double bond of the alkene isomers can be formed. CHCH 3 in (b), two different Width: 612 Height: 792 166 Chapter 8 8.18 Find the smallest repeating unit in each polymer and add a double bond. This is the monomer unit. Monomer Polymer (a) OCH, OCH, I H 2 C=CHOCH 3 yCHp (b) ci One radical ci CH2 I ci CH2 CH a a CH" a \ CH— CH— CH— CH — CH — CH—7- CIHC=CHCI 8.19 CH OCH, I abstracts a hydrogen atom from a second radical, and the remaining two electrons create a double bond. -|-CH= CH, -f-CH 2 8.20 — CHj — ^~ CH2 CH3 Look back to Figure 8. 12, which shows the reaction of (/?)-4-methyl-l-hexene with H3O" In a similar way, we can write a reaction mechanism for the reaction of H30+ with 1 ". (5")-4-methyl- 1 -hexene. PH3 V h.U CH >C— 3 v OHo o bottom t0 H % CH , 3 ^ h PH CH 3 (25,45)-4-Methyl-2-hexanol CH 3 H CH 3 H V OH (2tf,4S>4-Methyl-2-hexanol The products shown above are diastereomers and are formed in unequal amounts. The (2S,AS) stereoisomer is the enantiomer of the (2R,4R) isomer (shown in Figure 8.12), and the transition states leading to the formation of these two isomers are enantiomeric and of equal energy. Thus, the (2S,4S) and (2R,4R) enantiomers are formed in equal amounts. A similar argument can be used to show that the (2/?,45 )and (2S,4R) isomers are formed in equal amounts. The product mixture is optically inactive. Reactions and Synthesis Alkenes: 167 8.21 IR,3S 15,35 15,3/? IR,3R Two enantiomeric carbocations are formed. Each carbocation can react with H2O from bottom to yield a total of four stereoisomers. The same argument used in Problem 8.20 can be used to show that the (15,37?) and (l/?,35) enantiomers are formed in equal amounts, and the (15,35) and (IR,3R) isomers are formed in equal amounts. The result is a non-50:50 mixture of two racemic pairs. either the top or the Visualizing Chemistry 8.22 CH 3 RCO3H CH 3 I CHgC^ ^CHCH 2 CHCH 2 CHg — O CH 3 CH 3 CH 3 C = CHCH2CHCH2CH 3 KMnO, HqO + 2,5-Dimethyl-2-heptene I C=0 + CH 3 C0 2 H CH 3 I 0=CCH2 CHCH2 CH3 H3 C CH 3 H3C L °3 2. Zn, OH H3 C C=0 / H3C + 0=CHCH 2 CHCH 2 CH 3 168 Chapter 8 (b) HoC RCO3H HoC HO \ HoC HoC KMn0 4 HoC C= HoC OH 4 3,3-Dimethylcyclopentene C=0 HoC 1. 0, 2. Zn, CH 3 C0 2 H ,0 HoC 8.23 (a) CHo3 OH fH 3 CH0CHCCH0CH0 3 2 3 Hg(0Ac) 2 H 2 1 I I , CH 2 CH 3 H or 2-Ethyl-3-methyl- 1 -butene 2. (b) 1 . CH, , CH3 "OH 2. H2 1 Hg(OAc) 2 H 2 2. NaBH 4 or CH 3 BH 3 THF 2 , , rA/ HO^^^^ CH 3 4,4-Dimethylcyclopentene Both hydroboration/oxidation and oxymercuration yield the same alcohol product from the symmetrical alkene starting material. Alkenes: Reactions and Synthesis 169 8.24 Two possible alcohols might be formed by hydroboration/oxidation of the alkene shown. (not formed), and the other product results from addition to the bottom face of the double bond (formed). Addition from the top face does not occur because a methyl group on the bridge of the bicyclic ring system blocks approach of the borane. One product results from addition of BH 3 to the top face of the double bond 8.25 trans 4-Methylcyclohexene RCO3H = mefa-Chloroperoxybenzoic acid Since the hydroxyl groups in the diol product have a trans relationship, the product can only be formed by epoxide hydrolysis. (Treatment of the alkene with OSO4 yields a product in which the two -OH groups have a cis relationship.) Additional Problems Reactions of Alkenes 8.26 170 Chapter 8 H (c) HO CH 2 Os0 4 H CH 2 OH , NMO (d) H HO CH 2 H CH 2 CI CI, HoO (e) H CH 2 I 2 CH 2 , Zn/Cu (f) RC03H RCO3H = meta-Chloroperoxybenzoic acid 8.27 (a) CH 3 CH 3 CH 2 CH 2 CH 2 C=CH 2 2-Methyl- 1 -hexene CH 3 CH 2 CH 2 CH=C(CH 3 ) 2 2-Methyl-2-hexene CH 3 CH 2 CH=CHCH(CH 3 ) 2 2-Methyl-3-hexene CH 3 CH=CHCH 2 CH(CH 3 5-Methyl-2-hexene ) 2 H 2 C=CHCH 2 CH 2 CH(CH3 ) 2 CHo3 H 2 /Pd I CH 3 CH 2 CH 2 CH 2 CHCH 3 2-Methylhexane 5-Methyl-l -hexene (b) 3 ,3-Dimethylcyclohexene H 2 /Pd 4,4-Dimethylcyclohexene 1 , 1 -Dimethy lcyclohexane Reactions and Synthesis Alkenes: (c) Br CH 3 CH= CHCH 2 CH(CH 3 Br 2 ) Br I I CH 3 CH— CHCH 2 CH(CH 3 2 ) 2 2,3-Dibromo-5-methylhexane 5-Methyl-2-hexene (d) HCI H 2 C^— CHCHCH 2 CH 2 CH 2 CH 3 CHgCHCHCH 2 CH 2 CH 2 CHg CH 3 CH 3 2-Chloro-3-methylheptane 3-Methyl- 1 -heptene (e) OH _ CH3CH 2 CH 2 CH — CH 2 1. Hg(OAc) 2 H 2 2 NaBH 4 1-Pentene , CH3CH 2 CH 2 CHCH3 \ 2-Pentanol (f) CH 2 I 2 Zn/Cu , CH 2 8.28 (a) 1. c O, 2. Zn, II + H3 o H O (b) 2H KMnQ 4 CC0 C0 H 2 (c) H >CH 3 CH< 1 BH 3 THF 2. H2 , 2 ,"OH OH Remember that -H and -OH add syn across the double bond. (d) OH CH, 1. Hg(OAc) 2 H 2 Q 2. NaBH 4 , CH, 171 172 Chapter 8 8.29 H HBr Br Br secondary carbocation less stable HBr Br" CH 3 tertiary carbocation more stable Remember from Section 7.10 that a reaction that forms a more stable carbocation comparable reaction that forms a less stable carbocation intermediate. Thus, the reaction of 1-methylcyclohexene with HBr is faster than the reaction of cyclohexene with HBr. intermediate 8.30 is faster than a Recall the mechanism of hydroboration and note that the hydrogen added to the double bond comes from borane. The product of hydroboration with BD3 has deuterium bonded to the more substituted carbon; -D and -OH are cis to one another. 1 . 2. BD 3 THF , H2 2 , OH 1— H CH< 8.31 H^ c_ c -H H3C r CH 2 I 2 ^CH 3 Zn-Cu CH 2 I 2 H --C=C-" r ^CH H CH 2 HoC 13M v^n CHr3 cis- 1 ,2-Dimethylcyclopropane c/s-2-Butene H3C / Zn-Cu C c-cr^->" H trans- 1 ,2-Dimethylcyclopropane /ra«5-2-Butene The Simmons-Smith CH 2 HoC--,-> 3 ,< H' reaction occurs with syn stereochemistry. Only c/s-1,2is produced from c/s-2-butene, and only trans-1,2- dimethylcyclopropane dimethylcyclopropane is produced from ?ra«5-2-butene. Alkenes: Reactions and Synthesis 173 8.32 8.33 Step 1: Protonation of the double bond. Step 2: Nucleophilic attack of methanol on the carbocation. Step 3: Loss of proton. is the same as the mechanism shown in Section 8.4 with one exception: In this problem, methanol, rather than water, is the nucleophile, and an ether, rather than an alcohol, is the observed product. The above mechanism 174 Chapter 8 8.34 H (^PCH I >t-och 3 < 3 CI /^H— CI OCH< { H >=OCH rC' ( 3 Reaction with HC1 yields a cation Addition of pair electrons intermediate that the observed makes the double bond more can be stabilized by product. Conjugation with the oxygen lone the oxygen CI" leads to electrons. nucleophilic. There are two reasons why the other regioisomer is not formed: (1) Carbon 1 is less nucleophilic than carbon 2; (2) The cation intermediate that would result from protonation at carbon 1 can't be stabilized by the oxygen electrons. Synthesis Using Alkenes 8.35 (a) H Os0 4 a , ' NMO (b) OH OH OH 1 Hg(OAc) 2 H 2 Q , 2. NaBH 4 Acid-catalyzed hydration and hydroboration/oxidation are both additional routes to this product. H (c) CHCI 3 KOH , a* Reactions and Synthesis Alkenes: 175 (d) PH 3 H 2 SQ 4 H 2 Q OH , heat (e) HoC O 3 O _ CH— CHCHCH3 1 1 CH 3 2 II 3 • H3 Zn, . II | CH 3 CH + CH3CHCH + (f) CH 3 CHc 1 BH 3 THF 2. H2 CHgC -"" CH2 8.36 , CH 3 CHCH 2OH 2 ,"OH Because ozonolysis gives only one product, we can assume \/ c=c HoC d / CHod H3C HoC d 1.0o \ ^ n H3 , 2. Zn, \ I that the alkene is symmetrical. , ^++ » lg / c=o o=c\ + / I CHoJ CH 3 IgC!^ 2,3-Dimethyl-2-butene 8.37 Remember that alkenes can with KMn04 in acidic give ketones, carboxylic acids, and CO2 on oxidative cleavage solution. (a) CH 3 CH 2 CH=CH 2 KMn0 4 - q+ » CH 3 CH 2 C0 2 H + C0 2 (b) CH 3 CH 2 CH 2 CH= C(CH 3 KMn0 4 ) 2 u ^+ » H 3 CH 3 CH 2 CH 2 C02 H + (CH 3 ) 2 C=0 (c) C(CH 3 ) 2 KMnQ 4 H3 (d) CH 2 CH 3 (a) + (CH 3 ) 2 C= O O // 8.38 O + KMnO— —!» HoO CH 3 CH 2 CCH 2 CH 2 CH 2 CH 2 C0 2 H Addition of HI occurs with Markovnikov regiochemistry - iodine adds to the more substituted carbon. Hydroxylation of double bonds produces cis, not trans, diols. Ozone reacts with both double bonds of 1,4-cyclohexadiene. (d) Because hydroboration is a syn addition, the -H and the -OH added to the double bond must be cis to each other. (b) (c) Width: 612 Height: 792 176 8.39 Chapter 8 (a) This alcohol can't be synthesized selectively by hydroboration/oxidation. Consider the two possible starting materials. _ 1 - 1 . CHgCh^Ch^CH^- CH2 2. BH 3 THF , CH3CH2CH2CH2CH2OH _ H2 2 ,"OH 1-Pentene yields only the primary alcohol. OH 2. CH0CH9CH — CHCHo3 32 1 BH 3 THF . OH I , H2 2. 2 , CH2CH2CHCH2CH2 OH + CH 3 CH2CH2CHCH 3 2-Pentene yields a mixture of alcohols. (b) 1 (CH3)2C=C(CH 3 )2 . BH 3 THF , (CH 3 )2CHC(CH 3 )2 2H202 -QH 2,3-Dimethyl-2-butene yields the desired alcohol exclusively. (c) This alcohol can't be formed cleanly by a hydroboration reaction. The to a double bond must be cis to each other. -H and -OH added (d) The product shown is not a hydroboration product; hydroboration yields an alcohol which ~OH is bonded to the less substituted carbon. in Polymers 8.40 O HoC 3 CH, CH, II I CH, — C— CH — C— CH — C-^I H 2 C= C— COCH3 CH 2 2 2 C0 2 CH 3 C0 2 CH 3 C0 2 CH 3 8.41 many H2 CH CH 2 — CH— CH — CH— CH 2 — CH-^2 I N O N-Vinylpyrrolidone Poly(vinyl pyrrolidone) 8.42 CI H 2 C=CCI 2 Vinylidene chloride + H 2 C=CHCI Vinyl chloride \/ CI H \/ CICI W CI H \/ CI r or c /\ /\ HH HH HH H x /\ H /\ Saran The dashed bonds formed during the polymerization reaction. The structural fragments that lie between the dashed lines are the monomer units. Saran is a copolymer of vinylidene chloride and vinyl chloride. lines cross the Reactions and Synthesis Alkenes: 177 General Problems 8.43 (a) Compound A has three degrees of unsaturation. Because compound one double bond, the other two degrees of unsaturation must be rings. A contains only (b), (c) Other compounds containing two fused rings and a shared double bond also yield symmetrical diketone products. 8.44 Hydrocarbon A (C6H12) has one double bond or ring. Because A reacts with one equivalent of H2, it has one double bond and no (3) Compound A forms a diol (B) when reacted with OSO4. (1) (2) (4) When alkenes are oxidized with ring. KMn04 they give either carboxylic acids or ketones, depending on the substitution pattern of the double bond. (a) A ketone is produced from what was originally a disubstituted carbon in the double bond. (b) A carboxylic acid is produced from what was originally a monosubstituted carbon in the double bond. (5) One fragment from KMn04 oxidation is a carboxylic acid, CH3CH2CO2H. (a) This fragment was CH3CH2CH= (a monosubstituted double bond) in compound A. (b) It contains three of the six carbons of compound A. The other fragment contains three carbons. (b) It forms ketone C on oxidation. (c) The only three carbon ketone is acetone, 0=C(CH3)2. (6) (a) (d) (7) If This fragment was =C(CH3)2 in compound A. 5(a) with the one in 6(d), we join the fragment in CH3 CH 2CH=C(CH 3 )2 A we get: C 6 Hi2 The complete scheme: CH 3 CH 2 CH — CCH 3 A I * H 2 /Pd CHo3 1 . 2. Os0 4 pyridine NaHS0 3 H 2 fH 3 , CHoCHoCH^- CCHo ' 3 2, , B OH ,3 OH KMn0 4 H3O+ ] I CH 3 CH 2 CH 2 CHCH 3 CH 3 CH 2 C0 2 H + 0=C(CH 3 C ) 2 178 Chapter 8 8.45 The oxidative cleavage reaction of alkenes with O3, followed by Zn in acid, produces aldehyde and ketone functional groups at sites where double bonds used to be. On ozonolysis, these two dienes yield only aldehydes because all double bonds are monosubstituted. 1- Q3 H H3 2. Zn, O H Because the other diene 8.46 Try to solve this I H symmetrical, only one dialdehyde, is OCHCH2CHO, is produced. problem phrase by phrase. CioHigO has two double bonds and/or rings. (2) C10H18O must be an alcohol because it undergoes (1) reaction with H2SO4 to yield an alkene. When CioHisO is treated with dilute H2SO4, a mixture of alkenes of the formula produced. (4) Since the major alkene product B yields only cyclopentanone, C5H8O, on ozonolysis, B and A contain two rings. A therefore has no double bonds. (3) C10H16 is B 8.47 (a) :I— n=n=n: «* — *> :I— n— n=n: (b) mr\ -~ r * °^ va ence l ~ # of bonding electrons ' L electrons J L ? |~ ^ ' 2 _ :I— n=n=n: "123 -1 I— N=N=N -* — *- 2 I 3 A I— N— N= N B -1 electrons J Formal Charge :I— N— n=n: V nonbondingl [~# L B A +1 1 J N1 N2 +1 +1 -1 Formal charge calculations show a partial negative charge on Nl. +1 N3 -1 Alkenes: Reactions and Synthesis 179 (c) Addition of IN3 to the alkene yields a product in which -I is bonded to the primary carbon and -N3 is bonded to the secondary carbon. If addition occurs with Markovnikov + orientation, I must be the electrophile, and the reaction must proceed through an iodonium ion intermediate. Opening of the iodonium ion gives Markovnikov product for the reasons discussed in Problem 8.6. The bond polarity of iodine azide is: I— N 3 CA I-N3 CH3CH2CH I — CH2 CH3CH0CH + No 1 — CH2 CH3CH2CHCH2I :n 3 8.48 Bromoperoxidase bromonium ion y-Bisabolene 1 0-Bromo-a-chamigrene cyclic carbocation 8.49 2 H 2 /Pd 1.0o 2. Cyclooctane 1 ,5-Cyclooctadiene Zn,H 3 ft +> 2 ft HCCH 2CH 2 CH 180 Chapter 8 8.50 Focus on the stereochemistry of the three-membered ring. Simmons-Smith reaction of 1,1-diiodoethane with the double bond occurs with syn stereochemistry and can produce two isomers. In one of these isomers (A), the methyl group is on the same side of the three-membered ring as the cyclohexane ring carbons. In B, the methyl group is on the side of the three-membered ring opposite to the cyclohexane ring carbons. 8.51 CH 3 (CH2) 12 CH=CH(CH 2 7CH3 KMn0 4 ) 8.52 CH 3 (CH 2 ) 12 C0 2 H + CH 3 (CH 2) 7 C0 2 H five double bonds and/or rings. One of these double bonds reacts with H2/Pd. Stronger conditions cause the uptake of four equivalents of H2. CsHs thus contains four double bonds, three of which are in an aromatic ring, and one C=C double bond. good guess for C^R^ at this point is: CgHg has A CH=CH2 Reaction of a double bond with KMn04 yields cleavage products of the highest possible degree of oxidation. In this case, the products are C0 2 + C6H5CO2H. O II H 2 /Rh CH 2 CHg Alkenes: Reactions and Synthesis 181 8.53 .OH + Br \ HO^^-^N // Bromoperoxidase Laurediol .OH carbocation Prelaureatin 8.54 (a) Bromine dissolved in CH2CI2 has a reddish-brown color. When an alkene such as cyclopentene is added to the bromine solution, the double bond reacts with bromine, and the color disappears. This test distinguishes cyclopentene from cyclopentane, which does not react with Br2- Alternatively, each compound can be treated with H^/Pd. The alkene takes up H 2 and the alkane is unreactive. , (b) An aromatic compound such as benzene is unreactive to the B^/Ct^C^ reagent and can be distinguished from 2-hexene, which decolorizes Br2/CH2Cl2. Also, an aromatic compound doesn't take up H2 under reaction conditions used for hydrogenation of alkenes. 8.55 CI .0 tr 1 CI Cl— c: C: cr I : ci Q- CI ci + COc In step 1 carbon dioxide is lost from the trichloroacetate anion. In step 2, elimination of chloride anion produces dichlorocarbene. Step 2 is the same for both the above reaction and the base-induced elimination of HC1 from chloroform, and both reactions proceed , through the trichloromethanide anion intermediate. 182 Chapter 8 8.56 (a) u-Terpinene, C10H16, has three degrees of unsaturation. Hydrogenation removes only two degrees of saturation, producing a hydrocarbon C10H20, that has one ring. «-Terpinene thus has two double bonds and one ring, (b) (c) Glyoxal a-Terpinene 8.57 6-Methyl-2,5heptanedione cis and trans diols show that it is much easier to form a five-membered from the cis diol A than from the trans diol B. The cis periodate The models of the cyclic periodate intermediate is of lower energy than the trans periodate intermediate because of the lack of strain in the cis periodate ring. Because any factor that lowers the energy of a transition lowers AG* and increases the rate of reaction, diol cleavage should proceed more slowly for trans diols than for cis diols. state or intermediate also 8.58 CH, ^Br: trans- 1 -Bromo-3cis-l-Bromo-3methylcyclohexane methylcyclohexane CH, • -J Br trans- l-Bromo-2- methylcyclohexane c/s-l-Bromo-2methylcyclohexane HBr, two intermediate carbocations of approximately equal stability are formed. Both react with bromide ion from top and bottom faces to give four different products. In the reaction of 3-methylcyclohexene with :Br:^ +Rr H— Br Br The most stable cation intermediate from protonation of 3-bromocyclohexene is a cyclic bromonium ion, which is attacked by Br from the opposite side to yield trans product. Alkenes: Reactions and Synthesis 183 8.59 OAc / < \>— OCH 3 NaBH 4 Cyclohexyl methyl ether 8.60 C0 2 CH 3 OAc The mechanism involves the following steps: Addition of Hg(OAc)2 to one of the double bonds to form a cyclic mercurinium reaction Step 1: ion. Step 2: Reaction of a second double bond with the mercurinium ion to form a membered ring and six- a different carbocation. second cyclization forms the other ring and yields another carbocation. Step 3: A Step 4: Removal of -H gives a double bond. 184 Chapter 8 8.61 s~+-Br— Br CV / / H2C h2 c H2 C— CH 2OH Step Step Step 1. HC— CH 2? Br I H2C 2. I CH: HC— CHoBr 2 / \.. ^"6h HoC- I H 2C / :0H 2 lM CH 2 Formation of a cyclic bromonium ion. -OH on the bromonium + 3: Loss of H \ H 2? C +Q-rrf 3. | H2 C I CH 2 1: 2: Nucleophilic attack of ion. . The above mechanism is the In this case, the nucleophile 8.62 + \ HC— CH 2 HC— CHo* + H3 Br" same is as that for halohydrin formation, shown in Section 8.3. the hydroxyl group of 4-penten-l-ol. Hydroboration of 2-methyl-2-pentene at 160 °C is reversible. The initial organoborane intermediate can eliminate BH3 in either of two ways, yielding either 2-methyl-2-pentene or 4-methyl-2-pentene, which in turn can undergo reversible hydroboration to yield either 4-methyl-2-pentene or 4-methyl-l-pentene. The effect of these reversible reactions is to migrate the double bond along the carbon chain. A final hydroboration then yields the most stable (primary) organoborane, which is oxidized to form 4-methyl-l-pentanol. Reactions and Synthesis Alkenes: 185 8.63 CH 3 (a) equiv Br2 1 CH3CHCH2CH2 J^f C=Cv / H Br CH 3 CH 3 (b) CH3CHCH2CH2C^^ CH 2 equiv H 2 CH3CHCH2CH2CH2CH3 Pd/C Br (c) 1 equiv HBr I CH3CHCH2CH2C — CH2 Addition of one equivalent of HX or X2 to a triple bond occurs with Markovnikov regiochemistry to yield a product in which the two added atoms usually have a transrelationship across the double bond. 8.64 .0 O. Os o Os0 4 H ^C=C^' H o NMO C~ C \ H-^ HoC / HO OH V / " H-'/- Cv-H C"H HoC CHo cw-2-Butene OsN / o H~ ^C-C^ r^-' HqC H"^ H3 C H /ra«.s-2-Butene o C~ C C'CH 3 \ OsO, CH, / NMO^ OH HO \ ** / C_CC-CH H"^ H3C H 3 H Formation of the cyclic osmate, which occurs with syn stereochemistry, retains the cistrans stereochemistry of the double bond because osmate formation is a single-step reaction. Oxidation of the osmate does not affect the stereochemistry of the carbon-oxygen bond, and the diol produced from cw-2-butene is a stereoisomer of the diol produced from rra«s-2-butene. 8.65 A has four multiple bonds/rings. ^3^\ HO— ^ CH2CH2CH2 dil. ^CHCH2CHg HgCs,^ H 2 S0 4 1. 2-Phenyl-3-pentanol B is also an acceptable answer. OHCCH 2 CH 3 O, 2. Zn, A HoC H3 + Width: 612 Height: 792 Review Unit 3: Organic Reactions; Alkenes Major Topics Covered (with vocabulary): Organic Reactions: addition reaction reaction elimination reaction mechanism homolytic polar reaction curved arrow substitution reaction heterolytic rearrangement reaction homogenic heterogenic propagation termination electronegativity electrophile nucleophile carbocation initiation radical reaction polarizability Describing a Reaction: exergonic endergonic enthalpy entropy heat of reaction exothermic endothermic Keq AG bond dissociation energy reaction energy diagram transition state activation energy reaction intermediate Introduction to alkenes: degree of unsaturation methylene group vinyl group allyl group E,Z isomerism heat of hydrogenation hyperconjugation cis-trans isomerism Electrophilic addition reactions: electrophilic addition reaction carbocation rearrangement regiospecific Markovnikov's rule Hammond Postulate hydride shift Other reactions of alkenes: dehydrohalogenation dehydration anti stereochemistry bromonium ion halohydrin hydration oxymercuration hydroboration syn stereochemistry carbene stereospecific Simmons-Smith reaction hydrogenation hydroxylation diol osmate molozonide ozonide Polymerization reactions: polymer monomer chain branching radical polymerization cationic polymerization Types of Problems: After studying these chapters you should be able to: - Identify reactions as polar, radical, substitution, elimination, addition, or rearrangement reactions. - Understand the mechanism of radical reactions. Identify reagents as electrophiles or nucleophiles. to draw reaction mechanisms. Understand the concepts of equilibrium and rate. Calculate and AG of reactions, and use bond dissociation energies to calculate A#° of Use curved arrows reactions. - Draw reaction energy diagrams and - Calculate the degree of unsaturation of any compound, including those containing N, O, and halogen. Name acyclic and cyclic alkenes, and draw structures corresponding to names. Assign E,Z priorities to groups. - label them properly. Review Unit 3 - Assign cis-trans and E,Z designations to double bonds. Predict the relative stability of alkene double bonds. - Formulate mechanisms of electrophilic addition reactions. Predict the products of reactions involving alkenes. Choose the correct alkene starting material to yield a given product. Deduce the structure of an alkene from its molecular formula and products of cleavage. Carry out syntheses involving alkenes. - - Points to 187 Remember: all cases, a compound is of lower energy than the free elements of which it is composed. Thus, energy is released when a compound is formed from its component elements, and energy is required when bonds are broken. Entropy decreases when a compound is formed from its component elements (because disorder decreases). For two compounds of similar structure, less energy is required to break all bonds of the higher energy compound than is required to break all bonds of the lower energy compound. * In virtually * Calculating the degree of unsaturation is an absolutely essential technique in the structure determination of all organic compounds. It is the starting point for deciding which functional groups are or aren't present structure determination in a problem given compound, and eliminates many possibilities. When a is given, always calculate the degree of unsaturation first. * All cis-trans isomers can also be described by the E,Z designation, but not can be described by the cis-trans designation. * Bond dissociation * Not all E,Z isomers energies, described in Chapter 6, measure the energy required to homolytically break a bond. They are not the same as dissociation enthalpies, which measure the ability of a compound to dissociate heterolytically. Bond dissociation energies can be used to calculate dissociation enthalpies in the gas phase if other quantities are also known. all hydrogens bonded to carbons adjacent to a carbocation can take part in hyperconjugation at the same time. At any given instant, some of the hydrogens have bonds that lie in the plane of the carbocation and are not suitably oriented for hyperconjugative overlap. C-H Review Unit 3 188 Self-Test: HO OH \ / f? /C^ H3C H2 + CH3 HgC CHg A What type of reaction is or a radical mechanism? If K Would you expect A5° to occurring in eci A? Would you expect that the reaction occurs by for the reaction at K is 298 a polar 10~3 , what sign do you expect for AG°? be negative or positive? What about A//°? PH 3 CH, (CH 3 ) 3C n / OH CH 3 O' B C0 2 H H H C=C\ CH(CH 3 ) 2 Abscisic acid (a plant hormone) Give E,Z configurations for the double bonds in B. Provide a name for stereochemistry). Predict the products of reaction of with (a) 1 equiv HBr + C BH3, THF, then H2 2, HO (d) 3, then Zn, H3 C bond Pd/C (c) (include (b) H2 , . D isomeric compounds and E have the formula Q0H16. On hydrogenation, each reacts with two molar equivalents of H 2 Ozonolysis of each compound yields the following fragments: Two compound . How many rings/double bonds do D and E have? What are the structures of D and E? Review Unit 3 189 Multiple Choice: 1 . Which of the following molecules (a)BH 3 (b)NH 3 2 . (c) Which of the following (a) addition reaction (d) is not a nucleophile? H 2C=CH 2 reactions probably has the greatest entropy increase? (b) elimination reaction . 4 . 5 . rearrangement At a specific temperature T, a reaction has negative AS" and Keq > 1. What can you say about AG°andAtf°? (a) AG is negative and A#° is positive (b) AG° and A//° are both positive (c) AG° and AH° are both negative (d) AG is negative but you can't predict the sign of A//°. In which of the following situations is AG* likely to be smallest? (a) a slow exergonic reaction (b) a fast exergonic reaction (c) a fast endergonic reaction (d) a slow endergonic reaction What is (a) . the degree of unsaturation of a . compound whose molecular formula is Ci 1H13N? 4 (b)5 (c)6 (d)7 Two equivalents of H2 are needed to hydrogenate a hydrocarbon. It is also known that the 15 carbons. What is its molecular formula? compound contains two rings and has (a)C 15 H 22 (b)C 15 H 24 (c)C 15 H 28 7 (c) substitution reaction (d) 3 3 6 HO- What is the usual relationship (d) C 15 H 32 between the heats of hydrogenation of a pair of cis/trans alkene isomers? (a) Both have positive heats of hydrogenation (b) Both have negative heats of hydrogenation, and A//hydrog f° r tne c is isomer has a greater negative value (c) Both have negative heats of hydrogenation, and A//hydrog f° r the trans isomer has a greater negative value (d) Both have negative heats of hydrogenation, but the relationship between the two values of A//h y drog can 't be predicted. 8 . what is the relationship of the two transition states? both resemble the intermediate (b) the first resembles the starting material, and the second resembles the product (c) the first resembles the intermediate and the second resembles the product (d) there is no predictable relationship between the two transition In a two-step exergonic reaction, (a) states 9. For synthesis of an alcohol, acid-catalyzed hydration of an alkene 10. A reaction that produces a diol from an alcohol is a: is useful in all of the following instances except: (a) when an alkene has no acid-sensitive groups (b) when an alkene is symmetrical (c) when a large amount of the alcohol is needed (d) when two possible carbocation intermediates are of similar stability. (a) hydration (b) hydrogenation (c) hydroboration (d) hydroxylation Chapter 9 - Alkynes: An Introduction to Organic Synthesis Chapter Outline I. Introduction to alkynes (Section 9. 1-9.2). A. Naming alkynes (Section 1 . The rules for 9.1). naming alkynes are like the rules for alkenes (Sec. 7.3), with a few exceptions. a. b c . . The suffix -yne is used for an alkyne. Compounds with both double bonds and When triple d. bonds are enynes. bonds receive lower numbers than triple there is a choice in numbering, double bonds. Compounds can also contain alkynyl groups. B. Preparation of alkynes (Section 9.2). 1 Alkynes can be prepared by elimination reactions of 1,2-dihalides, using a strong . base. dihalides are formed by addition of 2 to alkenes. 3 Vinylic halides give alkynes when treated with a strong base. Reactions of alkynes (Sections 9.3-9.6). 2 . X The . II. A. General principles (Section 9.3). bonds result from the overlap of two sp-hybridized carbon atoms, a. One o bond and two n bonds are formed. 2 The length (120 pm) and strength (965 kJ/mol) of a -OC- bond make it the strongest carbon-carbon bond. 3 Alkynes are somewhat less reactive than alkenes in electrophilic addition reactions B. Addition of X 2 and HX. 1 HX adds to alkynes by an electrophilic addition mechanism. a. Addition of two equivalents of HX occurs if the acid is in excess. b Addition occurs with Markovnikov regiochemistry and with trans 1 . Alkyne triple . . . . stereochemistry. 2 3 . . X2 also adds in the same manner, and trans stereochemistry is observed. The intermediate in addition reactions is a vinylic carbocation, which forms less readily than an alkyl carbocation. 4 Mechanisms of some alkyne addition reactions are complex. C. Hydration reactions of alkynes (Section 9.4). . 1 . Hg(H)-catalyzed additions. a. The -OH group adds to the more substituted carbon to give Markovnikov b . c . d . product. The intermediate enol product tautomerizes to a ketone. The mechanism is similar to that of addition to alkenes, but no NaBH* is necessary for removal of Hg. mixture of products is formed from an internal alkyne, but a terminal alkyne A yields a methyl ketone. Hydroboration/oxidation of alkynes. a. Hydroboration/oxidation of alkynes gives an intermediate enol product that tautomerizes to a carbonyl product. Hydroboration of a terminal alkyne gives an aldehyde. i. Hydroboration of an internal alkyne gives a ketone. ii. b Hydroboration/ oxidation is complementary to Hg(II)-catalyzed hydration. D. Reduction of alkynes (Section 9.5). Complete reduction to an alkane occurs when H2/Pd is used. 1 2 Partial reduction to a cis alkene occurs with H2 and a Lindlar catalyst. 2. . . . Alkynes: 3 . Partial reduction with Li in The a. An Introduction to Organic Synthesis 191 NH3 produces a trans alkene. reaction proceeds through an anion radical — > vinylic radical — > vinylic anion. b The more stable trans vinylic anion is formed. E. Oxidative cleavage of alkynes (Section 9.6). 1 O3 or KMn04 cleave alkyne bonds to produce carboxylic acids or CO2 (terminal alkyne). 2 Oxidative cleavage reactions were formerly used for structure determinations, Alkyne acidity (Sections 9.7-9.8). A. Formation of acetylide anions (Section 9.7). 1 Terminal alkynes are weakly acidic (pA^a = 25). 2 Very strong bases CNH2) can deprotonate a terminal alkyne, yielding an acetylide anion, 3 Acetylide anions are stabilized by the large amount of "s character" of the orbital that holds the electron. B Alkylation of acetylide anions (Section 9.8). 1 Acetylide anions are strongly nucleophilic. 2. Acetylide anions can react with haloalkanes to form substitution products. a. The nucleophilic acetylide anion attacks the electrophilic carbon of a haloalkane to produce a new alkyne. b This reaction is called an alkylation reaction. c. Any terminal alkyne can form an alkylation product. 3 Acetylide alkylations are limited to primary alkyl bromides and iodides. a. Acetylide ions cause dehydrohalogenation reactions with secondary and tertiary . . . m. . . . . . . . halides. IV. Organic synthesis (Section 9.9). A. Reasons for the study of organic synthesis. In the pharmaceutical and chemical industries, synthesis produces 1 or better routes to important molecules. . new molecules, In academic laboratories, synthesis is done for creative reasons. 3 In the classroom, synthesis is a tool for teaching the logic of organic chemistry. Strategies for organic synthesis. 2. . B . 1 . Work backward from the 2 . Keep structure of the product, but - the structure of the starting material in mind. Solutions to Problems 9 . 1 naming alkynes are almost the same as the rules for naming alkenes. The used, and compounds containing both double bonds and triple bonds are enynes, with the double bond taking numerical precedence. The rules for suffix -yne is (a) CHo I CHo I CH 3 CHC= CCHCH 3 (b) CHo I HC= CCCH 3 CH 3 2,5-Dimethyl-3-hexyne 3,3-Dimethyl- 1 -butyne - 192 Chapter 9 (d) (c) CH 3 _ CH2CH2CC— CCH2CH2CH2 CHo ,3,3 CHo I CH3CH2CC — CCHCH3 CH 3 CH 3 2,5,5-Trimethyl-3-heptyne 3 ,3-Dimethyl-4-octyne m CH 3 CH= CHCH= CHC= CCH 3 2,4-Octadien-6-yne 6-Isopropylcyclodecyne (not 4,6-Octadien-2-yne) 9.2 Ch^Ch^C-r^Ch^C^ CH CH 3 CH2C= CCH2CH 3 Ch^Ch^Ch^C— CCH 3 1-Hexyne 3-Hexyne 2-Hexyne CHo3 fH 3 CH 3 CHCH 2 C= CH CH 3 CH 2 CHC=CH 3-Methy 1- 1 -penty ne I _ CH 3 CHC= CCH 3 4-Methyl- 1 -pentyne 4-Methyl-2-pentyne CH 3 CHoCC= CH 3 | CH 3 3,3-Dimethyl- 1 -butyne 9 3 . Markovnikov addition is observed with alkynes as well as with alkenes. (a) CH 3 CH 2 CH 2 C=CH CH 3 CH2CH2CCl2CHCl2 2CI2 + (b) CH 2 C=CH +1 HBr Br (c) CH 3 CH2CH2CH2 CH 3 CH2CH 2 CH 2 C= CCH 3 + 1 HBr / H Two products result from addition to GHgCH^CH^GHg ^H c=c *- an internal alkyne. \ CH 3 Br CH 3 Alkynes: An Introduction to Organic Synthesis 9.4 hhO CH3CH2CH2C— CCH2CH2CH3 • O + CHgCh^Ch^Ch^CCh^Ch^CHg HgS0 4 This symmetrical internal alkyne yields only one product. CH 3 P II _ CHqCHpCHpC^ CCHpCHCHq H 3Q 1 13 c d c I CH3CH2CH2CH2CCH2CHCH3 CH 3 J + O _ _ , HgS0 4 , L I CH3CH2CH2CCH2CH2CHCH3 Two ketone products result from hydration of 2-methyl-4-octyne. 9.5 (a) OH _ CHoCHpCHpC — CH 6 * * + H 3Q * CH3CH2CH2C — CH2 ^ HgS0 4 (b) O CH 3 CH2C — CCH 3 The 9 6 . CH 3 CH2CH2CCH 3 O CHgCh^CCh^CHg HgS0 4 + CH2CH2CH2CCH3 desired ketone can be prepared only as part of a product mixture. Remember that hydroboration yields aldehydes from terminal alkynes and ketones from internal alkynes. (a) O — C=CH a 1 . - // \ // 2. THF BHo, H2 II = 2 , CH 2 CH \ OH (b) O (CH 3 2 CHC= CCH(CH 3 ) 2 1 BH 3 THF 2. H2 , (CH 3 ) 2CHCH 2 CCH(CH 3 ), ) 2 ,"OH 9.7 (a) if C=CH Br H 3 Q+ > HgS0 4 1 BH 3 THF 2. H2 , 2 ,"OH XT* o. 193 194 9 8 . Chapter 9 The bond with correct reducing reagent gives a double (a) the desired geometry, CH3CH2CH2CH2CH2 Li/NHr CHgCh^Ch^Ch^Ch^C^^ CCH 3 CH 3 H 2-Octyne /rans-2-Octene (b) CH3CH2CH2 H2 CH3CH2CH2C — CCH2CH3 Lindlar / C=CN H 3-Heptyne CH2CH3 H c/5-3-Heptene (c) Li/NH, CH 3 CH 3 CH 2 CHC=CH 3-Methyl- 1 -pentyne 9.9 CH 3 ' / \ _ CH2 CH3CH2CHCH— r or H2 3-Methyl- 1 -pentene Lindlar A base that is strong enough to deprotonate acetone must be the conjugate base of an acid weaker than acetone. In this problem, only deprotonate acetone. Na+ ~C=CH is a base strong enough to 9.10 Remember that the alkyne must be a terminal alkyne and the halide must be primary. than one combination of terminal alkyne and halide may be possible. R'X (X=Br or Alkyne I) More Product (a) CH 3 CH2CH2C — CH CH 3 X CHjjCh^Ch^C— CCH 3 or HC=CCH 3 CH 3 CH2CH2X 2-Hexyne (b) (CH 3 ) 2 CHC= CH CH 3 CH2X (CH 3 )2CHC^ CCH2CH 3 2-Methyl-3-hexyne (c) C=CH CH 3 X C= CCH3 Products (b) and (c) can be synthesized by only one route because only primary halides can be used for acetylide alkylations. An Alkynes: 9.11 The cis Introduction to Organic Synthesis 195 double bond can be formed by hydrogenation of an alkyne, which can be synthesized by an alkylation reaction of a terminal alkyne. 1 CHoC=CH 3 . 2. NaNH 2 NHo , CH 3 Br,THF — Ho 2 CHoC=CCHo 3 3 H H \ »- Lindlar / / C=C\ CH 3 H3C catalyst ds-2-Butene 9.12 The starting material is the target molecule and (a) To reduce a triple CH3CH2CH2OCCH2CH2CH3. Look at the functional groups in work backward bond to a double to 4-octyne. bond with H 2 with Lindlar cis stereochemistry use catalyst. CH3CH2CH2 2 CHoCHoCHoC^r CCH9CH9CH0 2 2 3 2 2 3 ^Ch^Ch^CHg C~~ C ^ / Und|ar H H catalyst ds-4-Octene (b) An aldehyde is the product of double-bond cleavage of an alkene with O3. The starting material can be either c/s-4-octene or fram^-octene. CH3CH2CH2 ,C / ^CH20H2CHg — Q\ H H from (c) \ — H from \ 2 3 |_j Butanal (a) Addition of HBr to cw-4-octene [part CHqCHpCHp 3 CHqCHpCHpC CH C0 H -32 2. Zn, / (a)] yields CHpCHpCHo 2 2 3 HBr 4-bromooctane. I * CHgCH2CH2CHCH2CH2CH2CHg H 4-Bromooctane (a) Alternatively, lithium/ammonia reduction of 4-octyne, followed by addition of HBr, gives 4-bromooctane. (d) Hydration or hydroboration/oxidation of ds-4-octene [part (a)] yields 4-hydroxyoctane (4-octanol). 1. Hg(OAc) 2 H 2 Q , CH3CH2CH2 2 H 1.BH 3 ,THF f . C=C^ 2. from (a) NaBH 4 or / H 0H CH2 CH 2CH 3 H2 CH3CH2CH2CHCH2CH2CH2CH3 2 , OH ^ 4-Hydroxyoctane (4-Octanol) Width: 612 Height: 792 196 Chapter 9 (e) Addition of Cl 2 to 4-octene [part CH 3CH 2 CH 2 / CH 2 CH 2 CH 3 (a)] yields Cl 2 4,5-dichlorooctane. CI CI ^ ^C^ C^ H H from (f) CHgCH 2 CH 2 CHCHCH 2 CH 2 CH2 4,5-Dichlorooctane (a) KMn04 cleaves 4-octyne into two four-carbon fragments. KMnQ 4 ^ _ CH3CH 2 CH 2 C= CCH 2 CH 2 CH3 —^ ~ HoO 2 CH3CH 2 CH 2 C0 2 H Butanoic acid 9.13 The following syntheses are explained in detail in order to illustrate retrosynthetic logic the system of planning syntheses by working backwards. (a) 1 . An immediate . CH3CH2CH2CH2CH2CH2CH2CH2CH2CH3 might be CsHnCsCH, which can be reduced to decane by H2/Pd. precursor to an alkene or alkyne. Try 2 - The alkyne CgHiyC^CH can be formed by alkylation of + HC=C:~Na by C 8 Hi 7 Br, 1-bromooctane. - 3 . HOC: Na+ can be formed by treatment of HOCH with NaNH2 NH3 , HC=CH NaNH 2 > — NH3 HC=C:-Na+ —— C 8 H 17 Br THr ** C 8 H 17 C=CH H 2/Pd . m Decane C 8 Hi 7 Br = 1 -Bromooctane (b) 1 . An immediate CH3CH2 CH 2 CH 2 C(CH3)3 might be precursor to HC=CCH2CH2C(CH3)3, which, when hydrogenated, 2 . yields 2,2-dimethylhexane. HOCCH 2CH2C(CH3) 3 can be formed by alkylation of HOC:" Na+ (from a.) with HC=CH BrCH 2 CH 2 C(CH3)3. NaNH 2 NH 3 > HC=C:- Na+ BrCH 2 CH 2 C(CH3)3 HC=C:~Na^ y^p _ CCH CH C(CH3)3 — 2 2 2H P ^ ptj* CH3CH 2 CH 2 CH 2 C(CH3)3 2,2-Dimethylhexane Alkynes: (c) 1 . An Introduction to Organic Synthesis CH3CH2CH2CH2CH2CHO can be made by CH3CH2CH2CH2OCH treating 197 with borane, followed by H2O2. 2. CH 3 CH2CH2 CH2<>CH can be + HC-C:~Na synthesized from CH3CH2CH2CH 2Br and . HC=CH NaNH 2 — NH > HC=C:~Na+ 3 CH 3 CH 2 CH 2 CH 2 Br HC=C:-Na+ (d) 1 . 2. _ THF 1 CHgCh^Ch^Ch^C^^ CH 9 ' * ' . 2. BH 3 THF , H2 _ 2 , OH H _ * CH3CH2CH2CH2CH2CH Hexanal The desired ketone can be formed by mercuric-ion-catalyzed hydration of 1heptyne. 1-Heptyne can be synthesized by an alkylation of sodium acetylide by bromopentane. HC=CH NaNH 2 > — NH 1- HC=C:"Na + 3 HC=C:-Na+ + CH 3 CH2CH2CH 2 CH2Br CH 3 CH2CH 2 CH2CH 2 C= CH THF H ? S0 4 ,H 2 H gSO CH 3 CH 2 CH 2 CH2CH2C= CH II * * CH 3 CH 2 CH 2 CH 2 CH 2 CCH 3 2-Heptanone 198 Chapter 9 Visualizing Chemistry 9.14 (a) CH 3 H2 (i) — CHCH2CCH2CH3 Lindlar CH, CH, catalyst HC — CCH2CCH2CH3 CH 3 H3 (ii) 4,4-Dimethyl- 1 -hexyne O + CH, CH3CCH2CCH2CH3 HgS0 4 CHo (b) CH, CH3CHCH2 CH2CHCH3 H2 (i) CH 3 CH, CH 3 Lindlar H H CH 3 O catalyst CH3CHCH2C^^ CCH2CHCH3 M) 2,7-Dimethyl-4-octyne h3 o + II I CH 3 I CH3CHCH2CH2CCH2CHCH3 HgS0 4 9.15 (a) CHo3 CH 3 BH 3 THF - ~ ~ 1 I CH 3 CHCH 2 C=CH . , d. ri2U2, -* Un O I CH 3 CHCH 2 CH 2 CH 4-Methyl- 1 -pentyne An aldehyde is formed by reacting a terminal alkyne with borane, followed by oxidation, (b) CH, CH, C= CH 2 HCI ? CCH, , ether CI 9.16 First, draw the structure of each target compound. Then, analyze the synthetic route. (a) OH /C H 2 CH3CHCH2CH — CH2 (b) f? I H2C CHCH2CH2CCH3 structures for a An Alkynes: (a) The 199 and the right side might have double bonds as immediate precursors; the a Simmons-Smith carbenoid addition to an alkene, and the left result from hydration of an alkene. Let's start with 3-bromo-l-propene. left side right side side Introduction to Organic Synthesis may may result from /CH 2 CH2I2 BrCH 2 CH= CH 2 Zn(Cu) BrCH 2 CH— CH 2 HC=C:~Na+ / CH 2 \ HC=CCH2CH— CH 2 H2 Lindlar OH CH 2 / \ CHCH 2 CH—CHp2 (b) The right side /Q\H 2 HpC= CHCHpCH— CH. 2 2 catalyst I 1 . «* 2. —— Hg(OAc) ? H 2 ia 2 , NaBH 4 ' can result from Hg-catalyzed addition of H2O to a terminal alkyne. HC=C:-Na+ H 2 C= CHCH 2 CH 2 Br *- _ H 2 C= CHCH 2 CH 2 C= CH H3 + HgS0 4 o _ H2C^- CHCh^Ch^CCHg II 9.17 It's not possible to form a small ring containing a triple bond because the angle strain that result from bending the bonds of an sp-hybridized carbon to form a small ring is too would great. Additional Problems Naming Alkynes 9.18 (b) (a) fH 3 CCCHg CH3CH2C CHgC CCH2C CCH2CH3 CH 3 2,2-Dimethyl-3-hexyne CHo3 I CHo3 2,5-Octadiyne (d) fH 3 I HC= CCCH2 C= CH CH 3 CH= CC= CCHCH 3 CH 3 3 ,6-Dimethyl-2-hepten-4-y ne 3,3-Dimethyl- 1 ,5-hexadiyne ^ (e) H 2 C= CHCH= CHC= CH 1 ,3-Hexadien-5-yne CH 2 CH 3 ,3 CH 3 CH 2, 2CHC= CCHCHCH 3 CH2CH 3 CH 3 3 ,6-Diethyl-2-methy 1-4-octyne 200 Chapter 9 9.19 (b) (a) CHo3 3 I I CH 3 C= CC= CCHCH 2 CHC= CH CH3CH2CH2C— CCCH2CH3 CH 3 CH2CH2 3 ,3-Dimethyl-4-octyne 3-Ethyl-5-methyl- 1 ,6,8-decatriyne (d) (c) I _ CH, 3 I CH 2 C=CCH CHoCC— CCCHo3 3, ( CHg ^CHCH3 ^C^ CH3 CH2CH2CH2CH2 3 ,4-Dimethy Icy clodecy ne 2,2,5 ,5-Tetramethyl-3-hexyne (e) (f) CHo CI I CH 3 CH= CHCH= CHC= CH CHoCH2C^^ CCH2C CHCH _ — CH2 I CH 3 3,5-Heptadien- 1-yne 3-Chloro-4,4-dimethyl- 1 -nonen-6-yne (g) (h) CH3CHCH2CH3 ^( 5-te^Butyl-2-methyl-3-octyne 3-sec-Butyl- 1 -heptyne (a) H3 ^ CHgCH2CH2CHC^^ CCHCHg CHgCH2CH2CH2CHC^^ CH 9.20 CH 3)3 CH3 CH=CHC=CC=CCH=CHCH=CHCH=CH 2 . 1,3,5,1 l-Tridecatetraen-7,9-diyne Using E-Z notation: (3£',5£',1 1£)-1, 3,5,1 l-Tridecatetraen-7,9-diyne The parent alkane of this hydrocarbon (b) CH 3 C=CC=CC=CC=CC=CCH=CH 2 is . tridecane. l-Tridecen-3,5,7,9,1 1-pentayne This hydrocarbon also belongs to the tridecane family. Reactions of Alkynes Alkynes: An Introduction to Organic Synthesis 9.22 (a) Br 1 CHgCh^Ch^Ch^C^^ CH equiv_ HBr CH3CH2CH2CH2C — CH2 * ! CH3CH2CH2CH2 (b) 1 — CH3CH2CH2CH2C— CH * equiv_ ~ Cl" 2 ^ Cl — / f A H CI (c) = _ H2 CHqCHoCHoCHoC 2 2 — CH 3 2 CHoCHoCHoCHoCH^— CHo Lindlar catalyst (d) _ 1 CH3CH2CH2CH2C— CH . NaNH 2 NH 3 ^ ^ , CH3CH2CH2CH2C — CCHg 3 O (e) _ CH3CH2CH2CH2C — CH H 2 Q, H 2 SQ 4 * ~ §Q II CH3CH2CH2CH2CCH3 « a CH 3 CH2CH 2 CH 2 C= CH 2 ^!"' V » CHgCHgCHgCHgCCHg HCI | CI 9.23 (a) CH3CH2CH2CH2 CH 3 (CH 2 )3C= C(CH 2 )3CH 3 * Lindlar catalyst ^CH2CH2CH2CH3 C= C\ / H H 201 202 Chapter 9 ^ ChhCHoCHpCHp NH 3w Li in CH 3 (CH 2 )3C= C(CH 2 ) 3 CH 3 \ ** p H / — CH2CH2CH2CH3 H CH 3 CH2CH2CH2 CH 3 (CH 2 3C=C(CH 2 3 CH 3 ) e 1 V |j"' ) » ^Br C=C^ CH2CH2CH2CH 3 Br (d) 1 CHg(CH2)3C — C(CH2)3CH3 ~ BH 3 THF . if , H~ O — OH* CH3CH2CH2CH2CH2CCH2CH2CH2CH3 O (e) H 2°- H 2S0 CH 3 (CH2)3C=C(CH2) 3 CH3 4> > (f) v excess H2 ' CH 3 (CH 2 )3C= C(CH 2 )3CH 3 9.24 Mixtures of products are CH3CH2CH2CH2CH2CCH2CH2CH2CH3 HgS04 ** Ch^CH^Cr^ p^/Q sometimes formed since the alkynes are unsymmetrical. (a) CH 3 CH 2 CH 2 C= CCH 3 —2 equiv t —*" CH 63 CH d2 CH d2 C= CCH 63 —HBr 1 (b) CH 3 CH 2 CH2C(Br2)C(Br2)CH3 CH3CH2CH2 1 equiv ,7„ * *• / CH3CH2CH2 H C=CA rC=C\ + CH 3 Br Br / \ ' CH 3 H (c) CH 3CH 2 CH 2 C= CCH 3 excess HBr CH 3 CH2CH2C(Br2)CH 2 CH3 CH3CH2CH2CH2C(Br2)CH3 + (d) CH^CHpCHp Li in Nh^ NH, \ CH3CH2CH2C^^ CCH3 / H _ / C— C \ CH3 H (e) O _ CHoCHoCHoC^^ CCHo 3 2 2 3 H 2 Q, H 2 SQ 4 ^ HgS0 4 H CH0CH0CH0CCH0CH0 3 2 2 2 3 O || + CH3CH2CH2CH2CCH3 Alkynes: 9.25 Both KM11O4 and 3 An Introduction to Organic Synthesis 203 oxidation of alkynes yield carboxylic acids; terminal alkynes give C02 also. In (a), (b), and (c), the observed products can also be formed by KMnC>4 oxidation of the corresponding alkenes. (a) CH 3 (CH 2 (b) ^ss ) 5 C=CH .C=CCH3 KMnO^ CH 3 (CH 2 )5C0 2 H HoO + CH 3C02 H + HoO + C0 2 2H aC0 KMn0 4 + (c) KMn04 H0 2 C(CH 2 ) 8 C0 2 H HqO + Since only one cleavage product bond as part of a ring. is formed, the parent hydrocarbon must have contained a triple H f * CH 3 CH= CCH 2 CH 2 C= CH ,o3 f? - ' H3 + * * CH 3 CH0 + CH 3 CCH 2 CH 2 C0 2 H + C0 2 Notice that the products of this ozonolysis contain aldehyde and ketone functional groups, as well as a carboxylic acid and CO2. The parent hydrocarbon must thus contain a double and a triple bond. H 204 Chapter 9 Organic Synthesis 9.27 O CI A n A O 1. RCH HCI I RCHCH 3 O A RCH 2 CH 3 1 . / / 2. BH 3 THF , H2 _ » 2 , OH II RCH 2 CH R— C=CH \N RCH= CH 2 A . 2. _ c* R— C= NaNHp, NH 3 S CH 3 Br O O Rv / 1 ^ H A z c— C^ -* H 1 H 1 . 2. H 2 0, H 2 S0 4 RC0 3 H + H3 II RCCHg HgS0 4 9.28 (a) CH 3 CH 2 C— CH H 2 Q, H 2 SQ 4 HgS0 4 f? CH 3 CH 2 CCH 3 (b) CH 3 CH 2 C:^ CH (c) 1 BH 3 THF 2. H2 , 2 ,~OH CH 3 CH 2 CH 2 CHO ^^C=CH C=CCH3 1 NaNH 2 NH 3 CH 3 Br , 2. (d) C=CCH3 H2 I CH 3 Lindlar catalyst (e) CH 3 CH 2 C — CH KMn0 4 (0 CH 3 CH 2 CH 2 CH 2 CH — CH 2 H3 + Br2 CH 2 CI 2 CH 3 CH 2 C0 2 H C0 2 + CH 3CH 2 CH 2 CH 2 CH(Br)CH 2 Br 1 2 NaNH 2 NH 3 . [2. H3 + , CH 3 CH 2 CH 2 CH 2 C= CH Alkynes: An Introduction to Organic Synthesis 205 9.29 (a) _ H2 CH0CH0CH9C 2 — CH 3 2 ^ _ CHoCHoCHoCH^ CHo2 3 2 2 Lindlar 1. Q3 t**" + H3 2. Zn, CH0CH0CH0CHO 2 2 3 + catalyst (b) (CH 3 ) 2CHCH 2 _ 1 C= CH C H 2o NaNHo, NH 3 . _ CCH (CH 3 2 CHCH 2 C= ) NH 3 Li in H \ / / 2 CH 3 CH2 CH 3 c=c\ (CH 3 ) 2 CHCH 2 H 9.30 The product contains a cis-disubstituted cyclopropane ring, which can be formed from a CH2I2 with a cis alkene. The alkene with a cis bond can be produced from an alkyne by hydrogenation using a Lindlar catalyst. The needed alkyne can be formed from the starting material shown by an alkylation using bromomethane. Simmons-Smith reaction of _ 1 CH0CH0CH0CH0C — CH 3 2 2 2 . 2. NaNH 2 NH 3 _ CH 3 Br , _ CHqCHpCHpCHpC ^ — CCHq 3 2 2 H2 H H Lindlar catalyst \ / C H /\ rTjP CH3CH 2 CH 2 CH 2 \ %*H CH 2 I 2 CH 3 / H / c=c\ CH 3 CH 2 CH 2 CH 2 Zn (Cu) CH 3 9.31 C= CH Or CH. Br c CH 2 CI 2 '^^X 1 NaNH 2 NH 3 1 2.CH 3 Br 1 . . 2. BH 3 THF H2 2 ~OH , , , ^^.C^ ^T\^C=CCH3 ^CH 3 U in NH 3 aCH 2 CH The trans double bond in the second target molecule is a product of reduction of a triple bond with Li in NH3. The alkyne was formed by an alkylation of a terminal alkyne with bromomethane. The terminal alkyne was synthesized from the starting alkene by bromination, followed by dehydrohalogenation. Width: 612 Height: 792 206 Chapter 9 9.32 (a) Cl^ ch 2 J> 2 2 CH 3 CH 2 C= CH CH 3 CH 2 C(CI 2 )CHCI 2 1,1,2,2-Tetrachlorobutane (b) ci Hz CH 3 CH 2 C=CH CHCI > CH 3 CH 2 CH=CH 2 3, CH 3 CH 2 CH — CH 2 . KOH Lindlar N ,d l,l-Dichloro-2-ethyl- catalyst cyclopropane 9.33 In all of these problems, an acetylide ion (or an anion of a terminal alkyne) haloalkane. is alkylated by (a) HC= CH 1 . n 2. (b) _ HC — CH 1 . 2. NaNH 2 NHo * CHoCHpCHoC=CH * * on 2 3 2 CH 3 CH 2 CH 2 o Br , _ NaNH 2 NH 3 ^ CHqCHoC 6 d — CH CH 3 CH 2 Br , 1 . 2. NaNH 2 NH 3 ^ * CH^CHoC d d CH 3 CH 2 Br , _ — CCHoCHo d 6 (c) 1 HC=CH . n NaNHo, NHo* * » ^, (CH 3 2 CHCH 2 Br , , 2. (CHo) v 2 3 ' 2 CHCHpC=CH , , ) H2 Lindlar catalyst Line ' | (CH 3 ) 2 CHCH 2 CH^— ^ _ CHoCHpCHpC 2 — CH 3 2 from 1 . NaNH 2 NH 3 ~ ~~ 2. , ~~~ ; _ CH^CHpCHpBr * , or f Liir in NH 3 CH 2 _ 223 CHoCHoCHoC^: CCHoCHoCHo 3 2 2 (a) H 2 0, H 2 S0 4 I jHgS0 4 CH 3 CH 2 CH 2 CCH 2 CH 2 CH 2 CH 3 Hydroboration/oxidation can also be used to form the ketone from 4-octyne. ^ _ HC — CH 1 . 2. NaNH 2 NH 3 , CH 3 CHpCH 2 CH 2 Br CH 3 CH 2 CH 2 CH 2 C — CH 1 . 2. BH 3 THF , H2 2 , "OH CH3CH 2 CH 2 CH 2 CH 2 CHO Alkynes: An Introduction to Organic Synthesis 207 9.34 (a) D D2 CH 3 CH 2 C — CCH 2 CH 3 \ / Lindlar catalyst D / C=C\ CH 3 CH 2 CH 2 CH 3 (b) Li in CH 3 CH 2 C — CCH 2 CH 3 / ND 3 /C =cv D CH3CH2 (c) NaNH 5 CH 3 CH 2 CH 2 C — CH (d) . [CHgCHgCHsC^Cl-Na"^ NHo CH ^r\^C= v ^ CH 2 CH 3 —D + 3 CH 3 CH 2 CH 2 C= CD C= CD C=C:-Na' NaNHc 1 DqO" D2 I , Lindlar catalyst CD= CD2 9.35 HC=CH NaNH 2 NH 3 BrCH 2 (CH 2 6 CH 2 Br 2 1 2. , ) A dihalide is used to form the ring. 9.36 Muscalure is a C23 alkene. The only functional group present is the double bond between C9 and C io- Since our synthesis begins with acetylene, we can assume that the double bond can be produced by hydrogenation of a triple bond. 1 . HC=CH 2. NaNH 2 NH 3 , CH 3 (CH 2 ) 6 CH 2 Br CH 3 (CH 2 ) 7 C=CH 1 2. CH 3 (CH 2 6 CH 2 ) CH 3 (CH 2 )yC — C(CH 2 )-| 2 CH 3 Lindlar catalyst . H NaNH 2 NH 3 CH 3 (CH 2 )iiCH 2 Br , ^CH2(CH 2 )iiCH3 H (Z)-9-Tricosene 208 Chapter 9 General Problems 9.37 An acyclic alkane with eight carbons has the formula CsHis. CsHio has eight fewer hydrogens, or four fewer pairs of hydrogens, than CsHis. Thus, CsHio contains four degrees of unsaturation (rings/double bonds/triple bonds). (b) Because only one equivalent of H 2 is absorbed over the Lindlar catalyst, one triple (a) bond 9.38 9.39 is present. (d) Three equivalents of H2 are absorbed when reduction is done over a palladium catalyst; two of them hydrogenate the triple bond already found to be present. Therefore, one double bond must also be present. CgHio must therefore contain one ring. (e) Many (c) structures are possible. An Alkynes: Introduction to Organic Synthesis 209 9.40 (a) H Br Br^ CH2CI2 trans-5-Decem Br 2 NaNH 2 NH 3 , Lindlar cw-5-Decene catalyst (b) H Br Br 2 CH 2 CI; c/s-5-Decene Br 2 NaNH2 NH 3 , fra/w-5-Decene 9.41 OH O CH3CCH3 -— 1.HC=C:~Na+ I CH CHoCC= 3 | CHo CH 3 HoO H 2 S0 4 H 2 C= CC= CH I H2 Lindlar catalyst CH 3 f H 2 C~- CCH—— CH 2 2-Methyl- 1 ,3-butadiene 9.42 CH, OH •— C=CH CH3O Mestranol 210 Chapter 9 9.43 - :o: _ A CH3CH2CH2CH2CH2C'-V » _ :C=CH CH 3 CH2CH2CH2CH 2 CHCH= CH 2 9.44 H-A ^ 1 CH 3 CH 2 CH 2 CH 2 CH 2 CHC=CH CH 3 CH 2 CH 2 CH 2 CH 2 CHC= CH *indlar Cata| y st l-Octen-3-ol The -Q- addition of acetylide occurs by the same route as shown in Problem 9.41. (1) Erythrogenic acid contains six degrees of unsaturation (see Sec. 7.2 for the calculating unsaturation equivalents for compounds containing elements method of other than C and H). (2) One of these double bonds is contained in the carboxylic acid functional group -C0 2 H; thus, five other degrees of unsaturation are present. (3) (4) (5) Because five equivalents of H2 are absorbed on catalytic hydrogenation, erythrogenic acid contains no rings. The presence of both aldehyde and carboxylic acid products of ozonolysis indicates that both double and triple bonds are present in erythrogenic acid. Only two ozonolysis products contain aldehyde functional groups; these fragments must have been double-bonded to each other in erythrogenic acid. H 2 C=CH(CH 2 (6) )4 The other ozonolysis products result from cleavage of triple bonds. However, not enough information is available to tell in which order the fragments were attached. two possible structures are: The A H 2 C=CH(CH2 )4 C=C-C=C(CH 2 7 C02 H B H 2 C=CH(CH 2 4C=C(CH 2 ) 7 C=CC0 2 H ) ) One method of distinguishing between the two possible structures is to treat erythrogenic acid with two equivalents of 2 using Lindlar catalyst. The resulting trialkene can then be ozonized. The fragment that originally contained the carboxylic acid can then be identified. (A is the structure of erythrogenic acid.) H , 9.45 CH2CH2CH2CH3 t O II H2 CH 2 CH 2 CCHg Pd/C I CH 2 C^: CCHg ~ H 2 0, H 2 S0 4 HgS0 4 II 4 IKMn0 + H0 2 CCH 2 / H0 2 CCH 2 CH 2 CCH 2 CH3 H3 CH— CH 2C02 H O + HOCCH3 Alkynes: 9.46 This reaction mechanism is similar to the An Introduction to Organic Synthesis 211 mechanism of halohydrin formation. Step 1: Attack of n electrons on Br2Step 2: Opening of cyclic cation by H2O. Step 3: Deprotonation. Step 4: Tautomerization (for mechanism, see Problem 9.48 and Section 9.4). 9.47 jr bonds n bonds is pictured above. The carbons at the end of the cumulated double bonds are s/^-hybridized and form one n bond to the "interior" carbons. The interior carbons are sp-hybridized; each carbon forms two n bonds - one to an "exterior" carbon and one to the other interior carbon. If you build a model of this cumulene, you can see that the substituents all lie in the same plane. This cumulene can thus exhibit cis-trans This simplest cumulene isomerism, just as simple alkenes can. any compound with an odd number of adjacent double compounds can exhibit cis-trans isomerism. In general, the substituents of bonds lie in a plane; these 212 Chapter 9 Repeating The this process five first line represents the more times replaces all hydrogen atoms with deuterium atoms. mechanism for acid-catalyzed tautomerization of a ketone. Chapter 10 - Organohalides Chapter Outline I. Names and properties of alkyl halides A. Naming alkyl halides. 1 . (Section 10.1). Rules for naming alkyl halides: a. Find the longest chain and name it as the parent. If a double or triple bond is present, the parent chain must contain it i. b Number the carbon atoms of the parent chain, beginning at the end nearer the first substituent, whether alkyl or halo. c Number each substituent. i. If more than one of the same kind of substituent is present, number each, and use the prefixes di-, tri-, tetra- and so on. ii. If different halogens are present, number all and list them in alphabetical . . order. d. If the parent chain can be numbered from either end, start at the end nearer the substituent that has alphabetical priority. 2. Some alkyl halides are named by first citing the name of the alkyl group and then citing the halogen. B . Structure of alkyl halides. 1 Alkyl halides have approximately tetrahedral geometry. . 2 . 3 . Bond lengths increase with increasing size of the halogen bonded to carbon. Bond strengths decrease with increasing size of the halogen bonded to carbon. Carbon-halogen bonds are polar, and many halomethanes have dipole moments. 4. Alkyl halides behave as electrophiles in polar reactions. 5 Preparation of alkyl halides (Sections 10.2-10.5). A. Radical halogenation of alkanes (Section 10.2). 1 The sequence of steps: initiation, propagation, termination. 2 Complications of radical halogenation. . II. . . a. b . The reaction continues on to produce di- and poly substituted products. If more than one type of hydrogen is present, more than one type of monosubstituted product is formed. reactivity order of different types of hydrogen towards chlorination is: primary < secondary < tertiary. i. This reactivity order is due to the bond dissociation energies for formation of the alkyl radicals. ii. The stability order of alkyl radicals: primary < secondary < tertiary. B. Allylic bromination of alkenes (Sections 10.3-10.4). 1 Reaction of an alkene with NBS causes bromination at the position allylic to the double bond (Section 10.3). 2 This reaction occurs by a radical chain mechanism. a. Br- abstracts an allylic hydrogen. b The allylic radical reacts with Br2 to form an allylic bromide, plus Bp. 3 Reaction occurs at the allylic position because an allylic C-H bond is weaker than most other C-H bonds, and an allylic radical is more stable. 4. Reasons for stability of an allylic radical (Section 10.4). c. . . . . The 214 Chapter 10 a. b . The carbon with the unpaired electron is sp -hybridized, and its p orbital can overlap with the p orbitals of the double-bond carbons. The radical intermediate is thus stabilized by resonance. i This stability is due to derealization (spreading out) of the unpaired electron . over an extended n network. Reaction of the ally lie radical with Br2 can occur at either end of the n orbital system. mixture of products may be formed if the alkene is unsymmetrical. i. ii. These products aren't usually formed in equal quantities: reaction to form the more substituted double bond is favored. d Products of allylic brornination can be dehydrohalogenated to form dienes. C. Alkyl halides from alcohols (Section 10.5). 1 Tertiary alkyl chlorides, bromides or iodides can be prepared by the reaction of a tertiary alcohol with HC1, HBr or HI. a. Reaction of secondary or primary alcohols occurs under more drastic conditions, which may destroy other acid-sensitive functional groups. 2 Primary and secondary alkyl chlorides and bromides can be formed by treatment of the corresponding alcohols with SOCI2 or PBr3, respectively. a. Reaction conditions are mild, less acidic, and are less likely to cause acidcatalyzed rearrangements. 3 Alkyl fluorides can be prepared using either (CH3CH2)2NSF3 or HF in pyridine. Reactions of alkyl halides (Sections 10.6-10.7). A. Grignard reagents (Section 10.6). 1 Organohalides react with Mg to produce organomagnesium halides, RMgX. a. These compounds are known as Grignard reagents. 2. Grignard reagents can be formed from alkyl, alkenyl and aryl halides. a. Steric hindrance is no barrier to formation of Grignard reagents. 3 The carbon bonded to Mg is negatively polarized and is nucleophilic. 4 Grignard reagents react with weak acids to form hydrocarbons. B Organometallic coupling reagents (Section 10.7). 1 Alkyl halides can react with Li to form alkyllithiums. 2 These alkyllithiums can combine with Cul to form lithium diorganocopper compounds (RaCuLi), which are known as Gilman reagents. 3 R2CUO compounds can react with alkyl halides (except for fluorides) to form c . A . . . . III. . . . . . . . hydrocarbon products. Organometallic coupling reactions are useful for forming large molecules from small pieces. a. The reaction can be carried out on alkyl, vinyl and aryl halides. b The mechanism is not a typical polar nucleophilic substitution. related reaction is the Suzuki-Miyaura reaction - a palladium-catalyzed coupling 5 of aryl or vinyl organotin reagents with organohalides. IV. Oxidation and reduction in organic chemistry (Section 10.8). A. In organic chemistry, an oxidation is a reaction that results in a loss in electron density by carbon. 1 This loss may be due to two kinds of reactions: a. Bond formation between carbon and a more electronegative atom (usually O, or halogen). b Bond breaking between carbon and a less electronegative atom (usually H). 2 Examples include chlorination of alkanes and reaction of alkenes with Br2. reduction is a reaction that results in a gain of electron density by carbon. B 1 This gain may be due to two kinds of reactions: a. Bond formation between carbon and a less electronegative atom. b. Bond breaking between carbon and a more electronegative atom. 4 . . . A . N . . . A . Organohalides 2. 215 Examples include conversion of a Grignard reagent to an alkane, and reduction of an alkene with H2. C. Alkanes are at the lowest oxidation level, and CO2 is at the highest level. D. A reaction that converts a compound from a lower oxidation level to a higher oxidation level is an oxidation. A reaction that converts a compound from a higher oxidation level to a lower oxidation E. level is an reduction. Solutions to Problems 10.1 The (a) were given for naming alkanes in Section 3.4 are used for alkyl halides. same as an alkyl substituent but is named as a halo group. rules that halogen is treated the (b) CH3CH2CH2CH2I CH, CH3CHCH2CH2CI 1-Iodobutane 1 (c) -Chloro-3-methylbutane (d) f CH3CCH2CH2CI BrCH2CH 2 CH 2 CCH2Br CH 3 1 ( CI ,5-Dibromo-2,2-dimethylpentane e) I 1 ,3-Dichloro-3-methylbutane CO CHoCHoCI II CI I CHgCHCHCHpCHg 1 Br I CH3CHCH2CH2CHCH3 2-Bromo-5-chlorohexane -Chloro-3-ethyl-4-iodopentane 10.2 (a) (b) CI f I CH3CH2CH2C CHCHg CH3CH2CH2C CH 3 2- Chloro-3 ,3-dimethylhexane 9 H3 CHCH3 CI 3,3-Dichloro-2-methylhexane (d) CHgCh^CCh^CHg H3C Br HC I Br H-,C Br 3- Bromo-3-ethylpentane 1 1 , -Dibromo-4-isopropylcyclohexane (e) ® ? CH3CH2CH2CH2CH2CCH2CHCH3 (CH 3) 3C- C H gC H2 C H C H3 4-sec-B uty 1-2-chlorononane 1 , 1 -Dibromo-4-terr-butylcy clohexane A Width: 612 Height: 792 216 Chapter 10 10.3 Site Product of chlorination CHo3 I CH3CH2CH2CHCH2CI 1 -Chloro-2-methylpentane CI I CHgCH^CH^C^Hg^ 2- Chloro-2-methylpentane CI e d c b a CH 3 CH2CH 2 CH(CH3) 2 — q| I CHgCH2CHCH(CHg)2 2-Methylpentane 3- Chloro-2-methylpentane ? CHgCHCH2CH(CH3)2 2-Chloro-4-methylpentane CICH2CH2CH2CH(CH3)2 1 -Chloro-4-methylpentane Chlorination at sites b and e yields achiral products. The products of chlorination at sites c and d are chiral; each product is formed as a racemic mixture of enantiomers. 10.4 Type Number of -H CHg - CH2 _ C CHg d a c I of-H b c d 1 2 3 of each type H b Relative reactivity 1.0 5.0 3.5 1.0 Number times reactivity 6.0 5.0 7.0 3.0 24% 33% 14% 2-Methylbutane 29% Percent chlorination CH 3 CH 3 CHgCh^CCHg CH2CH2CHCH2CI H f 3 CHqCHpCHCHo 2 3 3 29% CI 24% * , hv CH 3 ^3 + CH3CHCHCH3 CI 33% + CICH 2CH 2CHCH 3 14% a, Organohalides 10.5 10.6 Abstraction of hydrogen by a bromine radical yields an Br ally lie radical. The allylic radical reacts with Br 2 to produce A and B. Br2 B Product B is favored because reaction at the primary end of the product with a tri substituted double bond. allylic radical yields a 10.7 5-Methylcycloheptene 3-Bromo-5-methylcycloheptene 3-Bromo-6-methylcycloheptene 217 218 Chapter 10 (b) CH 3 CH 3 CH — CHCH2CH3 I CH — CHCH2CH3 CH3C CH3C Br CH 3 CH 3 t \_ I CH 3 I CHgC -- CH _ CH3CHCH — CHCH2CH3 NBS C\^2p~~ CH -- CHCH2CH3 CHCH2CH3 Br and CH 3 4 hv CH 3 I I CH 3 CHCH= CH— CHCH 3 CH 3 CHCH= CH— CHCH 3 Br CH 3 CHo3 I I CH3CHCH— CH= CHCH 3 CH3CHCH— CH= CHCH 3 Br Two different allylic radicals can 10.8 Remember that halogen form, and four different bromohexenes can be produced. acids are used for converting tertiary alcohols to alkyl halides. PBr3 and SOCI2 are used for converting secondary and primary alcohols to alkyl halides. (CH3CH2)2NSF3 and HF in pyridine can be used to form alkyl fluorides. (a) CH 3 CHo3 I HCI CH3CCH3 CH3CCH3 ether OH (b) OH I CI CH 3 Br CH3CHCH2CHCH3 Gther * CH3CHCH2CHCH3 (c) CH 3 CH, PBrc ether OH HF i pyridine I BrCH2CH2CH2CH2CHCH3 HOCH2CH2CH2CH2CHCH3 (d) CH 3 PBr, I 219 Organohalides 10.9 9. 1 shows that the pKa of CH3-H is 60. Since CH 4 is a very weak acid, ~:CH 3 is a very strong base. Alkyl Grignard reagents are similar in base strength to ~:CH3, but alkynyl Grignard reagents are somewhat weaker bases. Both reactions (a) and (b) occur as Table written. (a) CH 3 MgBr H— C=C — + stronger CH 4 stronger acid base H— C=C — MgBr + weaker weaker acid base (b) CH 3 MgBr base 10.10 CH 4 weaker weaker acid base + R-MgX into R-H, they also R-MgX into R-D. In this case: Just as Grignard reagents react with proton donors to convert react with deuterium donors to convert D MgBr Br Mg I CH 3 CHCH 2 CH 3 10.11 H 2 N— MgBr NH 3 stronger acid + stronger D2 Q I CH 3 CHCH 2 CH 3 ~ I ^ CH 3 CHCH 2 CH 3 (a) The methyl group has an allylic relationship to the double bond. Thus, an organometallic coupling reaction between 3-bromocyclohexene and lithium dimethylcopper gives the desired product. 3-Bromocyclohexene can be formed by allylic bromination of cyclohexene with NBS. 3-Methylcyclohexene (b) We are asked to synthesize an eight-carbon product from a four-carbon starting material. Thus, an organometallic coupling reaction between 1-bromobutane and lithium is formed from 1- dibutylcopper gives octane as the product. The Gilman reagent bromobutane. 2 CH 3 CH 2 CH 2 CH 2 Br 4 Li 2 pentane CH 3 CH 2 CH 2 CH 2 Li Cul - » ether (CH 3 CH 2 CH 2 CH 2 ) 2 Cu + 2 LiBr CH 3 CH2CH2CH 2 Br + (CH CH CH2CH 2 )2Cu"Li 3 2 + + CH 3 (CH 2 6CH 3 ) Lil + LiBr Octane + CH 3 CH 2 CH 2 CH 2 Cu _ , Li 220 Chapter 10 (c) The synthesis in (b) suggests a route to the product. Decane can be synthesized from 1bromopentane and lithium dipentylcopper. 1-Bromopentane is formed by hydroboration of 1-pentene, followed by treatment of the resulting alcohol with PBr3. 1 BH 3 THF 2. H2 CHgCh^Ch^CH— CH2 , CH3CH2CH2CH2CH2OH 2 ,"OH PBr3 ether > I CHgCHgCHoCHgCHgBr 4 2 CHgCh^Ch^Ch^Cr^Br Li 2 CHaCHgCHgCHgCHgLi + 2LiBr pentane CuL ether I (CH 3 CH2CH2CH 2 CH2)2Cu"Li CH 3 CH2CH2CH2CH 2 Br + (C^CHgC^Ch^Cr^CifL^ + + Lil CH 3 (CH 2 8 CH 3 ) ether Decane + LiBr + 10.12 CH 3 CH2CH 2 CH2CH 2 Cu (a) As described in Worked Example 10.2, the oxidation level of a compound can be found by adding the number of C-O, C-N, and C-X bonds and subtracting the number of C-H bonds. Cyclohexane, the first compound shown, has 12 C-H bonds, and has an oxidation level of -12. Cyclohexanone has 2 C-O bonds (from the double bond) and 10 C-H bonds, for an oxidation level of -8. 1-Chlorocyclohexene has one C-Cl bond and 9 C-H bonds, and also has an oxidation level of -8. Benzene has 6 C-H bonds, for an oxidation level of -6. In order of increasing oxidation level: < CH 3 CH 2 NH 2 oxidation level 10.13 < = -4 H2NCH2CH2NH2 = -2 oxidation level < CH 3 CN oxidation level = (a) The aldehyde carbon of the reactant has an oxidation level of 1 (2 C-O bonds minus C-H bond). The alcohol carbon of the product has an oxidation level of -1 (1 C-O bond minus 2 C-H bonds). The reaction is a reduction because the oxidation level of the product 1 lower than the oxidation level of the reactant. The oxidation level of the upper carbon of the double bond in the reactant changes from to +1 in the product; the oxidation level of the lower carbon of the double bond changes from to -1. The total oxidation level, however, is the same for both product and reactant, and the reaction is neither an oxidation nor a reduction. is (b) Organohalides 221 Visualizing Chemistry 10.14 (a) (b) CH 3 I CI CH3C CH, cis- 1 -Chloro-3-methylcyclohexane CI I CHCHCH2CH2CH2 4-Chloro-2-methyl-2-heptene 10.15 (a) CH G CHo3 l_ CH3C — CHCHCH2CH3 CH 3 CH3CCH— CHCH2CH3 CH 3 _ CHCH2CH2CH3 CH3C— ' 2-Methyl-2-hexene J NBS * Br I CH3C — CHCHCH2CH3 CHo CH3CCH — CHCH2CH3 Br + CH 2 qq| CH 2 Br CHgC^- CHCH2CH2CH3 CH3C— CHCH2CH2CH3 + CH, CH3C t CHCH2CH2CH3 H 2C CH3C Br CHCH2CH2CH3 (b) NBS hv CCI4 Br 10.16 The name of the compound is (/?)-2-bromopentane. Reaction of (5')-2-pentanol with PBr3 form (/?)-2-bromopentane occurs with a change in stereochemistry because the configuration at the chirality center changes from S to R. to 222 Chapter 10 Additional Problems Naming Alkyl Halides 10.17 (a) H3C I I (b) CH 3 Br Br I I CH 3 CH — CHCH 2 CHCH 3 CH 3 CHCHCHCH 2 CHCH 3 5-Iodo-2-hexene 3 ,4-Dibromo-2,6-dimethylheptane (c) Br CI I (d) CH 3 I CH 2 Br I CH 3 CCH2 CHCHCH 3 1 CH 3 CH 2 CHCH 2 CH 2 CH 3 CH 3 3-(Bromomethyl)hexane 2-Bromo-4-chloro-2,5-dimethylhexane (e) CICH 2 CH 2 CH 2 C= CCH 2 Br 1 -Bromo-6-chloro-2-hexyne 10.18 (a) (b) CH3 Br CI I I I CH 3 CH 2 CHCHCHCH 3 CH 3 I CH 3 CH 2 CCH 2 CHCH 3 CH 2 CH 3 CI 4-Bromo-4-ethyl-2-methylhexane 2,3-Dichloro-4-methylhexane (d) (c)

Ether (e) MgBr Br I Mg CH0CH2CHCH2 I CH3CH2CHCH3- * Ether (f) H 2Q ^* A 4 Li » 2 CHgCHgCHgCHgBr pentan 2 CH3CH2CH2CH2U CH0CH2CH2CH3 B Cul (CH 3 CH2CH 2 CH2)2CuLi (g) CH 3 CH 2 CH2CH2Br + (CH 3 2 CuLi ) » CH 3 CH2CH 2 CH 2 CH 3 + CH 3 Cu + LiBr Organohalides 10.22 Abstraction of hydrogen by Br» can produce either of two allylic radicals. resulting from abstraction of a secondary hydrogen, CH 3 CHCH= CHCH 3 -* is »- CH 3 CH=CHCHCH 3 * CH3CH2CHCH — CH2 more likely to The 225 first radical, be formed. (identical resonance forms) and CHgCH 2 CH — CHCH2 * Reaction of the radical intermediates with a bromine source leads to a mixture of products: Br I CH3CHCH = CHCH 3 cis- and frans-4-Bromo-2-pentene cis- and fra>w-l-Bromo-2-pentene and CH 3 CH 2 CH= CHCH 2 Br f and CH 3 CH 2 CHCH=CH 2 The major product is 3-Bromo-l-pentene 4-bromo-2-pentene, instead of the desired product, l-bromo-2- pentene. 10.23 Three different allylic radical intermediates can be formed. Bromination of these intermediates can yield as many as five bromoalkenes. This is definitely not a good reaction to use in a synthesis even if the products could be separated. Br (allylic - T (allylic - secondary hydrogen ~ - secondary hydrogen 3-Bromo-2-methylcyclohexene abstracted) . hv abstracted) Br 3-Bromo- 1 -methylcyclohexene (ally he - primary hydrogen abstracted) l-(Bromomethyl)cyclohexene 3-Bromo-3-methylcyclohexene 2-Bromomethylenecyclohexane Width: 612 Height: 792 226 Chapter 10 10.24 NBS I/TV CCI4 hv CCI4J — CH2 CH 3 CH= CHCHCH= CH 2 .t _ — CHCHCH2CH — CH2 CHoCH= CHCH= CHCH 2 Ch^CH^— CHCH2CH H2C NBS CHgCH — CHCH2CH — CH2 l |Br2 . t . LCH 3 CHCH= CHCH= CH 2 BrCH 2 CH= CHCH 2 CH= CH 2 J Br 2 I 6-Bromo- 1 ,4-hexadiene + _ CH 3 CH= CHCH= CHCH 2 Br Br I h^C^- CHCHCH2CH l-Bromo-2,4-hexadiene _ — CH2 5r C h 3ch = CHCHCH 3-Bromo- 1 ,5-hexadiene = CH2 3-Bromo- 1 ,4-hexadiene Br + CH 3 CHCH=CHCH=CH 2 5-Bromo- 1 ,3-hexadiene The intermediate on the right is more stable because the unpaired electron is delocalized over more atoms than in the intermediate on the left, and the resulting products should predominate. 10.25 Two allylic radicals can form: and Organohalides The second radical it is much more likely to form because it is both allylic 227 and benzylic, and yields the following products: Oxidation and Reduction 10.26 Remember that the oxidation level is found by subtracting the number of C-H bonds from the number of C-O, C-N, and C-X bonds. The oxidation levels are shown beneath the structures. In order of increasing oxidation level: CH3CH — CHCh^ CH3CH2CH — CH2 < -8 -8 CH3CH2CH2CH < CH3CH2CH2COH -4 -6 f? CH3CH2CH2NH2, -6 CH 3 CH 2 CH 2 Br < -6 BrCH 2 CH 2 CH 2 CI < CH 3 CCH 2 CI -2 -4 10.27 O O o -6 -6 All of the 10.28 compounds except 3 (which (a) This reaction (b) The reactant (c) reaction is the is more oxidized) have the -6 same oxidation level. neither an oxidation nor a reduction because the oxidation level of the same This reaction is -6 an oxidation. is is -4 as the oxidation level of the product. a reduction. 228 Chapter 10 General Problems 6.3 shows that the bond dissociation energy for C6H5CH2-H is 375 kJ/mol. This value is comparable in size to the bond dissociation energy for a bond between carbon and an allylic hydrogen, and thus it is relatively easy to form the CerlsCr^* radical. The high 10.29 Table bond C$15; 472 kJ/mol, indicates the bromination on The only product of reaction with NBS is CeHsCt^Br. dissociation energy for formation of the benzene ring will not occur. 10.30 p c: c- a10.31 (a) CH 3 CH= CHCH= CHCH= CHCH 2 CHoCH= CHCH— CHCHCH= CH? , t CHoCH= CHCHCH= CHCH= CH ? . t CH 3 CHCH= CHCH= CHCH= CH 2 (c) CH 3 C=N=0 CH 3 C=N— O: CH 3 C=N— O: 10.32 CH2CH2CH 3 Br S 2, -CH 3 CH2CH 3 hv 2CH2CH 3 R ^•CH 3 CH2CH 3 H V'Br CH2CH 3 Abstraction of a hydrogen atom from the chirality center of (5)-3-methylhexane produces an achiral radical intermediate, which reacts with bromine to form a 1 : 1 mixture of R and enantiomeric, chiral bromoalkanes. The product mixture is optically inactive. S Organohalides 229 10.33 H H if CH 3 p ^c~> CH 3 o /J,C % /C-. H V H 3C C H ^c^ H3C CI \f I Cl H 2» /7v CI CI HH H H + CH 3 p s,c^ ^c-. V H H3C CI CI H \f I + I H H other products Abstraction of a hydrogen atom from carbon 4 yields a chiral radical intermediate. Reaction of this intermediate with chlorine does not occur with equal probability from each side, and the two diastereomeric products are not formed in 1 : 1 ratio. The first product is optically active, 10.34 and the second product is a meso compound. All these reactions involve addition of a dialkylcopper reagent [(CH3CH2CH2CH2)2CuLi] same alkyl halide. The dialkylcopper is prepared by treating 1-bromobutane with lithium, followed by addition of Cul: to the 2 CH 3 CH 2 CH 2 CH 2 Br 4 Li * pent 2 CH 3 CH 2 CH 2 CH 2 Li Cul -|—*- (CH 3 CH 2 CH 2 CH 2 ) 2 CuLi 230 Chapter 10 10.35 (a) Fluoroalkanes don't usually form Grignard reagents, (b) Two allylic radicals can be produced. Instead of a single product, as (c) 10.36 many as four bromoalkene products may result. Dialkylcopper reagents don't react with fluoroalkanes. A Grignard reagent can't be prepared from a compound containing an acidic functional group because the Grignard reagent is immediately quenched by the proton source. For example, the -CO2H, -OH, -NH2, and RC=CH functional groups are too acidic to be used for preparation of a Grignard reagent. BrCH2CH2CH2NH2 is another compound that doesn't form a Grignard reagent. 10.37 Reaction of the ether with HBr can occur by either path A or path B. Path because its cation intermediate can be stabilized by resonance. A is favored Organohalides 231 10.38 hv (C 4 H 9 ) 3 SnH (C 4 H 9 ) 3 Sn- + (C 4 H 9 ) 3 SnH + (C 4 H 9 ) 3 Sn- + H- RX (C 4 H9 ) 3 SnX + R" R- (C 4 H 9 ) 3 Sn- + RH 10.39 CH 3 1 . 2. CH 3 BH 3 THF , H2 2 per. OH , (CH 3 ) 2 CuLi + in Chapter 7, tertiary carbocations (RsC ) are more stable than either secondary or primary carbocations, due to the ability of the three alkyl groups to stabilize positive charge. If the substrate is also allylic, as in the case of 2C=CHC(CH 3 ) 2 Br, positive charge can be further delocalized. Thus, 2 C=CHC(CH3)2Br should form a carbocation faster than (CH3)3CBr because the resulting carbocation is more stable. 10.40 As we saw H H 10.41 H3 CH 3 :h 3 f .. H 3 C— C—Br: H O3 C—+ H3 c-f-q^ I CH H 3C + H 3 C— C— OH ^H. CH 3 + H3 :Br: H + H 10.42 •25 R— O:" R— R— :0: alkoxide ion :0 carboxylate ion Carboxyhc acids are more acidic than alcohols because the negative charge of a carboxylate ion is stabilized by resonance. This resonance stabilization favors formation of carboxylate anions over alkoxide anions, and increases Ka for carboxylic acids. Alkoxide anions are not resonance stabilized. 232 Chapter 10 10.43 The Suzuki-Miyaura reaction is a Pd-catalyzed coupling reaction between an aromatic boronic acid and an aromatic (or vinyl) halide. Two possible coupling reactions can yield the target product. Other halogens can also be used. Chapter 11 - Reactions of Alkyl Halides: Nucleophilic Substitutions and Eliminations Chapter Outline I. Substitution Reactions (Sections 11 . 1-1 1 .6). A. Sn2 reactions (Sections 11.1-11.3). The discovery of S N 2 reactions (Section 1 . a. b . c. Walden discovered 11.1). and (-) malic acid could be interconverted (Section 11.1). This discovery meant that one or more reactions must have occurred with inversion of configuration at the chirality center. Nucleophilic substitution of tosylate ion by acetate ion occurs with inversion of configuration. that (+) malic acid Nucleophilic substitution reactions of primary and secondary alkyl halides always proceed with inversion of configuration. S N 2 reaction (Section 11.2). i. 2. The a. Kinetics. i The kinetics of a reaction measure the relationship between reactant . ii. iii. b . concentrations and product concentrations and the rate of reaction. In an Sn2 reaction, reaction rate depends on the concentration of both alkyl halide and nucleophile (bimolecular reaction). (a). This type of reaction is a second-order reaction. In a second-order reaction, rate = k x [RX] x [Nu]. (a). The constant, k, is the rate constant. Mechanism. i. ii. The reaction takes place in a single step, without intermediates. The nucleophile attacks the substrate from a direction directly opposite to the leaving group. (a). This type of attack accounts for inversion of configuration. iii. In the transition state, the new bond forms at the same time as the old bond breaks. iv. Negative charge is shared between the attacking nucleophile and the leaving group. v. In the transition state, the three remaining bonds to carbon are in a planar arrangement. vi. Both substrate and nucleophile are involved in the step whose rate is measured. 3 . Characteristics of the a. Changes Sn2 reaction (Section 11.3). in the energy levels of reactants or of the transition state affect the reaction rate. b . Changes in the substrate. i. Reaction rate ii. Substrates, in order of increasing reactivity: tertiary, neopentyl, secondary, is decreased if the substrate is bulky. primary, methyl. iii. iv. reactions can occur only at relatively unhindered sites. Vinylic and aryl halides are unreactive to Sn2 substitutions. Sn2 234 Chapter 11 c . Changes in the nucleophile. i. Any ii. The iii. In general, nucleophilicity parallels basicity. species can act as a nucleophile if it has an unshared electron pair, (a). If the nucleophile has a negative charge, the product is neutral, (a). If the is neutral, the product is positively charged. dependent on reaction conditions. down a column of the periodic table. Negatively charged nucleophiles are usually more reactive than neutral nucleophiles. Changes in the leaving group. i. In general, the best leaving groups are those that best stabilize negative charge. Usually, the best leaving groups are the weakest bases. ii. iii. Good leaving groups lower the energy of the transition state. iv. Poor leaving groups include F~, HCT. RCT, and rfcN (a). Poor leaving groups can be converted to better leaving groups. iv. v d. nucleophile reactivity of a nucleophile is Nucleophilicity increases going . . e. Changes i. ii. Sn2 reactions by lowering the reactivity of the nucleophile. Polar, aprotic solvents raise the ground-state energy of the nucleophile and make f in the solvent. Polar, protic solvents slow it more reactive. A summary: . i. Steric hindrance in the substrate raises the energy of the transition state, ii. increasing AG*, and decreasing the reaction rate. More reactive nucleophiles have a higher ground-state energy, decreasing iii. iv. AG*, and increasing the reaction rate. Good leaving groups decrease the energy of the transition state, decreasing AG*, and increasing the reaction rate. Polar protic solvents solvate the nucleophile, lowering the ground-state energy, increasing AG ? and decreasing the reaction rate. Polar aprotic , solvents don't solvate the nucleophile, raising the ground-state energy, decreasing AG*, and increasing the reaction rate. B. S N 1 Reactions (Sections 11.4-11.5). 1 The S N 1 reaction (Section 11.4). a. Under certain reaction conditions, tertiary halides are much more reactive than primary and methyl halides. These reactions must be occurring by a mechanism other than Sn2. i. b Kinetics of the Sn 1 reaction. i. The rate of reaction of a tertiary alkyl halide with water depends only on the . . concentration of the alkyl halide (unimolecular reaction). reaction is a first order process, with reaction rate = k x [RX]. iii. rate expression shows that only is involved in the slowest, or ratelimiting, step, and the nucleophile is involved in a different, faster step. iv. The rate expression also shows that there must be at least two steps in the ii. The The RX reaction. In an SnI reaction, slow dissociation of the substrate is followed by rapid reaction with the nucleophile. Stereochemistry of SnI reactions. i. An SnI reaction of an enantiomer produces racemic product because an SnI reaction proceeds through a planar, achiral intermediate. v. c. Reactions of Alkyl Halides: Nucleophilic Substitutions and Eliminations ii. Few S N 1 iii. The ion pair formed by 235 reactions proceed with complete racemization. the leaving group and the carbocation sometimes from attack before the leaving group can shields one side of the carbocation diffuse away. 2. Characteristics of the a. b. As SnI reaction (Section 1 1.5). Sn2 reactions, factors that lower AG* favor Changes in the substrate. i. The more stable the carbocation intermediate, ii. in faster reactions. the faster the SnI reaction. Substrates, in order of increasing reactivity: methyl, primary, secondary and benzyl, tertiary. iii. Allylic and benzylic substrates are also reactive Changes in the leaving group. and allyl c. i. e. Sn2 reactions. best leaving groups are the conjugate bases of strong acids. SnI reactions, water can act as a leaving group. Changes in the nucleophile have no effect on SnI reactions. Changes in the solvent. ii. d. The in i. ii. In Polar solvents (high dielectric constant) increase the rates of SnI reactions. Polar solvents stabilize the carbocation intermediate more than the reactants and lower AG*. iii. f Polar solvents stabilize by orienting themselves around the carbocation, with electron-rich ends facing the positive charge. A summary: . i . ii. iii. iv. The best substrates are those that form stable carbocations. Good leaving groups lower the energy of the transition state leading to carbocation formation and increase the reaction rate. The nucleophile doesn't affect the reaction rate, but it must be nonbasic. Polar solvents stabilize the carbocation intermediate and increase the reaction rate. C. Biological substitution reactions (Section 11.6). Both SnI and Sn2 reactions occur often in biochemical pathways. 1 2. In SnI reactions, the leaving group is often an organodiphosphate. 3 Sn2 reactions are involved in biological methylations. Elimination reactions (Sections 1 1.7-1 1. 1 1). A. Introduction (Section 11.7). 1 In addition to bringing about substitution, a basic nucleophile can also cause elimination of from an alkyl halide to form a carbon-carbon double bond. 2 mixture of double-bond products is usually formed, but the product with the more substituted double bond is the major product. a. This observation is the basis of Zaitsev's rule. 3 Double-bond formation can occur by several mechanistic routes, but at this point, we will study only three mechanisms. B The E2 reaction (Sections 1 1 .8-1 1 .9). 1 General features (Section 1 1.8). a. An E2 reaction occurs when an alkyl halide is treated with strong base. b. The reaction occurs in one step, without intermediates. c E2 reactions follow second-order kinetics. d. E2 reactions show the deuterium isotope effect i. In a reaction in which a C-H bond is cleaved in the rate-limiting step, substitution of -D for -H results in a decrease in rate. Because this effect is observed in E2 reactions, these reactions must involve ii. C-H bond breaking in the rate-limiting step. . . II. . HX . A . . . . Width: 612 Height: 792 236 Chapter 11 e. E2 reactions always occur with periplanar geometry. is required because of the need for overlap of the sp* Periplanar geometry i. become n orbitals in the product. Anti periplanar geometry is preferred because it allows the substituents of the two carbons to assume a staggered relationship. Syn periplanar geometry occurs only when anti periplanar geometry isn't orbitals of the reactant as they ii. iii. possible. The preference for anti periplanar geometry results in the formation of double bonds with specific E,Z configurations. 2. Elimination reactions and cyclohexane conformations (Section 1 1.9). a. The chemistry of substituted cyclohexanes is controlled by their conformations. b The preference for anti periplanar geometry for E2 reactions can be met only if the atoms to be eliminated have a trans-diaxial relationship. c. Neomenthyl chloride reacts 200x faster than menthyl chloride because the groups to be eliminated are trans diaxial in the most favorable conformation, and the Zaitsev product is formed. d. For menthyl chloride, reaction must proceed through a higher energy conformation, and non-Zaitsev product is formed. C. The El and ElcB reactions (Sections 11.10-11.11). An El reaction occurs when the intermediate carbocation of an SnI loses H"*" to 1 form a C=C bond. 2. El reactions usually occur in competition with SnI reactions. f. . . 3. El reactions show first-order kinetics. for the groups to be eliminated, and the most formed. D. The ElcB reaction (cB = conjugate base). 1 The ElcB reaction takes place through a carbanion intermediate. 2 The rate-limiting step involves base-induced abstraction of a proton. 3 Often the leaving group is poor. 4. A carbonyl group stabilizes the anion. 5 The ElcB is fairly common in biochemical pathways (Section 11.11). Summary of reactivity (Section 11.12). A. Primary halides. 1 Sn2 reaction is usually observed. 2. El reaction occurs if a strong, bulky base is used. ElcB reaction occurs if the leaving group is two carbons away from a carbonyl 3 group. 4 . There is no geometric requirement stable (Zaitsev) product is . . . . III. . . B . Secondary halides. 1 Sn2 and E2 reactions occur in competition. 2. Strong bases promote E2 elimination. 3 Secondary halides (especially allylic and benzylic halides) can react by SnI and El routes if weakly basic nucleophiles and protic solvents are used. 4. ElcB reaction occurs if the leaving group is two carbons away from a carbonyl . . group. C. Tertiary 1 . 2. 3 . halides. Under basic conditions, E2 elimination is favored. SnI and El products are formed under nonbasic conditions. ElcB reaction occurs if the leaving group is two carbons away from group. a carbonyl Reactions of Alkyl Halides: Nucleophilic Substitutions and Eliminations 237 Solutions to Problems 11.1 As described in Worked Example 11.1, identify the leaving group and the chirality center. Draw the product carbon skeleton, inverting the configuration at the chirality center, and replace the leaving group (bromide) with the nucleophilic reactant (acetate). O H II OCCH3 CH 3 CO" 11.2 Use the suggestions in the previous problem to draw the correct product. CH 3 HoC is HO— cZmun HO: + :Br: V CH 2 CH 3 11.3 H CH 3 Br H CH 3 H SH H V HS" HoC 3 C s ^ HoC GHt Br" CHo3 i\ HH 11.4 V OR H H All of the nucleophiles in this problem are relatively reactive.See Table 11.1. (a) CH 3 CH2CH 2 CH2Br + Nal CH 3 CH 2 CH 2 CH 2 I CH 3 CH 2 CH2CH 2 Br + KOH CH 3 CH 2 CH 2 CH 2 OH CH 3 CH 2 CH 2 CH 2 Br + HC=C"Li H — CH 3 CH 2 CH 2 CH 2 Br + NH 3 CH 3 CH 2 CH 2 CH 2 NH 3 + NaBr + (b) (c) (d) 11.5 + KBr CH 3 CH 2 CH 2 CH 2 C= CH (a) (CHa^N^ is more (b) (CH3)3N is more nucleophilic than (CHaJaB. (CH3)3B has no lone electron pair. (c) H2S is more nucleophilic than column of the periodic table. + LiBr Br nucleophilic because it is more basic than (CH3)2NH and because a negatively charged nucleophile is more nucleophilic than a neutral nucleophile. H2O because is non-nucleophilic because nucleophilicity increases in going down it a 238 Chapter 11 11.6 In this problem, we are comparing two effects - the effect of the substrate and the effect of the leaving group. Tertiary substrates are less reactive than secondary substrates, less reactive than primary substrates. Most reactive Least reactive (CH 3) 3 CCI which are < (CH 3 2 CHCI < CH 3 Br < CH 3 OTos ) secondary carbon tertiary carbon good excellent leaving leaving group group 11.7 3 ~7// \ \\ AGpa AG n* Reaction progress Polar protic solvents (curve 1) stabilize the charged transition state by solvation and also stabilize the nucleophile by hydrogen bonding. Polar aprotic solvents (curve 2) stabilize the charged transition state by solvation, but do not hydrogen-bond to the nucleophile. Since the energy level of the nucleophile is higher, AG* is smaller and the reaction is faster in polar aprotic solvents than in polar protic solvents. Nonpolar solvents (curve 3) stabilize neither the nucleophile nor the transition state. AG ? is therefore higher in nonpolar solvents than in polar solvents, and the reaction rate is slower. Benzene, ether, and chloroform are in this category. 11.8 CH3 (CH 2 ) 3 CH29 H 3 O S/C— OCCHo3 CH2CH 3 ,-. CH 3 (CH 2 3 CH29 H 3 c CH 3 (CH 2 3 CH2 rM ) ) — C CI CH2CH 3 + Cl~ + CH 2 CH 3 o II CH 3 CO" H 3p, CH 2( CH 2)3 CH 3 is CH 3 CO-C R \ CH2CH 3 In this SnI reaction, attack by acetate can occur on either side of the planar, achiral carbocation intermediate, resulting in a mixture of both the R and S enantiomeric acetates. The ratio of enantiomers is probably close to 50:50. Reactions of Alkyl Halides: Nucleophilic Substitutions and Eliminations 11.9 had proceeded with complete inversion, the product would have had a specific If reaction rotation of +53.6°. If complete racemization The observed was 239 rotation 11.10 The S substrate is 1 % reacts with water to would have been zero. form a mixture of R and S alcohols. The ratio of close to 50:50. HO A CHo3 Br > 90. [cx]d = 0.099, 9.9% of the original tosylate C +53.6 of the product must have been racemized. +5.3°. Since The remaining inverted. enantiomers was had occurred, +5.3° i CHoCH 3 S |_| S 2o CH 3 OH HoC / CH0CH3 R CH0CH3 11.11 SnI reactivity is related to carbocation stability. Thus, substrates that form the most stable carbocations are the most reactive in SnI reactions. Most reactive Least reactive « CH CH H 2 C=CHBr 3 f 2 Br secondary primary vinylic f < CH3CHCH3 < H 2 C=CHCHCH 3 allylic 11.12 Br I _ -Br" CHgCHCH — CH 2 ^ CH3CHCH — CH 2 * * CHgCH — CHCH2 The two bromobutenes form 11.13 Both substrates reaction have mechanism the same allylic Br" CH 3 CH= CHCH 2 Br carbocation in the rate-limiting step. groups and might react either by an S N 1 or an S N 2 route. The determined by the leaving group, the solvent, or the nucleophile. allylic is This reaction probably occurs by an SnI mechanism. HC1 converts the poor -OH + leaving group into an excellent -OH2 leaving group, and the polar solvent stabilizes the carbocation intermediate. (a) a negatively charged nucleophile in a polar, aprotic very likely that the reaction occurs by an Sn2 mechanism. (b) This reaction takes place with solvent. It is 240 Chapter 11 11.14 Redraw linalyl diphosphate so that has the same orientation as limonene. OPP After dissociation of PPj, the cation cyclizes by attack of the double bond Removal of an -H by base yields limonene. 11.15 Form the double bond by removing possible. n electrons. HX from the alkyl halide reactant in as many ways as The major elimination product in each case has the most substituted double bond (Zaitsev's rule). (a) Br CH3 ' ' CH3CH2CHCHCH3 CH3 BasG C-H3 ' ^ ' Ch^CH^CH^ CCH3 major (trisubstituted double bond) + CH^CH^ CHCHCH3 minor (disubstituted double bond) Reactions of Alkyl Halides: Nucleophilic Substitutions and Eliminations (b) CH 3 I CH 3 CI r I CH 3 CHCH2 C— CHCH 3 CH 3 CH 3 r ease *- CH 3 I CHo3 I CH 3 CHCH 2 C= CCH 3 241 I CH 3 CHCH= CCHCH 3 + CH 3 major (tetrasubstituted double minor bond) (trisubstituted double bond) CH 3 + ^^3 CH 3 CHCH2 CCHCH 3 CH 2 minor (disubstituted double bond) (c) Br CHCH 3 BaSG > /=CHCH 3 ( + { 11.16 For maximum — CH=CH double bond) (monosubstituted double bond) yield, the alkyl halide reactant should not give a mixture of products on elimination. CH 3 CH 3 CH3 CHoCHCHoCHoCHCHoCHoBr 2 2 3 2 2 The 2-bromo isomer yields ( b) CHo3 Br CH 3 ^ CH 3 CH 2 OH CH 3 CHoCHCHoCHoCHCHoCH^- CHo2 3 2 a mixture of alkene products. _ KOH CH 3 CH 2 OH 2 minor major (trisubstituted ) ^CH< ' \^ N CHr 2 2 242 Chapter 11 11.17 Draw the reactant with correct stereochemistry. H Ph v R >Br ! C (IR,2R)- 1 ,2-Dibromo- 1 ,2-diphenylethane R I H this drawing into a Newman projection, and draw the conformation having anti periplanar geometry (staggered) for -H and -Br. Convert Ph BrP h Ph H Br Ph I H H The alkene resulting from E2 elimination is (Z)-l-bromo-l,2-diphenylethylene. On 11.18 As in the previous problem, draw the structure, convert it to a Newman projection, and -H and -Br to be eliminated have an anti periplanar rotate the groups so that the (staggered) relationship. Br H Br V Br HH V CH2CH3 HoC"!"^ H H3CH The major product CH3CH2 sy CH 3 H,C'l^ H CH3 is (Z)-3-methyl-2-pentene. A small amount of 3-methyl-l-pentene is also formed. CH2CH3 H3C CH3CH2 -y^V CH3 — c=C (Z)-3-Methyl-2-pentene Reactions of Alkyl Halides: Nucleophilic Substitutions and Eliminations 243 11.19 Br Br (CH 3 3 C (CH 3 ) 3 C ) trans The more two isomers are pictured above; the larger always equatorial in the more stable conformation. The cis isomer reacts conditions because -Br and -H are in the anti periplanar arrangement that stable conformations of each of the te/t-butyl group faster under E2 favors CIS is E2 elimination. 11.20 (a) Ch^Ch^Ch^Ch^Br + CH 3 CH2CH2CH2N 3 NaN, primary substitution product The is reaction occurs by an Sn2 mechanism because the substrate nonbasic, and the product is a substitution product. (b) is primary, the nucleophile CI I CH 3 CH 2 CHCH 2 CH 3 secondary + KOH - strong base CH 3 CH2CH — CHCH 3 elimination product is an E2 reaction since a secondary halide reacts with a strong base to yield an elimination product. This (c) ff OCCH 3 CI + CH 3 C0 2 H CHo CH, substitution product tertiary This is an S N 1 reaction. Tertiary substrates form substitution products only by the SnI route. NaOH ethanol This is an group. ElcB reaction because the leaving group is two carbons away from a carbonyl 244 Chapter 11 Visualizing Chemistry 11.21 (a) (i) (ii) CH 3 CH 2 CI Na+ "SCH 3 + CH 3 CH 2CI + Both reactions yield Sn2 CH 3 CH 2 OH "OH Na + CH 3 CH 2 SCH 3 + + NaCI NaCI substitution products because the substrate is primary and both nucleophiles are strong. (b) ( " n CH 2 CH 3 H 3C CI / I CH 3 CCH 2 CH 3 + SCH3 Na / C=CHCH 3 H3C CH 3 + H 2C=C + (ii) CH 3 minor major HSCH 3 + NaCI HoC CI CH 3 CCH 2 CH 3 + Na + / C=CHCH3 "OH H3C CH 3 + H 2 C=C minor major + H2 + CH 2 CH 3 CH 3 NaCI substrate is tertiary, and the nucleophiles are basic. Two elimination products are expected; the major product has the more substituted double bond, in accordance with The Zaitsev's rule, (c) (i) CI + Na+ + Na SCH, SCH3 + NaCI (ii) CI + OH OH + HoO NaCI In (i), the secondary substrate reacts with the good, but weakly basic, nucleophile to yield substitution product. In (ii), is a poorer nucleophile but a stronger base, and both NaOH substitution and elimination product are formed. 11.22 O II CH 3 CO" Br CH< CH 3 Reaction of the secondary bromide with the weakly basic acetate nucleophile occurs by an Sn2 route, with inversion of configuration, to produce the R acetate. Reactions of Alkyl Halides: Nucleophilic Substitutions and Eliminations 245 11.23 HH 1/ HoC Q C ' c\ if CH 2 OH + NaCN HoC „ C NaCI NCH HCI The S CH 2 OH substrate has a secondary allylic chloride group and a primary hydroxyl group. reaction occurs at the secondary carbon to give the poor leaving group. 11.24 HoO, CH 3 CH 2 NaOH NaCI . left side of the molecule so that the groups to be ehrninated have an anti periplanar relationship. The double bond in the product has the E configuration. Rotate the Additional Problems Nucleophilic Substitution Reactions 11.25 Most reactive Least reactive CH 3 CHgCh^Ch^Ch^Br > CHgCHCh^Br Br > CHgCh^CHCHg Br > CHgCCHg CH 3 1-Bromobutane l-Bromo-2methylpropane Sn2 R cyano product because hydroxide is a 2-Bromobutane 2-Bromo-2methylpropane Width: 612 Height: 792 246 Chapter 11 11.26 An converted to an ether by two different routes in this series of reactions. The have identical structural formulas but differ in the sign of specific rotation. Therefore, at some step or steps in these reaction sequences, inversion of configuration at the chiral carbon must have occurred. Let's study each step of the series to alcohol is two resulting ethers find where inversion is occurring. © OH J/ 1 CH3CHCH2 [<% = OTos TosCI ch 3 chch 2 [a] D +33.0 ©J 0"K + — = +31.1° © K > , (f \=Sy CH 3 CH 2OH, heat I OCH 2CH 3 I CH 3 CHCH 2 - CH3CHCH2 [a] D = -19.9 C ©J CH CH2Br 3 OCH 2CH 3 CH3CHCH2 [a] D = +23.5 In step 1, the alcohol reacts with potassium metal to produce a potassium alkoxide. Since the bond between carbon and oxygen has not been broken, no inversion occurs in this step. The potassium alkoxide acts as a nucleophile in the Sn2 displacement on CH^Cr^Br in It is the C-Br bond of bromoethane, however, not the C-0 bond of the alkoxide, step 2. No inversion carbon chirality center occurs in step 2. because the O-H bond, rather than the C-0 bond, of the alcohol is broken, no inversion occurs at this step. Inversion must therefore occur at step 4 when the "OTos group is displaced by CH3CH2OH. The C-0 bond of the tosylate (-OTos) is broken, and a new C-O bond is formed. Notice the specific rotations of the two enantiomeric products. The product of steps 1 and 2 should be enantiomerically pure because neither reaction has affected the C-0 bond. Reaction 4 proceeds with some racemization at the chirality center to give a smaller absolute value of [a]othat is broken. The 11.27 (a) (b) at the starting alcohol reacts with tosyl chloride in step 3. Again, CH3I CH^Br because - V is a better leaving group than Br~. dimethylsulfoxide (DMSO) than in ethanol. Ethanol, a protic solvent, hydrogen-bonds with hydroxide ion and decreases its reacts faster than CH3CH2I reacts faster with OH in reactivity. (c) Under the Sn2 conditions of this reaction, CH3CI reacts faster than (Cr^CCl. Approach of the nucleophile to the bulky (CH3)3CC1 molecule is hindered. (d) H2C=CHCH2Br reacts faster because vinylic halides such as unreactive to substitution reactions. H2C=CHBr are Reactions of Alkyl Halides: Nucleophilic Substitutions and Eliminations 247 11.28 To predict nucleophilicity remember these guidelines: , (1) In comparing nucleophiles basicity. In other words, a that (2) Nucleophilicity increases in (3) have the same attacking atom, nucleophilicity more basic nucleophile going down a is a more parallels effective nucleophile. column of the periodic table. A negatively charged nucleophile is usually more reactive than a neutral nucleophile. More Nucleophilic Reason Less Nucleophilic NH 3 Rule 1 or 3 H2 Rule 1 or 3 r BF 3 BF3 (d) (CH 3 ) 3 P (CH 3) 3 N Rule 2 (e) r cr Rule 2 (f) "C=N ~och 3 Reactivity chart, (a) "NH 2 (b) CH 3 C02 (c) " is not a nucleophile Section 11.3 11.29 CH 3 CHo3 I CH 3 CH 2 CHCH 2I + "CN CH 3 CH 2 CHCH 2 CN + F primary halide This is an Sn2 reaction, whose rate depends on the concentration of both alkyl halide and nucleophile. Rate = k x [RX] x [Nu:~] (a) Halving the concentration of cyanide ion and doubling the concentration of alkyl halide doesn't change the reaction rate. The two effects cancel. (b) Tripling the concentrations of both cyanide ion increase in reaction rate. and alkyl halide causes a ninefold 248 Chapter 11 11.30 CH 3 ^p^3 CH 3 CH 2 CCH 3 + CH 3 CH 2 OH CH 3 CH 2 CCH 3 HI + OCH 2 CH 3 I tertiary halide This is an SnI reaction, whose rate depends only on the concentration of 2-iodo-2methylbutane. Rate = k x [RX]. (a) Tripling the concentration of alkyl halide triples the rate of reaction. (b) Halving the concentration of ethanol by dilution with diethyl ether reduces the polarity of the solvent and decreases the rate. 11.31 (a) CH 3 Br + Na + ~C= CCH(CH 3 2 CH 3 C= C~ Na+ Not + elimination, producing (b) H 3 C— Br + BrCH(CH 3 ) 2 CH 3 C=CCH(CH 3 *~ ) . H 3 C— »- ) NaBr The strong base CH 3 C= C" brings about CH 3C=CH and H 2 C= CHCH 3 ~0— C(CH 3 3 2 + ) . O— C(CH 3 3 + ) NaBr or (CH 3 3 ) C— CI + (CH 3 ) 3 C— CH 3 OH O— CH3 + HCI + (CH ) 3 2 C=CH2 (minor) (major) (c) CH 3 CH 2 CH 2 CH 2 Br + NaCN ^ CH 3 CH 2 CH 2 CH 2 CN + NaBr (d) CH 3 CH 2 CH 2 Br + excess NH 3 CH 3 CH 2 CH 2 NH 2 + NH 4 + Br major 11.32 (a) difference in this pair of reactions is in the leaving group. Since ~OTos is a better leaving group than ~C1 (see Section 1 1.5), Sn2 displacement by iodide on CH3-OT0S The proceeds faster. (b) The (c) Ethoxide ion and cyanide ion are different nucleophiles. Since CN~ is more reactive than CH3CH20 in Sn2 reactions, Sn2 displacement on 2-bromopropane by CN~ proceeds at a faster rate. (d) The solvent in each substrates in these two reactions are different. Bromoethane is a primary bromoalkane, and bromocyclohexane is a secondary bromoalkane. Since Sn2 reactions proceed faster at primary than at secondary carbon atoms, Sn2 displacement on bromoethane is a faster reaction. reaction is different. The Sn2 reaction on bromoethane aprotic acetonitrile proceeds faster than the reaction in nonpolar benzene. in polar, Reactions of Alkyl Halides: Nucleophilic Substitutions and Eliminations 11.33 (/?)-2-Bromooctane is Sn2 reactions proceed 249 a secondary bromoalkane, which undergoes Sn2 substitution. Since with inversion of configuration, the configuration at the carbon chirality center is inverted. (This does not necessarily mean that all R isomers become S isomers after an Sn2 reaction. The R,S designation refers to the priorities of groups, which may change when the nucleophile is varied.) h H 1 '>CH Nu — C H 3 C^» Nu: C— Br + 3 + Br * / C 6H13 C6 H 13 (R)-2-Bromooctane Product Nucleophile (a) NC— CN S (b) '>CH 3 X C 6 H 13 H CH 3 C02 CH 3 C0 2 — S v C 6 H 13 (C) CH 3 S CH 3 S C C 6 H 13 11.34 After 50% 50% R starting material is of the starting material has reacted, the reaction mixture consists of bromooctane and 50% (.S)-2-bromooctane. At this point, the completely racemized. H H3C K \ R Br + C— Br C 6 H 13 < ^ H S '>CH 3 Br— + C 6Hi3 Br" (R)-2- 250 Chapter 11 Elimination Reactions 11.35 (a) CH 3 CH 2 Br CHoCHoCHCHo 6 6 * — "OEt / H H 3C C=C\ H + / H H C=C\ + / CH 3 H CH 3 H3C C=C\ H H (b) H 2 C=CHBr, undergo nucleophilic like other vinylic organohalides, does not substitutions. CO H r^^J H C HgCHg flip I H This alkyl halide gives the less substituted cycloalkene (non-Zaitsev product). Elimination to form the Zaitsev product does not occur because the -CI and -H involved cannot assume the anti periplanar geometry preferred for E2 elimination. (d) (CH 3 ) 3 C— 0° OH *- HCI + (CH 3 3 ) C— CI H2 + 11.36 Because 1-bromopropane is a primary haloalkane, the reaction proceeds by either a Sn2 or E2 mechanism, depending on the basicity and the amount of steric hindrance in the nucleophile. (a) CH 3 CH 2 CH 2 Br NaNH 2 + »- Propene (elimination product) also is CH 3 CH 2 CH 2 NH 2 formed because NaBr + NaNrb is a strong base. (b) CH 3 CH 2 CH 2 Br + K + OC(CH 3 1 ~OC(CH 3 3 is a some substitution. strong, CH 3 CH 2 CH 2 Br + Nal CH 3 CH 2 CH 2 Br + NaCN CH 3 CH 2 CH 2 Br + Na FC " ) ) 3 *- CH 3 CH=CH 2 + CH 3 CH 2 CH 2 OC(CH 3 + HOC(CH 3 ) 3 ) 3 bulky base that brings about elimination as well as (c) »- CH 3 CH 2 CH 2 I + NaBr (d) ** CH 3 CH 2 CH 2 CN + NaBr (e) ® CH 3 CH 2 CH 2 Br + Mq + C=CH CH 3 CH 2 CH 2 MgBr CH 3 CH 2 CH 2 C= CH H2 ^— + CH 3 CH 2 CH 3 NaBr KBr Reactions of Alkyl Halides: Nucleophilic Substitutions and Eliminations 251 11.37 CI Ph Ph H A Both Newman projections -H place and -CI in the correct anti periplanar geometry for E2 elimination. 6- 6- Cl Cl Ph—^V- H Ph Ph H Ph H § H trans- 1 ,2-Diphenylethylene OCH 3 OCH 3 5- 6- Either transition state A* or B* can form when l-chloro-l,2-diphenylethane undergoes E2 elimination. Crowding of the two large phenyl groups in A* makes this transition state (and the product resulting from it) of higher energy than transition state B*. Formation of the product from B* is therefore favored, and ?ra«5-l,2-diphenylethylene is the major product. 11.38 H3C CH3 —HOAc — CH 3 CHCBr HoC d \ *- heat I CHo6 t c=c \ / CH2CH3 H3C CH2CH3 The alkene shown above has Zaitsev's rule, is the CH 3 the most highly substituted double bond, and, according major product. The following minor products may also form. CH 3 CH 3 I CH 3 CH H / CHr ch 3 ch CH 3 CtH H f C=Cx / H3C CH3 CH3CH2 H to 252 Chapter 11 11.39 Diastereomer 8 reacts much more slowly than other isomers in an E2 reaction because no pair of hydrogen and chlorine atoms can adopt the anti periplanar orientation preferred for E2 elimination. General Problems 11.40 (a) Substitution is does not take place with secondary alkyl halides when a strong, bulky base H2C=CHCH2CH 3 and used. Elimination occurs instead and produces CH 3 CH=CHCH 3 . undergo S N 2 reactions because is a poor leaving group. primary and secondary alcohols to chlorides by an Sn2 mechanism. 1 -Methyl- 1-cyclohexanol is a tertiary alcohol and does not undergo Sn2 substitution. Instead, E2 ehmination occurs to give 1-methylcyclohexene. (b) Fluoroalkanes don't (c) SOCI2 11.41 SnI in pyridine converts reactivity: Most reactive Least reactive ( a) NH 2 CH3CH2CHCH3 CH 3 < HoC— C— CI 3 I CHo most stable carbocation (b) (CH 3 ) 3 COH (CH 3 ) 3 CCI < (CH 3) 3 CBr best leaving group Reactions of Alkyl Halides: Nucleophilic Substitutions and Eliminations 253 Most reactive Least reactive (c) aCH f aCHCHa 2 Br CBr < < most stable carbocation 11.42 S N 2 reactivity: Most reactive Least reactive (a) CH,£ CI I I HoC— C— CI 3 < C HgCH 2 C H C Hg CH3CH2CH2CI I primary substrate CH 3 (b) CHo3 fH 3 CH 3 CHCH 2 Br I H3 C— C— CH 2 Br < CHgCHCHCHg CHo least sterically Br hindered substrate (c) CH3CH2CH2OCH3 CH 3 CH 2 CH 2 OTos CH 3 CH2CH2Br best leaving group 11.43 According Cahn-lngold-Prelog rules (Section 5.5), the nucleophile (~CN) has a lower group (~OTos). Thus, even though the reaction proceeds with inversion of configuration, the priorities of the substituents also change, and the configuration remains S. to priority than the leaving 4 ^ hhC 2 1 H OTos ''c; CH 2 OCH 3 2 NaCN NC H3 C 3 CH20CH 3 1 254 Chapter 11 11.44 O: OCH, CH 3 I This is + r an excellent method of ether preparation because iodomethane is very reactive in Sn2 displacements. .OCH< CH 3 p: + CH3OH + r Reaction of a secondary haloalkane with a basic nucleophile yields both substitution and elimination products. This is a less satisfactory method of ether preparation. 11.45 HO \ HO Br rotate / C_CC-CH H'> HoC \ 180°' 3 H H— o: ^C- H /> H —C— U H a HoC Rotate around the C-C bond so that -OH and -Br are 180° apart. /> \ Br O CH, / \ C H-* H3 C \ HoC + removal of proton This reaction is UBr HoC Br H2 S N2 displacement of -Br- by-O" C- C V-H CH. + Br" an intramolecular Sn2 displacement. 11.46 H H CH3 ~,;c=C'' H 3 C^ ^H Br2 HoO —c— u H j HoC O CH 3 ,'--H \ Br NaOH^ Ethanol / \ H^ C_C VCH 3 H3 C H + Br" Reactions of Alkyl Halides: Nucleophilic Substitutions and Eliminations 11.47 Draw 4-bromo- 1 -butanol in an orientation that resembles the product. CH 2 CHo HoC CH 2 H2 C 255 H2 C H CH2 :o: :och 3 'Br: + \CH 2 HoC"" \ 2. HoC —/ o: + HOCH 3 :bt: Tetrahydrofuran Step Step 1: 2: Methoxide removes a proton from the hydroxy 1 group of 4-bromo- 1 -butanol. S N 2 displacement of Br~ by (X yields the cyclic ether tetrahydrofuran (THF). 11.48 The first SnI displacement is dissociation of the substrate to form a planar, sp hybridized carbocation and a leaving group. The carbocation that would form from step in an dissociation of this alkyl halide can't become planar because of the 2- rigid structure of the ring skeleton. Because it's not possible to form the necessary carbocation, an SnI reaction can't occur. In addition, approach by a nucleophile from the back side of the alkyl halide is blocked by the rigid ring system, and Sn2 displacement can't take place either. 11.49 2 In a molecule containing a double bond, all atoms bonded to the sp carbons must he in a common plane. For this compound, planar geometry at the "bridgehead" of the ring system is not possible because the rigid ring framework won't allow it. Thus, E2 elimination does not take place because the product containing a bridgehead double bond can't form. 11.50 H OTos HgC^^OTos .OTos ^ CHj Ph*| H3C H H Draw a H Newman projection projection until the Even though in CH3 Ph this which -OTos H of the tosylate of (2/?,35)-3-phenyl-2-butanol, and rotate the -OTos and the -H on the adjoining carbon atom are conformation has several gauche interactions, and -H are 180° apart. 5- it is anti periplanar. the only conformation It OTos OTos Ph CH 3 - OEt —- Ph H3C H3C \ ^ H c II Ph H H OEt 6- Elimination yields the Z isomer of 2-phenyl-2-butene. CH 3 (Z)-2-Phenyl-2-butene Width: 612 Height: 792 256 Chapter 11 11.51 Using the same argument from the previous problem, you can show that elimination from the tosylate of (2/?,3/?)-3-phenyl-2-butanol gives the £-alkene. H OTos i OTos H 3 Cfc. R R H 3 C^| ^Ph H H H3C H 6- OTos OTos H3C ^r Ph H3C "OEt ^fOr H3 C \ Ph H3C H3C H C *H Ph (£)-2-Phenyl-2-butene OEt 6- The (25,35) isomer also forms the E alkene; the (25,3/?) isomer yields the Z alkene. 11.52 H H I nng flip H C cis H V trans diaxial Br*> . y E2 reactions require that the two atoms to be eliminated have a periplanar relationship. Since it's impossible for bromine at CI and the hydrogen at C2 to be periplanar, elimination occurs in the non-Zaitsev direction to yield 3-methylcyclohexene. 11.53 This tertiary bromoalkane reacts by SnI and El routes to yield alcohol and alkene products. The alcohol products are diastereomers. Reactions of Alkyl Halides: Nucleophilic Substitutions and Eliminations 11.54 Step 1: NAD + oxidizes an alcohol to a ketone. H # NH 3 + NH 3 + H 2 + NADH Adenine : Base S-Adenosylhomocysteine 2: Base brings about an leaving group. Step ElcB elimination reaction that has homocysteine as the NH 3+ H H— A H NH 3 + " HS " C0 2 Homocysteine Adenine OH 11.55 :N=C:— W>-C— I :n=c— u n v + I H Formal charge = -1 t ;C=n: v y + CH 3 / :c=N— c-v u + f A T H Formal charge = +1 Sn2 attack by the lone pair electrons associated with carbon gives the nitrile product. Attack by the lone pair electrons associated with nitrogen yields the isonitrile product. 257 258 Chapter 11 11.56 H HOoC H C0 2 H H0 2 C CI (£)-2-Chloro-2-butene- 1 ,4-dioic acid (Z)-2-Chloro-2-butene- 1 ,4-dioic acid are anti to each other in the Z isomer and are syn in the E isomer. Since the Z isomer reacts fifty times faster than the E isomer, elimination must proceed more favorably when the substituents to be eliminated are anti to one another. This is the same stereochemical result that occurs in E2 eliminations of alkyl halides. Hydrogen and chlorine 11.57 is a secondary alcohol, substitution can occur by either an SnI or Sn2 depending on reaction conditions. Two factors favor an SnI mechanism in this case. (1) The reaction is run under acidic conditions in a polar, protic solvent (water). (2) Dilute acid converts a poor leaving group (~OH) into a good leaving group (OH2), which Since 2-butanol route, dissociates easily. 3' Protonation of the hydroxy 1 oxygen.. + followed by loss of water to form a planar H2 is carbocation. [I "H 2 CH2CH3 Attack of water from either side of the planar carbocation yields racemic product. CH2CH3 CH2CH3 CH2CH3 CH2CH3 + H3O- RC HCT" v H CH 3 SC H'], H3 C OH Reactions of Alkyl Halides: Nucleophilic Substitutions and Eliminations 11.58 The chiral tertiary alcohol (/?)-3-methyl-3-hexanol reacts with HBr by 259 an S N 1 pathway. HBr protonates the hydroxyl group, which dissociates to yield a planar, achiral carbocation. Reaction with the nucleophilic bromide anion can occur from either side of the carbocation to produce (±)3-bromo-3-methylhexane. 11.59 Since carbon-deuterium bonds are slightly stronger than carbon-hydrogen bonds, more required to break a C-D bond than to break a C-H bond. In a reaction where either a carbon-deuterium or a carbon-hydrogen bond can be broken in the rate-limiting step, a higher percentage of C-H bond-breaking occurs because the energy of activation for C-H breakage is lower. energy is 6- 8- Br Br D Transition state A* is of higher energy than transition state B* because more energy is required to break the C-D bond. The product that results from transition state B* is thus formed in greater abundance. 11.60 5Br CH 3 Br*^ ' 1* ^Ph C II CHgCH2 CHg H (2/?,3S>2-Bromo-3methyl-2-phenylpentane Base 6- The (25,3/?) isomer also yields E product. (£)-3-Methyl-2phenyl-2-pentene 260 Chapter 11 11.61 NH 2 H C0 2 " '• Base Base removes a proton, yielding an ElcB carb- anion intermediate. Arginine, the protonated leaving group, departs, forming fumarate. Arginine 11.62 One of the steric requirements of E2 elimination is the need for periplanar geometry, which optimizes orbital overlap in the transition state leading to alkene product. Two types of periplanar arrangements of substituents are possible syn and anti. model of the deuterated bromo compound shows that the deuterium, bromine, and the two carbon atoms that will constitute the double bond all lie in a plane. This arrangement of atoms leads to syn elimination. Even though anti elimination is usually preferred, it doesn't occur for this compound because the bromine, hydrogen, and two carbons can't achieve the necessary geometry. — A 11.63 We concluded in Problem 1 1 .62 that E2 elimination in compounds of this bicyclic structure occurs with syn periplanar geometry. In compound A, -H and -CI can be eliminated via the syn-periplanar route. Since neither syn nor anti periplanar elimination is possible for B, elimination occurs by a slower, El route. Reactions of Alkyl Halides: Nucleophilic Substitutions and Eliminations 261 11.64 The two pieces of evidence indicate that the reaction proceeds by an S N 2 mechanism: S N 2 much faster in polar aprotic solvents such as DMF, and methyl esters react faster than ethyl esters. This reaction is an Sn2 displacement on a methyl ester by reactions proceed iodide ion. Other experiments can provide additional evidence for an Sn2 mechanism. We can determine if the reaction is second-order by varying the concentration of Lil. We can also vary the type of nucleophile to distinguish an Sn2 mechanism from an SnI mechanism, which does not depend on the identity of the nucleophile. 11.65 Because CT is a relatively poor leaving group and acetate is a relatively poor nucleophile, a two groups proceeds at a very slow rate. 1~, however, is both a good nucleophile and a good leaving group. 1-Chlorooctane thus reacts preferentially with iodide to form 1-iodooctane. Only a small amount of 1-iodooctane is formed (because of the low concentration of iodide ion), but 1-iodooctane is more reactive than 1-chlorooctane toward substitution by acetate. Reaction with acetate produces 1-octyl acetate and regenerates iodide ion. The whole process can now be repeated with another molecule of 1-chlorooctane. The net result is production of 1-octyl acetate, and no iodide is consumed. substitution reaction involving these 11.66 Two optically inactive compounds X are possible structures for X strong base | compound X. 262 Chapter 11 11.67 CTos-p CI :Cl: CH 2CI IB ^ + Pyr:H Sx + + "OTos 1 At lower temperatures, a tosylate is formed from the reaction of p-toluenesulfonyl chloride and an alcohol. The new bond is formed between the toluenesulfonyl group and the oxygen of the alcohol. At higher temperatures, the chloride anion can displace the -OTos group, which is an excellent leaving group, to form an organochloride. 11.68 H Br X, <3v, OH NaOH H HO Co: H ho: |h 3 o + HO H OH Two inversions of configuration equal a net retention of configuration. 11.69 + W H3C H3C y CH O 3fi> y C\ R CF3CO- V s N H 3 C /CH 3 CF 3 C0 2 - ^ .CrVc + U M3U N 1. I H H 2. CF 3 C0 2 H + Baser OH n H 3C 3. C=CH 2 c+ H«aC 1: Protonation. 2: Elimination of leaving group. 3: Removal of proton by base. This reaction proceeds by an El mechanism. I 1 H 3 C V CH 2 N / HoC Step Step Step r + O N I H R Reactions of Alkyl Halides: Nucleophilic Substitutions and Eliminations 11.70 Notice that the chiral methyl group has the (R) configuration in both Nmethyltetrahydrofolate and in methionine. This fact suggests that methylation proceeds with two inversions of configuration which, in fact, has been shown to be the case. 11.71 CH 3 CH 2 CH 2 CH 2 CH 2 NH 2 + CH 3 CH 2 CH 2 CH 2 CH 2 N(CH 3 )3 excess CHoI 2-*- ^ T Ag 20, H 2 — - _ T + CH 3 CH2 CH 2 CH 2 CH 2 N(CH 3 )3 »- CH 3 CH 2 CH 2 CH= CH 2 + : N(CH 3 ) 3 The intermediate is a charged quaternary ammonium compound that results from Sn2 substitutions on three CH3I molecules by the amine nitrogen. E2 elimination occurs because the neutral N(CH3)3 molecule is a good leaving group. 11.72 (a) electrophile (b) 263 Review Unit 4: Alkynes; AlkyI Halides; Substitutions and Eliminations Major Topics Covered (with vocabulary): Alkynes: alkyne enyne vicinal tautomer acetylide anion Lindlar catalyst alkylation Organic Synthesis. Alky I halides: allylic position derealization Grignard reagent Gilman reagent Suzuki -Miyaura reaction Oxidation and reduction in organic chemistry: Substitution reactions: nucleophilic substitution reaction Walden inversion reaction rate kinetics second-order reaction rate constant reaction bimolecular nucleophilicity leaving group solvation Sn 1 reaction first-order reaction rate-limiting step ion pair dielectric polarization Elimination reactions: E2 reaction syn periplanar geometry deuterium isotope effect El reaction ElcB reaction Zaitsev's rule anti periplanar geometry Types of Problems: After studying these chapters, you should be able to: - Predict the products of reactions involving alkynes. Choose the correct alkyne starting material to yield a given product. Deduce the structure of an alkyne from its molecular formula and products of cleavage. Carry out syntheses involving alkynes. - Draw, name, and synthesize alkyl halides. Understand the mechanism of radical halogenation and the stability order of radicals. Prepare Grignard reagents and dialkylcopper reagents and use them in synthesis. Predict the oxidation level of a compound. - - Formulate the mechanisms of S N 2, SnI and elimination reactions. - Predict the effect of substrate, nucleophile, leaving group and solvent elimination reactions. - Predict the products of substitution and elimination reactions. Classify substitution and elimination reactions by type. on substitution and Review Unit 4 Points to 265 Remember: Although it is very important to work backwards when planning an organic synthesis, don't forget to pay attention to the starting material, also. Planning a synthesis is like solving a maze from the middle outward: keeping your eye on the starting material can keep you from mnning into a dead end. The reagent L1/NH3 is used to reduce an alkyne to a trans alkene; the reagent used to form an acetylide anion. It is easy to confuse the two reagents. NaNth/NHa is by the IUPAC system, remember that a halogen is named as a on an alkane. When numbering the alkyl halide, the halogens are numbered same way as alkyl groups and are cited alphabetically. In naming alkyl halides substituent in the The definition of oxidation and reduction given in Chapter 10 expands the concept to reactions you might not have considered to be oxidations or reductions. As you learn new reactions, to classify them as oxidations, reductions or neither. that try outcome of substitutions and eliminations is only straightforward in certain For primary halides, Sn2 and E2 reactions are predicted. For tertiary halides, SnI, E2 and El (to a certain extent) are the choices. The possibilities for secondary halides are more complicated. In addition, many reactions yield both substitution and elimination products, and both inversion and retention of configuration may occur in the same reaction. Predicting the cases. Self-Test: CH2CH3 CHgCh^CCh^C^^ CH CH 3 A B Terbinafine (an antifungal) What is the configuration of the double bond in the side chain of A? What products result from treatment of A with KMn0 4 H3O (neither the aromatic ring nor the amine are affected)? How might the triple bond have been introduced? , Pd/C Provide a name for B. Predict the products of reaction of B with (c) BH3 THF, then H 2 2 HO (d) O3, then Zn, H3O+ , , (a) 1 equiv HBr (b) H?, Width: 612 Height: 792 Review Unit 4 266 CH 3 CI HC= CCCH= CHCI CHgCH^CHCHCHg OH Br E D c Ethchlorvynol (a sedative) Name C. Draw all products of reaction of the major product); (d) of (a) stereoisomers of C, label them, and describe their relationship. Predict the C with:(a) NaOH; (CH 3 CH 2 )2CuLi. (b) Mg, then H2O; (c) product of (b) + Br2, hv (show Draw the R enantiomer of D. Predict the products of reaction of D with: (a) HBr; (b) product + aqueous ethanol. Describe the reactivity of the -CI atom in substitution and elimination reactions. How might E be synthesized from the appropriate alkylbenzene? From the appropriate alcohol? Predict the reactivity of E in substitution and elimination reactions. Multiple Choice: 1 . An enol is a tautomer of an: (a) alcohol 2 . (c) . . . (d) ketone reaction proceeds through a vinylic radical? Hg-catalyzed hydration of an alkyne (b) Li/NH3 reduction of an alkyne catalytic hydrogenation of an alkyne (d) treatment of an alkyne with a strong base reagents is not used in a Suzuki-Miyaura reaction? aromatic boronic acid (b) lithium (c) Pd catalyst (d) potassium carbonate Monochlorination of 2,3-dimethylbutane yields what percent of 2-chloro-2,3dimethylbutane? (a) 5 alkene Which of the following (a) 4 (c) Which (a) 3 (b) alkyne 16% (b) 35% (c) 45% (d) 55% How many monobromination products can be formed by NBS bromination of 2-ethyl- 1pentene? Include double-bond isomers. (a) 3 (b)4 (c)5 (d)6 6 . Which of the following (a) hydroxylation reactions (b) hydration is an oxidation? hydrogenation (c) (d) addition of HBr Review Unit 4 7 . All of the following are true of S N 2 reactions except: (a) The rate varies with the concentration of nucleophile (b) The rate varies with the type of nucleophile (c) The nucleophile is involved in the rate-deterrnining step (d) The rate of the Sn2 reaction of a substrate and same two compounds. 8 . a nucleophile Which base (a) 10. the same as the rate of the E2 reaction of the is step. best for converting 1-bromohexane to 1-hexene? (CH 3 ) 3 CO- (b) -CN (c) -OH (d) -C^CH Which of the following (a) 11. is Which of the following is true of SnI reactions? (a) The rate varies with the concentration of nucleophile (b) The rate varies with the type of nucleophile (c) The rate is increased by use of a polar solvent, (d) The nucleophile is involved in the rate-determining 9. 267 -OH (b) -CN is both a good nucleophile and a good leaving group? (c)-Cl (d)-I In the reaction of (2/?,35>3-methyl-2-pentanol with tosyl chloride, what is the configuration of the product? (a) a mixture of all four possible stereoisomers (b) (2R,3S) and (2S,3S) (c) (2R,3S) (d) (2S,3S) Chapter 12 - Structure Determination: Mass Spectrometry and Infrared Spectroscopy Chapter Outline I. Mass Spectrometry (Sections 12.1-12.4). A. General features of mass spectrometry (Section 12.1). 1 Purpose of mass spectrometry. a. Mass spectrometry is used to measure the molecular weight of a compound. b Mass spectrometry can also provide information on the structure of an unknown . . compound. 2. Technique of mass spectrometry. a. A small amount of sample is vaporized into the ionization source and is bombarded by a stream of high-energy electrons. b An electron is dislodged from a molecule, producing a cation radical. c Most of the cation radicals fragment; the fragments may be positively charged or . . neutral. d In the deflector, a strong magnetic field deflects the positively charged fragments, which are separated by m/z ratio. detector records the fragments as peaks on a graphic display. e. Important terms. a. The mass spectrum is presented as a bar graph, with masses (m/z) on the x axis and intensity (relative abundance) on the y axis. b. The base peak is the tallest peak and is assigned an intensity of 100%. + c. The parent peak, or molecular ion (M ), corresponds to the unfragmented cation . A 3 . radical. In large molecules, the base peak is often not the molecular ion. B. Interpreting mass spectra (Sections 12.2-12.4). 1. Molecular weight (Section 12.2). a. Mass spectra can frequently provide the molecular weight of a sample. Double-focusing mass spectrometers can provide mass measurements i. accurate to 0.0005 amu. + ii. is not seen. Some samples fragment so easily that b If you know the molecular weight of the sample, you can often deduce its molecular formula. 13 c. There is often a peak at M+l that is due to contributions from C and 2 H. 2 Fragmentation patterns of hydrocarbons. a. Fragmentation patterns can be used to identify a known compound, because a given compound has a unique fragmentation "fingerprint". b Fragmentation patterns can also provide structural information. Most hydrocarbons fragment into carbocations and radicals. i. ii. The positive charge remains with the fragment most able to stabilize it. d. M . . . iii. It is often difficult to assign structures to fragments. For hexane, major fragments correspond to the loss of methyl, propyl, and butyl radicals. 3. Fragmentation patterns of common functional groups (Section 12.3). iv. a. ethyl, Alcohols. i. Alcohols can fragment by alpha cleavage, in which a C-C bond next to the -OH group is broken. (a). The products are a cation and a radical. ii. Alcohols can also dehydrate, leaving an alkene cation radical with a mass 18 units less than M*. Structure Determination: Mass Spectrometry and Infrared Spectroscopy b 269 Amines also undergo alpha cleavage, forming a cation and a radical. Carbonyl compounds. i. Aldehydes and ketones with a hydrogen 3 carbons from the carbonyl group can undergo the McLafferty rearrangement. (a). The products are a cation radical and a neutral alkene. Aldehydes and ketones also undergo alpha cleavage, which breaks a bond ii. between the carbonyl group and a neighboring carbon. (a). The products are a cation and a radical. . c. C. Mass spectrometry in biological systems: TOF instruments (Section 12.4). 1 Time-of-flight (TOF) instruments are used to produce charged molecules with . little fragmentation. 2 . The a. be either ESI or MALDI. ESI source, the sample is dissolved ionizer can In an through a As the sample exits, i. in a polar solvent and sprayed steel capillary tube. it is subjected to a high voltage, which protonates the sample. The ii. b . i. ii. 3 4 . . solvent is evaporated, yielding protonated sample molecules. MALDI source, the sample is absorbed onto a matrix compound. In a The matrix compound The matrix compound The samples is ionized by a burst of laser light. transfers energy to the sample, protonating it. are focused into a small packet and given a burst of energy. a. Each molecule moves at a velocity that depends on the square root of its mass. The analyzer is an electrically grounded tube that detects the charged molecules by velocity. H Spectroscopy and the electromagnetic spectrum (Section 12.5). A. The nature of radiant energy. 1 Different types of electromagnetic radiation make up the electromagnetic spectrum. 2 Electromagnetic radiation behaves both as a particle and as a wave. 3 Electromagnetic radiation can be characterized by three variables. a. The wavelength (X) measures the distance from one maximum to the next. b The frequency (v) measures the number of wave maxima that pass a fixed point . . . . per unit time. 4. The amplitude is the height measured from the midpoint Wavelength times frequency equals the speed of light. c. to the maximum. is transmitted in discrete energy bundles called quanta. energy per photon. b. Energy varies directly with frequency but inversely with wavelength. c E = 1 .20 x 10" 2 kJ/mol k (cm) for a "mole" of photons. Electromagnetic radiation and organic molecules. When an organic compound is struck by a beam of electromagnetic radiation, it 1 absorbs radiation of certain wavelengths, and transmits radiation of other wavelengths. 2 If we determine which wavelengths are absorbed and which are transmitted, we can obtain an absorption spectrum of the compound. a. For an infrared spectrum: i. The horizontal axis records wavelength. 5 . Electromagnetic energy a. e = hv, where e is . B . . . ii. iii. . 4 . vertical axis records percent transmittance. baseline runs across the top of the spectrum. is a downward spike (low percent transmittance). The energy a molecule absorbs is distributed over the molecule. There are many types of spectroscopies that differ in the region of the iv. 3 The The Energy absorption electromagnetic spectrum that is being used. 270 Chapter 12 in. Infrared Spectroscopy (Sections 12.6-12.8). A. Infrared radiation (Section 12.6). 1 . The infrared (IR) region of the electromagnetic spectrum extends tolO^m. from 7.8 x 10 -7 m -5 m Organic chemists use the region from 2.5 x 1CT6 to 2.5 x 10 m. b Wavelengths are usually given in \xm, and frequencies are expressed in wavenumbers, which are the reciprocal of wavelength. -1 -1 this corresponds to c. The useful range of IR radiation is 4000 cm to 400 cm energies of 48.0 kJ/mol to 4.80 kJ/mol. 2. IR radiation causes bonds to stretch and bend and causes other molecular a. . ; vibrations. 3 . Energy is absorbed at a specific frequency that corresponds to the frequency of the vibrational motion of a bond. 4. If we measure the frequencies at which IR energy is absorbed, we can find out the compound contains and identify functional groups. B. Interpreting IR spectra (Sections 12.7-12.8). General principles (Section 12.7). 1 a. Most molecules have very complex IR spectra. kinds of bonds a . This complexity means that each molecule has a unique fingerprint that allows it to be identified by IR spectroscopy. ii. Complexity also means that not all absorptions can be identified. Most functional groups have characteristic IR absorption bands that change very i. b. c. little from one compound The significant regions of i. to another. - 4000 cm ^2500 cm-1 N-H bonds. _1 IR absorptions : corresponds to absorptions by C-H, O-H, and - 2500 cm -2000 cm corresponds to triple-bond stretches, _1 -1 hi. 2000 cm -1500 cm corresponds to double bond stretches. -1 iv. The region below 1500 cm is the fingerprint region, where many complex bond vibrations occur that are unique to a molecule. d. The frequency of absorption of different bonds depends on two factors: i. The strength of the bond. The difference in mass between the two atoms in the bond. ii. IR spectra of some common functional groups (Section 12.8). ii. 2. a. Alkanes. 1 i . ii b C-C absorbs at 800- 300 cm" -1 C-H absorbs at 2850-2960 cm 1 . , .Alice ncs i . ii. iii. iv. c. =C-H absorbs at 3020-3 1 00 cm" C=C absorbs at 1650-1670 cm-1 1 . . RCH=CH2 absorbs at 910 and-1990 cm-1 R 2C=CH 2 absorbs at 890 cm . . Alkynes. -1 i . ii. d . -C=C- absorbs at 2 1 00-2260 cm -1 =C-H absorbs at 3300 cm . . Aromatic compounds. i. =C-H absorbs at 3030 cm" 1 ii. Ring absorptions occur at 1660-2000 . cm-1 and at 1450-1600 -1 cm . Structure Determination: Mass Spectrometry and Infrared Spectroscopy 271 -1 e. f . g. The alcohol O-H bond absorbs at 3400-3650 cm -1 The N-H bond of amines absorbs at 3300-3500 cm . . Carbonyl compounds. Saturated aldehydes absorb at 1730 i. 1705 cm" -1 cm cm unsaturated aldehydes absorb at cm-1 ; unsaturated ketones absorb at 1690 . cm-1 Saturated esters absorb at 1735 iii. ; . Saturated ketones absorb at 1715 ii. -1 cm 1 ; unsaturated esters absorb at 1715 _i . Solutions to Problems 12.1 If the isotopic masses of the atoms C, H, and O had integral values of 12 amu, 1 amu and 16 amu, many molecular formulas would correspond to a molecular weight of 288 amu. Because isotopic masses are not integral, however, only one molecular formula is associated with a molecular ion at 288.2089 amu. To reduce the number of possible formulas, assume that the difference in molecular weight between 288 and 288.2089 is due mainly to hydrogen. Divide 0.2089 by 0.00783, the amount by which the atomic weight of one 'H atom differs from 1. The answer, 26.67, gives a "ballpark" estimate of the number of hydrogens in testosterone. Then, divide 288 by 12, to determine the maximum number of carbons. Since 288 -r 12 = 24, we know that testosterone can have no more than 22 carbons if it also includes hydrogen and oxygen. Make a list of reasonable molecular formulas containing C, H and O whose mass is 288 and which contain 20-30 hydrogens. Tabulate these, and calculate their exact masses using the exact atomic mass values in the text. The only possible formula for testosterone is C19H28O2. Isotopic Mass Molecular formula of carbons C 20 H 32 O mass Mass Mass Mass of molecular ion of oxygens of hydrogens 240.0000 amu 32.2504 2 228.0000 28.2191 31.9898 288.2089 c i8 H 24°3 216.0000 24.1879 47.9847 288.1726 C 19 H 28 amu 15.9949 amu 288.2453 amu 12.2 7 CHgCH2CH^= CH 3 CHgCh^Ch^CH—- CHCHg C CH 3 2-Hexene 2-Methyl-2-pentene CHo + * + CH 2 CH=C m/z =69 CH 3 CH 2 CH=CHCH 3 m/z = 55 weakest carbon-carbon bonds, producing a relatively stable cation. Spectrum (a), which has a dominant peak at m/z = 69, corresponds to 2-methyl-2-pentene, and spectrum (b), which has m/z = 55 as its base peak, corresponds to 2-hexene. Fragmentation occurs to a greater extent at the 272 Chapter 12 12.3 In a mass spectrum, the molecular ion is both a cation and a radical. When it fragments, two kinds of cleavage can occur. (1) Cleavage can form a radical and a cation (the species observed in the mass spectrum). Alpha cleavage shows this type of pattern. (2) Cleavage can form a neutral molecule and a different radical cation (the species observed in the mass spectrum). Alcohol dehydration and the McLafferty rearrangement show this cleavage pattern. For each compound, calculate the mass of the molecular ion and identify the functional groups present. Draw the fragmentation products and calculate their masses. (a) r o II r Alpha^ ' HgC J CH 2 CH 2 CHo, HoC— C=0 cleavage CH2CH2CH3 L M+ = 86 m/z = 43 O II Alpha ( HgC M+ = 86 0=C— CH 2 CH2 CH3 + HoC" CH 2 CH 2 CHo, J cleavage J m/z =71 In theory, alpha cleavage can take place on either side of the carbonyl group to produce cations with m/z = 43 and m/z = 7 1 In practice, cleavage occurs on the more substituted side of the carbonyl group, and the first cation, with m/z = 43, is observed. . (b) r Dehydration M+= HoO m/z 100 =82 Dehydration of cyclohexanol produces a cation radical with m/z = 82. (c) H CH, HoC McLafferty CH 3 CHCH 2 CCHo, M+= ? .C, rearrangement H 3C-c<-VC^ C u H 100 m/z CH- CH, =58 + / \ H H (fH2 .c. HoC The « cation radical fragment resulting r CH 2 CH 3 from McLafferty rearrangement has m/z = , +- CH 2 CH 3 Alpha 1 H 3 C~I" CH 2 CH 2 CH 3 HoC" + cleavage H2C M+=101 Alpha cleavage of triethylamine CH 2 CHg m/z = 86 yields a cation with m/z = 86. H 58. Structure Determination: Mass Spectrometry and Infrared Spectroscopy 12.4 273 Identify the functional groups present in the molecule and recall the kinds of fragmentations those functional groups produce. 2-Methyl-2-pentanol produces fragments that result from both dehydration and from alpha cleavage. Two different alpha cleavage products are possible. CHo3 CH, Dehydration I CHgCH2CH CHgCh^Ch^CCHg OH M+ = —C HoO CH 3 J _ mlz = 84 102 Alpha Alpha cleavage cleavage I OH Ha CH3CH2CH2C CH3CH2CH2" c CH, 1 CH, n 3 mlz = 87 mlz Peaks might appear 12.5 We know that: is M+ = 102 (molecular ion), 87, 84, 59. is energy increases as wavelength decreases, and (2) the wavelength of Xwe estimate that an X -34 £ = hv = hclk; h = 6.62 x 10" J " J-s; c = -6 for X = 1 x 10 m (infrared radiation): £ = [6.62 x IO-34 1.0 for £ = 59 smaller than the wavelength of infrared radiation. Thus, of higher energy than an infrared ray. radiation ray (1) at . X= = v-9 3.0 x 10 (6.62 x IO- J-s) 3.00 x 10° m/s (3.00 x 10 8 m/s) x 10" 6 = 19 2.0 x 10J = 6.6 x 10- 17 J m m (X radiation): 34 J-s) (3.00 x 10 8 m/s) 3.0 x 10- 9 m Confirming our estimate, the calculation shows that an X ray is of higher energy than infrared radiation. Convert radiation in m to radiation in Hz by the equation: 3.00 x 10 8 m/s 9.0 x 10~ 6 The equation e = 4.0 x 10 9 3.3 x 10 13 Hz = hv shows that the greater the value of v, the greater the energy. Thus, 13 6 = 3.3 x 10 Hz (A. = 9.0 x 10" m) is higher in energy than radiation with radiation with v v m = Hz. 274 Chapter 12 12.6 (a) E = 1.20 x _ l(HkJ/mol 1.20 x 11 5.0 x 10" X (in m) = (b) E = (c) v 6 2.4 x id kJ/mol for a 4 X = c = . 3.0 x v X E gamma ray. 4.0 x 10 kJ/mol for an c_ 1.20 x lO^kJ/mol X ray. 10 8 m/s 6.0 x 10 lO^kJ/mol = 15 = 5 x 10 -8 m Hz 2.4 x 10 3 kJ/mol for ultraviolet light 5.0 x 10- 8 12.7 2 (d) E 2.8 (e) E 6.0 kJ/mol for infrared radiation (f) E 4.0 x 10 (a) x 10 kJ/mol for visible -2 light. kJ/mol for microwave radiation. A compound with a strong absorption at 17 10 cm 1 contains a carbonyl group and is either a ketone or aldehyde. (b) A compound with a nitro group has (c) A compound showing both carbonyl (1720 cm-1 absorptions 12.8 is a strong absorption at 1540 ) and -OH cm (2500-3000 cm -1 broad) a carboxylic acid. To use IR spectroscopy to distinguish between isomers, find a strong IR absorption that present in one isomer but absent in the other. (a) CH 3 CH 2 OH CH3OCH3 Strong hydroxyl band No band in the region at 3400-3640 cm"! 3400-3640 cm-i (b) CH3CH2CH2CH2CH— CH2 Alkene bands at 3020-3100 cm- 1 and at 1640-1680- 1 No bands CH3CH2CO2H HOCH 2 CH 2 CHO . Strong, broad at band 2500-3100 cm" 1 in alkene region. Strong band at 3400-3640 cm" 1 is Structure Determination: Mass Spectrometry and Infrared Spectroscopy 12.9 Based on what we know at this point, (a) Absorptions in the region 1450 motions. (b) The absorption (c) Absorptions in the range 3000 at 2100 cm-1 we can identify four absorptions in this spectrum. cm_1 -1600 cm -1 due to a is 275 are due to aromatic ring -C=C- -C=C- stretch. -1 _1 cm -3100 cm due are to aromatic ring =C-H stretches. 12.10 (d) The absorption (a) An at at 3300 ester next to a double 1640-1680 cm4 -1 cm is due to a bond absorbs =C-H at stretch. 1715 cm-1 The . alkene double bond absorbs . -1 (b) The aldehyde carbonyl group absorbs at 1730 cm The alkyne C=C bond absorbs -1 2100-2260 cm and the alkyne H-C= bond absorbs at 3300 cm" . at 1 . , (c) The most important absorptions for this compound are due to the alcohol group (a -1 broad, intense band at 3400-3650 cm ) and to the carboxylic acid group, which has 2500-3100 1450-1600 cm-1 cm . a -1 1710-1760 cm and a broad O-H absorption in the range -1 Absorptions due to the aromatic ring [3030 cm (w) and C=0 absorption-1 in the range (m)] may also be seen. 12.11 H The compound contains nitrile and ketone groups, as well as a carbon-carbon double -1 bond. The nitrile absorption occurs at 2210-2260 cm The ketone shows an absorption -1 at 1690 cm a lower value than usual because the ketone is next to the double bond. The double bond absorption occurs at 1640-1680 cm . , . Width: 612 Height: 792 276 Chapter 12 Visualizing Chemistry 12.12 Compound Due to: Significant IR Absorption 1540 cm- 1 nitro 1730 cm-' aldehyde (2) O H3C V aromatic ring C-H(3) 1735 cm" 1 ester (1) 3020-3100 cm- 1 vinylic stretch aromatic ring C=C(3) II C 2 CH 3 ^CC ' C-H(2) 910 cm- 990 cm- 1 1 , 3 CH 2 1640-1680 (c) cm" 1 C=C ketone (1) 3400-3650 cm- C C=CH 2 bend(3) alkene 1715 cm- 1 HoC (1) 3030 cm- 1 1450-1600 cm- 1 , (b) group 1 alcohol (2) CH, /\ HH 12.13 (a) The mass spectrum of this ketone shows fragments resulting from both McLafferty rearrangement and alpha cleavage. McLafferty rearrangement: H O II CHgCH 2 CHCCH 2 CHo McLafferty i rearrangement CH 3 M+= H- H 114 H 2 C=CH 2 CH 2 CH 3 ps H CH 3 ? + C H CH 2 CHg m/z = 86 Structure Determination: Mass Spectrometry and Infrared Spectroscopy 277 Alpha cleavage: CH 3 |_ CH 3 CH 2 CH CH 2 CH3_ ; M+ = cleavage —Alpha * M+= CHoCHoCH- + cleavage CH 2 CH 3 _ j m/z Two different fragments 3 s CH2CH 3 0=C— CHoCHo 114 CH A N— W C * m/z = 85 I (b) CH 3 CH 2 CH— C=0 114 CH 3 CH 3 CH 2 CH 0x10 8hz V 3.0 x 10 8 = , A m 1.0 Hz l-20xl0^kJ/mol = E = j 20 x 1(H kJ/mol Increasing the spectrometer frequency from 200 energy needed for resonance. MHz to 300 MHz increases the amount of 13.3 CH 3 b H / c a C=C\ H CI 2-Chloropropene has three kinds of protons. Protons b and c differ because one the chlorine and the other is cis to is trans. 13.4 = Observed chemical 8 o 200 =im (a) 8 (C) 6 = shift (in Hz) MHz = 7 27 6forCHCl3 - OTJ = 3.46 6 forCH 3 OH \ ? CH 3 CHCH2CH2CI Nuclear Magnetic Resonance Spectroscopy Structure Determination: 13 C NMR 309 Spectroscopy 13.46 4^ H HoC ring-flip |J || ring-flip H3 C ds-l,3-Dimethylcyclohexane h3 C H H CH 3 H V^ng CH, trans- 1 ,3-Dimethylcyclohexane is a meso compound. Because of symmetry, it shows 5 spectrum, trans- 1 ,3-Dimethylcyclohexane exists as a pair of absorptions in its C enantiomers, which, at room temperature, undergo ring-flips that average the absorptions due to nonequivalent carbons. Like the cis isomer, the racemic mixture of trans 13 enantiomers shows 5 absorptions in its C spectrum. cw-l,3-Dimethylcyclohexane NMR 13.47-13.48 Number of 13 C Compound Absorptions Carbons Showing Peaks Positive Peaks in DEPT-135 Negative Peaks 13 C NMR Spectrum No Peaks (a) 1 1 CHo HoC carbon X 1 carbons 3,4,5 carbon 2 (b) 3 2 1 CH3CH2OCH3 (c) carbons 1,3 carbon 2 carbons 1,3 carbons 4,5,6 1 CHo3 1 H3C I 1 $2 CHs carbon 2 Chapter 13 310 Number Carbons Showing Peaks of 13 C Compound Absorptions Positive Peaks in DEPT-135 13 Negative Peaks C NMR Spectrum No Peaks (d) 3 CH 3 6 5 4l 2 carbon 2 carbons 1,3,4,6 carbon 5 carbons 1,2 carbons 3,4 1 CH 3 CH 2 CHC=CH carbons 2,3,4 13.49 carbon 1 Either *H NMR or 13C NMR can be used to distinguish among these isomers. In either case, first it is necessary to find the number of different kinds of protons or carbon atoms. Kinds of Compound Protons Kinds of Carbon atoms Number of l H NMR peaks Number of 13 C NMR peaks C-H2 H^C^ CH2 h^C^ CHCH2CH3 CHgCH — CHCHg (CH 3 ) 2 C=CH 2 13 2 2 3 3 C NMR is the simplest method for identifying these compounds because each isomer number of absorptions in its 13 C NMR spectrum. H NMR can also be used differs in the to distinguish among the isomers because differ in their splitting patterns. the two isomers that show two *H NMR peaks Nuclear Magnetic Resonance Spectroscopy Structure Determination: 311 13.50 Number of Distinguishing Peaks Absorptions Two 13C 1H vinylic peaks Unsplit vinylic peak, relative area 1 One vinylic peak 13C Split vinylic peak, relative area NMR 2 13 both H and C NMR. In 13 addition, the distinguishing absorptions in the vinylic region of both the *H and C spectra make it possible to identify each isomer by its spectrum. -1 The ketone IR absorption of 3-methyl-2-cyclohexenone occurs near 1690 cm because the double bond is one bond away from the ketone group. The ketone IR absorption of 3cyclopentenyl methyl ketone occurs near 1715 cm the usual position for ketone The two isomers have numbers of peaks different in ] NMR , absorption. 13.51 Carbon OCH 2 CH 3 14 61 3 166 4" 5 Ethyl benzoate 6 (ppm) 2 1 127-133 (4 peaks) 6 7 General Problems 13.52 (a),(b) C3H6O contains one double bond H 2 C- m 14.2 CH 3 CH=CHCH=CH 2 1,3-Pentadiene Name Product Results from : CI I CH 3 CH= CHCHCH 3 4-Chloro-2-pentene 1,2 addition 1 CI CHgCh^CHCH _ — CH2 addition ,4 I 3-Chloro- 1 -pentene 1,2 addition -Chloro-2-pentene 1,4 addition CH 3 CH 2 CH=CHCH 2CI 1 14.3 6+ 5+ CH3CH2CH D CH CH2 protonation on carbon 4 protonation H+ / \ CH=CHCH=CH CH 3 J on carbon 3 CHgCHCH^CH — CH2 C 6+ 6+ E-J CH 3 CH— CH— CHCH3 protonation A on carbon 2 . 1 protonation on carbon 2 N H+ » + CH 3 CH= CHCH 2 CH 2 B A and D, which are resonance-stabilized, are formed in preference to B and C, which are not. The positive charge of allylic carbocation A is delocalized over two secondary carbons, while the positive charge of carbocation D is delocalized over one secondary and one primary carbon. We therefore predict that carbocation A is the major intermediate formed, and that 4-chloro-2-pentene predominates. Note that 1,2 and 1,4 addition. this product results from both 322 Chapter 14 14.5 CH 3 CHCH=CH 2 ^t|CH 3 CHCH= CH ]^ CH CH= CHCH CH 3 CH=CHCH2 ; 3-Bromo- 1 -butene 3 2 Br l-Bromo-2-butene Br~ (l,2adduct) (1,4 adduct) Ally lie halides can undergo slow dissociation to form stabilized carbocations (SnI reaction). Both 3 -bromo-1 -butene and l-bromo-2-butene form the same allylic carbocation, pictured above, on dissociation. Addition of bromide ion to the allylic carbocation then occurs to form a mixture of bromobutenes. Since the reaction is run under equilibrium conditions, the thermodynamically more stable l-bromo-2-butene predominates. 14.6 X 1 1 2 / H 2 C^~ CHCH^- CH 2 + 1 H 2 CCHCH=CH 2 monosubstituted double bond X X2 * 14 \V-^-V HoCCH= CCH = CHCH 2 2 disubstituted double bond X 1,4 adducts are more stable than 1,2 adducts because disubstituted double stable than monosubstituted double 14.7 bonds are more bonds (see Chapter 7). Draw the reactants in an orientation that shows where the new bonds will form. Form the new bonds by connecting the two reactants, removing two double bonds, and relocating bond so that it lies between carbon 2 and carbon 3 of the diene. The on the dienophile retain their trans relationship in the product. The product is a the remaining double substituents racemic mixture. Conjugated Compounds and Ultraviolet Spectroscopy pCH 3 323 OCH, OCH, CH 3 H 14.8 Good dienophiles have an electron-withdrawing group Good dienophiles: conjugated with a double bond. (d) (a) O II H 2 C=CHCCI Poor dienophiles: (c) (b) (e) O O H 2 C= CHCH 2 CH 2 COCH 3 Compound (a) and (d) are good dienophiles because they have electron- withdrawing groups conjugated with a carbon-carbon double bond. Alkene (c) is a poor dienophile because it has no electron- withdrawing functional group. Compounds (b) and (e) are poor dienophiles because their electron- withdrawing groups are not conjugated with the double bond. 14.9 (a) This diene has an s-cis conformation and should undergo Diels-Alder cycloaddition. an s-trans conformation. Because the double bonds are in a fused ring not possible for them to rotate to an s-cis conformation. (b) This diene has system, (c) it is Rotation can occur about the single bond of this s-trans diene. The resulting s-cis conformation, however, has an unfavorable steric interaction of a methyl group with a hydrogen at carbon 1 Rotation to the s-cis conformation is possible but not favored. . H f C H 3 C' C C *C{3 *CH; H3C' V I H H s-trans (more stable) 5-C1S (less stable) 324 Chapter 14 14.10 Rotation of the diene to the s-cis conformation must occur in order for reaction to take place. O """"•-tiv. -C s OCH 3 OCH, o 5-trans 14.11 The 5-C1S may be either a radical or a cation. Diene polymerization is a 1 ,4 addition that forms a polymer whose monomer units have a 4 carbon chain that contains initiator process a double bond every 4 bonds. 14.12 etc. 14.13 200 x 10-9 m = nm 400 nm 200 forx 400 x 10-9 2x 10- 7 1.20 x E m 2x = 4x 10-7 10-7 m: KHkJ/mol 1.20 x 10^kJ/mol = 6.0 x 10 2 kJ/mol = 3.0 x 10 2 kJ/mol 2.0 x 10- 7 k (in m) 4 x 10-7 m: forX 1.20 x E 10^kJ/mol X (in m) The energy of electromagnetic nm is 300-600 kJ/mol. (in The energy required NMR transitions. for 10^ kJ/mol -7 radiation in the region of the spectrum 300-600 kJ/mol) 1.20 x 4.0 x 10 UV Energy m IR 4.8-48 *H from 200 nm to 400 NMR (at 200 MHz) 8.0 x 10-5 UV transitions is greater than the energy required for IR or *H Conjugated Compounds and Ultraviolet Spectroscopy 14.14 8 A = CX Where 8 = molar absorptivity (in L/mol-cm) / A = absorbance In this problem: 8 / = 50,100 = = A = 1.00 5.01 x 10 4 L/mol / cm c cm - concentration (in mol/L) 0.735 0.735 8 14.15 = sample pathlength (in cm) X / 5.01 x 104 = L/mol-cm x 1.00 1.47 x 10- 5 M cm All compounds having alternating single and multiple bonds should show ultraviolet absorption in the range 200^100 nm. Only compound (a) is not UV-active. All of the compounds pictured below show UV absorptions. O C0H H 2 C=CHC =N cx OCCHo3 II o 325 Width: 612 Height: 792 326 Chapter 14 14.17 5-trans \ / 5-C1S H ? C= CH c=o HgC 14.18 would have to rotate to an s-cis arrangement. In an s-cis conformation, however, the two circled methyl groups experience steric strain by being too close to each other, preventing the molecule from adopting this conformation. Thus, Diels-Alder reaction doesn't occur. In order to undergo Diels-Alder reaction, this s-trans diene H3C CH, 5-C1S 5-trans 14.19 CH, CHo3 CH, I /CH 3 HqC or or HoC- ? H C^ "CH 2 CH 3 I H 4-Methyl- 1 ,3-hexadiene 3-Methyl-2,4-hexadiene H / + \ CH, I I CH2CH3 C "I H 2-Ethyl- 1 ,3-pentadiene CH 3 H Br \ / f Br" HgC C H3 H HoC 6 I I C CH2CHg CH2CHg + Br > HgC CH2CH3 C H H HgC HqC 3 I HoC C.s. Br \ / * C«^ CH2CHg H 2-Bromo-4-methyl-3-hexene 4-Bromo-4-methyl-2-hexene Conjugated Compounds and Ultraviolet Spectroscopy Additional Problems Conjugated Dienes 14.20 All of these (a> compounds can exhibit E/Z isomerism. (b) CH 3 I CH 3 CH=CCH=CHCH 3 H 2 C=CHCH =CHCH =CHCH 3 3-Methyl-2,4-hexadiene 1,3,5-Heptatriene (d) (c) CH 3 CH= C= CHCH= CHCH 3 CH 3 CH=CCH=CH 2 3-Propyl- 1 ,3-pentadiene 2,3,5-Heptatriene 14.21 Excluding double-bond CH2CH2CH 3 isomers: CH 3 Conjugated dienes: CH 3 CH= CHCH= CH 2 H2C — CHC — CH2 2-Methyl- 1 ,3-butadiene 1.3- Pcntadiene Cumulated dienes: CH 3 CH= C= CHCH 3 Ch^Ch^CH^- C^— CH2 2,3-Pentadiene 1,2-Pentadiene H2C — C — C(CH 3 3-Methyl- 1 ,2-butadiene Nonconjugated diene: — CHCH2CH — CH2 1.4- Pentadiene 14.22 a (a) 1 mol Br2 (b) 1. O. 2. Zn, H3 O. + H^O H (c) 1 mol HCI Ether )2 Br Br 327 328 Chapter 14 HO H enantiomers 14.23 CHo 5+1 BrCH 2 — C— CH— CH A tertiary/primary allylic CH 3 + Br 6+ * 2 addition to carbon H3 — C — CH _ — CH 2 Br+ I * 6+ CH 2 ? carbon 4 carbocation Br B addition to 1 6+ — C— *CH— CH 2 secondary/primary allylic carbocation A Tertiary/primary allylic carbocation is more stable than secondary/primary allylic carbocation B. Since the products formed from the more stable intermediate predominate, 3,4-dibromo-3-methyl-l-butene is the major product of 1,2 addition of bromine to isoprene. In both cases, the product with the more substituted double bond (1,4 addition product) predominates. 14.24 Any unsubstituted cyclic 1,3-diene cyclic diene gives the addition. For example: same product from 1,2- and 1,4 Conjugated Compounds and Ultraviolet Spectroscopy 329 14.25 2 1 3 4 CH=CHCH=CH2 1 Protonation of carbon -Phenyl- 1 ,3-butadiene ci 1: CH2CHCH _ — CH2 I 6+ CH2CH 5 CH + cr 3- Chloro-4-phenyl- CH2 1- butene CH 2 CH= CHCH 2 CI cr A allylic isomers 1 Protonation of carbon E/Z -Chloro-4-phenyl2- butene CI 2: CHCH2CH _ — CH2 + I CHCh^CH^— CH2 cr 4- Chloro-4-phenyl- 1-butene Protonation of carbon 3: + aCH— CH= CHCH 2 CH 2 CI CHCH2CH2 E/Z isomers cr 4-Chloro- 1 -phenyl1-butene C Protonation of carbon 4: 6+ ? CH=CHCHCH 3 cr 5+ E/Z isomers 3-Chloro- 1 -phenyl1- butene CH— CH— CHCH 3 CI I CHCH= CHCH 3 D allylic 1 D E/Z isomers cr -Chloro- 1 -pheny 12- butene Carbocation is most stable because it can use the x systems of both the benzene ring and the side chain to delocalize positive charge. 3-Chloro-l -phenyl- 1-butene is the major product because it results from cation and because its double bond can be conjugated with the benzene ring to provide extra stability. D 330 Chapter 14 Diels-Alder Reaction 14.26 If two equivalents of cyclohexadiene are present for each equivalent of dienophile, you can also obtain a second product: 14.27 This conformation of 2,3-di-te/t-butyl-l,3-butadiene, cis relationship, suffers in which the tert-butyl groups have a due to the bulky substituents. Instead, the conformation, which relieves the strain but does not allow from steric strain molecule adopts the s-trans Diels-Alder reaction to take place. 14.28 The diene rotates to the s-cis conformation. The in the dienophile is preserved in the product. trans relationship of the two ester groups Conjugated Compounds and Ultraviolet Spectroscopy 331 14.29 H H \ HoC / C=CH 2 C=CH 2 H c=c / / \ c=c\ H3 C H H ds-l,3-Pentadiene H trans- 1 ,3-Pentadiene Both pentadienes are more stable in s-trans conformations. To undergo Diels-Alder must rotate about the single bond between the double bonds assume s-cis conformations. reactions, however, they H \ H H \ / C=C \ / / —C to H / C=C\ H H \\ C— H c/ H ds-l,3-Pentadiene When H 3C trans- 1 ,3-Pentadiene ds-l,3-pentadiene rotates to the s-cis conformation, steric interaction occurs between the methyl-group protons and a hydrogen on CI. Since it's more difficult for 1,3-pentadiene to assume the s-cis conformation, it is less reactive in the Diels-Alder cis- reaction. 14.30 HC=CC=CH can't be used as a Diels-Alder diene because it is linear. The end carbons are too far apart to be able to react with a dienophile in a cyclic transition state. Furthermore, the product of Diels-Alder addition would be impossibly strained, with two sp-hybridized carbons in a six-membered ring. 14.31 och 2 ch3 och 2 ch 3 Two different orientations of the dienophile ester group are possible, and two different products can form. 332 Chapter 14 14.32 The most reactive dienophiles contain electron-withdrawing groups. Most reactive Least reactive (NC) 2 C=C(CN) 2 > Four electronwithdrawing groups H 2 C=CHCHO One > electron- withdrawing group The methyl groups of 2,3-dimethyl-2-butene H 2 C=CHCH 3 > (CH 3 2 ) One electron- C= C(CH 3 2 ) Four electrondonating groups donating group also decrease reactivity for steric reasons. 14.33 The difference in reactivity of the three cyclic dienes is due to steric factors. As the nondiene part of the molecule becomes larger, the carbon atoms at the end of the diene portion of the ring are forced farther apart. Overlap with the n system of the dienophile in the cyclic transition state is poorer, and reaction is slower. 14.34 Although an electron-withdrawing group increases the reactivity of a dienophile, it decreases the reactivity of a diene. 14.35 First, find the cyclohexene ring formed by the Diels-Alder reaction. After you locate the able to identify the diene and the dienophile. new bonds, you should then be A .0 O V bonds formed diene dienophile CN r diene dienophile A V 1 I o diene dienophile (d) C0 2 CH 3 C0 2 CH 3 ? H dienophile diene Conjugated Compounds and Ultraviolet Spectroscopy 333 Diene Polymers 14.36 A vinyl branch in a diene polymer is the result of an occasional polymer chain, rather than the usual polymerization for the same reason. the 1 ,4 addition. 1,2 double bond addition to Branching can also occur in cationic 14.37 Ozone causes oxidative cleavage of the double bonds in rubber and breaks the polymer chain. 14.38 Polycyclopentadiene to a is the product of successive Diels-Alder additions of cyclopentadiene growing polymer chain. Strong heat causes depolymerization of the chain and reformation of cyclopentadiene UV monomer units. Spectroscopy 14.39 Only compounds having alternating multiple bonds show st -* nt* ultraviolet absorptions in the 200-400 nm range. Of the compounds shown, only pyridine (b) absorbs in this range. 14.40 To absorb in the bonds of allene 200—400 nm range, an alkene must be conjugated. aren't conjugated, allene doesn't absorb light in the Since the double UV region. 334 Chapter 14 14.41 The value of in the ultraviolet spectrum of dienes becomes larger with increasing inversely related to ^max> the energy needed to produce ultraviolet absorption decreases with increasing substitution. energy substitution. Since is # of-CH3 groups Diene V ax ( nm ) Vax ~ Kim (butadiene) H H 2C* C ^ H H ? C^ 217 CH 2 CH 3 .C^ 1 220 3 1 223 6 2 226 9 2 227 10 232 15 240 23 H v r /C^ ^C^ CH2 C HgC H CHo3 I /C. H 2 C. CH 2 CH 3 H H I I .C^ .C^. . H 3 C^ I H CHq CHo3 I H CHq ,3,3 HoC C f CH' H CH 3 H I I ^C.% ^C. ^CH3 HoC 3 C I H Each 4 ^C I CH 3 alkyl substituent causes an increase in V of 3-6 nm. Conjugated Compounds and Ultraviolet Spectroscopy 335 14.42 H H I H CH, I <>^v. HoC^ 2 H2 C I ^. C C I I H H CH, 2,3-Dimethyl- 1 ,3,5-hexatriene 1,3,5-Hexatriene \nax = 258 CH 2 nm ^max ra 268 nm In Problem 14.41, we concluded that one alkyl group increases Xmax of a conjugated diene by approximately 5 nm. Since 2,3-dimethyl-l,3,5-hexatriene has two methyl substituents, its 14.43 (a) UV ^max should be about nm longer than the Xmax of 1 ,3,5-hexatriene. 10 B-Ocimene, Q0H16, has three degrees of unsaturation. Catalytic hydrogenation yields a hydrocarbon of formula C10H22. 6-Ocimene thus contains three double bonds and no rings. (b) (c) The ultraviolet absorption at 232 nm indicates that 6-ocimene The carbon skeleton, as determined from hydrogenation, is: is conjugated. CH 3 CH3 CHgCHpCHCHpCHpCHpCHCHg 2,6-Dimethyloctane Ozonolysis data are used to determine the location of the double bonds. The acetone fragment, which comes from carbon atoms 1 and 2 of 2,6-dimethyloctane, fixes the position of one double bond. Formaldehyde results from ozonolysis of a double bond at the other end of fi-ocimene. Placement of the other fragments to conform to the carbon skeleton yields the following structural formula for 8-ocimene. CHg CH3 H 2 C= CHC= CHCH 2 CH= CCH 3 p-Ocimene (d) I H 2 C= CHC= CHCH 2 CH= CCH 3 1. H2 1 3 H3 + I CH3CH 2 CHCH 2 CH 2 CH 2 CHCH3 Pd p-Ocimene 2. Zn, CHo3 CHo 9H3 CH,v 2,6-Dimethyloctane CH 3

(b) Phenol > benzene > (c) Aniline > benzene > bromobenzene > toluene > benzene > nitrobenzene chlorobenzene > benzoic acid benzaldehyde 1 in the text for the directing effects of substituents. You should the effects of the most important groups. As in Worked Example 16.2, identify the directing effect of the substituent, and draw the product. Refer to Figure 16. 1 memorize The -NO2 group is a meta-director. CI Chemistry of Benzene: Electrophilic Aromatic Substitution 367 Br No catalyst is necessary because aniline is highly 16.10 An acyl substituent is deactivating. reactive to further substitution. substituted ring is more An activating. Once an aromatic alkyl substituent ring has been acylated, is activating, it is much less however, so an alkyl- reactive than an unsubstituted ring, and polysubstitution occurs readily. 16.11 (Trifluoromethyl)benzene is less reactive toward electrophilic substitution than toluene. electronegativity of the three fluorine atoms causes the trifluoromethyl group to be electron-withdrawing and deactivating toward electrophilic substitution. The electrostatic potential map shows that the aromatic ring of (trifluoromethyl)benzene is more electronpoor, and thus less reactive, than the ring of toluene (red). The 16.12 more favored For acetanilide, resonance derealization of the nitrogen lone pair electrons to the aromatic ring is less favored because the positive charge on nitrogen is next to the positively polarized carbonyl group. Resonance derealization to the carbonyl oxygen is favored because of the electronegativity of oxygen. Since the nitrogen lone pair electrons are less available to the ring than in aniline, the reactivity of the ring toward electrophilic substitution is decreased, and acetanilide is less reactive than aniline toward electrophilic substitution. 368 Chapter 16 16.13 Ortho attack: fi H cf H — -,. i N II Least :o: :0: N N :o: stable Meta attack: : 't + : .? e N Para attack: E , \\ Least s / II II :0: :o: stable The x< , two positive charges thus favored. circled resonance forms are unfavorable, because they place adjacent to each other. The intermediate from meta attack is 16.14 (a) OCH< OCH ix Br Both groups are ortho, para directors and direct substitution two groups for steric reasons. to the same positions. Attack doesn't occur between the NH, Br E + Both groups are ortho, para directors, but direct to different positions. Because group is a more powerful activator, substitution occurs ortho and para to it. -NH2 Chemistry of Benzene: Electrophilic Aromatic Substitution 369 Both groups are deactivating, but they orient substitution toward the same positions. 16.15 N0 2 Although both groups are ortho, para directors, the methyl group directs the orientation of the substituents because it is a stronger activating group than bromine. N0 2 The methoxyl group directs substitution to the positions ortho and para to it. 370 Chapter 16 16.16 Hydroxide is used to form the nucleophilic phenoxide anion. OCH 2 CH 3 '•?.'• Oxyfluorfen Meisenheimer complex Step 1: Addition of the nucleophile. Step 2: Elimination of fluoride ion. The nitro group makes the ring electron-poor and vulnerable to attack by the nucleophilic It also stabilizes the negatively charged Meisenheimer complex. RCT group. 16.17 H- HoO "OH OH + Br Br" p-Methylphenol /7-Bromotoluene CH< CH< HoO "OH 6r- Br m-Bromotoluene + m-Methylphenol o-Methylphenol OH /n-Methylphenol CH. + Br" /?-Methylphenol Treatment of m-bromotoluene with NaOH leads to two possible benzyne intermediates, which react with water to yield three methylphenol products. Chemistry of Benzene: 16.18 Oxidation takes place at the Electrophilic Aromatic Substitution 371 benzylic position. /?-ter?-Butylbenzoic acid Treatment with untouched. KMn04 oxidizes the methyl group but leaves the terf-butyl group 16.19 Bond Bond dissociation energy CH 3 CH 2 —H .CH 2 421 kJ/mol —H 375 kJ/mol H 2 C=CHCH 2 —H 369kJ/mol Bond dissociation energies measure the amount of energy that must be supplied to cleave a bond into two radical fragments. A radical is thus higher in energy and less stable than the compound from which it came. Since the C-H bond dissociation energy is 421 kJ/mol for ethane and 375 kJ/mol for a methyl group C-H bond of toluene, less energy is required to form a benzyl radical than to form an ethyl radical. A benzyl radical is thus more stable than a primary alkyl radical by 46 kJ/mol. The bond dissociation energy of an allyl C-H bond is 369 kJ/mol, indicating that a benzyl radical is nearly as stable as an allyl radical. 16.20 Br 16.21 372 Chapter 16 16.22 (a) In order to synthesize the product with the correct orientation of substituents, benzene nitrated before it is chlorinated. must be HNO, Clc H 2 S0 4 FeCI< CI m-Chloronitrobenzene (b) Chlorine can be introduced into the correct position if benzene is first acylated. The chlorination product can then be reduced. . CHo CH2CH3 O II CH3CCI AICI3 m-Chloroethylbenzene followed by chlorination, reduction, and nitration, is the only route that gives a product in which the alkyl group and chlorine have a meta relationship. (c) Friedel-Crafts acylation, O II CH3 CH 2 CCI AICI< H2 Pd/C 4-Chloro- 1 -nitro-2-propy lbenzene pathway, remember that the ring must be sulfonated after Friedel-Crafts alkylation because a sulfonated ring is too deactivated for alkylation to occur. Performing the reactions in this order allows the first two groups to direct bromine (d) In planning this to the same position. SO3H — CHoCI AlClr 3-Bromo-2-methylbenzenesulfonic acid Chemistry of Benzene: 16.23 Electrophilic Aromatic Substitution (a) Friedel-Crafts acylation, like Friedel-Crafts alkylation, does not occur at an aromatic ring carrying a strongly electron-withdrawing group. (b) There are two problems with this synthesis as it is written: Rearrangement often occurs during Friedel-Crafts alkylations using primary 1 . halides. 2. Even if /?-chloropropylbenzene could be synthesized, introduction of the second -CI group would occur ortho, not meta, to the alkyl group. A possible route to this compound: , o Ch^CHg CH 2 CH 3 O^. Ch^Ch^CHg i 1 ii CH3 CH 2 CCI r AIGIg \ ci 2 Jl FeCI 3 T CI Visualizing Chemistry 16.24 (a) (1) The methoxyl group is an ortho-para director. ^^OCH ^S^OCHg OCH< 3 Br c FeBr, Br p-Bromomethoxybenzene (2) a OCH< CH3CCI CH< /7-Methoxyacetophenone (b) o-B romomethoxybenzene AICk Both functional groups direct substituents to the o-Methoxyacetophenone same position. (1) Br 2 FeBrc 3-Bromo-4-methylbenzaldehyde 373 374 Chapter 16 (2) H. H. O O 3-Acetyl-4-methylbenzaldehyde 16.25 In the lowest-energy conformation of this biphenyl, the aromatic rings are tilted. If the rings had a planar relationship, steric strain between the methyl groups and the ring hydrogens on the second ring would occur. Complete rotation around the single bond doesn't take place because the repulsive interaction between the methyl groups causes a barrier to rotation. 16.26 C0 2 H CH3 KMn0 4 CHoCI *-9 AlCIo OoN 16.27 Imagine two routes for synthesis of m-nitrotoluene: Alkylation of benzene, followed by nitration, doesn't succeed because an alkyl group is (1) an o,p-director. (2) Nitration of benzene, followed by alkylation, doesn't succeed because nitrobenzene is unreactive to Friedel-Crafts alkylation. Thus, it isn't possible to synthesize m-nitrotoluene by any route that we have studied in this chapter. Additional Problems Reactivity and Orientation of Electrophilic Substitutions 16.28 Group: (a) -j-N(CH 3 ) Identification: 2 e>,/7-activator Reason: Reaction intermediates are stabilized by electron donation by the amine (b) nitrogen. 0,p-activator Reaction intermediates are stabilized by the electron donating inductive effect of the alkyl group. 0,/?-activator Reaction intermediates are stabilized by electron donation by the ether oxygen. m-deactivator Reaction intermediates are destabilized by electron with- drawal by the carbonyl oxygen. Chemistry of Benzene: Electrophilic Aromatic Substitution 16.29 o-B romonitrobenzene /?-Bromonitrobenzene m-Dinitrobenzene m-Nitrobenzenesulfonic acid o-Methoxynitrobenzene Only methoxybenzene 16.30 Most reactive reacts faster than /?-Methoxynitrobenzene benzene (See Figure 16.1 1). > Least reactive Benzene > Chlorobenzene > o-Dichlorobenzene Phenol > Nitrobenzene > p-Bromonitrobenzene (c) o-Xylene > Fluorobenzene > Benzaldehyde (d) p-Methoxybenzonitrile > p-Methylbenzonitrile > Benzonitrile (a) (b) 375 Width: 612 Height: 792 376 Chapter 16 16.31 (a) a 1 ch 3 ci a:- AICk o-Bromotoluene ^s HON s /7-Bromotoluene H0\ .Br ^\ / Y< Br Y CH< 5-Bromo-2-methylphenol Both groups direct substitution to the same 3-Bromo-4-methylphenol position. (c) No reaction. A1C1 3 combines with form a complex -NH2 t0 that deactivates the ring toward Friedel-Crafts alkylation. (d) CH3CI No reaction. The ring is deactivated. AICI. CI (e) OH XX OH v XX CH3CI AICk cr CI ci 2,4-Dichloro-6-methylphenol C0 2 H (f) CH3CI AICk (g) No reaction.The ring is deactivated. No reaction.The ring is deactivated. ^\^S0 Hr fir 3 7T CH3CI AICk (h) CH< Br 1 ,4-Dibromo-2,5-dimethylbenzene Alkylation occurs in the indicated position because the methyl group is more activating than bromine, and because substitution rarely takes place between two groups. Electrophilic Aromatic Substitution Chemistry of Benzene: 16.32 OsN^^Ts^OH CI, FeCI- v XJ 2N * ^ cr 4-Chloro-3-nitrophenol The -OH group ci 2-Chloro-5-nitrophenol directs the orientation of substitution, (b) ^s.CH CH, 3 ci. FeCI, CH, CH 3 CI 4-Chloro-l,2dhnethylbenzene (c) CI Clr W\/ l-Chloro-2,3dimethylbenzene C ° 2H C0 2 H FeCI, 2N 2N 3-Chloro-4-nitro- benzoic acid Both groups are deactivating to a similar extent, 2-Chloro-4-nitrobenzoic acid and both possible products form. S0 3 H CI, FeCI- 4-B romo-3 -chlorobenzenesulfonic acid 16.33 (a) SO, H 2 S0 4 HO3S v p-Fluorobenzenesulfonic acid v SO3H o-Fluorobenzenesulfonic acid (b) H 2 S0 4 HO3S SO3H 2-Bromo-4-hydroxy- 4-Bromo-2-hydroxy- benzenesulfonic acid benzenesulfonic acid 377 378 Chapter 16 (c) CI SO, H 2 S0 4 H0 3 \^^ CI S^^ 2,4-Dichlorobenzenesulfonic acid (d) A S0 3 H OH SO, H 2 S0 4 Br Br " Br 3,5-Dibromo-2-hydroxybenzenesulfonic acid > 16.34 Most reactive Least reactive Phenol > Toluene > p-Bromotoluene > Bromobenzene Aniline and nitrobenzene don't undergo Friedel-Crafts alkylations. 16.35 (a) f? Pd NH 2 NOc o-Ethylaniline Catalytic hydrogenation reduces both the aromatic ketone and the nitro group. (b) Br ^\ 1 |1 HNO3, H 2 SQ 4 2. Fe, H3 + Br 3,4-Dibromoaniline Nitration, followed by reduction with Fe, produces substituted KMnO, H2 or C0 2 H oBenzenedicarboxylic acid (Phthalic acid) Aqueous 2,3-Dibromoaniline KMn04 oxidizes alkyl side chains to benzoic acids. anilines. Chemistry of Benzene: Electrophilic Aromatic Substitution 379 (d) 5-Chloro-2-methoxypropylbenzene 4-Chloro-2-isopropyl- methoxybenzene directs substitution because it is a more powerful activating group. Rearranged and unrearranged side chains are present in the products. The methoxyl group 16.36 (d) '^Tss .N(CH 2 CH 3 ) 2 N(CH 2 CH 3 2 ) SO N(CH 2 CH 3 2 ) + H 2 S0 4 H0 3 S SO3H 380 Chapter 16 Mechanisms of Electrophilic Substitutions 16.37 1 ^ : |_|^ Base H 6+ 8" — CI because chlorine is a more electronegative element than IC1 can be represented as I iodine. Iodine can act as an electrophile in electrophilic aromatic substitution reactions. 16.38 S0 3 H k H fa H— OSO3H 2> + is the reverse of the sulfonation S0 3 H H + "OSO3H This mechanism + mechanism illustrated in the text. H+ is the electrophile in this reaction. 16.39 H— OPO3H2 CH 3 C= CH 2 (CH 3 3 C+ ) C(CH 3 3 + ~OP0 3 H 2 ) C(CH 3 ): V :Base Phosphoric acid protonates 2-methylpropene, forming a tert-butyl carbocation. This carbocation acts as an electrophile in a Friedel-Crafts reaction to yield rm-butylbenzene. Chemistry of Benzene: 16.40 When an Electrophilic Aromatic Substitution electrophile reacts with an aromatic ring bearing a 381 + (CH3)3N - group: This is a destabilizing resonance form because two positive charges are next to each other. Meta attack: This form is destabilizing. The AWAf-trimethylarnmonium group has no electron-withdrawing resonance effect because it has no vacant p orbitals to overlap with the n orbital system of the aromatic The (CH3)3N +- group is inductively deactivating, however, because it is positively charged. It is ring. meta-directing because the cationic intermediate resulting from meta attack somewhat more 16.41 The stable than those resulting from ortho or para is attack. aromatic ring is deactivated toward electrophilic aromatic substitution by the combined electron-withdrawing inductive effect of electronegative nitrogen and oxygen. The lone pair of electrons of nitrogen can, however, stabilize by resonance the ortho and para substituted intermediates but not the meta intermediate. Ortho attack: 382 Chapter 16 Meta attack: :o: :o: HE HE :o: HE HE 16.42 CHCI 3 + AICI3 « * + CHCI 2 AICI4" (Dichloromethyl)benzene (Dichloromethyl)benzene can react with two additional equivalents of benzene by the same to produce triphenylmethane. mechanism Chemistry of Benzene: Electrophilic Aromatic Substitution 383 16.43 structures show that bromination occurs in the ortho and para positions of the The positively charged intermediate formed from ortho or para attack can be stabilized by resonance contributions from the second ring of biphenyl, but this Resonance rings. stabilization is not possible for meta attack. 384 Chapter 16 16.44 HO— OH — acid , , » catalyst HO— OHo* reactive electrophile PH HO-OHo H ^ :OH 2 OH Attack of jt electrons Loss of proton on reactive electrophile The reactive electrophile (protonated H2O2) is equivalent to + OH. Organic Synthesis 16.45 C0 2 H CH 3 IH 3 CH3C1 FeBr< AlCIo oBromobenzoic (b) acid OH CH3CI 1 AlCIo 2. NaOH CH 3 Br CH 3 p-Methoxytoluene The reactions in (b) can be performed in either order, (c) CH< C0 2 H CH< N0 2 OoN 2N CH3CI HNO< KMnO/i AlCIo H 2 S0 4 HoO NOc N0 2 2,4,6-Trinitrobenzoic acid Chemistry of Benzene: Electrophilic Aromatic Substitution 385 m-Bromoaniline 16.46 When synthesizing substituted aromatic rings, it is necessary to introduce substituents in group that is introduced out of order will not have the proper directing the proper order. effect. A Remember that in many of these reactions a mixture of ortho and para isomers be formed. /7-Chloroacetophenone ra-Bromonitrobenzene THF H 2 2 "OH , X CHoCHo' is a substituted phenol, whose -OH group directs the orientation of the -C(CH3)3 groups. The precursor to MON-0585 is synthesized by a Friedel-Crafts 16.56 The product alkylation of phenol by by the appropriate hydrocarbon halide. This compound is synthesized NBS bromination of the product of alkylation of benzene with 2-chloropropane. MON-0585 C(CH 3) 3 Electrophilic Aromatic Substitution Chemistry of Benzene: 391 16.57 + (1) ' OH :OH ^H— OS03 H 11 H H' Formaldehyde is H H + H protonated to form a carbocation. (2) H ? clv CI- CH 2 OH Ylb) CH 2 OH Hv:Base CI jf^ CI cation acts as the electrophile in a substitution reaction at the "6" position of 2,4,5-trichlorophenol. The formaldehyde H CH 2 — OH 2 CI CI H-rOS0 3 H CI CI The product from CI step 2 is + ~:OS0 3 Hj protonated by strong acid to produce a cation. (4) Hexachlorophene This cation is attacked by a second molecule of 2,4,5-trichlorophenol to produce hexachlorophene. 392 Chapter 16 16.58 :o: II heat a? 9 N=N Benzyne + The boron atom C0 2 + N2 phenylboronic acid has only six outer-shell electrons and is a resonance forms for phenylboronic acid in which an electron pair from the phenyl ring is delocalized onto boron. In these resonance forms, the ortho and para positions of phenylboronic acid are the most electron-deficient, and substitutions occur primarily at the meta position. trivalent Lewis in acid. It is possible to write Electrophilic Aromatic Substitution Chemistry of Benzene: 16.60 Resonance forms Br for the intermediate H Br Br from attack at H C 1 Br H Br 393 H Br H Br H H + Resonance forms for the intermediate Br from attack Br at C2: Br + Br There are seven resonance forms for attack at CI and six for attack at C2. Look carefully at the forms, however. In the first four resonance structures for CI attack, the second ring is still fully aromatic. In the other three forms, however, the positive charge has been delocalized into the second ring, destroying the ring's aromaticity. For C2 attack, only the first two resonance structures have a fully aromatic second ring. Since stabilization is lost when aromaticity is disrupted, the intermediate from C2 attack is less stable than the intermediate from CI attack, and CI attack is favored. 394 Chapter 16 16.61 I :OCH 3 :o: ci OCH 3 L. Meisenheimer complex Step 1: Addition of the nucleophile -OCH3. Step 2: Elimination of -Cl~. The carbonyl oxygens make the chlorine-containing ring electron-poor and open to attack by the nucleophile ""OCH3. They also stabilize the negatively charged Meisenheimer complex. 16.62 •:Base + : cr N(CH 3) 2 (CH 3 2 NH ) 3. 2. N N' Step 1: Attack of the nucleophile diethylamine. Step 2: Loss of proton. Step 3: Loss of CI". This reaction is an example of nucleophilic aromatic substitution. Dimethylamine a is nucleophile, and the pyridine nitrogen acts as an electron- withdrawing group that can stabilize the negatively-charged intermediate. 16.63 Step Step The 1: 1: Abstraction of proton and elimination of Br Addition of NH3 to the benzyne intermediate to form two aniline products. . reaction of an aryl halide with potassium amide proceeds through a benzyne Ammonia can then add to either end of the triple bond to produce the intermediate. methylanilines observed. two Chemistry of Benzene: Electrophilic Aromatic Substitution 16.64 Protonation of the cyclic ether creates a carbocation intermediate that can react in a Friedel-Crafts alkylation. OH The intermediate alkylates benzene, forming an alcohol product. Protonation of the alcohol, followed by loss of water, generates a second carbocation. (d) This carbocation undergoes internal alkylation to yield the observed product. 395 Width: 612 Height: 792 396 Chapter 16 16.65 Step 1: Formation of primary carbocation. Step 2: Rearrangement to a secondary carbocation. Step 3: Attack of ring n electrons on the carbocation. + Step 4: Loss of H . This reaction takes place despite the fact that an electron- withdrawing group the ring. Apparently, the cyclization reaction is strongly favored. is 16.66 W_ .. :C=0 + * Carbon monoxide The HCI,AICI 3 is » : H— C=0+ " AICI 4 protonated to form an acyl cation. acyl cation reacts with benzene by a Friedel-Crafts acylation mechanism. attached to Electrophilic Aromatic Substitution Chemistry of Benzene: 397 16.67 OH H H f~ : Base + (CH 3 ) 2 C JC3H 2 (CH 3 2 C^~ CH2 ) Loss of H C(CH 3) 3 ) 3 proton Loss of Protonation of aromatic ring 16.68 Both C(CH 3 tert-butyl carbocation of these syntheses test ^ your ability to carry out steps in the correct order. CHo CHo CH< Br HNO 3^ H 2 S0 4 CH 3 CI AlCIo Br 2 FeBrc N0 2 NOc + ortho isomer (b) 0>, C ^CH(CH 3 ) 2 c o II (CH 3 2 CHCQ * AiCk ) FeCI CH 2 CH(CH 3 ) 2 S0 3 H CI ^CH(CH 3 ) 2 398 Chapter 16 16.69 Problem 16.51 shows the mechanism of the addition of HBr to 1-phenylpropene and shows how the aromatic ring stabilizes the carbocation intermediate. For the methoxylform can be drawn in which the cation is by the electron-donating resonance effect of the oxygen atom. For the nitrosubstituted styrene, the cation is destabilized by the electron-withdrawing effect of the substituted styrene, an additional resonance stabilized nitro group. Thus, the intermediate resulting from addition of HBr to the methoxyl- substituted styrene is more stable, and reaction of /7-methoxystyrene is faster. 16.70 HoC — S — CH (CH 3 2 S ) arc - H 2. Base + Br 1: Sn2 displacement takes place when the negatively charged oxygen of dimethyl sulfoxide attacks the benzylic carbon of benzyl bromide, displacing Br~. Step Step 2: Base removes a benzylic proton, and dimethyl sulfide is eliminated in an reaction. 16.71 NH, H = 1.52 D strong electron-withdrawing -NH 2 has a strong inductive effect. resonance n = 1.53 D -Br has a electron-donating effect. NH, Br [i = D 2.91 The polarities of the two groups add to produce a net dipole moment almost equal to the sum of the individual moments. E2 Chemistry of Benzene: Electrophilic Aromatic Substitution 399 16.72 O (a) CH3CH2COCI, AICI3; (b) H2 , Pd/C; (c) Br2 FeBr 3 , ; (d) NBS, (PhC0 2 ) 2 ; (e) KOH, ethanol 16.73 An electron-withdrawing substituent destabilizes a positively charged intermediate (as in electrophilic aromatic substitution) but stabilizes a negatively charged intermediate. dissociation of a phenol, an For the -N02 group stabilizes the phenoxide anion by resonance, thus lowering AG and pKa In the starred resonance form for /7-nitrophenol, the negative charge has been delocalized onto the oxygens of the nitro group. . 16.74 For the same reason described in the previous problem, a methyl group destabilizes the AG° and pA^, making this phenol less acidic. negatively charged intermediate, thus raising Review Unit 6: Conjugation and Aromaticity Major Topics Covered (with vocabulary): Conjugated dienes: derealization 1 ,4-addition allylic position thermodynamic control kinetic control vulcanization Diels-Alder cycloaddition dienophile endo product exo product s-cis conformation Ultraviolet spectroscopy: highest occupied molecular orbital absorptivity (HOMO) lowest unoccupied molecular orbital Aromaticity: aromatic arene phenyl group benzyl group Hiickel 4/i + 2 rule antiaromatic heterocycle (LUMO) molar ortho, meta, para substitution degenerate polycyclic aromatic compound ring current Chemistry of aromatic compounds: sulfonation F-TEDA-BF4 Friedel-Crafts alkylation Friedel-Crafts acylation ortho- and para-directing activator ortho- and para- electrophilic aromatic substitution polyalkylation directing deactivator meta-directing deactivator aromatic substitution inductive effect resonance effect nucleophilic Meisenheimer complex benzyne benzylic position Types of Problems: After studying these chapters, you should be able to: - Predict the products of electrophilic addition to conjugated molecules. Understand the concept of kinetic vs. thermodynamic control of reactions. Recognize diene polymers, and draw a representative segment of a diene polymer. Predict the products of Diels-Alder reactions, and identify compounds that are good dienophiles and good dienes. Calculate the energy required for UV absorption, and use molar absorptivity to calculate concentration. - Predict - Name - if and where a compound absorbs in the ultraviolet region. and draw substituted benzenes. structures and molecular Draw resonance orbital diagrams for benzene and other cyclic conjugated molecules. Use Huckel's rule to predict aromaticity. Draw orbital pictures of cyclic conjugated molecules. data to deduce the structures of aromatic compounds. Use NMR, IR and UV Predict the products of electrophilic aromatic substitution reactions. Formulate the mechanisms of electrophilic aromatic substitution reactions. Understand the activating and directing effects of substituents on aromatic rings, and use inductive and resonance arguments to predict orientation and reactivity. Predict the products of other reactions of aromatic compounds. Synthesize substituted benzenes. Review Unit 6 Points to * It's 401 Remember: not always easy to recognize Diels-Alder products, especially bond of the product has been hydrogenated. if the carbon-carbon double no hydrogenation has taken place, look for a double bond in a six-membered ring and at least one electron-withdrawing group across the ring from the double bond. When a bicyclic product has been formed, it has probably resulted from a Diels-Alder reaction in which the diene is cyclic. * To be initial If aromatic, a molecule must be planar, cyclic, conjugated, and electrons in its it must have 4n + 2 n system. * The carbocation * Nucleophilic aromatic substitution reactions and substitution reactions proceeding through benzyne intermediates take place by different routes. In the first reaction, the substitution takes place by an addition, followed by an elimination. In the second case, the substitution involves an elimination, followed by an addition. Virtually all substitutions are equivalent to an addition and an elimination (in either order). * Activating groups achieve their effects by making an aromatic ring more electron-rich and reactive toward electrophiles. Ortho and para directing groups achieve their effects by intermediate of electrophilic aromatic substitution loses a proton to yield the aromatic product. In all cases, a base is involved with proton removal, but the nature of the base varies with the type of substitution reaction. Although this book shows the loss of the proton, it often doesn't show the base responsible for proton removal. This doesn't imply that the proton flies off, unassisted; it just means that the base involved has not been identified in the problem. from ortho or para addition of an electrophile to the aromatic ring. The intermediate resulting from addition to a ring with an ortho or para director usually has one resonance form that is especially stable. The intermediate resulting from addition to a ring with a meta director usually has a resonance form that is especially unfavorable when addition occurs ortho or para to the functional group. Meta substitution results because it is less unfavorable than ortho or para substitution. stabilizing the positive charge that results Self-test: a-Farnesene a-Farnesene (A), an important biological intermediate in the synthesis of many natural products, has double bonds that are both conjugated and unconjugated. Show the products you would expect from conjugate addition of HBr; of Eto. What products would you expect from ozonolysis of A? Give one or more distinctive absorptions that you might see in the IR spectrum of A and distinguishing features of the H NMR of A. Would you expect A to be UV-active? ] Review Unit 6 402 C B D Paroxypropione Describe the n orbitals in the ring of B. Might this ring be described as aromatic? Paroxypropione (C) is a hormone inhibitor. Predict the products of reaction of C with: (a) Br2 , FeBr 3 (b) CH3CI, A1C1 3 (c) KMn0 4 , H3O; (d) H 2 Pd/C. If the product of (d) is treated with the reagents in (a) or (b), does the orientation of substitution change? What significant information can you obtain from the IR spectrum of C? Name D. Plan a synthesis of D from benzene. Describe the *H of D (include spin-spin splitting). Where might D show an absorption in a UV spectrum? ; , ; NMR Multiple choice: 1 . What (a) 2. are the hybridizations of the carbons in sp 2 , sp 2 , sp 2 , sp 2 more (c) which the sp 2 sp 2 sp 2 sp 3 , , , 1 ,2-butadiene, starting with (c) sp 2 sp, sp 2 sp 3 , , CI? (d) sp, sp, sp 2 sp 3 , product is formed at lower temperature, and the formed at higher temperature: AGras ° > AGi s ° and AGms * > AGis* (b) AG ms ° > AGi s ° and AGi s * > AGms * ° ° * < and AG > * AGms (d) AGms ° < AGi s ° and AGi s * > AGms * AGi s AGi s ms In a reaction in (a) (b) less stable (Is) stable product (ms) is Note: In this problem, a large value for AG° means a large ne gative value. 3 . Which of the following combinations is most likely to undergo a successful Diels-Alder reaction? (a) 4 . (b) Which of the following (d) (c) groups, when bonded to the terminal carbon of a conjugated k system, probably affects the value of Xmax the least? (a)-NH 2 (b)-Cl (c)-OH (d)-CH 3 5 . value of Xmax for an unsubstituted diene is approximately 220 nm, and each additional double bond increases the value of Xmax by 30 nm, what is the minimum number of double bonds present in a compound that absorbs in the visible range of the electromagnetic spectrum? (a) 6 (b)7 (c)8 (d)9 If the Review Unit 6 6 . 7 . Which of the following compounds 9 . . 10 (b) 11 (c) 12 (d) 14 Which of the following functional groups isn't (a)-N0 2 (b)-CONHCH 3 (c)-N(CH 3 ) 3 + Which of the following compounds substitution reaction that (a) 10. aromatic? How many benzene isomers of CvH6Br2 can be drawn? (a) 8 is m-Cresol we have can't a meta-directing deactivator? (d) -NHCOCH3 be synthesized by an electrophilic aromatic studied? (b) p-Chloroaniline (c) 2,4-Toluenedisulfonic acid compounds can you reduce Which compound is it? In only one of the following reducing the side chain. (a) p-Bromoanisole (d) Phenylacetylene (b) (d) m-Bromotoluene the aromatic ring without also Acetophenone (methyl phenyl ketone) (c) Styrene 403 Chapter 17 - Alcohols and Phenols Chapter Outline I. Naming alcohols and phenols (Section 17.1). A. Alcohols are classified as primary, secondary or organic groups bonded to the -OH carbon. B Rules for naming simple alcohols. 1 The longest chain containing the -OH group tertiary, depending on the number of . . name replaces 2 . 3 . is the parent chain, and the parent -e with -ol. Numbering begins at the end of the chain nearer the -OH group. The substituents are numbered according to their position on the chain and cited in alphabetical order. II. C. Phenols are named according to rules discussed in Section 15.1 for aromatic compounds. Properties of alcohols and phenols (Section 17.2). A. Hydrogen-bonding of alcohols and phenols. 1 Alcohols have sp hybridization and a nearly tetrahedral bond angle. 2. Alcohols and phenols have elevated boiling points, relative to hydrocarbons, due to hydrogen-bonding a. In hydrogen-bonding, an -OH hydrogen is attracted to a lone pair of electrons on another molecule, resulting in a weak electrostatic force that holds the . molecules together. These weak forces must be overcome in boiling. Acidity and basicity of alcohols and phenols. 1 Alcohols and phenols are weakly acidic as well as weakly basic. 2 Alcohols and phenols can be reversibly protonated to form oxonium ions. Alcohols and phenols dissociate to a slight extent to form alkoxide ions and 3 phenoxide ions. b B . . . . 4. Acidity of alcohols. a. b. c. Alcohols are similar in acidity to water. Alkyl substituents decrease acidity by preventing solvation of the alkoxide ion. Electron- withdrawing substituents increase acidity by delocalizing negative charge. d. 5 . Alcohols don't react with weak bases, but they do react with alkali metals and strong bases. Acidity of phenols. Phenols are a million times a. more acidic than alcohols and are soluble in dilute NaOH. b. c. III. Phenol acidity is due to resonance stabilization of the phenoxide anion. Electron-withdrawing substituents increase phenol acidity, and electrondonating substituents decrease phenol acidity. Alcohols (Sections 17.3-17.8). A. Preparation of alcohols (Sections 17.3-17.5). 1 . Familiar methods (Section 17.3). a. Hydration of alkenes. i. Hydroboration/oxidation yields non-Markovnikov products. ii. Oxymercuration/reduction yields Markovnikov products. b. 1,2-diols can be prepared by OSO4 hydroxylation, followed by reduction. This reaction occurs with syn stereochemistry. i. Ring-opening of epoxides produces 1,2-diols with anti stereochemistry. ii. Alcohols and Phenols 2. 405 Reduction of carbonyl compounds (Section 17.4). Aldehydes are reduced to primary alcohols. a. b. Ketones are reduced to secondary alcohols. Either NaBH4(milder) or LiAlH4(more reactive) can be used to reduce 1 aldehydes and ketones. c. Carboxylic acids and esters are reduced to primary alcohols with LLA.1H4. i. These reactions occur by addition of hydride to the positively polarized carbon of a carbonyl group. ii. Water adds to the alkoxide intermediate during workup to yield alcohol . 3 . product. Reaction of carbonyl compounds with Grignard reagents (Section 17.5). a. RMgX adds to carbonyl compounds to give alcohol products. i. Reaction of RMgX with formaldehyde yields primary alcohols. ii. Reaction of RMgX with aldehydes yields secondary alcohols. iii. Reaction of RMgX with ketones yields tertiary alcohols. iv. Reaction of RMgX with esters yields tertiary alcohols with at least two identical R groups bonded to the alcohol carbon. v No reaction occurs with carboxylic acids because the acidic hydrogen quenches the Grignard reagent. b Limitations of the Grignard reaction. Grignard reagents can't be prepared from reagents containing other reactive i. . . functional groups. Grignard reagents can't be prepared from compounds having acidic hydrogens. Grignard reagents behave as carbon anions and add to the carbonyl carbon, A proton from water is added to the alkoxide intermediate to produce the i. ii. c. alcohol. B. Reactions of alcohols (Sections 17.6-17.8). 1 Conversion to alkyl halides (Section 17.6). a. Tertiary alcohols (ROH) are converted to RX by treatment with HX. The reaction occurs by an SnI mechanism. i. b. Primary alcohols are converted by the reagents PBr3 and SOCI2. i. The reaction occurs by an Sn2 mechanism. 2 Conversion into tosylates. a. Reaction with /?-toluenesulfonyl chloride converts alcohols to tosylates. b Only the O-H bond is broken. . . . Tosylates behave as halides in substitution reactions. d Sn2 reactions involving tosylates proceed with inversion of configuration. Dehydration to yield alkenes. a. Tertiary alcohols can undergo acid-catalyzed dehydration with warm aqueous c. . 3 . H2SO4. i. ii. iii. Zaitsev products are usually formed. The severe conditions needed for dehydration of secondary and primary alcohols restrict this method to tertiary alcohols. Tertiary alcohols react fastest because the intermediate carbocation formed in this El reaction is more stable. Secondary alcohols are dehydrated with POCI3 in pyridine. This reaction occurs by an E2 mechanism. i. ii. Pyridine serves both as a base and as a solvent. Conversion into esters. Oxidation of alcohols (Section 17.7). a. Primary alcohols can be oxidized to aldehydes or carboxylic b Secondary alcohols can be oxidized to ketones. b. 4 . 5 . . acids. Width: 612 Height: 792 406 Chapter 17 c. Tertiary alcohols aren't oxidized. d. Oxidation to ketones and carboxylic acids can be carried out with KMnC>4, Cr0 3 , or Na 2 Cr2 7 Oxidation of a primary alcohol to an aldehyde is achieved with the Dess-Martin . e. periodinane. The Dess-Martin periodinane is also used on sensitive alcohols. Oxidation occurs by a mechanism closely related to an E2 mechanism. Protection of alcohols (Section 17.8). a. It is sometimes necessary to protect an alcohol when it interferes with a reaction involving a functional group in another part of a molecule. b The following reaction sequence may be applied: i. f 5. . . Protect the alcohol. i. Carry out the reaction. ii. iii. A c. Remove the protecting group. trimethylsilyl (TMS) ether can be used for protection. i. ii. iii. TMS ether formation occurs by an Sn2 route. TMS ethers are quite unreactive. TMS ethers can be cleaved by aqueous acid or by F~ to regenerate the alcohol. IV. Phenols (Sections 17.9-17.10). A. Preparation and uses of phenols (Section 17.9). Phenols can be prepared by treating chlorobenzene with NaOH. Phenols can also be prepared from isopropylbenzene (cumene). a. Cumene reacts with 2 by a radical mechanism to form cumene hydroperoxide. b Treatment of the hydroperoxide with acid gives phenol and acetone. The mechanism involves protonation, rearrangement, loss of water, i. readdition of water to form a hemiacetal, and breakdown to acetone and phenol. 3 Chlorinated phenols, such as 2,4-D, are formed by chlorinating phenol. 4. BHT is prepared by Friedel-Crafts alkylation of /7-cresol with 2-methylpropene. B. Reactions of phenols (Section 17.10). 1 Phenols undergo electrophilic aromatic substitution reactions (Chapter 16). a. The -OH group is a o,/?-director. 2. Strong oxidizing agents convert phenols to quinones. Reaction with Fremy's salt to form a quinone occurs by a radical mechanism. a. b The redox reaction quinone -* hydroquinone occurs readily. c Ubiquinones are an important class of biochemical oxidizing agents that function as a quinone/hydroquinone redox system. V. Spectroscopy of alcohols and phenols (Section 17.1 1). A. IR spectroscopy. -1 1 Both alcohols and phenols show -OH stretches in the region 3300-3600 cm -1 a. Unassociated alcohols show a peak at 3600 cm -1 b Associated alcohols show a broader peak at 3300-3400 cm -1 2. Alcohols show a C-0 stretch near 1050 cm -1 3 Phenols show aromatic bands at 1500-1600 cm -1 4. Phenol shows monosubstituted aromatic bands at 690 and 760 cm 1 . 2. . . . . . . . . . . . . . . B . NMR spectroscopy. In C NMR spectroscopy, carbons bonded to -OH groups absorb in the range 1 1 . 2. 50-80 6. *H NMR. a. Hydrogens on carbons bearing -OH groups absorb in the range 3.5-4.5 The hydroxyl hydrogen doesn't split these signals. i. b. D2 exchange can be used to locate the O-H signal. 6. 407 Alcohols and Phenols Spin-spin splitting occurs between protons on the oxygen-bearing carbon and neighboring -H. d Phenols show aromatic ring absorptions, as well as an O-H absorption in the range 3-8 6. C. Mass Spectrometry. Alcohols undergo alpha cleavage to give a neutral radical and an oxygen-containing 1 c. . . cation. 2 . Alcohols also undergo dehydration to give an alkene radical cation. Solutions to Problems 17.1 The parent chain must contain the hydroxyl group, and the hydroxyl group(s) should receive the lowest possible number. (a) OH OH I (b) (c) OH HO I CH3CHCH2CHCHCH3 ^^^s^^^* CH2CH2CCH3 CHo CH< CH 3 CH 3 5 -Methy 1-2 ,4-hexanediol (d) 2-Methy l-4-phenyl-2-butanol (e) 4,4-Dimethylcyclohexanol (f) HoC --Br or H (1 5,25)-2-Bromocyclopentanol 4-Bromo-3-methylphenol 2-Cyclopenten- 1 -ol 17.2 (a) HoC \ / (c) CHoOH 2 c=c / H \ CH2CH2 (Z)-2-Ethyl-2-buten- 1 -ol 3-Cyclohexen- 1 -ol rrafls-3-Chlorocycloheptanol and enantiomer (<*) OH (e) (f) I CH3CHCH2CH2CH2OH H3C ^^^ 1,4-Pentanediol CH 2CH 2 OH CH3 2,6-Dimethylphenol ^^ o-(2-Hydroxyethyl)phenol 408 Chapter 17 17.3 In general, the boiling points of a series of isomers decrease with branching. The more nearly spherical a compound becomes, the less surface area it has relative to a straight A chain compound of the same molecular weight and functional group type. smaller surface area allows fewer van der Waals interactions, the weak forces that cause covalent molecules to be attracted to each other. In addition, branching in alcohols makes it more difficult for hydroxyl groups to approach each other to form hydrogen bonds. given volume of 2-methyl-2-propanol therefore contains fewer hydrogen bonds than the same volume of 1-butanol, and less energy is needed to break them in boiling. A 17.4 Most acidic Least acidic (a) HC=CH < (CH 3 2 CHOH < ) alkyne hindered < (CF 3) 2 CHOH CH 3 OH alcohol with electron- alcohol withdrawing groups alcohol < (b) /7-Methylphenol Phenol /?-(Trifluoromethyl)phenol phenol with electrondonating groups (c) phenol with electronwithdrawing groups < Benzyl alcohol 17.5 < Phenol phenol alcohol /?-Hydroxybenzoic acid carboxylic acid We saw in Chapter 16 that a nitro group is electron- withdrawing. Since electronwithdrawing groups stabilize anions, p-nitrobenzyl alcohol is more acidic than benzyl alcohol. The methoxyl group, which is electron-donating, destabilizes an alkoxide ion, making p-methoxybenzyl alcohol less acidic than benzyl alcohol. 17.6 (a) CH3CH2 / CHod 1 c=c\ / . 2. CHo3 BH 3 THF , H2 2 , I CH3CH2CHCHCH3 OH CHo H OH 2-Methyl-3-pentanol In a hydroboration/oxidation reaction, the hydroxyl group is bonded to the less substituted H3C 1 Hg(OAc) 2 H 2 Q , 2. NaBH 4 2-Methyl-4-phenyl-2-butanol Markovnikov product results from oxymercuration/reduction. (c) H \ HO H / 1 \ 2. Q— Q / C4H9 C4H9 . Os0 4 NaHS0 3 H 2 \ OH / ,-C— C-, u A , = k C4H9 C4H9 meso-5 ,6-Decanediol Hydroxy lation results in a diol with syn stereochemistry. HO P4.H0 \ ijvr ,C— H'A C4H9 \ OH Alcohols and Phenols 409 17.7 (a) O O II II OH 1.NaBH 4 CH 3 CCH 2 CH 2 COCH3 NaBH4 reduces ( H3 2. O II I CH 3 CHCH 2 CH 2 COCH3 + aldehydes and ketones without interfering with other functional groups. b) OH 1 II II LiAIH 4 . ~~ CH 3 CCH 2 CH 2 COCH 3 d. H3 /-«+ I * CH 3 CHCH 2 CH 2 CH 2 OH LiAlH4, a stronger reducing agent, reduces both ketones and esters, (c) )H 1 UAIH4 2. H3 + LiAlH4 reduces carbonyl functional groups without reducing double bonds. 17.8 f? OH ^ ' Benzyl alcohol may be the reduction product of an aldehyde, a carboxylic NaBH4 may be used to reduce the aldehyde. acid, or an ester. (b) C" [^jf CH 3 1 . LiAIH, + 2H3 Reduction of a ketone yields the secondary alcohol. NaBH4 may also be used here and in (c). OH (c) LiAIHj^ + 2 -H 3 1 ( r T— d) 1 (CH 3 2 CHCHO or ) (CH 3) 2 CHC0 2 H or (CH 3 2 CHC0 2 R . UAIH4 (CH 3 ) 3 ) 2 CHCH 2 OH 410 Chapter 17 17.9 All of the products have an ketone carbon. 17.10 type of alcohol. If the alcohol is primary, it can only be synthesized from formaldehyde plus the appropriate Grignard reagent. If the alcohol is secondary, it is synthesized from an aldehyde and a Grignard reagent. (Usually, there are two combinations of aldehyde and Grignard reagent). A tertiary alcohol is synthesized from a ketone and a Grignard reagent. If all three groups on the tertiary alcohol are different, there are often three different combinations of ketone and Grignard reagent. If two of the groups on the alcohol carbon are the same, the alcohol may also be synthesized from an ester and two equivalents of Grignard reagent. -OH and a methyl group bonded to what was formerly a First, identify the (a) 2-Methyl-2-propanol is a tertiary alcohol. To synthesize a tertiary alcohol, start with a ketone. OH O II CHoCCHo d d 1 . CH 3 MgBr_ 2.H 3 t + ^ ' CHoCCHo d d | CH 3 two or more alkyl groups bonded to the carbon bearing the -OH group are the same, an alcohol can be synthesized from an ester and a Grignard reagent. If OH O II CH 3 COR 1 . 2 CH 3 MgBr 2.H 3 t + ^ CHoCCHo d 6 j | CH 3 2-Methyl-2-propanol Alcohols and Phenols (b) Since 1-methylcyclohexanol is a tertiary alcohol, start 411 with a ketone. .0 1 2. CH 3 MgBr H3 + -Methylcyclohexanol 1 (c) 3-Methyl-3-pentanol is a tertiary alcohol. When two of the three groups bonded to the alcohol carbon are the same, either a ketone or an ester can be used as a starting material. II 1 CH3CH2CCH2CH3 . £ CH 3 MgBr |_| 0+ OH or I CH3CH2CCH2CH3 1 CH 3 CH 2 CCH 3 . CH 3 CH 2 MgBr^ ~l CH 3 2 H ^ 0+ 3-Methyl-3-pentanol or O 1 CH3COR (d) . 2. 2 CH 3 CH 2 MgBr + H3 Three possible combinations of ketone plus Grignard reagent can be used to synthesize this tertiary alcohol. CH 2 CH 3 i 1 2. CHa MgBr H3 + > HO or CH 1 . 2. or CH 3 CH 2 MgB H3 + II 1 2. C 6 H 5 MgB H3 + v U CH2CH3 2-Phenyl-2-butanol o CH3CH2CCH3 CHo6 \ / r 412 Chapter 17 (e) Formaldehyde must be used to synthesize this primary alcohol, ^s^CH2OH f? C 6 H 5 MgBr 2. H3 H 1 H + Benzyl alcohol (f) As in (e), use formaldehyde B H' 17.11 1 . primary alcohol. (CH 3 2 CHCH 2 CH 2 MgBr ) 2.H 3 H to synthesize a (CH 3 2 CHCH2CH2CH 2 OH + ) 4-Methyl- 1 -pentanol of the alcohol. This alcohol, 1-ethylcyclohexanol, is a tertiary alcohol that can be synthesized from a ketone. Only one combination of ketone and Grignard reagent is possible. First, interpret the structure OH 1 2. 17.12 CH 3 CH 2 MgBr H3 aCHoCHo + Recall from Chapter 1 1 that -OH is a very poor leaving group in reactions run under Sn2 conditions. toluenesulfonate, however, is a very good leaving group, and reaction of the toluenesulfonate of the alcohol with ~CN proceeds readily under Sn2 conditions to give the desired product with inversion of configuration at the chirality center. A Alcohols and Phenols 413 17.13 (a) OH —— CH3CH2 POCI3 CH 3 CH2 CHCH(CH 3 2 ) K ^-^3 /C =C^ ,0=0 + CHg H ^CH(CHg)2 H H3C minor major ^CH(CH3 )2 + / C=C\ H H minor The major product has the more substituted double bond. POCI3 pyridine " CH 3 3-Methylcyclohexene In E2 elimination, dehydration proceeds have an most readily when the two groups to be eliminated compound, the only hydrogen with the proper anti periplanar relationship. In this stereochemical relationship to the is formed. -OH group is at C6. Thus, the non-Zaitsev product 3- methylcyclohexene POCIg _ pyridine 1 -Methylcyclohexene Here, the hydrogen at C2 is trans to the hydroxy 1 group, and dehydration yields the Zaitsev product, 1 -methylcyclohexene. 414 Chapter 17 (d) H3 f CH3CH c=c\ / H3C CH3 CH0CHCCH0CH0 ... + H 2-Ethyl-3-methyl- 1 -butene minor major PQCI3 =cN CH3CH2 2,3-Dimethyl-2-pentene OH HoC Jri /C _ ' pyridine I CHo HoC H3p CHgCH / CH3CH ^CHg c=c\ H3 C H C=C + / \ H3C H CH3 (E)-3 ,4-Dimethyl-2-pentene (Z)-3,4-Dimethyl-2-pentene minor minor Four different products (including E,Z isomers) can result from dehydration of 2,3dimethyl-2-pentanol. The major product has the most substituted double bond, according to Zaitsev's rule. (e) OH CH 3 CH 2 POCI3 CH3CH2CH2CCH3 _ pyridine / C=C\ / + rj CH3CH2CH2C — CH2 CH 3 H CHo CHo CH 3 2-Methyl-2-pentene 2-Methyl- 1 -pentene minor major 17.14 Aldehydes are synthesized from oxidation of primary alcohols, and ketones are synthesized from oxidation of secondary alcohols. CH- (b) CHo3 CHo3 I CH 3 CHCH 2 OH (c) Periodinane CH2CI2 OH H HoO + I CH3CHCHO Alcohols and Phenols 415 17.15 Cr02 Starting material (a) H3 + Product Periodinane Product CH3CH 2 CH 2 CH 2 CH 2 CHO CH 3 CH 2 CH 2 CH 2 CH 2 CH 2 OH CH3CH 2 CH 2 CH 2 CH 2 C0 2 H (b)

.CH< C H OH ds-3-Methylcyclohexanol (S)- 1 -Cyclopentylethanol OH N0 2 H 3C 4-Methyl-3-nitrophenol 17.21 The reduction product is a racemic mixture. Reaction of the (S) enantiomer is (a) (5>5-Methyl-2-hexanol Dess-Martin periodinane Na*" "O H H CI (iii) I shown, Alcohols and Phenols 417 418 Chapter 17 Dess-Martin periodinane > CH2CI2 ^ CH 3 CH 3 O I.UAIH4 f*- II I CH3CHCH2CH2COCH3 * CH 3 1 CH3CHCH2CH2COCH3 ^ . 2 + HOCH3 3 O II I I CH3CHCH2CH2CH2OH — f OH CH 3 CHoCHoMgBr 2 ' ' » CH3CHCH2CH2CCH2CH3 3 + HOCH3 ^ CH2CH3 17.24 VF CH 3 ^CHo6 .C.S / CH /S C 2 H3C H H3C H H3C H 1 . \f CHoCHoMgBr 2 -H 3 + *~ .C. if „ ^ CHo /^C.S.CHo S CH3 CH 2 CHoCHp's HO CH 2 CH 3 (3/e,45)-3,4-Di- methyl-3-hexanol The product + CH 3 CH 2 OH (35,45)-3,4-DImethyl-3-hexanol is a mixture of the (3R,4S) and (35,45) diastereomers. The diastereomers are formed in unequal amounts, and the product mixture is optically active. We can't predict which diastereomer will predominate. Alcohols and Phenols 419 Additional Problems Naming Alcohols 17.25 (a) (b) CHo3 I HOCH 2 CH 2 CHCH 2 OH CH3CHCHCH2CH3 CH2CH2CH3 HO 2-Methyl- 1 ,4-butanediol ( d) & OH H 3-Ethyl-2-hexanol cis- 1 ,3-Cyclobutanediol m (e) N=C ds-3-Phenylcyclopentanol ds-2-Methyl-4-cyclohepten- 1 -ol 2-Bromo-4-cyanophenol or 3-Bromo-4-hydroxybenzonitrile 17.26 None of these alcohols has multiple bonds or rings. OH CH3CH2CH2CH2CH2OH 1-Pentanol J CH 3 CH2CHCH2CH 3 2-Pentanol 3-Pentanol I CH 3 CH2CCH 3 CH 3 CH 3 2-Methyl- 1-butanol I CH 3 CH2CH2CHCH 3 OH CH 3 CH 2 CHCH 2 OH OH 2-Methyl-2-butanol OH I CH 3 CHCHCH 3 * I CHo 3-Methyl-2-butanol CHo3 I HOCH 2 CH 2 CHCH 3 CH 3 3-Methyl- 1 -butanol CH 3 CCH 2 OH CH 3 2,2-Dimethyl- 1 -propanol 2-Pentanol, 2-methyl- 1-butanol and 3-methyl-2-butanol have chiral carbons (starred). 420 Chapter 17 17.27 Primary alcohols react with CrOs in aqueous acid to form carboxylic acids, secondary alcohols yield ketones, and tertiary alcohols are unreactive to oxidation. Of the eight alcohols in the previous problem, only 2-methyl-2-butanol is unreactive to CrO, + CH 3 CH2CH2CH2C02H 3j + CH 3 CH2CH2CCH 3 3] + CH 3 CH2CCH2CH 3 CH3CH2CH2CH2CH2OH H3 OH I CrQ CH3CH2CH2CHCH3 H3 H ? CrQ CH3CH2CHCH2CH3 H3 CH 3 CH 2 CHCH 2 OH CH 3 O03j H3 + OH CH3CHCHCH3 CH 3 H3 CrQ HOCH 2 CH 2 CHCH 3 CH 3 O CrQ I CH 3 CH 2 CHC02 H II 3] + CH 3 CCHCH 3 CH 3 3) HoO+ H0 2 CCH 2 CHCH 3 CH 3 fH 3 CH 3CCH 2 OH CH 3 CHo3 CrQ I 3) + HoO CH 3 CC0 2 H CH 3 CH 3 17.28 Bombykol ( 0E, 1 2Z)- 1 0, 1 2-Hexadecadien- 1 -ol 17.29 Carvacrol 5-Isopropyl-2-methylphenol C1O3 oxidation. Alcohols and Phenols Alcohols Synthesizing 17.30 some of these problems, different combinations of Grignard reagent and carbonyl compound are possible. Remember that aqueous acid is added to the initial Grignard In adduct to yield the alcohol. (a) CH3CHO H CH 3 CH 2 MgBr + CHgCHCH 2 CHo or i CH 3 CH 2 CHO + CH 3 MgBr 2-Butanol (b) OH CH 3 CH 2 CHO CHgCH 2 CHCH 2 CHo CH 3 CH 2 MgBr + ( 3-Pentanol (c) CHo3 CHo3 I I ^Cx H2 C + MgBr CH 2 H2C CH 2 OH 2-Methyl-2-propen- 1 -ol MgBr HO or MgBr OR + 2 Triphenylmethanol 422 Chapter 17 (e) CCH 3 CH 3 MgBr or =\ HO \ / ff COR ^ + + CHo6 2CH 3 MgBr // or n MgBr (f) + CH3CCH3 2-Phenyl-2-propanol CH 2 OH MgBr + CH 2 17.31 Carbonyl precursor( s) Alcohol (a) CH 3 CHo CHo I I CH3CH2CH2CH2CCHO CH3CH2CH2CH2CCH2OH CH 3 CH3CH2CH2CH2CCO2H CH 3 CHg CHo3 I CH3CH2CH2CH2CCO2R CH 3 (b) OH I (CH 3 3 CCHCH 3 (CH 3 ) 3 CCCH 3 ) o (c) CHCH2CH 3 o II CCH2CH 3 17.32 Grignard Reagent + (a) Carbonyl Compound Product (after dilute acid workup) o II CH 3 MgBr + CH 3 CCH 3 o 2 CH 3 MgBr CH 3 CH 2 MgBr + (CH 3 ) 3 COH CH 3 COR OH CH2CH 3 + Alcohols and Phenols Carbonyl Compound Grignard Reagent + (c) Product {after dilute acid workup) o -o II CH 3 CH 2 MgBr CH 3 CH 2 C + or CH 3 CH 2 MgBr 2 O ROC" + OH \ / I CH3CH2CCH2CH3 or MqBr d \— MgBr CH 3 CH 2 CH 2 MgBr + CH3CH2CCH2CH3 + + CH 3 CH 2 CH 2 C y — MgBr , OH v —f I CH 3 CH2CH2CCH 3 y _ or (? 7 (f O or CH 3 MgBr CH 3 C + CH 3 CH 2 CH 2 CCH 3 ^^^CH 2 MgBr (e) XJ + CH 2 CH 2 OH H2C=0 HoC (0 O II CH 3 MgBr + CH2CCH 3 or O OH II 2 CH 3 MgBr + CH 2 COR CH 3 or ft CH 2 MgBr CH2CCH 3 CH 3 CCH 3 423 424 Chapter 17 17.33 All of these syntheses involve a Grignard reaction at some step. Both the carbonyl compound and the Grignard reagent must be prepared from alcohols. (a) Mg PBiv CH 3 CH 2 OH CH 3 CH 2 Br » CH 3 CH 2 MgBr ether .OH OH CrO, CHo3 CH< CHr. PBro I Periodinane ch 2 c 2 ' Mg ' 3* CH 3 CH 2 CH 2 CHCH 2 Br CH 3 CH 2 CH 2 CHCH 2OH CH 3° H CH 3 CH 2 MgB r 2.H 3 + + H3 (b) 1. I CH 3 CH 2 CH 2 CHCH 2 MgBr ether H * c=0 9 H3 CH 3 CH 2 CH 2 CHCH 2 MgBr CH< + ether - " ~+ 2. H 3 1 H 2 C=0 . I » CH 3CH 2 CH 2CHCH 2CH 2 OH (c) MgBr Br< Mg FeBrc ether OH CrO, CH 3 CH 2 CH 2 CHCH 3 H3 CH 3 CH 2 CH 2 CCH 3 + HO \ / O MgBr 1 CH 3 CH 2 CH 2 CCH 3 ( d) CH 3 22 CHoCHCHoCHoOH 3 CH 3 I ether 2-H 3 CH 3 ^ . Periodinane *- CH 2 CI 2 CH 3 CHCH 2 CH CH 2 CH 2 CH 3 + O ^ ^ ^ CHoCHCHoCH 2 3 O CH 3 II 1. + CH 3 CH 2 MgBr from (a) CHo3 2 ether H 3° + I OH I CH 3 CHCH 2 CHCH 2 CH3 Alcohols and Phenols Reactions of Alcohols 17.34 ( a) CH3CH2CH2CH2CH2OH (b) CH3CH2CH2CH2CH2OH PBr 3 CHgCH^CH^CH^CH^Br soci, CH3CH2CH2CH2CH2CI (c) CrOc CH3CH2CH2CH2CH2OH + H3 (d) CH3CH2CH2CH2CH2OH Periodinane CH3CH2CH2CH2CO2H CH3CH2CH2CH2CHO CH2CI2 17.35 (a) aCH ^\^CH=CH 2 2 CH 2 OH POCI3 pyridine 2-Phenylethanol (b) Styrene ^^CH CHO ^\^CH CH OH 2 2 2 Periodinane CH2CI2 Phenylacetaldehyde (c) ^^CH C0 CH 2 CH 2 OH 2 CrQ 2H 3| HoO + Phenylacetic acid (d) CH 2 CH 2 OH KMnO/ HoO Benzoic acid (e) ^v^CH=CH CH2CH3 2 He Pd from (a) Ethylbenzene 425 Width: 612 Height: 792 426 Chapter 17 (f) .^/CHO CH=CH 2 2. Zn, from H3 + L^JJ Benzaldehyde (a) (g) CH— CHo >^s. ^, CHCHq Hg(OAc) 2 H 2 Q , 2. from (h) NaBH 4 1-Phenylethanol (a) ^s^ C ^\^CH CH OH 2 2 H 2 CH 2 Br PBrc 1 -Bromo-2-phenylethane 17.36 (a) OH ^^CHCH /\ 3 XCHo CrO< H3 + Acetophenone 1-Phenylethanol (b) OH I XHCHo r KMnO, .CH 2 OH .C0 2 H 1 if HoO . LiAIHx 2-H3 +" r ii ^Jl Benzyl alcohol (c) aC0 from 2H FeBo m-Bromobenzoic acid (b) (d) o aCCH HO (a) CHod \ / 3 1 . 2. from .C0 2 H Br CH 3 MgBr H3 ^CH< + 2-Phenyl-2-propanol Alcohols and Phenols 427 17.37 from Remember (c) that hydroboration added have a (a) (b) cis relationship. proceeds with syn stereochemistry, and the -H and -OH 428 Chapter 17 (c) CH 3 H 2 S04 OH O (d) CH 3 Na 2 Cr2 7 no reaction OH Tertiary alcohols aren't oxidized by sodium dichromate. Mechanisms 17.39 HgC^ ^CHg HgC^ ^CHg CH< HoC d CH< C H3 C / \ H HOS020— H> L. :OH C H P H2 ' CH 3 HoO + + H3 C I CH3 Step 1: Protonation. Step 2: Loss of Step 3: Alkyl H 2 0. shift to Step 4: Loss of H3 form the + . tertiary carbocation. + Alcohols and Phenols 17.40 This mechanism consists of the same steps as are seen in two different cycloalkenes. Problem 17.39. alkyl shifts result in two different alkyl shifts Step 1: Sn2 reaction of Grignard reagent. Step 2: Protonation of alkoxide oxygen. The methyl group and the hydroxy 1 group have a trans relationship. Two different 429 430 Chapter 17 17.42 CH 3 1. CH 3 OH 2 HO- H CH 3 2. J CH 3 CH 3 HO^ H CH 3 H3 C CHg HgC Step 1: Protonation. Step 2: Addition of H 2 0. + Step 3: Loss of H . 17.43 O H 3 C^ HOH HO H ^C^ 1 CH 3 C . NaBH 2 H3 + v \! 4. H sC . H H ^ "CH3 S "C^CH + H H Reaction of 2-butanone with 3 H H NaBH4 produces a racemic mixture of (/?)-2-butanol and 2-butanol. Spectroscopy 17.44 2.32 6 — HoC 2.43 6 7.10 6,7.17 6 4.50 6 /?-Methylbenzyl alcohol 17.45 (a) OH a= c (b) 0.93 6 OH b a= 1.42 6 b = 2.43 6 c = 4.80 6 d = 7.32 6 I CH3 CH2CHCH2CH 3 a b d b 3-Pentanol a b = 1.42 5 c= 1.83 6 d = 3.41 6 I CHCH 3 c 1-Phenylethanol a (S)- Alcohols and Phenols 17.46 3 . C8H18O2 has no double bonds or rings, based on degree of unsaturation. 1 The IR band at 3350 cm" shows the presence of a hydroxyl group. The compound is symmetrical (simple NMR). 4 . There 1 . 2. is no 431 splitting. CHo3 CHo3 I ~*r- 1.24 6 I HOCCH 2 CH 2 COH 1.95 5 1 N CHo3 CH n3 ^ 1.56 6 2,5-Dimethyl-2,5-hexanediol 17.47 e d CHo6 H \ / C= C 3-Methyl-3-buten-3-ol CH 2 CH 2 OH H a b c f The peak absorbing at 1.76 6 (3 the allylic region of the spectrum, H) is due to the d protons. This peak, which occurs The peak absorbing at 2.13 6 (1 H) is due to the -OH proton a. The peak absorbing at 2.30 6 (2 H) is due to protons c. The peak splitting in is unsplit. is a triplet because of by the adjacent b protons. The peak absorbing at 3.72 6 (2 H) is due to the b protons. The adjacent oxygen causes the peak to be downfield, and the adjacent -CH2- group splits the peak into a triplet. The peaks 17.48 (a) at 4.79 6 and 4.85 6 (2 H) are due to protons e and C 5 H, 2 0, C 4 H 8 2, f. C3H4O3 The H NMR data show that the compound has twelve protons. -1 shows that the compound is an alcohol. (c) The IR absorption at 3600 cm (d) The compound contains five carbons, two of which are identical. (e) C5H12O is the molecular formula of the compound. (b) (f) , (g) b a = 0.9 6 H ? b =i. 08 CH 3 CCH 2 CH 3 c d I CH 3 1 9 § 2-Methyl-2-butanol a d = 1.4 5 c 17.49 c OH = 1.41 5 = 2.24 6 = 5.00 6 = 6.97 5 432 Chapter 17 General Problems 17.50 In these present. compounds you want to reduce some, but not all, of the To do this, choose the correct reducing agent. functional groups H2 with a palladium catalyst hydrogenates carbon-carbon double bonds without affecting carbonyl double bonds. LiAlH4 reduces carbonyl groups without affecting carbon-carbon double bonds. 17.51 433 Alcohols and Phenols 17.52 Remember that electron- withdrawing groups acidity. Electron-donating stabilize phenoxide anions and increase groups decrease phenol acidity. Most acidic Least acidic CH 3 electron- electron- electron- donating withdrawing by inductive by resonance group withdrawing effect 17.53 + ^^CH -CI H I c 2 H -:o- ^:Base ci" 1. C, ry i :0 UH + O 2. C 76^ H 1: Sn2 f H :Base Step H substitution. Step 2: E2 elimination. 17.54 H 3 QH H 3 CH CH3 :OH / C~ C \ H 3 C-7 CCH 3 HgC OH3 Pinacol H3 + + Pinacolone Step 1: Protonation. Step 2: Loss of H 2 Step 3: Alkyl shift. + Step 4: Loss of H . CHr we Alcohols and Phenols 437 17.62 The hydroxyl group is axial in the cis isomer, which is expected to oxidize faster than the trans isomer. (Remember that the bulky tert-butyl group is always equatorial in the more stable isomer.) OH CrO< (CH 3 3C- + H3 ) ds-4-tert-Butylcyclohexanol O faster (CH 3 3 C ) OH CrQ (CH 3 3C) 3> + H3 slower Jrafts-4-tert-Butylcyclohexanol 17.63 1 2. UAIH4 H3 + PBr ' Mg, ether 1 .Cyclohexanone 2. H3 MgBr + B icy clohexy lidene 17.64 An alcohol adds to an aldehyde by a mechanism that we will study in a later chapter. The hydroxyl group of the addition intermediate undergoes oxidation (as shown in Section 17.7), and an ester is formed. :o9 y OH \\ OH I I HCCHo CHoCHoO^ 3 2 S CH 3 v CH 3 CHoCHoO^ 3 2 H CH 3 CH 2 OH "0 Cr 3 H. Aj O ? CH 3 CH 2 0^ O E2 CrO>' H C or enantiomer CH 3 CH 2 0" £ ^tT^lBese CH. elimination H:Base + CH 3 CH 2 + Cr0 3 CH 3 2- 438 Chapter 17 17.65 (a) H 3 + (b) PBr3 (c) Mg, ether, then CH2 (d) Dess-Martin periodinane, (e) C 6H 5 CH 2MgBr, then H 3 CT (f) POCl 3 pyridine NaBH4 CH 2 C12 , then , 17.66 2. NAD OH UDP UDP UDP-Glucose UDP-Galactose Step 1: Base deprotonates the C4 hydroxyl group while UDP NAD+ oxidizes the alcohol to a ketone. Step 2: When the ketone the starred carbon is is reduced by the NADH formed in Step and UDP-glucose is formed. 1, the configuration at inverted, 17.67 (a) OH c I CHCH^CHg d 1 b a -Phenyl- 1 -propanol a = 0.88 6 a = 2.60 b = 1.8 6 b = 3.76 6 c = 2.32 6 c = 4.53 6 d = 4.54 6 d = 6.85 6 e = 7.24 6 e = 7.23 6 p-Methoxybenzyl alcohol 6 Alcohols and Phenols 439 17.68 Structural formula: CgHioO contains 4 multiple bonds and/or rings. 1 3500 cm" indicates a hydroxyl group. The absorptions at -1 1500 cm" and 1600 cm" are due to an aromatic ring. The absorption at 830 cm shows Infrared: The broad band 1 at 1 that the ring is p-disubstituted. 1 Compound A is probably a phenol. H NMR: The triplet at 1.16 6 (3 H) is coupled with the quartet at 2.55 6 (2 H). These two absorptions are due to an ethyl group. The peaks at 6.74 6-7.02 6 (4 H) are due to aromatic ring protons. The symmetrical splitting pattern of these peaks indicate that the aromatic ring is /?-disubstituted. The singlet absorption at 5.50 6 (1 H) is due to an -OH proton. Compound A p-Ethylphenol 17.69 +. Step Step 1: 2: The nucleophile CN adds to the positively polarized carbonyl carbon. The tetrahedral intermediate is protonated to give the addition product. 17.70 r The reaction is an Sn2 displacement of iodide by phenoxide ion. Chapter 18 - Ethers and Epoxides; Thiols and Sulfides Chapter Outline I. Acyclic ethers (Sections 18.1-18.4). A. Naming ethers (Section 18.1). Ethers with no other functional groups are named by 1 substituents and adding the word "ether". . citing the two organic When other functional groups are present, the ether is an alkoxy substituent. Properties of ethers. Ethers have the same geometry as water and alcohols. 1 2 Ethers have a small dipole moment that causes a slight boiling point elevation. Ethers can react slowly with oxygen to give explosive peroxides. 3 2. B . . . . C. Synthesis of ethers (Section 18.2). Symmetrical ethers can be synthesized by acid-catalyzed dehydration of alcohols, 1 This method is used only with primary alcohols. i. . 2. Williamson ether synthesis. a. Metal alkoxides react with primary alkyl halides and tosylates to form ethers. b. The alkoxides are prepared by reacting an alcohol with a strong base, such as NaH. c. i. Reaction of the free alcohol with the halide can also be achieved with Ag20. The reaction occurs via an Sn2 mechanism. The halide component must be primary. i. ii. In cases where one ether component is hindered, the ether should be synthesized from the alkoxide of the more hindered reagent and the halide of the less hindered reagent. 3 . Alkoxymercuration of alkenes. Ethers can be formed from the reaction of alcohols with alkenes. a. b . c. The reaction is carried out in the presence of mercuric trifluoroacetate. The mechanism is similar to that for hydration of alkenes. i. NaBH4 is used for demercuration of the intermediate. d Many different types of ethers can be prepared by this method. D. Reactions of ethers (Sections 18.3-18.4). 1 Ethers are relatively unreactive and often used as solvents. 2. Acidic cleavage (Section 18.3). Strong acids can be used to cleave ethers. a. Cleavage can occur by Sn2 or SnI routes. b i. Primary and secondary alcohols react by an Sn2 mechanism, . . . halide attacks the ether at the less hindered in which the site. route selectively produces one halide and one alcohol. and allylic ethers react by either an SnI or an El route. Claisen rearrangement (Section 18.4). (a). This ii. 3 . a. b . c. II. Tertiary, benzylic, The Claisen rearrangement is specific to allyl aryl ethers or aryl vinyl The result of Claisen rearrangement is an o-allyl phenol. The reaction takes place in a single step by a pericyclic mechanism, ethers. i. Inversion of the allyl group is evidence for this mechanism. Cyclic ethers (Sections 18.5-18.7). A. Epoxides (oxiranes) (Sections 18.5-18.6). 1 The three-membered ring of epoxides gives them unique chemical reactivity (Section 18.5). 2 The nonsystematic name -ene oxide describes the method of formation. 3 The systematic prefix epoxy- describes the location of the epoxide ring. . . . Ethers and Epoxides; Thiols and Sulfides 4. 441 Preparation of epoxides. Epoxides can be prepared by reaction of an alkene with a peroxyacid RCO3H. a. i. The reaction occurs in one step with syn stereochemistry. b. Epoxides are formed when halohydrins are treated with base. i This reaction is an intramolecular Williamson ether synthesis. Ring-opening reactions of epoxides (Section 18.6). a. Acid-catalyzed ring opening. Acid-catalyzed ring opening produces 1,2 diols. i. ii. Ring opening takes place by back-side attack of a nucleophile on the protonated epoxide ring. (a) trans-l, 2-diol is formed from an epoxycycloalkane. (b) If is used, the product is a trans halohydrin. iii. When both epoxide carbons are primary or secondary, attack occurs primarily at the less hindered site. iv. When one epoxide carbon is tertiary, attack occurs at the more highly . 5 . . . A HX substituted v. site. The mechanism is midway between Sn2 and SnI routes. (a). The reaction occurs by back-side attack (Sn2), but positive charge stabilized b. by a is tertiary carbocation-like transition state (SnI). Base-catalyzed ring-opening. Base-catalyzed ring opening occurs because of the reactivity of the strained i. ii. iii. epoxide ring. Ring-opening takes place by an Sn2 mechanism, in which the nucleophile attacks the less hindered epoxide carbon. Other nucleophiles can bring about ring opening. (a) Epoxides react with Grignard reagents to form a product with two more carbons than the starting alkyl halide. (b) Epoxide rings also react with amines in a ring-opening reaction. . . B. Crown ethers (Section 1 . 2 . 3 . 18.7). Crown ethers are large cyclic ethers. Crown ethers are named as x-crown-y, where x = the ring size and y = # of oxygens. Crown a. b. c. d. ethers are able to solvate metal cations. crown ethers solvate different cations. Complexes of crown ethers with ionic salts are soluble in organic solvents. This solubility allows many reactions to be carried out under aprotic conditions. The reactivity of many anions in Sn2 reactions is enhanced by crown ethers. Different sized IV. Thiols and sulfides (Section 18.8). A. Naming thiols and sulfides. Thiols (sulfur analogs of alcohols) are named by the same system as alcohols, with the suffix -thiol replacing -ol. a. The -SH group is a mercapto- group. 2. Sulfides (sulfur analogs of ethers) are named by the same system as ethers, with sulfide replacing ether. a. The -SR group is an alkylthio- group. B. Thiols. 1 Thiols stink! 2 Thiols may be prepared by Sn2 displacement with a sulfur nucleophile. 1 . . . The a. 3 . reaction may proceed to form sulfides. Better yields occur when thiourea is used. Thiols can be oxidized by Br2 or I2 to yield disulfides, RSSR. a. The reaction can be reversed by treatment with zinc and acid. b b . . The thiol-disulfide interconversion is an important biochemical interconversion. Chapter 18 442 C. Sulfides. 1 Treatment of a thiol with base yields a halide to form a sulfide. . 2 3 . . 4. III. thiolate anion, which can react with an alkyl Thiolate anions are excellent nucleophiles. Dialkyl sulfides can react with alkyl halides to form trialkylsulfonium salts, which are also good alkylating agents. a. Many biochemical reactions use trialkylsulfonium groups as alkylating agents. Sulfides are easily oxidized to sulfoxides (R2SO) and sulfones (R2SO2). a. Dimethyl sulfoxide is used as a polar aprotic solvent. Spectroscopy of ethers (Section 18.9). A. IR spectroscopy. 1 . Ethers are difficult to identify by IR spectroscopy because -1 occur at 1050-1 150 cm where ethers absorb. spectroscopy. spectroscopy. many other absorptions , B . NMR H NMR 1 . a. b. Hydrogens on a carbon next to an ether oxygen absorb downfield (3.4-4.5 6). Hydrogens on a carbon next to an epoxide oxygen absorb at a slightly higher field (2.5-3.5 6). 13 2. C a. NMR spectroscopy. Ether carbons absorb downfield (50-80 6). Solutions to Problems 18.1 Ethers can be bonded to named either as alkoxy-substituted compounds or by citing the two groups oxygen, followed by the word "ether". Diisopropyl ether p-Bromoanisole Propoxycyclopentane or or Cyclopentyl propyl ether p-Bromomethoxybenzene ® '3 1-Methoxycyclohexene 18.2 Ethyl isobutyl ether H 2 C= CHCH 2 OCH= CH 2 Allyl vinyl ether first step of the dehydration mechanism is protonation of an alcohol. Water is then displaced by another molecule of alcohol to form an ether. If two different alcohols are present, either one can be protonated and either one can displace water, yielding a mixture of products. If this procedure were used with ethanol and 1-propanol, the products would be diethyl The and dipropyl ether. If there were equimolar amounts of the were of equal reactivity, the product ratio would be diethyl ether ether, ethyl propyl ether, alcohols, and if they ethyl propyl ether : dipropyl ether = 1:2: 1. : Ethers and Epoxides; Thiols and Sulfides 18.3 443 Remember that the halide in the Williamson ether synthesis should be primary or methyl, in order to avoid competing elimination reactions. The alkoxide anions shown are formed by treating the corresponding alcohols with NaH. (a) CH 3 CH 2 CH 2 0~ + CH 3 Br + CH 3 0~ or CH 3 CH 2 CH 2 Br CH 3 CH 2 CH 2 OCH 3 + Br" + Br" Methyl propyl ether (b) OCH 3 CH 3 Br Methyl phenyl ether (Anisole) (c) CHo3 CH. I CH 3 CHO" CH 2 Br CH 3 CHOCH 2 + Benzyl isopropyl ether (d) CH 3 CHo3 I CH 3 CCH 2 0" CHo CH 3 CH 2 Br CH 3 CH3CHCH3 » primary secondary aryl halide halide halide (not reactive) (b) CH 3 CH 2 Br > CH3CH 2 CI » + NaBr Tetrahydrofuran Ho The compounds most reactive in the Williamson ether any Sn2 reaction (review Chapter 1 1 if necessary). CH 3 CH 2 Br CHp on 2 HoC^ /,CH O + "O Na + 18.6 3 \ / / Br ) must be used. HpC NaH I (CH 3 .sec-Butyl tert-butyl ether synthesis CH2 \ 6 2 NaBH 4 The Williamson / — CH0CHCH0CH0 (CFoCOp)pHg, CH 3 CH=CHI better poorer vinyhc leaving group leaving group (not reactive) most reactive in Width: 612 Height: 792 446 18.7 Chapter 18 (a) First, notice the substitution pattern of the ether. Bonded to the ether oxygen are a primary alkyl group and a tertiary alkyl group. When one group is tertiary, cleavage occurs by an SnI or El route to give either an alkene or a tertiary halide and a primary alcohol. CH- 0^ HBr tertiary Br + CH 3 OH methyl - problem, the groups are primary and secondary alkyl groups. Br attacks at the less hindered primary group, and oxygen remains with the secondary group, to give a secondary alcohol. (b) In this CH 3 CH3 HBr ft I CH 3 CH 2 CH— O— CH 2 CH 2CH 3 I ^ CH 3 CH 2 CHOH Br— CH2CH 2CH3 + primary secondary 18.8 HoC \ / s CH 3 H3C\ CHod i= II CHo H 3 C^ H— + HX The first step of acid-catalyzed ether cleavage is protonation of the ether oxygen to give an intermediate oxonium ion, which collapses to form an alcohol and a tertiary carbocation. The carbocation then loses a proton to form an alkene, 2-methylpropene. This is an example of El elimination. The acid used for cleavage is often trifluoroacetic acid. 18.9 H— Kb ( ^ \._ R— X + HO— R' H :x: HX first protonates the oxygen atom, and halide then brings about a nucleophilic displacement to form an alcohol and an organic halide. The better the nucleophile, the more effective the displacement. Since I~ and Br~ are better nucleophiles than Cl~, ether cleavage proceeds more smoothly with HI or HBr than with HC1. Ethers and Epoxides; Thiols and Sulfides 447 18.10 Draw the ether with the groups involved in the rearrangement positioned as they will appear in the product. Six bonds will either be broken or formed in the product they are shown as dashed lines in the transition state. Redraw the bonds to arrive at the intermediate enone, which rearranges to the more stable phenol. ; 2-Butenyl phenyl ether o-(l-Methylallyl) phenol intermediate transition state 18.11 Epoxidation by use of ra-chloroperoxybenzoic acid (RCO3H) is a syn addition of oxygen to a double bond. The original bond stereochemistry is retained, and the product is a meso compound. ,H :c=cl r H. l 3 C RCOoH ^CHo ° \o / /ic-c^ Hn M V "if CH 3 H3C ds-2-Butene c/s-2,3-Epoxybutane In the epoxide product, as in the alkene starting material, the methyl groups are cis. •"k _ ,- CH 3 RCO3H O R H3 C trans-2-Butene / ^ O Ft S S H H CH 3 trans-2 ,3 -Epoxy butane Reaction of rra/w-2-butene with m-chloroperoxybenzoic acid yields trans-2,3epoxybutane. A mixture of enantiomers is formed because the peroxyacid can attack either the top or bottom of the double bond. 448 Chapter 18 18.12 As discussed epoxide ring opening occurs primarily at the one of the epoxide carbons is tertiary. In both parts of this problem, one epoxide carbon is tertiary. in this section, acid-catalyzed more hindered carbon if OH CI HCI CH, HgC ether major tertiary (b) CHg O CI CH 2 OH HCI ether major tertiary 18.13 Notice the relationship of the hydroxyl groups in the two diols. In diol (a), the two hydroxyls are cis, and in (b) they are trans. Since ring-opening of epoxides forms trans 1,2-diols, only diol (b) can be formed by this route. The cis- 1,2-diol in (a), results from treatment of 1-methylcyclohexene with OSO4. The enantiomers of the diols are also formed. 1 Os0 4 pyridine NaHS0 3 H 2 . , 2. 1 , (b) RCO3H OH .CHg aCHc OH + — HoO H 18.14 (a) Attack of the basic nucleophile occurs at the less substituted epoxide carbon. O A HoC — / CH 2 CH 3 OH N£|8 OH HoC-cr" 18 CH< Ho 18 OH CH2CH3 HoO W OCH CCH CH .18 2 CH 3 2 3 Ethers and Epoxides; Thiols and Sulfides Under (b) of the carbons HoC— substituted epoxide carbon is tertiary. 18 CH 2 CH 3 / / \ more acidic conditions, ring-opening occurs at the when one 449 18 H3 + OH HOCH 2 (j)CH2CH3 CH, CHo Addition of a Grignard reagent takes place at the less substituted epoxide carbon. (c) ,CH2CH3 H3C s /> CH 3 H 1. 2. -=\ I + H3 CHo OH 3 \>— CH-CCH C 6 H 5 MgBr I 2 CH 3 CH, 18.15 . / \ ^q. ; %Q" . o> .X. o | -or ^.o'" o 15-Crown-5 Bases on ionic radii, the 12-Crown-4 ion-to-oxygen distance in 15-crown-5 is about 40% longer than the ion-to-oxygen distance in 12-crown-4. 18.16 Thiols are named by the same rules as alcohols, with the suffix -ol replaced by the suffix named by the same rules as ethers, with "sulfide" replacing "ether". thiol. Sulfides are (a) CHo3 (b) I CHo3 SH I (c) CHo3 I I CH 3 CCH2CHCH2CHCH 3 CH 3 CH2CHSH CH 3 (d) CHo3 I CH 3 CHSCH2CH 3 2-Cyclopentene- 1 -thiol 2,2,6-Trimethyl4-heptanethiol 2-Butanethiol (e) m SCH< o a. SCH 3 Ethyl isopropyl sulfide o-(Dimethylthio)benzene SCH2CH 3 3-(Ethylthio)cyclohexanone 450 Chapter 18 18.17 Thiourea is used to prepare thiols from alkyl halides. O LLWJHa o t| ^fr 2.H II CH 3 CH = CHCOCH3 „ CH H 3 CH= CHCH 2 OH PBr3 CH 3 CH= CHCH 2 Br 3 Methyl 2-butenoate 1 . (H 2 N) 2 C=S 2."OH, H 2 H 2 C-~ CHCH HBr CH 2 CH 3 CH=CHCH 2 Br 1 . (H 2 N) 2 C=S _ 2. "OH, H 2 CH 3 CH= CHCH 2 SH 2-Butene-l -thiol 1,3-Butadiene 18.18 O / \ CH 3 CH 2 C-C rH H H d,e a= 1.0 5 b = 1.5 6 c = 2.9 6 c 1,2-Epoxybutane d,e = 2.5 6, 2.75 Visualizing Chemistry 18.19 (a) (b) OCH 2 CH 3 H 3 C^/ C Cn7CH f=\ HoC 3 H Br 2s-2-(oBromophenyl)-2,3-epoxybutane cis- 1 -Ethoxy-3-methylcyclohexane (c) C i*. H SH (5)- 1 -Cyclopentylethanethiol 18.20 Ring-opening state. occurs at the tertiary carbon to give carbocation-like stability to the transition the C-OH bond, as it would in an Sn2 reaction. Bromine approaches 180° from 6+ O R i_N R C H-J° C-C 6 H 5 H3 C CH 3 OH Hj H3C r—rP b+ H5 f C"C CH 6 ( :B*r: 3 H ' H3 C S\ Br Ethers and Epoxides; Thiols and Sulfides 18.21 The Grignard 451 reagent attacks the epoxide at the less hindered carbon in an Sn2 reaction. to the tertiary carbon. The oxygen remains bonded secondary 1 . 2- 18.22 CH 3 MgBr, H3 ether + A molecular model shows that approach to the upper face of the double bond is hindered by a methyl group. Reaction with RCO3H occurs at the lower face of the double bond to produce epoxide A. and H2O, the intermediate bromonium ion also forms at the lower water yields a bromohydrin which, when treated with base, forms In the reaction of Br2 face. Reaction with epoxide B. 452 Chapter 18 Additional Problems Naming Ethers 18.23 (a) CH2CH3 CH3CH2OCHCH2CH3 Ethyl 1-ethylpropyl ether Di(/?-chlorophenyl) ether O (d) (c) 3,4-Dimethoxybenzoic acid (e) OCH, Cyclopentyloxycyclohexane CH2CH — CH2 4-Allyl-2-methoxyphenol 18.24 (b) (a) aOCH (c) 3 OCH< Cyclohexyl isopropyl sulfide (d) o-Dimethoxybenzene (e) O CHr. 1,2-Epoxycyclopentane (f) CHr CH 3 CH— O SH —<^j N0 2 2-Methyltetrahydrofuran Cyclopropyl isopropyl ether o-Nitrobenzenethiol or Isopropoxycyclopropane (g) (h) CHo I CHo I I ,3 CH0CH0CHCHCHSCHCH0 3 2, CH3 2-(Isopropylthio)-3,4- dimethylhexane (i) OCHo3 CH3CCH3 OCHo 2,2-Dimethoxypropane a SCHr SCH, l,l-(Dimethylthio)- cyclohexane Ethers and Epoxides; Thiols and Sulfides Synthesizing Ethers (b) CH 3 CH — CH2 1 C 6 H 5 OH, Hg(OCOCF 3 )2^ 2. NaBH 4 CH(CH 3 (c) H 3C ^ r H .- H ) 2 O RCOoH / \ H 3 C"i XH 3 C" C C"H CHo H [RCO3H = raeta-Chloroperoxybenzoic acid] (d) 1. OH Hg(OCOCF 3 )2, (CH 3 ) 2 C(CH 3 C=CH2 2. ) 3 O 5 NaBH 4 H (e) "OCH. RC0 3 H O H OH 1 HOCH 3 H ' . NaH 2.CH 3 I H OCH 3 OCH3 + enantiomer (f) + enantiomer H H OH 1.BD< -OCH- ' 1 NaH 1 2.H 2 2. 2 ,"OH CH31 D H + enantiomer -OCHj H + enantiomer 453 454 Chapter 18 18.26 (a) 0"Na+ OH OCH, /\ XHCHo CH,* CHCHo fY f^Y Na H> XHCHo NaBr Methyl 1-phenylethyl ether (b) O OH /\ aCH— I CHCH 3 POCI< CH 2 RCOo,H C ^ T CH— CH2 pyridine Phenylepoxyethane Styrene (c) OC(CH 3 a CHCH 3 1 Hg(OCOCF3 )2, (CH 3) 2 C= CH 2_ 2. NaBH 4 aCHCH ) 3 3 tert-Butyl 1-phenylethyl ether SH Br ^^CHCH I ^\.CHCH 3 1. (H 2 N) 2 C=S 2. "OH, H 2 1 U 3 -Phenylethanethiol 18.27 CHo CH< CH 3 C-r- CH 2 ^ ^*H T A U CHo3 CHo3 I CH3 CCH 3 •:o I H I CH 3 CCH 3 & H^ C. : CH 3 CCH 3 R Base Step 1: Protonation. Step 2: Attack of alcohol oxygen on carbocation. Step 3: Loss of proton. Notice that (Problem this reaction is the reverse 18.8). of acid-catalyzed cleavage of a tertiary ether Ethers and Epoxides; Thiols and Sulfides 455 18.28 n. r H 1 c/s-2-Chlorocyclohexanol enol ,2-Epoxycyclohexane Cyclohexanone -OH and -CI are in the trans orientation that allows epoxide formation to occur as described in Section 18.5. Epoxidation can't occur for the cis isomer, however. Instead, the base ~OH brings about E2 elimination, producing an enol, which tautomerizes to cyclohexanone. In the trans isomer, the Reactions of Ethers and Epoxides 18.29 The enol tautomerizes to an aldehyde. Width: 612 Height: 792 456 Chapter 18 (d) LIT (CH 3 ) 3 » CCH 2 OCH 2 CH3 (CH 3 3 CCH 2 OH + ) ^ q CH 3 CH 2 I 18.30 (a) CHoCHo Hg(OCOCF3 ) 2 CH 3 CH 2 OH 1 , 2. (b) NaBH 4 OH OCHo H HoCi H PBr3 HI HoCi H2 -Br HoC CH 3 I + H H H H (C) O .O RCQ 3 H 'OH H2 T C(CH 3 ) C(CH 3 3 ) C(CH 3 3 ) C(CH 3 3 ) 3 (d) He CH 3 CH 2 CH 2 CH 2 C — CH CH 3 CH 2 CH 2 CH 2 CH^- CH 2 Lindlar catalyst 1 . 2. CH3CH2CH2CH2CH2CH2OCH3 ^ BH 3 THF H 2 2 "OH , , CH3CH2CH2CH2CH2CH2OH CH"~l" 3 (e) _ CH0CH0CH0CH0CH — CHo from(d) 1. Hg(OCOCF3 ~ 2. NaBH 4 18.31 O. HI HoO ICH 2 CH 2 CH 2 CH 2 OH ) 2 , CH 3 OH^ ^ OCH 3 I CH0CH0CH0CH0CHCH0 Ethers and Epoxides; Thiols and Sulfides 18.32 c«-5,6-Epoxydecane C H'j° C~ H (C 4 H 9 (C4H9) ) protonation of epoxide oxygen tt + QH a attack of " H2 b "Vl ft" H (C4H9) attack at carbon a ( V/ attack of C 4 H 9> H2 attack at carbon b H 2 b: H OH +/ HCT*) V H (C 4 H 9 ), H 2 or Vc * H C— / HO loss of proton tt H H (C4 H 9 ) + 1 HoO" Y< \R_ />(C4 Hg) / (C 4 H 9 )"^r OH The product of acid hydrolysis of ds-5,6-epoxydecane S,S diols. \^~^ H H2 H (C 4 H 9 k \S ^OH (C 4 H 9 ) (C 4 H 9 ) It H >(C4h9) HO HO is R \ OH a racemic mixture of R,R and 457 458 Chapter 18 18.33 r H-OH 2 trans-5 ,6-Epoxy decane ft C C*(C 4 H 9 H (C 4 H 9 ) ) protonation of epoxide oxygen it OH + attack of - H2 attack at carbon a a r-r (C4H9) V/ b attack of H2 attack at carbon H b Hob: OH H (C4Hg)^*» _ V^^Hg) +/ H^T*) H 2 or (C4H9) _ I^H / ^(pH (C4 H 9 ) H H H2 H V It It loss of proton HO OH H (C 4 Hg).\S \R / F ^ Cv"(C4Hg) HO + H3 H .-C— + ?4 H 9 _'^H (C4 H 9 ) is a meso diastereomer of the products formed in the previous problem. The product of acid hydrolysis of frans-5,6-epoxy decane OH compound 18.34 OH H HoO + Z-^^H H C(CH3 O ) 3 cw-3-tert-Butyl- 1 ,2-epoxycyclohexane The hydroxyl groups in the product have a trans-diaxial relationship. that is a Ethers and Epoxides; Thiols and Sulfides 459 18.35 (a)(b) Hj PH2CH3 HO O R r-r R ,C— C^ CH3 N w CH 2 CH 3 H'i H3 C (2#,3#)-2,3-Epoxy- OH (2/?,35)-3-Methyl2,3-pentanediol 3-methylpentane Reaction with aqueous acid causes ring opening to occur at C3 because the positive charge of the transition state is more stabilized at the tertiary carbon. Ring opening produces a diol in which the hydroxyl groups have a trans-diaxial relationship. (c) Since ring opening occurs exclusively at C3, the product is the 2R,3S isomer and opening occurred equally at either carbon, the product would be a mixture of chiral enantiomers). is chiral. (If ring (d) The product is optically active because only one enantiomer is produced. 18.36 H H3O+ * 2. 1. H Step 1: Attack of the hydride nucleophile. Step 2: Protonation of the alkoxide anion. The reaction is an Sn2 epoxide cleavage with ":H" as the nucleophile. The exact nature of the attacking nucleophile is not clear. 18.37 OH :0: H H3 + H H H D D + enantiomer Deuterium and -OH have a trans-diaxial relationship in the product. 460 Chapter 18 Spectroscopy 18.38 M + =116 corresponds to a sulfide of molecular formula C6H12S, indicating one degree of The IR absorption unsaturation. at 890 cm 1 is due to a a a= 1.74 5 CHo3 b = 2.11 6 c = 2.27 5 d = 2.57 6 R2C=CH2 group. I H2 C — CCH 2CH 2 SCHg e c d b e = 4.73 6 2-Methyl-4(methylthio)- 1 -butene 18.39 Chemical Peak H 3 C— H Anethole Multiplicity shift Split by: a 1.84 6 doublet b 3.76 6 singlet c 6.09 6 two d 6.36 5 doublet e 6.82 doublet f doublet e f 6, 7.23 6 quartets c a,d c 18.40 (a) (b) b H3 a = 0.99 6 CH0CH0CSH d b = 1.34 6 a c: d I CH 3 c,d OCH 2 CH 2 CH 2 Br cab = 1.61 5 b General Problems 18.41 (b) CHo3 CHo3 1. I CH3CHCH 2 CH 2 CH 2 Br 2. (c) Brc (H 2 N) 2 C=S "OH,H 2 I CH3CHCH 2 CH 2 CH 2 SH ~0 a= 2.31 6 b = 3.58 6 c = 4.08 6 d = 6.90-7.25 6 Ethers and Epoxides; Thiols and Sulfides 461 (d) CH 2 CH 3 h2 2 , CH 2 CH3 H2 18.42 OCH- NaBH 4 HoO + OCH< Anethole c H3 C CHgCH 2 H CHgCH 2 two functional groups - an ether and a hydrocarbon side chain with a double bond. The ether is synthesized first - by a Williamson ether synthesis from phenol and CH3I. The hydrocarbon side chain results from a Friedel-Crafts acylation of the ether. Reduction of the ketone, bromination and dehydrohalogenation are used to introduce the The anethole ring has double bond. 18.43 + O" Na NaH + (b) He CH 2 Br CH3CI (PhC0 2 AlCIo (c) - M„ + 0"Na CH 2 Br ) 2 CH 2 0. + from (a) from (b) Benzyl phenyl ether NaBr 462 Chapter 18 18.44 CHp HpC H2C HO HO 3. CHp HpC H 2C HpC \^CH 3 1 N / CV UH 3 O The + H2n I CHp H 2 C \../G- .. + H3 CH 3 HO^ H— OSOoH Step Step Step C\ H 2C ,(X + /-Tch i 1. / k H2C CHo / \*CH< / CHp vhGCHq HpC CHp HpC \^CH3 / + ch 3 :OH 2 Protonation of the tertiary hydroxyl group. Loss of water to form a tertiary carbocation. 3: Nucleophilic attack on the carbocation by the second hydroxyl group. 1: 2: tertiary carbocation hydroxyl group is more is more likely to be eliminated because the resulting stable. 18.45 CH 3I This reaction is an Sn2 displacement and can't occur aprotic solvent that increases the rate of an Sn2 at DMF an aryl carbon. is a polar by making anions more reaction nucleophilic. 18.46 CH 3 CH 2 i 0(CH 2 CH 3 ) 2 + 0(CH 2 CH 3 ) 2 CH 3 CH 2>V CH3 CH 2 >^ :OH I. o: :Base 2. Step 1: Attack of the alcohol on the triethyloxonium cation, with loss of diethyl Step 2: Loss of proton. ether. Trialkyloxonium salts are more reactive alkylating agents than alkyl iodides because a neutral ether is an even better leaving group than an iodide ion. Ethers and Epoxides; Thiols and Sulfides 463 18.47 0"Na + OH o- CH 2 O OH H 2 C= 2 NaH CHCH 2 CI AICk +2NaBr + 2H, Safrole 18.48 The mechanism of Grignard addition to oxetane is the same as the mechanism of Grignard addition to epoxides, described in Section 18.6. The reaction proceeds at a reduced rate because oxetane is less reactive than ethylene oxide. The four-membered ring oxetane is less strained, and therefore more stable, than the three-membered ethylene oxide ring. 18.49 HoO + 2 HBr BBr 3 forms Br a Lewis acid complex with to acts as a nucleo- phile in an form CH 3 Br + S N 2 reaction CH 3 Br. B(OH) 3 Water cleaves the Lewis acid complex. the ether. Step Step Step BBr3 forms a Lewis acid complex with the ether. Br~ acts as a nucleophile in an Sn2 reaction to form CH3Br. 3: Water cleaves the Lewis acid complex. 1: 2: 18.50 — —— = 1.06 g vanillin ; * ; , , n„ _ 6.97 x 1A 10 J3 mol 6.81 x 10-3 mol Agl , vanillin 152 g/mol 1.60 gAgl = 234.8 g/mol 6.81 x 10" 3 mol 3 6.81 x 10~ Agl mol 6.81 x 10" 3 CH 3 I I- mol 6.81 x 10" 3 mol -OCH3 Thus, 6.97 x 10 mol of vanillin contain 6.81 x 10 mol of methoxyl groups. Since the ratio of moles vanillin to moles methoxyl is approximately 1:1, each vanillin contains one ~ methoxyl group. CH3O CHO Vanillin 464 Chapter 18 18.51 Disparlure, C19H38O, contains one degree of unsaturation, at 2.8 5 identifies as an epoxide which the ! H NMR absorption ring. KMnO/ HO Undecanoic acid 6-Methylheptanoic acid 18.52 NaNH HC=C 7:C=C + Br C=C H2 , Lindlar catalyst I RCO3H + Br" Ethers and Epoxides; Thiols and Sulfides 465 18.53 CX+ HoO H— OH 2 /lOH :0 / \ Ph 2 C— PhoC— CH CH 2 H Step 1: Protonation. Step 2: Epoxide opening. Step 3: Hydride shift. Step 4: Loss of proton. Reaction occurs by this route because of the stability of the intermediate tertiary carbocation. 18.54 Use the aldehyde-forming reaction shown in the previous problem. o-Hydroxyphenylacetaldehyde 18.55 H CH 3MgBr, ether; ~OH, H 2 0. (a) (b) H 2 S0 4 H2 0; , (c) NaH, then CH 3 I; (d) m-ClC 6 H 4 C0 3 H; (e) Width: 612 Height: 792 466 Chapter 18 18.56 an Sn2 reaction because the rate depends on the concentrations of both reagents, reaction is a Williamson ether synthesis, in which an alkoxide displaces a halogen. In this reaction, KOH is used to form the phenoxide anion. (a) This is Sn2 (b) This 18.57 The reaction complex is is a nucleophilic aromatic substitution. The intermediate Meisenheimer by the -NO2 group. stabilized Step 1: Addition of phenoxide. Step 2: Elimination of fluoride. 18.58 (a) b OCH< CHoCH 3 a |c OCHo3 b (b) a= 1.27 6 b = 3.31 5 c = 4.57 6 CH= CHOCH3 a= 3.71 6 b = 5.17 5 c = 6.08 6 d = 7.1-7.6 5 Ethers and Epoxides; Thiols and Sulfides 18.59 (a) pCH 2 CH3 O CH 3 CH 2OH H + OH OCH 2 CH 3 CH 3 CH 2OH H catalyst + OCH 2 CH 3 catalyst hemiacetal acetal + OCH 2 CH 3 r*;ocH 2 cHo +H2 ° - rn. \^ 2 I. HOCH 2 CH 3 - 3 - tl OCH 2 CH 3 OCH 2 CH 3 OCH 2 CH 3 f J^OCH 2CH3 4. acetal + H3 + OH, Step 1: Protonation. Step 2: Loss of water. Step 3: Addition of ethanol. Step 4: Loss of proton. 18.60 O / \ CH 2 Br Br" Step Step 1: 2: Addition of hydride to the ketone. Displacement of bromide by the alkoxide anion. The intermediate resulting from addition of H: is similar to the intermediate in a Williamson ether synthesis. Intramolecular reaction occurs to form the epoxide. 468 Chapter 18 Review Unit Major Topics Covered The and Related Compounds 7: Alcohols, Ethers, (with vocabulary): -OH group: alcohol phenol glycol wood alcohol hydrogen bonding alkoxide ion phenoxide ion acidity constant Alcohols: Grignard Reagent Dess-Martin periodinane Phenols: cumene hydroperoxide quinone Acyclic ethers: Williamson ether synthesis Cyclic ethers: oxirane epoxide tosylate hydroquinone protecting group TMS ether ubiquinone Claisen rearrangement vicinal glycol Thiols and sulfides: Thiol sulfide mercapto group sulfoxide sulfone peroxyacid crown ether alkylthio group disulfide 18-crown-6 thiolate ion trialkylsulfonium salt Types of Problems: After studying these chapters, you should be able to: - Name and draw structures of alcohols, phenols, ethers, thiols and sulfides. Explain the properties and acidity of alcohols and phenols. Prepare all of the types of compounds studied. Predict the products of reactions involving alcohols, phenols and ethers. Formulate mechanisms of reactions involving alcohols, phenols and ethers. Identify alcohols, phenols and ethers by spectroscopic techniques. Points to Remember: great biochemical importance of hydroxy 1 groups is due to two factors:(l) Hydroxy 1 groups make biomolecules more soluble because they can hydrogen-bond with water. (2) Hydroxyl groups can be oxidized to aldehydes, ketones and carboxylic acids. The presence of a hydroxyl group in a biological molecule means that all functional groups derived from alcohols can be easily introduced. * The * Carbon-carbon bond-forming reactions are always more skeleton. The difficult to learn than functional group often difficult to recognize the components that form a carbon product of a Grignard reaction contains a hydroxyl group bonded to at least one transformations because it is two or three ). When looking at a product that might have been formed by a Grignard reaction, remember that a tertiary alcohol results from the addition of a Grignard reagent to either a ketone or an ester (the alcohol formed from the ester has two identical -R groups), a secondary alcohol results from addition of a Grignard reagent to an aldehyde, and a alkyl group (usually 470 Review Unit 7 primary alcohol results from addition of a Grignard reagent to formaldehyde or to ethylene Remember that any molecule taking part in a Grignard reaction must not contain functional groups that might also react with the Grignard reagent. oxide. Ethers are quite unreactive, relative to many other functional groups we study, and are often used as solvents for that reason. Concentrated halogen acids can cleave ethers to alcohols and halides. Remember that the halide bonds to the less substituted alkyl group when the ethers are primary or secondary alkyl ethers. Epoxide rings can be opened by both acid and base. In basic ring-opening of an unsymmetrical epoxide (and in ring-opening using a Grignard reagent), attack occurs at the less substituted carbon of the epoxide ring. In acidic ring opening, the position of attack depends on the substitution pattern of the epoxide. When one of the epoxide carbons is tertiary, attack occurs at the more substituted carbon, but when the epoxide carbons are both primary or secondary, attack occurs at the less substituted carbon. * The most useful spectroscopic data for these compounds: (1) A broad IR absorption in the range 3300 cm_1 -3600 cm -1 shows the presence of the -OH group of an alcohol or a phenol. (2) Hydrogens bonded to the -O-C- carbon of an alcohol or ether absorb in the range 3.5-4.5 6 in an H NMR spectrum or in the range 50-80 8 in a 13 C NMR spectrum. ! Self-Test: *2^ H3C-7 HCI 10 (d) (c) H 3 C-> H3 C H3C-7 H3 C OH HCI Ethers are stable to all of the following reagents except: (a) nucleophiles (b) bases (c) strong acids (d) dilute acids ~OH Preview of Carbonyl Compounds Chapter Outline The carbonyl I. group. A. Kinds of carbonyl compounds. 1 . 2. All carbonyl compounds contain an acyl group (R-C=0). The groups bonded to the acyl group can be of two types: a. b. B . Groups that can't act as leaving groups. Examples: aldehydes and ketones. i. Groups that can act as leaving groups. ii. Examples: carboxylic acids, esters, amides, acid halides, lactones, acid anhydrides, lactams. Nature of the carbonyl group. 1 The carbonyl carbon is s/? 2 -hybridized. . A k bond is formed between carbon and oxygen. Carbonyl compounds are planar about the double bond. 2 The carbon-oxygen bond is polar. The carbonyl carbon acts as an electrophile. a b. The carbonyl oxygen acts as a nucleophile. Reactions of carbonyl compounds. A. Nucleophilic addition reactions of aldehydes and ketones. 1 A nucleophile adds to the carbonyl carbon. a. b. . . II. . 2 . The resulting tetrahedral intermediate has two fates: a The negatively charged oxygen can be protonated to form an b Loss of water leads to formation of a C=Nu double bond. . alcohol. . B . Nucleophilic acyl substitution reactions. nucleophile adds to the carbonyl carbon. 1 2 The resulting tetrahedral intermediate expels a leaving group to form a . A . new carbonyl compound. 3 C . . This type of reaction takes place with carbonyl compounds other than aldehydes and ketones. Alpha substitution 1 . reactions. Reaction can occur at the position next to the carbonyl carbon ( a position). a. This type of reaction is possible because of the acidity of alpha hydrogens. b Reaction with a strong base forms an enolate anion, which behaves as a . nucleophile. 2 All carbonyl compounds can undergo a substitution reactions. D. Carbonyl condensation reactions. 1 Carbonyl condensation reactions occur when two carbonyl compounds react with each other. 2. The enolate of one carbonyl compound adds to the carbonyl group of a second . . compound. Solutions to Problems 1 . According to the electrostatic potential maps, the carbonyl carbon of acetyl chloride is more electrophilic and the oxygen of acetone is more nucleophilic. This makes sense, because acetyl chloride has two electron-withdrawing groups that make its carbonyl carbon electron-poor and thus electrophilic. Because acetyl chloride has two electron- withdrawing groups, neither group is as nucleophilic as the carbonyl oxygen of acetone. Preview of Carbonyl Compounds The reaction of cyanide ion with acetone is OH HoC Cyanide anion adds + CN H3 C to the positively polarized carbonyl intermediate. This intermediate (a) H3 I H 3C j HqC* is CN carbon to form a tetrahedral protonated to yield acetone cyanohydrin. is a nucleophilic acyl substitution. Ammonia adds to acetyl chloride, and eliminated, resulting in formation of an amide. This reaction chloride is (b) In this nucleophilic addition reaction, addition of the nucleophile is followed by loss of water. (c) 473 a nucleophilic addition reaction. O" "CN 1 Two molecules of cyclopentanone react in this carbonyl condensation. Chapter 19 - Aldehydes and Ketones: Nucleophilic Addition Reactions Chapter Outline I. General information about aldehydes and ketones (Sections 19.1-19.3). A. Naming aldehydes and ketones (Section 19.1). 1 . Naming a. b . c. d 2 . . aldehydes. Aldehydes are named by replacing the -e of the corresponding alkane with -al. The parent chain must contain the -CHO group. The aldehyde carbon is always carbon 1. When the -CHO group is attached to a ring, the suffix -carbaldehyde is used. Naming ketones. Ketones are named by replacing the -e of the corresponding alkane with -one. b Numbering starts at the end of the carbon chain nearer to the carbonyl carbon. c. The word acyl is used when a RCO- group is a substituent. B. Preparation of aldehydes and ketones (Section 19.2). a. . Preparation of aldehydes. a. Oxidation of primary alcohols with Dess-Martin periodinane. b Partial reduction of carboxylic acid derivatives. 2 Preparation of ketones. a. Oxidation of secondary alcohols. b. Ozonolysis of alkenes with at least one disubstituted unsaturated carbon. c. Friedel-Crafts acylation of aromatic compounds. d. Preparation from carboxylic acid derivatives. C. Oxidation of aldehydes and ketones (Section 19.3). 1 Aldehydes can be oxidized to carboxylic acids by many reagents. a. Cr03 is used for normal aldehydes. b Oxidation occurs through intermediate 1 1 -diols. 2 Ketones are generally inert to oxidation, but can be oxidized to carboxylic acids with strong oxidizing agents. Nucleophilic addition reactions of aldehydes and ketones (Sections 19.4-19.13). A. Characteristics of nucleophilic addition reactions (Section 19.4). Mechanism of nucleophilic addition reactions. 1 a. nucleophile attacks the electrophilic carbonyl carbon from a direction 105° 1 . . . . , . . II. . A b . c. d . opposite to the carbonyl oxygen. 2 3 rehybridizes from sp to sp and a tetrahedral alkoxide intermediate is produced. The attacking nucleophile may be neutral or negatively charged. i. Neutral nucleophiles usually have a hydrogen atom that can be eliminated. The tetrahedral intermediate has two fates: i. The intermediate can be protonated to give an alcohol. ii. The carbonyl oxygen can be eliminated as -OH to give a product with a The carbonyl group C=Nu 2. , double bond. Relative reactivity of aldehydes and ketones. Aldehydes are usually more reactive than ketones in nucleophilic addition a. reactions for two reasons: nucleophile can approach the carbonyl group of an aldehyde more readily i because only one alkyl group is in the way. A Aldehydes and Ketones: Nucleophilic Addition Reactions 475 Aldehyde carbonyl groups are more strongly polarized and electrophilic because they are less stabilized by the inductive effect of alkyl groups, Aromatic aldehydes are less reactive than aliphatic aldehydes because the electron-donating aromatic ring makes the carbonyl carbon less electrophilic. ii. b . B. Nucleophilic addition reactions (Section 19.5-19.13). 1. Hydration (Section 19.5). a. Water adds to aldehydes and ketones to give 1,1-diols (often referred to as gemdiols or hydrates). b . c. d . The reaction compound. Reaction is is reversible, but generally the equilibrium favors the carbonyl slow is catalyzed by both aqueous acid and base. an addition of -OH, followed by protonation in pure water, but i. The base-catalyzed ii. of the tetrahedral intermediate by water. In the acid-catalyzed reaction, the carbonyl oxygen neutral water adds to the carbonyl carbon. The i. catalysts Base Acid reaction have different is is protonated, and effects. catalysis converts water to a better nucleophile. ii. catalysis makes the carbonyl carbon a better electrophile. Reactions of carbonyl groups with H-Y, where Y is electronegative, are reversible; the equilibrium favors the aldehyde or ketone. Cyanohydrin formation (Section 19.6). a. HCN adds to aldehydes and ketones to give cyanohydrins. i. The reaction is base-catalyzed and proceeds through a tetrahedral e. 2. intermediate. Equilibrium favors the cyanohydrin adduct. b is one of the very few protic acids that add to a carbonyl group c. Cyanohydrin formation is useful for the transformations that the -CN group can undergo. The -CN group can be reduced, to form an amine. i. ii. The -CN group can be hydrolyzed, to produce a carboxylic acid. Addition of hydride and Grignard reagents (Section 19.7). ii. . 3 . HCN a. Hydride addition. i. LiAlHi and NaBH4 act as if they are H:~ donors and add to carbonyl compounds to form tetrahedral alkoxide intermediates. ii. In a separate step, water is added to protonate the intermediate, yielding an b Addition of Grignard reagents. i. Mg 2+ complexes with oxygen, making the carbonyl group more alcohol. . electrophilic. ii. iii. R:~ adds to the carbonyl carbon to form a tetrahedral intermediate. in a separate step to protonate the intermediate, yielding an Water is added alcohol. Grignard reactions are irreversible because R:~ is not a leaving group. Addition of amines (Section 19.8). a. Amines add to aldehydes and ketones to form imines and enamines. b An imine (R2C=NR) is formed when a primary amine adds to an aldehyde or iv. 4. . ketone. i. The process ii. A proton transfer converts the initial adduct to a carbinolamine. iii. Acid-catalyzed elimination of water yields an imine. is acid-catalyzed. Width: 612 Height: 792 476 Chapter 19 iv. The reaction rate enough maximum occurs at pH = 4.5. At this pH, to catalyze elimination of water, but [H low enough so + ] is that the high amine is nucleophilic. v . Some imine derivatives Enamines (R2N=CR-CR2) c. and ketones, produced when aldehydes and ketones react with are useful for characterizing aldehydes are secondary amines. i 5 . . The mechanism is similar to that of imine formation, except the a carbon is lost in the dehydration step. Addition of hydrazine: the Wolff-Kishner reaction (Section 19.9). a. Hydrazine reacts with aldehydes and ketones in the presence of a proton from KOH to form alkanes. i. b 6. The intermediate hydrazone undergoes base-catalyzed bond of N2 and protonation to form the alkane. migration, loss The Wolff-Kishner reduction can also be used to convert an acylbenzene to an alkylbenzene. Addition of alcohols: acetal formation (Section 19.10). a. In the presence of an acid catalyst, two equivalents of an alcohol can add to an aldehyde or ketone to produce an acetal. The initial intermediate, a hemiacetal (hydroxy ether), is formed when the i. first equivalent of alcohol is added. Protonation of -OH, loss of water, with formation of an oxonium ion, and ii. addition of a second molecule of yields the acetal. b Since the reaction is reversible, changing the reaction conditions can drive the . ROH . reaction in either direction. Because acetals are inert to many reagents, they can be used as protecting groups in syntheses. i. Diols are often used as protecting groups, forming cyclic acetals. c. 7 . The Wittig reaction (Section 19.1 1). a. The Wittig reaction converts an aldehyde b . or ketone to an alkene. Steps in the Wittig reaction: i. An alkyl halide reacts with triphenylphosphine to form an ii. iii. alkyltriphenylphosphonium salt. Butyllithium converts the salt to an ylide (phosphorane). The ylide adds to an aldehyde or ketone to form a dipolar betaine. (a). In c. some iv. The i. The Wittig cases, the addition is a one-step cycloaddition. betaine forms a four-membered ring intermediate (oxaphosphatane), which decomposes to form the alkene and triphenylphosphine oxide. Uses of the Wittig reaction. reaction can be used to produce mono-, di-, and trisubstituted alkenes, but steric hindrance keeps tetrasubstituted alkenes ii. 8. The Wittig from forming. reaction produces pure alkenes of known stereochemistry (excluding E,Z isomers). Biological reductions (Section 19.12). a. b . The Cannizzaro reaction is unique in that the tetrahedral intermediate of addition of a nucleophile to an aldehyde can expel a leaving group. Steps in the Cannizzaro reaction. i. HCT adds to an aldehyde with no a hydrogens to form a tetrahedral intermediate. ii. c. FT is The expelled and adds to another molecule of aldehyde. which one molecule of aldehyde is oxidized and a second molecule is reduced. The Cannizzaro reaction isn't synthetically useful, but it resembles the mode of action of the enzyme cofactor NADH. iii. result is a disproportionation reaction, in Aldehydes and Ketones: Nucleophilic Addition Reactions 9 . Conjugate addition to ^-unsaturated aldehydes and ketones (Section 477 19. 13). Steps in conjugate addition. a. Because the double bond of an a,/3-unsaturated aldehyde/ketone is conjugated with the carbonyl group, addition can occur at the y3 position, which is an electrophilic site. i. Protonation of the a carbon of the enolate intermediate results in a product having a carbonyl group and a nucleophile with a 1,3 relationship. Conjugate addition of amines. ii. b . Primary and secondary amines add to a,/3-unsaturated aldehydes and i. ketones. ii. The conjugate addition product Conjugate addition of water. c. is often formed exclusively. Water can add to yield ^-hydroxy aldehydes and ketones. Conjugate addition of water also occurs in living systems. d Conjugate addition of organocopper reagents. Conjugate addition of organocopper reagents (R^CuLi) alkylates the double i. bond of a,/3-unsaturated ketones. ii. This type of addition doesn't occur with other organometallic reagents. iii. Primary, secondary, tertiary, aryl, and alkenyl groups can be added. iv. The mechanism may involve conjugate addition of the diorganocopper anion, followed by transfer of an -R group. Spectroscopy of aldehydes and ketones (Section 19.14). A. IR spectroscopy. The C=0 absorption of aldehydes and ketones occurs in the range 1660-1770 1 i. ii. . III. . cm -1 . The exact position of absorption can be used a. to distinguish between an aldehyde and a ketone. b . The position of absorption also gives information about other structural and angle strain. absorption values are constant from one compound to another. c. The -1 Aldehydes also show absorptions in the range 2720-2820 cm spectroscopy. spectroscopy. features, such as unsaturation 2 B . . . NMR H NMR 1 . a. b. 2. 13 a. b. c. Aldehyde protons absorb near 10 6, and show spin-spin coupling with protons on the adjacent carbon. Hydrogens on the carbon next to a carbonyl group absorb near 2.0-2.3 6. Methyl ketone protons absorb at 2. 1 6. i. C NMR spectroscopy. carbonyl-group carbons absorb in the range 190-215 The These absorptions characterize aldehydes and ketones. Unsaturation lowers the value of 6. 6. C. Mass spectrometry. 1 . Some aliphatic a. aldehydes and ketones undergo McLafferty rearrangement. A hydrogen on the y carbon is transferred to the carbonyl oxygen, the bond between the a carbon and the p carbon produced. The remaining cation radical is broken, and a neutral alkene fragment is is detected. b Alpha cleavage. a. The bond between the carbonyl group and the a carbon is cleaved. b. The products are a neutral radical and an acyl cation, which is detected. . 2 . 478 Chapter 19 Solutions to Problems 19.1 Remember that the principal chain must contain the aldehyde or ketone group and that an aldehyde group occurs only at the end of a chain. The aldehyde carbon is carbon 1 in an acyclic compound, and the suffix -carbaldehyde is used when the aldehyde group is attached to a ring. (b) (a) ^\.CH CH CHO 2 2 f? CH3CCH 2 CH 2 CH 2 CCH 2 CH3 CHgCH2CCH(CHg)2 2-Methyl-3-pentanone 3-Phenylpropanal 2,6-Octanedione (e) CH 3 CH= CHCH 2 CH 2 CHO 4-Hexenal /ra«s-2-Methylcyclo- ds-2,5-Dimethylcyclohexanone hexanecarbaldehyde 19.2 (b) (c) o CI CH 2 CHO I CHgCHCh^CCHg 4-Chloro-2-pentanone (e) CHO ds-3-tert-Butylcyclo- (f) CHo3 CHo3 I CHoCHCI 3 I H 2 C=CCH 2 CHO 3-Methyl-3-butenal hexanecarbaldehyde 19.3 Phenylacetaldehyde 2-(l-Chloroethyl)-5methylheptanal We have seen the first two methods of aldehyde preparation in earlier chapters. (a) Dess-Martin CH3CH 2 CH 2 CH 2 CH 2 OH 1-Pentanol periodinane CH 2 CI 2 ^ I CH3 CH 2 CHCH 2 CH 2 CHCHO CH3CH 2 CH 2 CH 2 CHO Aldehydes and Ketones: Nucleophilic Addition Reactions (b) Or 1. CH3CH2CH2CH2CH — CH2 ~^ + Zn H * CH3CH2CH2CH2CHO + CH2O 1-Hexene (c) CH3CH2CH2CH2CO2CH3 2. (d) _ 1 CH3CH2CH2CH — CH2 . 2. 1-Pentene 19.4 THF BH3, H2 CH3CH2CH2CH2CHO + H3 CH3CH2CH2CH2CH2OH z 2 ,"OH (a) All of these methods are familiar, (a) O HoO + CH3CH2O— CCH2OH3 II CH3CH2CH2CCH2CH3 HgS0 4 (b) n CH3COCI Br CH3 Br2 *» AICk FeBro O^ HO. (c) ^CH< CHCH 3 MgBr Br Mg 1 CH3CHO . 2. H3 CrO< l^^J + (d) H3 + ^ H 1 BH3, V CH3 THF CrO : 2. H2 2 , "OH HoO !^H OH O 19.5 ^HCo; H :o: :CN HO CN CN 6+ + "OH 2. 1. cyanohydrin Step 1: Cyanide anion adds to the positively polarized carbonyl carbon to form a tetrahedral intermediate. Step 2: This intermediate is protonated to yield the cyanohydrin. 479 480 Chapter 19 19.6 CH 3 electron-withdrawing electron-donating The electron-withdrawing nitro group makes the aldehyde carbon of /7-nitrobenzaldehyde more electron-poor (more electrophilic) and more reactive toward nucleophiles than the aldehyde carbon of p-methoxybenzaldehyde. 19.7 :o:s- OH :o: I CI 3 C'^ H CIqC C H -oh 2 ' OH H :OH, Chloral hydrate 19.8 Q :o: :o: II I R R OH OH OH, R R :OH, The above mechanism is mechanisms we have we can write the above mechanism in reverse to similar to other nucleophilic addition studied. Since all steps are reversible, show how labeled oxygen is incorporated into an aldehyde or ketone. +. OH C r ~> R O* C OH. I Sh This exchange II is H2 + R J * A O'. R \j- R R very slow in water but proceeds more rapidly when either acid or base is present. 19.9 O NC NC O" CH 3 H3C CH 3 + "CN H3 C HCN OH CH 3 CH 3 + "CN 2,2,6-Trimethylcyclohexanone is an equilibrium process. Because addition of ~CN to 2,2,6trimethylcyclohexanone is sterically hindered by the three methyl groups, the equilibrium lies toward the side of the unreacted ketone. Cyanohydrin formation Aldehydes and Ketones: Nucleophilic Addition Reactions 481 of a ketone or aldehyde with a primary amine yields an imine, in which C=0 has been replaced by C=NR. Reaction of a ketone or aldehyde with a secondary amine yields an enamine, in which C=0 has been replaced by C-NR2, and the double bond has moved. 19.10 Reaction CH 2 CH 3 O H + CH 3 CH 2 NH 2 1 mine : N(CH 2 CH 3 2 ) H + + (CH 3 CH 2 2 NH H2 ) enamine 19.11 carbinolamine Step 1: Protonation of nitrogen. Step 2: Addition of water. Step 3: Loss of proton. Step 4: Proton transfer. Step 5: Loss of amine. 19.12 The structure is an enamine, which the amine and ketone is prepared from a ketone and a secondary amine. Find components and draw the reaction. from diethylamine from cyclopentanone 482 Chapter 19 19.13 O CH 3 CHo CHgC — CHCCHg (b) l_ H 2 NNHj CH3C — CHCH2CH3 KOH H2 (a) H2 (c) Pd/C Pd/C O CHo3 (c) II I CH3CHCH2CCH3 19.14 Formation CHo3 H 2 NNH; I CH3CHCH2CH2CH3 KOH of the hemiacetal is the first step, (a) H— CI CH0CH02 C"OH V11 / :OH . OH OH ^ ^ R *\ ^ + HoO 3 OH+ :OH 2 I I D -- C<+- ^ H • CH 2 CH 2 OH CH 2 CH 2 OH hemiacetal Step 1: Protonation of oxygen. Step 2: Addition of -OH. Step 3: Loss of proton. (b) :OH H ^ CI R^ R' .. +^ 9 ,CH 2 I o: O: 1. R' R' CH 2 CH 2 OH CH, 2. :0- 3. -c I CH 2 CH 2 OH H H HH V R .CH 2 I H3 + + 0^ c' i\ H H acetal Step 1: Protonation. Step 2: Loss of H 2 0. Step 3: Addition of -OH. Step 4: Loss of proton. Protonation of the hemiacetal hydroxyl group is followed by loss of water. Attack by the second hydroxyl group of ethylene glycol forms the cyclic acetal ring. Aldehydes and Ketones: Nucleophilic Addition Reactions 19.15 Locate acetal. by =0 the two identical -OR groups to identify the alcohol that was used to form the (The illustrated acetal was formed from methanol.) Replace these two -OR groups to find the carbonyl compound. 2 19.16 Locate 483 CH 3 OH + bond that is formed by the Wittig reaction. The simpler or less component comes from the ylide, and the more substituted component comes the double substituted from the aldehyde or ketone. Triphenylphosphine oxide is a byproduct of all these reactions. (a) from ylide CH 3 cr + + (Ph) 3 P - — CHCH3 from ketone from aldehyde (c) ?l CH3CCH3 from ketone from ylide + (Ph) 3 P— CHCH 2CH2 CH3 484 Chapter 19 (d) H ^\zC^' O .CH 3 II from CH3CCH3 CHo ketone from ylide (e) CHo CH, (Ph) 3 P-CH O from ketone The Z isomer is from ylide also produced, (f) OVCH 2 from ylide .0 (Ph) 3 from ketone 19.17 CHO p-Ionylideneacetaldehyde + P-CH 2 Aldehydes and Ketones: Nucleophilic Addition Reactions 19.18 (To: :OH OH H H C _ / \ H :0: 3. ti O C "OH |^J^ CH H 3 Q+ OC 2 OH .h c / \ HO H Step 1: Addition of "OH. Step 2: Expulsion, addition of ~H. Step 3: Proton transfer. Step 4: Protonation. This is an intramolecular Cannizzaro reaction. 19.19 Addition of the pro-R hydrogen of NADH takes place at the Re face of pyruvate. Re face t NADH H3 C \_/ C02 " \ pro-R I HO H ^ \ r H 3C _ co 2 + C // (^-Lactate — NHo • NAD+ H C-NH 2 7/ Width: 612 Height: 792 486 Chapter 19 19.20 The -OH group adds to the Re face at carbon 2, and -H + adds to the Re face at carbon 3, to yield (2tf,3S)-isocitrate. CO. "0 C2 2^ COH 2C H ' Re facet H2 "OH ofC2 \ Re f "0 2 C^ face sj* HO of C3 ^CO H (2/?,35)-Isocitrate 19.21 The product is formed by 1,4 addition of CN, followed by H— CN protonation. _ O :o: :CN nucleophilic addition CN rV ^^^CN + ~CN CN protonation Aldehydes and Ketones: Nucleophilic Addition Reactions 19.22 To choose the reactants that form a conjugate addition product, follow these steps: Give to the aldehyde or ketone carbon the number "1", and count two carbons away from the carbonyl carbon. The double bond in the a, -unsaturated starting material connected the carbons numbered "2" and "3". (2) The grouping bonded to the "3" carbon (circled here) came from the alkyllithium (1) (3 reagent. (a) o _ H 2 C — CHCCHg II o 1 2 . - Li(CH 3 CH 2 CH2)2Cu^ *^ + H 3° II \ CH3CH2CH2PH2CH2CCH3 N*, 3 2 2-Heptanone 1 487 488 Chapter 19 19.23 2-Cyclohexenone 3 -Methylcyclohexanone l-Methyl-2cyclohexen-l-ol 2-Cyclohexenone is a cyclic (^-unsaturated ketone whose carbonyl IR absorption occurs -1 at 1685 cm If direct addition product A is formed, the carbonyl absorption vanishes and -1 a hydroxyl absorption appears at 3300 cm If conjugate addition produces B, the 1 carbonyl absorption shifts to 1715 cm" where 6-membered-ring saturated ketones . . , absorb. 19.24 Find (a) the type of aldehyde or ketone and check Table 19.2 for absorptions. H 2C=CHCH 2 COCH 3 absorbs at 1715 cm" 1 . (4-Penten-2-one is not an aJ3-un saturated ketone.) (b) CH3CH=CHCOCH3 is an ^-unsaturated ketone and absorbs (d) (c) -1 at 1685 cm . (e) (c) 2,2-Dimethylcyclopentanone, a five-membered-ring ketone, absorbs (d) m-Chlorobenzaldehyde shows an absorption cm" and 2820 cm"1 at 1750 cm . 1 at 1705 cm" and two absorptions at 2720 1 . (e) 3-Cyclohexenone absorbs (f) CH 3 CH 2CH2 CH=CHCHO at is 1715 cm" 1 . an ^-unsaturated aldehyde and absorbs at 1705 -1 cm . Aldehydes and Ketones: Nucleophilic Addition Reactions 19.25 489 In mass spectra, only charged particles are detected. The McLafferty rearrangement produces an uncharged alkene (not detected) and an oxygen-containing fragment, which is a cation radical and is detected. Alpha cleavage produces a neutral radical (not detected) and an oxygen-containing cation, which is detected. Since alpha cleavage occurs primarily on the more substituted side of the aldehyde or ketone, only this cleavage is shown. (a) H 3 (X H I ^ H HoC« OH CH McLafferty H C H CH 2 rearrangement CH, I II C CHo3 / \ I CH 3 H CH 3 m/z = 72 H 3 C, o c C H \ / \ CHc cleavage CHod III c H- I r H' CH 3 H O H 3 C. Alpha H CH 3 3-Methyl-2-hexanone CH 3 m/z = 43 m/z = 114 H HoCX H- I * H HoC» PL o II rearrangement A H H h3 c OH CH McLafferty I CH CHs HoC CH 3 H2 C m/z = 58 H HoC^ I M O C H- H3 C Alpha cleavage I C^n ^ch- CHc H O III H- H 3C / \ H H 3 C, c ' ^CHo I CH 3_ m/z = 43 4-Methyl-2-hexanone m/z = 114 Both isomers exhibit peaks at m/z = 43 due to a-cleavage. The products of McLafferty rearrangement, however, occur at different values of m/z and can be used to identify each isomer. 490 Chapter 19 (b) H H3C HoC. FCH McLafferty H rearrangement HC c. 2 H v_»n2 -»n2 CHoCH HoC 2^ CHoCHo v m/z = 72 H H H 3 C^L,H or H 3 C. I o H- H. cleavage C H / \ H S, if CHoCHo s ^ O CHc Alpha ? CH2CH2 CHc H m/z = 57 m/z = 114 3-Heptanone H OH CHc' McLafferty II I rearrangement CH0CH0CH0 d d C ' H CHc H2 * / \ C CH2CH2CH2 H H m/z = 86 H H^I^H O C H C H CH 2 CH 2 CHo n / \ H I H- cleavage ' + III 11 1 -1 O CHo3 Alpha C / H ^CH 2 - I CH2CH2CH3 _ H m/z = 71 m/z = 114 4-Heptanone The isomers can be distinguished on the basis of both a-cleavage products (m/z = 57 vs m/z = 71) and McLafferty rearrangement products (m/z = 72 vs m/z = 86). (c) -1 HoC. I X 5 H HoC O C H OH CH McLafferty H / \ H + rearrangement II CHc I C^ H I CH 3 2-Methylpentanal m/z = 100 CH 3 m/z = 58 Aldehydes and Ketones: Nucleophilic Addition Reactions H O HoC. Alpha H C H H \ O CH 2 cleavage III c / \ CH 3 H I H CH 3 2-Methylpentanal m/z = 100 m/z = 29 H HoC. I .H H 3 C, o McLafferty rearrangement C u r H CH OH II I CH H H HoC HoC' H m/z = 44 H O C H- II I -a v,a H3 C Alpha cleavage O CH. III H. H3C / \ H H 3 C, C CHo H I H m/z = 29 3-Methylpentanal m/z = 100 The fragments from McLafferty rearrangement, which occur serve to distinguish the two isomers. at different values -1 of m/z, 19.26 IR: The only important IR absorption for the compound is seen at 1750 cm where 5membered ring ketones absorb. Mass spectrum: The products of alpha cleavage, which occurs in the ring, have the , same mass m/z as the molecular ion. =140 m/z - 84 491 492 Chapter 19 Visualizing Chemistry 19.27 It helps to Look know that all of these substances were prepared from aldehydes or ketones. for familiar groupings of atoms to identify the starting materials. Notice that the substance pictured is a cyclic acetal. The starting materials were a diol (because cyclic acetals are prepared from diols) and an aldehyde (because an -H is bonded to the acetal carbon). Replace the two -OR groups with =0 to identify the aldehyde starting (a) material (acetaldehyde). acetal (b) We know that the product is an imine because it contains a carbon-nitrogen double bond. The carbon that is part of the C=N bond came from a ketone, and the nitrogen came from a primary amine. (c) The product is an enamine, formed from a ketone and a secondary amine. Nitrogen bonded to the carbon that once bore the carbonyl oxygen. is H— enamine (d) The secondary alcohol product might have been formed by either of two routes reduction of a ketone or by Grignard addition to an aldehyde. - by Aldehydes and Ketones: Nucleophilic Addition Reactions 493 19.28 The intermediate results from the addition of an amine to a ketone. The product is an enamine because the amine nitrogen in the carbinolamine intermediate comes from a secondary amine. C OH o N— H + N— II CH(CH 3 \ H3 C C / CH(CH 3) 2 ) 2 CH 3 C(CH 3 2 ) // N— 3-Methyl-2-butanone 19.29 The nitrogen atom (a) is sp -hybridized, and the geometry (b) A p orbital holds the lone-pair electrons of nitrogen. (c) The p orbital holding the lone-pair electrons is of nitrogen planar. is aligned for overlap with the n electrons of the enamine double bond. With this geometry, the nitrogen lone-pair electrons can be conjugated with the double bond, thus lowering energy. CHo _-. I ft H3 C H3C 1/ /N^ ^CH2 CHo Additional Problems Naming Aldehydes and Ketones 19.30 (a) (b) O X II CH 3 CCH 2 Br HoC Bromoacetone (d) CHO (c) CHO s OH CH 3 CH 2 CH 2 CH 2 CCH(CH 3 2-Methyl-3-heptanone (5')-2-Hydroxypropanal (f) (e) CH 3 HO^l^H CH 3 C^— CHCCH 3 (CH 3) 3 CCC(CH 3 3 ) » O II s VT ^OH O 2,2,4,4-Tetramethyl- 4-Methyl-3-penten-2-one 3-pentanone ; CH 2OH (2S,3/?)-2,3,4-Tri- hydroxybutanal (h) (g) (i) O OHCCH 2 CH 2 CHO Butanedial CH=CHCHO 3 -Pheny 1-2-propenal 6,6-Dimethyl-2,4cyclohexadienone ) 2 494 Chapter 19 O (j) OoN- p-Nitroacetophenone 19.31 Only 2-methylbutanal is chiral. CH< CH 3 CH 2 CHCHO CH 3 CH 2 CH 2 CH 2 CHO 2-Methylbutanal Pentanal (CH 3 3 CCHO CH3 CHCH 2 CHO ) 2,2-Dimethylpropanal 3-Methylbutanal O O ft CH 3 CH 2 CH 2 CCH 3 CH 3 CHCCH 3 CH 3 CH 2 CCH 2 CH 3 CH 3 3-Pentanone 2-Pentanone 3-Methyl-2-butanone 19.32 (b) (a) QHO H^C-OH i CH 2 OH 3-Methyl-3-cyclo- hexenone (/?)-2,3-Dihydroxypropanal (D-Glyceraldehyde) (d) (e) I CHO O II CH 3 CHCH 2 CH OHC CHo 2-Methy 1- 3 -pentanone 19.33 (a) The a,B-unsaturated ketone 2-cyclohexenone (f) OH CH 3 CHCCH 2 CH 3 5-Isopropyl-2-methyl- 3 -Hydroxy butanal C6HsO contains one p-Benzenedicarbaldehyde ring. Possible structures include: HoC CH 3 H 3C Cyclobutenones and cyclopropenones are also possible. Aldehydes and Ketones: Nucleophilic Addition Reactions (b) O O II II CH 3 C — CCH 3 and many other 495 structures. (c) f? C H3C ^CH3 CH 2 CHg CH. CHo HoC (d) CHO s^^J and many other structures. Reactions of Aldehydes and Ketones 19.34 Reactions of phenylacetaldehyde: (b) (a) (c) OH N O aCH (d) II II OH (e) OCHo3 (f) I I CH 2 CHCHg CH 2 CHg CH 2 CHOCH 3 (h) (g) CH 2 CH CH 2 COH 2 CH 2 OH OH I CH 2 CHCN CH 2 CH^- CH2 Reactions of acetophenone: (c) (b) (a) H OH OH f \ / no reaction Width: 612 Height: 792 496 Chapter 19 HO (d) CHo6 (e) HoCO \ / a G, CH, (g) (f) OCHod \ / CH2CH3 CH< CH 2 II 19.35 Remember: RCH 2 — (C 6 H 5 ) 3 P: Triphenyl alkyl halide (C 6H 5 ) 3 PCH 2 R X" phosphonium salt phosphine (C 6 H 5 ) 3 PCH 2 R X phosphonium salt (C 6 H 5 ) 3 P-CHR + - CH3CH2CH2CH2" :+ Li 1 / o=c + \ aldehyde or ketone Aldehyde/ketone Alkyl halide (a) CH 2 Br O O P-CHR ylide Butyllithium ylide CH 2 Br (C 6 H 5 ) 3 R s / C=C \ h' alkene Product Aldehydes and Ketones: Nucleophilic Addition Reactions 19.36 Remember from Chapter 497 17: Primary alcohols are formed from formaldehyde + Grignard reagent. Secondary alcohols are formed from an aldehyde + Grignard reagent. Tertiary alcohols are formed from a ketone (or an ester) + Grignard reagent. Aldehyde/ Ketone (a) Grignard Product (after acidic workup) reagent O II CH 3 CH OH CH 3 CH 2 CH 2 MgBr O I or CH 3 CH 2 CH 2 CHCH 3 CH 3 CH 2 CH 2 CH CH 3 MgBr CH 2 CH 3 CH 2 CH 2 MgBr (b) CH 3 CH 2 CH 2 CH 2 OH OH C 6 H 5 MgBr C6 H 5 C 6 H 5 MgBr or 2 19.37 C 6 H 5 MgBr In general, ketones are less reactive than aldehydes for both steric (excess crowding) and electronic reasons. If the keto aldehyde in this problem were reduced with one equivalent NaBH4, the aldehyde functional group would be reduced in preference to the ketone. For the same reason, reaction of the keto aldehyde with one equivalent of ethylene glycol selectively forms the acetal of the aldehyde functional group. The ketone can then be reduced with NaBH4 and the acetal protecting group can be removed. of O O 1 II . equiv ^ 1 CH3CCH2CH2CH2CHO NaBH 4 ^ II CH3CCH2CH2CH2CH2OH 3 o HOCH0CH0OH 2 2 o- — ' - / nLJ ^' CH OH 3 CCH 2 CH 2 CH 2 C^H ' ' acid catalyst 11. 2. NaBH, H3 O' + OH I CH3 CHCH 2 CH 2 CH 2 CHO P H2 | CHc 498 19.38 Chapter 19 Aldehydes and Ketones: Nucleophilic Addition Reactions 499 19.39 CHO ^s^CH CH 2OH 1. NaBH 4 2. H3 2 Br PBr, + . aCHoCHoOH CH 2 CHO Periodinane CH2CI2 1. CH 2 2. H3 Mg, ether + The product resembles the starting material in having an aldehyde group, but a -CHfebetween the aldehyde and the aromatic ring. The product aldehyde results from oxidation of an alcohol that is the product of a Grignard reaction between formaldehyde and benzylmagnesium bromide. The Grignard reagent is formed from benzyl bromide, which results from treatment of benzyl alcohol with PBr3. Reduction of benzaldehyde group lies yields the alcohol. An alternate route: H I (Ph) 3 a*0 P-CH 2 ' a 1 i CH 2 CHO Periodinane . 2. I BH 3 THF H 2 2 ,"OH , aCH 2 CH 2 OH CH 2 CI 2 from hydroboration of a double bond that introduced by a Wittig reaction between benzaldehyde and methylenetriphenylphosphorane. In this scheme, the intermediate alcohol results (b) 1 p | . CH 3 MgBr 2.H 3 + CHr is 500 Chapter 19 When you see a secondary amine and a double bond, you should recognize an enamine. is formed from the amine and acetophenone. Acetophenone, in turn, results The enamine from reaction of benzaldehyde with methylmagnesium bromide, followed by oxidation. (c) aCH from The CH-P(Ph) 3 2 Br 1 (Ph) 3 P: 2. BuLi Cyclopentanone (a) trisubstituted double bond suggests a Wittig reaction. Reaction of cyclopentanone with the Wittig reagent formed from benzyl bromide (formed from benzaldehyde in (a)) yields the desired product. 19.40 H OH NHCH 3 C6H 5 (f) no reaction (h) I O. \ .0 Aldehydes and Ketones: Nucleophilic Addition Reactions 501 19.41 HO CH 3 H3 + CH 3 MgBr 1 2. + H3 1 -Methylcy clohexene The methyl group introduced by a Grignard reaction with methylmagnesium bromide. Dehydration of the resulting tertiary alcohol produces 1-methylcyclohexene. (b) X is HQ C 6 H 5 MgB 1. 2. H3 ^6 H 5 C6H5 y 1 -p6 H 5 1.BH 3 H 3 Ot r + 2. H2 OH 2 OH , and enantiomer I 1 Cr0 3 + HoO 3* 6 H5 O 2-Phenylcyclohexanone Reaction with phenylmagnesium bromide yields a tertiary alcohol that can be dehydrated. resulting double bond can be treated with BH3 to give an alcohol that can be oxidized to produce the desired ketone. The (c) H 1 2. OH H NaBH, H3 1 + . 2. Os0 4 NaHS0 3 OH , H2 cis- 1 ,2-Cyclohexanediol Reduction, dehydration and hydroxylation yield the desired product, (d) MgBr )H 1.PBr 3 2. from Mg, ether (c) A Grignard reaction forms 1 . 2- Cyclohexanone^ * H3 + 1 -Cyclohexylcyclohexanol 1 -cyclohexylcyclohexanol. 502 Chapter 19 Spectroscopy 19.42 Use Table 19.2 if you need help. Only carbonyl absorptions Due to: Absorption: 1750 1685 1720 (a) (b) are noted. cm-l cm-l cm-l 5-membered ring ketone a^-unsaturated ketone 5-membered ring and aromatic ketone -l 1750 cm -1 1705 cm 2720 1715 cm" 1 (c) (d) , 5-membered cm -1 , 2820 ring ketone aromatic aldehyde aliphatic ketone -1 cm Compounds in parts (b)-(d) also show aromatic ring IR -1 cm - 1600 cm" and in the range 690-900 cm" 1 absorptions in the range 1450 1 . 19.43 HoO + or C 6R n H5 6 n5 B 3-Hydroxy-3-phenylcyclohexanone Compound A is a cyclic, nonconjugated enone whose carbonyl infrared absorption occur at 1715 cm" Compound B is an a,8-unsaturated, cyclic ketone; additional conjugation with the phenyl ring should lower its IR absorption below 1685 cm" Because the actual IR absorption occurs at 1670 cm" B is the correct structure. should . 1 . 1 , 19.44 (b) NMR that the unknown is a ketone, and indicates that the carbonyl C flanked by a secondary carbon and a tertiary carbon. unknown is an aldehyde and contains an isopropyl group. IR shows (a) group is The The IR absorption shows that this compound is an a,p-unsaturated ketone, and the 13 spectrum indicates 3 molecular formula shows 3 degrees of unsaturation. The C 2 s/? -hybridized carbons and 3 secondary carbons. (c) NMR O (a) (b) (c) II (CHg^CHCCH^CHg (CH 3 2 CHCH 2 CHO ) A O A has 4 degrees of unsaturation and is a five-membered ring ketone. The ^C NMR spectrum has only three peaks and indicates that A is very symmetrical. 1 19.45 Compound O O Compound A Aldehydes and Ketones: Nucleophilic Addition Reactions 19.46 As always, calculate the degree of unsaturation assign the principal functional groups. (b) (a) p ell a first, 503 then use the available IR data to Q a b CH 3 CHCCH 3 c || b (CH 3 3 CCH2CCH 3 ) CI a= 1.62 6 a = 1.02 6 b = 2.33 6 b = 2.12 6 c = 4.32 6 c = 2.33 6 General Problems 19.47 (a) + (Ph) 3 P: + BrCH 2 OCH 3 + BuLi (Ph) 3 PCH 2 OCH 3 Br" (Ph) 3 - P— CHOCH 3 + LiBr (b) H-OHj ..+ C/° H— .C CH 3 H d2 H CH HoV^ oV° C p -h 3 + 2. 3. M HO H3 + H 2o: + 5. 6. HOCH 3 Steps 1,4: Protonation. Step 2: Addition of water. Steps 3,6: Deprotonation. Step 4: Loss of CH 3 OH. + 4 H 504 Chapter 19 19.48 4-Hydroxybutanal forms a cyclic hemiacetal when the hydroxyl oxygen adds to the aldehyde group. /**H— HO X." H2 C H 2 C— / , •:o: H2 C 1. ^OH R H A \ H 2 C. II I H HH+H 3 H H hemiacetal Step 1: Protonation. Step 2: Addition of -OH. Step 3: Loss of proton. Methanol reacts with the cyclic hemiacetal to form 2-methoxytetrahydrofuran. H 2 C— Q: \„oh 7 HyA H 2 C, A \\4 A " HH + H2 \< OHc / H2 C L H 2 C— <^> HoC— o:-) ^> H2C A H HH ). HH HOCHo 3. tl H 2 C— O: / HoO + + \ HoC. /\ H H H 2 C— O: H OCH< / \ H H 2 C, A £pCH 3 HH H a cychc acetal Hob:-* Step 1: Protonation. Step 2: Loss of water. Step 3: Addition of methanol. Step 4: Loss of proton. 2-Methoxytetrahydrofuran is a cyclic acetal. The hydroxyl oxygen of 4-hydroxybutanal reacts with the aldehyde to form the cyclic ether linkage. + Aldehydes and Ketones: Nucleophilic Addition Reactions 505 19.49 H0:-n H H— HO: ^6 H 5 intermediate, which loses Br~ 0=C\ CeHsCHB^ on + + HoO * Br C6 H 5 6 n5 + Br" substitution of hydroxide ion Sn2 s- C\J yields an unstable bromoalcohol to give benzaldehyde. 19.50 :CN H3c CN CN HCN I ^c==0 H 3 C'y CHgCHj C ^0- CHgCH 2 C ^OH :CN OH :CN CH3CH2 H3C CH 3 CH 2< ^ CH3CH2 >s H 3 C'y HCN CHgCHp^ - I T CN ^:CN CN Attack can occur with equal probability on either side of the planar carbonyl group to yield a racemic product mixture that is optically inactive. 19.51 H :0: I I I CL H2 H H /J SCoA H 2 o:-^ I :o: 1 ^SCo A + 1. H H H2 + 2: Conjugate addition of water. Proton shift. HO C :o: II C /\ H H :o: H 1: v I _ Step Step H II SCoA SCoA Width: 612 Height: 792 506 Chapter 19 19.52 Hv (PLP) R'O" V ~C0 2 ~ ~T V( pLp (PLP) ) N COc H H Step 1: Elimination to form the unsaturated imine. Step 2: Conjugate addition of cysteine to the imine. 19.53 (a) Grignard addition to a conjugated ketone yields the 1,2 product, not the 1,4 product. to an alcohol. The correct scheme: LiAlH4 reduces a ketone 1 . 2- H 2 NNH; Li(CH 3 ) 2 Cu^ H3 + KOH (b) Oxidation of an alcohol with acidic Cr03 converts primary alcohols to carboxylic acids, not to aldehydes. The correct scheme: C6 H 5 CH=CHCH 2 OH Periodinane ^ ^ » C 6 H 5 CH=CHCHO Treatment of a cyanohydrin with correct scheme: (c) H30 + produces OH HCN^ CHgCCHg I CH3CCH3 CHoOH ^j-** C H CH=CHCH(OCH — 6 5 a carboxylic acid, not an amine. OH 1.LiAIH 4 ~ _ CH3CCH3 ~ ~ I 2 CN 3)2 CH 2 NH 2 The Aldehydes and Ketones: Nucleophilic Addition Reactions 507 19.54 CH2 CH3CCH2CH2COCH2 HOCH 2 CH 2 OH ^ ,0 N fp CH3CCH2CH2COCH2 + 1 . 2. 1. H2C / CH2 2. DIBAH + H3 H2C \ Periodinane H3 + CH2 \ / CH 3 CCH 2 CH 2 CHO UAIH4 CH 3 CCH2CH2CH20H CH2CI2 (C 6 H 5 ) 3 P-C(CH 3 )2 H2C ^^2 O 4 ,0 CH 3 CCH 2 CH 2 CH=C(CH 3 2 H HoO 3 ) * • CH 3 CCH 2 CH 2 CH— C(CH 3 ) 2 6-Methyl-5-hepten-2-one The reaction sequence involves protecting the ketone, converting the ester to an aldehyde, using a Wittig reaction to introduce a substituted double bond, and deprotecting the ketone. 19.55 The same series of steps used to form an ^OH acetal is followed in this mechanism. HO^SCH 2CH 3 3. hemithioacetal Step 1: Protonation. Step 2: Addition of RSH. Step 3: Loss of proton. + CH 3 CH 2 S^) HO: SCH2CH3 H 2° HSCH2CH 3 1. hemithioacetal 3 IT tl CH 3 CH2S» «SCH2CH 3 CH 3 CH2S» - H-*^- :a" SCH2CH 3 HA + thioacetal Step 1: Protonation. Step 2: Loss of water. Step 4: Loss of proton. Step 3: Addition of RSH. 508 Chapter 19 19.56 Even though the product looks unusual, this reaction is made up of steps with which you are familiar. :q: I — S(CH CH 2 :o— CH 2 3) 2 CH2S(CHg)2 + + :S(CH 3 ) 2 2. 1. Step 1: Addition of ylide. Step 2: Sn2 displacement of dimethyl sulfide by O". 19.57 :o: :OH II =^ c 1. H3C Step Step 1: H 3 C 6r CN /C^ 2. + CH 3 H3C Deprotonation by OH. of ~CN. 2: Elimination This sequence Ok is the reverse of the :o: :CN mechanism shown in Section 19.6. HO CN CN HoO + 3. 4. Step 3: Nucleophilic addition of cyanide Step 4: Protonation of tetrahedral intermediate. This step is a nucleophilic addition of cyanide. HO" :cn Aldehydes and Ketones: Nucleophilic Addition Reactions 509 19.58 O CCH2CH3 1? CH 3 CH 2 CCI / \ AICI 3 1 UAIH4 2. H3 + OH \ CHCH^CHg MgBr 1 2. Br2 , FeBr^ ( H H2CHQ 2 ^ Q+ Mg, ether OH ^" pheny 1' 1 -p r°p ano1 MgBr P CHCH2CH3 PBr 3> VSs r"^^ N^' aCHCH2CH CHCH2CH3 3 Mg, ether •J MgBr aCHCHoCHo 1 . C=0 2- (CH 3 2 NCH2CH 2 (CH 3 ) ether H3 ) + CH2CH 3 HO— C— C— 2 NCH 2 CH 2 CH2CH 3 c=c (CH 3 )2NCH 2 CH 2 When you Tamoxifen see a product that contains a double bond, and you also know that one of the is a ketone, it is tempting to use a Wittig reaction for synthesis. In this starting materials case, however, the tetrasubstituted double bond can't be formed by a Wittig reaction because of steric hindrance. The coupling step is achieved by a Grignard reaction between the illustrated ketone and a Grignard reagent, followed by dehydration. The Grignard reagent is synthesized from 1 -phenyl- 1-propanol. which can be prepared from benzene by either of two routes. 510 Chapter 19 19.59 H\ H— u :0^ II t. H HoC H3 C ^ q=c ^^ 2 V CH 3 ^ CH 3 ^ :0: H :OH c - 3 - tl H HA + ,:a- CH 3 1 :o C H ^ H ( , 0+ O: C C o: I H 3 C/ . I ^o" C ^H 5. H 3 C^/ H -o" " ^H 4 " CHo CHo H Paraldehyde Step 1: Protonation makes the carbonyl carbon more electrophilic. Steps 2,3,4: Three successive additions of the carbonyl oxygen of acetaldehyde to the electrophilic carbonyl carbon, followed by loss of a proton (Step 5), give the cyclic product. 19.60 :o: \/ " II Al CH3CCH3 C(CH 3 2 ) ,_ r 2. Aluminum, a Lewis acid, complexes with the carbonyl oxygen. 2: Complexation with aluminum makes the carbonyl group more electrophilic and facilitates hydride transfer from isopropoxide. Step Step 1: Step 3: Treatment of the reaction mixture with aqueous acid cleaves the aluminum-oxygen bond and produces cyclohexanol. Both the Meerwein-Ponndorf-Verley reaction and the Cannizzaro reaction are hydride transfers in which a carbonyl group is reduced by an alkoxide group, which is oxidized. Note that each aluminum triisopropoxide molecule is capable of reducing three ketone molecules. Aldehydes and Ketones: Nucleophilic Addition Reactions 19.61 (a) 511 Nucleophilic addition of one nitrogen of hydrazine to one of the carbonyl groups, followed by elimination of water, produces a hydrazone. HoC 3 ^ T ff / C ^ C>^ ft if -« CHo3 • C^. HoC 1 1. 9f* ^ c« T 53 R HH +^h H3 C H H H 2N~NH 2 / HH H 2 G + h 2 n: 2. H 2 N— NH 2 _ 3. tl :a ("OH. .£>H-A :OH I? f? I c H3 C CH 3 C H 3C 5. >A CH3 ff H H HH N 4. — NH 2 H3 C \ CH3 HN— NH2 & |_j |_| H HoN \: n: C.\ HoC 3 C y ^ v. + HA CH /\ hydrazone H H manner, the other nitrogen of hydrazine can add to the other carbonyl group of 2,4-pentanedione to form the pyrazole. (b) In a similar H— A LA : ^ u H" NH 2 \ ^:q: H 2 0+ HO + x/ CHo3 A HH HoC 1 A HH CHg 2 . CH. JP-H-, « tl' \ r :N— n: y C\ / C v £C CH 3 5. H 3C H H CH 3 /\ 4. HoC HH C CH, H H :n— n: w / H 3C C H H H If H3C 1 /\ 1^ n: H3C A" \ n: C HoC 3 : C + HA 3,5-Dimethylpyrazole CH 3 H The driving force behind this reaction is the formation of an aromatic ring. The reactions in both parts of this problem are nucleophilic addition of a primary amine (Step 2), followed by elimination of water to yield an imine or enamine (Step 5). All of the other steps are protonations (Steps 1,4) and deprotonations (Steps 3,6). 512 Chapter 19 19.62 The same sequence of steps used in the previous problem leads to the formation of 3,5dimethylisoxazole when hydroxylamine is the reagent. Loss of a proton in the last step of (b) results in a ring that is aromatic. (a) H-A B C HoC CHo3 M l HoC H3 C A H H HH £ H H \ CH3 H 2N" 0H H 2 N— OH H2 + HQ C ^ A" ^ f? H3 C : f? CHo 5 H H H3 C C H H A :6h CH 3 IN— OH I H tl HO. n: HoC 3 + HA C CHo3 H H oxime A 4. H3 C C H H h— \ CH 3 HN— OH Aldehydes and Ketones: Nucleophilic Addition Reactions 513 19.63 Go: R u :p: H 1,1 rotate ' H • R (Ph) 3 j I H (Ph) 3 P w H R' ^C-rC" 180° (Ph) 3 P :p: / c=c / 3. 2. H \ H R \ R' + P:^ (Ph) 3 P=0 Step 1: Addition of the phosphine nucleophile. Step 2: Rotation of C-C bond. Step 3: Elimination of triphenylphosphine oxide. final step is the same as the last step in a Wittig reaction. converts a trans alkene to a cis alkene. The The same series of steps 19.64 HO— O— H + :pH Hydrogen peroxide and hydroxide :p— OH react to + H 2p: form water and hydroperoxide anion. "OH Conjugate addition of hydroperoxide anion (Step 1) is followed by elimination of hydroxide ion, with formation of the epoxide ring (Step 2). 514 Chapter 19 19.65 NAD + : Base NADH ^ HUNH2 "0 ' 2C + c + oxidation of CO, the amine by "0 2C CO. NAD+ Glutamate H 2 0:- — imine addition of water to form a tetrahedral J intermediate :NHc " C0 2 "OoC proton HO- shift "0 2C HoO + CO, loss of ammonia : I "0 Base Q+ : 2C deprotonation CO, + NH 3 COc "OoC a-Ketoglutarate of Compound A shows that the molecular formula of A is C5H10O (one degree of unsaturation), and the IR absorption shows that is an aldehyde. The uncomplicated is that of 2,2-dimethylpropanal. 19.66 The molecular weight A H NMR (CH 3 ) 3 CCH T 1.2 6 9.7 5 Compound A 19.67 The IR of Compound B shows H NMR a ketone absorption. The splitting pattern of the spectrum indicates an isopropyl group and indicates that the compound is a methyl ketone. 2.4 5 O CHgCHCCHg 2.1 6 CHo t 1.2 6 Compounds Aldehydes and Ketones: Nucleophilic Addition Reactions 515 H NMR at the spectrum, we know that the compound of formula C9H10O has 5 degrees of unsaturation, and we know from the IR spectrum that the unknown is an aromatic ketone. The splitting pattern in the spectrum shows an ethyl group next to a ketone, according to chemical shift values. 19.68 Before looking H NMR 19.69 The IR two absorption is that of an aldehyde that isn't conjugated with the aromatic ring. The H NMR spectrum are due to two adjacent methylene groups. triplets in the O aCh^Ch^CH 19.70 (a) a = 1.44 6 b = 4.08 5 c,d = 6.98 5, 7.81 6 e = I 9.87 6 a= a b OCH 2 CH 3 b,c 1.86 6 = 6.00 d = 6, 6.31 6 9.57 6 (b) O II (CH 30)2CHCH 2 CCH3 d c b a= 2.18 5 b = 2.47 6 b = 2.74 6 c = 3.66 6 c d = 7.28 5 d = = 3.37 6 4.79 6 a Width: 612 Height: 792 516 Chapter 19 19.72 CH3O..O + CH 3 NH 2 CH 3 NH 2 H3 C Q ! CU.' CH 3 NH\^ C P —x HoC H3 C ^ CH3O CH 3°\ CH 3<\ CHoO O CH3O o: 3. t( CH3OH H3C CH3OH N O H 3 C. H3 C t C. 2H CHo 3-Methyl-2-butenoic acid 1,3-Cyclopentadienecarboxylic acid OH I o H3C H (5)-3-Cyclopentyl-2methylpropanoic acid 20.18 (b) C0 2 H C0 2 H (CH 3 2 N ) (a) /7-Bromobenzoic acid is more acidic than benzoic acid because the electron-withdrawing bromine stabilizes the carboxylate anion. (b) This /^-substituted aminobenzoic acid is less acidic than benzoic acid because the electron-donating group destabilizes the carboxylate anion. 20.19 H H HoC 3 CHo 3 H H HoC CHo3 v V V HO C C0 2 H HO HH HI C K Br H H be used to synthesize the above carboxylic acid because the tertiary halide precursor (shown on the right) doesn't undergo Sn2 substitution with cyanide. Grignard carboxylation also can't be used because the acidic hydroxyl hydrogen interferes Nitrile hydrolysis can't with formation of the Grignard reagent. If the hydroxyl group Grignard carboxylation can take place. 20.20 The electrostatic potential maps show is protected, however, that the aromatic ring of anisole (red) than the aromatic ring of thioanisole, indicating that the is more electron-rich is more methoxyl group strongly electron-donating than the methylthio group. Since electron-donating groups decrease acidity, /?-(methylthio)benzoic acid methoxybenzoic acid. is likely to be a stronger acid than p- Width: 612 Height: 792 526 Chapter 20 Additional Problems Naming Carboxylic Acids and Nitriles 20.21 (a) COoH 2 I COoH 2 (b) CHo3 I I CH 3 CC0 2 H CHgCHCH 2 CH 2 CHCH3 CH 3 2,2-Dimethylpropanoic acid 2,5-Dimethylhexanedioic acid (d) (c) C0 2 H C0 2 H m-Cyanobenzoic acid (e) CHo (£)-2-Cyclodecenecarboxylic acid ® CH 2 C02 H CH3CH 2 CH 2 CHCH 2 CH3 CHoCCN 3 | CH 3 3-Ethylhexanoic acid 2,2-Dimethylpropanenitrile (g) (h) Br CN I BrCH 2 CHCH 2 CH 2 C02H 4,5-Dibromopentanoic acid 2-Cyclopentenecarbonitrile 20.22 (a) H (b) H0 2 CCH 2 CH 2 CH 2 CH 2 CH 2 C0 2 H \/T^C02 H Heptanedioic acid H cis- 1 ,2-Cyclohexanedicarboxylic acid (d) (c) CH 3 C= CCH= CHC0 2 H 2-Hexen-4-ynoic acid CHgCH 2 (pH 2 CH 2 CHg CH3CH 2 CH 2 CH 2 CHCH 2 CHC0 2 H 4-Ethyl-2-propyloctanoic acid Carboxylic Acids and Nitriles (e) (f) C0 2 H (C 6 H 5 ) 3 CC02 H Triphenylacetic acid C0 2 H 3-Chlorophthalic acid (h) (g) CN m-Benzoylbenzonitrile 2-Cyclobutenecarbonitrile 20.23 (a) CH 3 CH2CH2CHCH2CO2H CH2CH2CH2CHCO2H CH2CH2CH2CH2CH2CO2H 2-Methylpentanoic acid Hexanoic acid 3-Methylpentanoic acid CH2OH3 CHo3 THF CH2CO2H OH H3 H3 + + CHgC^C^C-~ CH2CH2OH OH CH, CH 3 CH 2 CH 2 C= CHC0 2 H 3-Methyl-2-hexenoic acid As in all of these more complex syntheses, other routes to the target compound are was chosen because the Grignard reaction introduces a double bond possible. This route without removing functionality at carbon 3. Dehydration occurs in the desired direction produce a double bond conjugated with the carboxylic acid carbonyl group. to Spectroscopy 20.43 The peak at 1.08 6 is acid group. due to a terr-butyl group, The compound is and the peak at 11.2 6 is due to a carboxylic 3,3-dimethylbutanoic acid, (CH3)3CCH2CC>2H. Carboxylic Acids and Nitriles 20.44 U NMR Either C or carboxylic acids. *H 535 NMR can be used to distinguish among these three isomeric Compound Number of !H NMR Splitting C NMR absorptions absorptions signals Number of 13 ] CH 3 (CH 2 3 C02 H ) of H NMR 1 triplet, peak area 3, 1.0 5 1 triplet, peak area 2, 2.4 5 2 multiplets, peak area 4, 1.5 5 (CH3) 2 CHCH 2 C02H peak area 12.0 6 1 singlet, 1 doublet, peak area 6, 1.0 6 1 doublet, peak area 2, 2.4 6 1 multiplet, 1 singlet, peak area 1, 1 singlet, peak area 9, 1.3 6 1 singlet, peak area 1, (CH 3 3 CC0 2 H 1, peak area 1, 1.6 5 12.0 5 ) 20.45 In all 12.1 5 of these pairs, different numbers of peaks occur in the spectra of each isomer, Use (a), (b) either ! H NMR or C NMR to distinguish between the isomers. 1 Number of Compound 13 (a) C NMR Number of ] H NMR absorptions absorptions 3 3 C0 2 H H0 2 C H02 C C0 2 H (b) H0 2 CCH 2 CH 2 C0 2 H CH 3 CH(C0 2 H) 2 (c) Use *H NMR to distinguish between these two compounds. The carboxylic acid proton of CH3CH2CH2CO2H absorbs near 12 6, and the aldehyde proton of HOCH2CH2CH2CHO absorbs near 10 5 and is split into a triplet. Width: 612 Height: 792 536 Chapter 20 H13 NMR and 13 C NMR spectra. (CH3)2C=CHCH2C0 2H shows six absorptions in its C NMR and five in its H NMR spectrum; one of the H NMR signals occurs in the vinylic region (4.5 - 6.5 C NMR spectrum of the unsaturated acid also shows two 6) of the spectrum. The (d) Cyclopentanecarboxylic acid shows four absorptions in both its ! absorptions in the C=C bond region (100-150 20.46 The compound has one degree 6). of unsaturation, which absorption seen in the IR spectrum. a b c d CH3CH2OCH2CO2H a = 1.26 5 b = 3.64 5 c _ 4 14 § d = 11.125 is due to the carboxylic acid General Problems a tertiary alkyl halide and ~CN is a base. Instead of the desired Sn2 reaction of cyanide with a halide, E2 elimination occurs and yields 2-methyl-2pentene. 20.47 2-Chloro-2-methylpentane is CH3CH2CH2CCH3 20.48 Ibuprofen Carboxylic Acids and Nitriles 20.49 (a) Use CO2 instead of NaCN to form the carboxylic acid, or eliminate scheme and form the acid by nitrile hydrolysis. 537 Mg from this reaction (b) Reduction of a carboxylic acid with LiAU-L* yields an alcohol, not an alkyl group. (c) Acidic hydrolysis of the nitrile will also dehydrate the tertiary alcohol. hydrolysis to form the carboxylic acid. Use basic 20.50 OH Step 1: Protonation of acetal oxygen. Step 2: Loss of cyanohydrin. Step 3: Addition of water, followed by deprotonation. ":CN Deprotonation of the cyanohydrin hydroxyl group butanone. is followed by loss of CN, forming 2- 538 Chapter 20 20.51 H 2 0: f-H— OH2 :OH 2 R— C= NH R— c=Nr H H3 + 3 - ll :6h 2 :0 :o: NH 2 R ^ NH R 2. 4. NH 2 R ^NH 2 R amide Step 1: Protonation. Step 3: Proton transfer. The first Step 2: Addition of water. Step 4: Deprotonation. equivalent of water adds to a nitrile to produce an amide. .s H — OH 2 H H :0' II R^j NH 2 R FT NH 2 2. H 2 o: NH/ + O OH R Step Step Step ^ n I :nh 3 Proton transfer. 5: Deprotonation R^ "bH Step Step / HO: Addition of water. 4: Loss of ammonia to the R 4. 2: The second equivalent of water adds amide ""-NH 2 3 - H C 5. 1: Protonation. 3: ^h / H2 + TWa ^ to yield a carboxylic acid, plus ammonium ion. 20.52 C0 2 H HNQ 3 KMnO, H 2 S04 HoO C0 2 H 1. Fe, 1 2. H 3 Q+ "OH PABA Notice that the order of the reactions is very important. If toluene is oxidized first, the nitro group will be introduced in the meta position. If the nitro group is reduced first, oxidation to the carboxylic acid will reoxidize the -NH2 group. Carboxylic Acids and Nitriles 539 20.53 Br c FeBrv — 1 ^ ^_ C H 2 OH . CH 2 Q MgBr Periodinane CH2CI2 \ // 2. H3 + Fenclorac to this compound are possible. The illustrated route was chosen because it introduced the potential benzylic functional group and the potential carboxylic acid in one step. Notice that the aldehyde functional group and the cyclohexyl group both serve to direct the aromatic chlorination to the correct position. Also, reaction of the hydroxy acid + with SOCl 2 converts -OH to -CI and -C0 2 to -COC1. Treatment with 3 regenerates the carboxylic acid. Other routes H H 20.54 Substituent pKa -PCI 2 3.59 -OS02CH 3 3.84 -CH= CHCN 4.03 -HgCH 3 4.10 -H 4.19 -Si(CH 3 3 ) 4.27 Acidity Most acidic *E.A.S. reactivity Least reactive (most deactivating) Least acidic Most reactive (least deactivating) *Electrophilic aromatic substitution Recall from Section 20.4 that substituents that increase acidity also decrease reactivity in electrophilic aromatic substitution reactions. Of the above substituents, only -Si(CH3)3 is an activator. 540 Chapter 20 Again, other routes to this compound are possible. The above route was chosen because it has relatively few steps and because the Grignard reagent can be prepared without competing reactions. Notice that nitrile hydrolysis is not a possible route to this compound because the halide precursor is tertiary and doesn't undergo Sn2 substitution. The product results from two Grignard to this compound. reactions. As in (a), nitrile hydrolysis is not a route 20.56 As we have seen throughout this book, the influence of substituents on reactions can be resonance effects. For m-hydroxybenzoic acid, the negative charge of the carboxylate anion is stabilized by the electron-withdrawing inductive effect of -OH, making this isomer more acidic. For p-hydroxybenzoic acid, the negative charge of the anion is destabilized by the electron-donating resonance effect of -OH that acts over the due to inductive effects and/or k electron system of the ring but is not important for m-substituents. Carboxylic Acids and Nitriles 541 O" H2 + and other resonance forms 20.57 C0 2 H CHO (a) H3 BH 3 + ); THF, then H 2 2 OH"; (b) PBr3 (c) Mg, then C0 2 + (d) LiAlH 4 then H 3 (e) Dess-Martin periodinane, , , , , ; ; then + H3 CH2 C12 ; XN, then H2 NNH2 ,KOH (or (f) 20.58 :OH :o: :.OH • H 3C Br HoC h3C \^ Nucleophilic addition observed product. (1), alkyl shift (2), 3 C0 2 H T) and displacement of bromide + Br" (3) lead to the 20.59 + g *\t~> "/ C<^ C\ /CH2 OP2 O i + P04 3 " Hoc II : C0 2 C C H H H H 35 3-Phosphomevalonate 5 -diphosphate * ^C^ /CH 2OP2 f It H H H 35 Isopentenyl diphosphate 542 Chapter 20 20.60 A compound with the formula C4H7N has two degrees of unsaturation.The IR absorption -1 at 2250 cm a identifies this b c CH 3 CH 2 CH 2 C=N 20.61 Both compounds nonequivalent). compound as a nitrile. a= 1-06 6 b = 1.68 5 c = 2.31 6 contain four different kinds of protons (the H2C= protons are acid proton absorptions are easy to identify; the other three The carboxylic absorptions in each spectrum are more complex. It is possible to assign the spectra by studying the methyl group absorptions. The methyl group peak of crotonic acid is split into a doublet by the geminal (CH3CH=) proton, while the methyl group absorption of methacrylic acid is a singlet. The first spectrum (a) is that of crotonic acid, and the second spectrum (b) is that of methacrylic acid. 20.62 (a) From the formula, we know compound has 2 degrees of unsaturation, one of 13 group that absorbs at 183.0 6. The C NMR spectrum that the which is due to the carboxylic acid also shows that no other sp carbons are present in the sample and indicates that the other degree of unsaturation is due to a ring, which is shown to be a cyclohexane ring by symmetry and by the types of carbons in the structure. (b) The compound has 5 degrees of unsaturation, and is a methyl-substituted benzoic The symmetry shown by the aromatic absorptions identifies the compound as p- acid. methylbenzoic acid. 20.63 O II HgC Step 1: Deprotonation Step 2: Decarboxylation + "OH CH3 Step 3: Protonation. This reaction proceeds because of the loss of CO2 and the stability of the enolate anion. Carboxylic Acids and Nitriles 20.64 The following steps take place in the Ritter reaction: Step 1: Protonation of the alkene double bond; Step 2: Attack of the nitrogen lone pair electrons on the carbocation; Step 3: Attack of water on the nitrile carbon; Step 4: Deprotonation; Step 5: Tautomerization to the ketone. 543 Chapter 21 - Carboxylic Acid Derivatives: Nucleophilic Acyl Substitution Reactions Chapter Outline I. Introduction to carboxylic acid derivatives (Sections 21.1-21.2). A. Naming carboxylic acid derivatives (Section 21.1). 1 . Acid b 2 . halides. acyl group is named first, followed by the halide. For acyclic compounds, the -ic acid or -oic acid of the carboxylic acid name is replaced by -oyl, followed by the name of the halide. i. There are eight exceptions, in which -yl is used c. For cyclic compounds, the -carboxylic acid ending is replaced by -carbonyl, followed by the name of the halide. Acid anhydrides. a. Symmetrical anhydrides are named by replacing acid by anhydride. b Unsymmetrical anhydrides are named by citing the two acids alphabetically, followed by anhydride. a. The . . 3. Esters. a. named by Esters are first identifying the alkyl group and then the carboxylic acid group, replacing -oic acid by -ate. 4. Amides. 5 Amides with an unsubstituted -NH2 group are named by replacing -oic acid or —ic acid by -amide or by replacing -carboxylic acid with -carboxamide. b If nitrogen is substituted, the nitrogen substituents are named in alphabetical order, and an N- is put before each. Thioesters. a. . . a. Thioesters are named like esters, using the prefix thio- before the name of the ester derivative of the carboxylic acid. 6. Acyl phosphates. a. Acyl phosphates are named by citing the acyl group and adding the word phosphate. B. Nucleophilic acyl substitution reactions (Section 21.2). 1 Mechanism of nucleophilic acyl substitution reactions. a. nucleophile adds to the polar carbonyl group. b The tetrahedral intermediate eliminates one of the two substituents originally . A . bonded c. to it, resulting in a net substitution reaction. Reactions of carboxylic acid derivatives take this course because one of the groups bonded to the carbonyl carbon is a good leaving group. The addition step is usually rate-limiting. Relative reactivity of carboxylic acid derivatives. d. 2 . a. Both steric and electronic factors determine relative reactivity. group decreases reactivity. i. Steric hindrance in the acyl ii. More polarized acid derivatives are more reactive than less polarized derivatives. hi. The effect of substituents on reactivity is similar to their effect on electrophilic aromatic substitution reactions. b. It is i. 3 . possible to convert more reactive derivatives into less reactive derivatives. In order of decreasing reactivity: acid chlorides thioesters > esters > amides. > acid Only esters, amides, and carboxylic acids are found ii. Kinds of reactions of carboxylic acid derivatives: a. anhydrides in nature. Hydrolysis: reaction with water to yield a carboxylic acid. > Carboxylic Acid Derivatives: Nucleophilic Acyl Substitution Reactions b . c. 545 Alcoholysis: reaction with an alcohol to yield an ester. Aminolysis: reaction with ammonia or an amine to yield an amide. Reduction. i. Reaction with a hydride reducing agent yields an aldehyde or an alcohol, ii Amides are reduced to yield amines. Reaction with an organometallic reagent to yield a ketone or alcohol. e. Reactions of carboxylic acids and their derivatives (Section 21.3-21.9). A. Nucleophilic acyl substitution reactions of carboxylic acids (Section 21.3). Carboxylic acids can be converted to acid chlorides by reaction with SOCI2. 1 a. The reaction proceeds through a chlorosulfite intermediate. 2 Acid anhydrides are usually formed by heating the corresponding carboxylic acid to remove 1 equivalent of water. d. II. . . 3 . Conversion to esters. a. Conversion can be effected by the Sn2 reaction of a carboxylate and an alkyl halide. b . Esters can be produced an alcohol. by the acid-catalyzed reaction of a carboxylic acid and This reaction is known as a Fischer esteriflcation. Mineral acid makes the acyl carbon more reactive toward the alcohol. iii. All steps are reversible. iv. The reaction can be driven to completion by removing water or by using a large excess of alcohol. v Isotopic labelling studies have confirmed the mechanism. 4. Conversion to amides. a. Amides are difficult to form from carboxylic acids because amines convert carboxylic acids to carboxylate salts that no longer have electrophilic carbons. b The reagent DCC (dicyclohexylcarbodiirnide) can be used; it is used in the laboratory to form peptide bonds. 5 Reduction of carboxylic acids. Reduction to alcohols can be achieved by use of L1AIH4. a. b BH3 in THF easily reduces carboxylic acids to alcohols. B. Chemistry of carboxylic acid halides (Section 21.4). Carboxylic acid halides are prepared by reacting carboxylic acids with either SOCI2 1 or PBr3 to form the corresponding acyl halide. 2. Acyl halides are very reactive. a. Most reactions occur by nucleophilic acyl substitution mechanisms. 3. Hydrolysis. Acyl halides react with water to form carboxylic acids. a. b The reaction mixture usually contains a base to scavenge the HC1 produced. i. ii. . . . . . . 3 . 4. Anhydride formation. Acid halides react with carboxylate ions a. to form anhydrides. Alcoholysis. Acyl halides react with alcohols to form esters. a. b. Base is usually added to scavenge the HC1 produced. Primary alcohols are more reactive than secondary or tertiary alcohols, It's often possible to esterify a less hindered alcohol selectively. i. Aminolysis. Acid chlorides react with ammonia and amines to give amides. a. b Either two equivalents of ammonia/amine must be used, or NaOH must be present, in order to scavenge HC1. Reduction. a. LiAlH4 reduces acid halides to alcohols. i. The reaction is a substitution of H~ for CF that proceeds through an c. 5 . . 6. intermediate aldehyde, which is then reduced. Width: 612 Height: 792 546 Chapter 21 Reaction with organometallic reagents. Reaction with Grignard reagents yields tertiary alcohols and proceeds through a. an intermediate ketone. b Reaction with diorganocopper (Gilman) reagents yields ketones. i. Reaction occurs by a radical mechanism. ii. This reaction doesn't occur with other carboxylic acid derivatives. C. Chemistry of carboxylic acid anhydrides (Section 21.5). 1 Acid anhydrides can be prepared by reaction of carboxylate anions with acid 7 . . . chlorides. 2 . a. Both symmetrical and unsymmetrical anhydrides can be prepared by this Acid anhydrides react more slowly than acid chlorides. a. Acid anhydrides undergo most of the same reactions as acid chlorides. route. Acetic anhydride is often used to prepare acetate esters. In reactions of acid anhydrides, half of the molecule is unused, making anhydrides inefficient to use. D. Chemistry of esters (Section 21.6). Esters can be prepared by: 1 a. Sn2 reaction of a carboxylate anion with an alkyl halide. b. Fischer esterification. c. Reaction of an acid chloride with an alcohol, in the presence of base. 2 Esters are less reactive than acid halides and anhydrides but undergo the same types b. c. . . of reactions. 3. Hydrolysis. a. Basic hydrolysis (saponification) occurs through a nucleophilic acyl substitution mechanism. i. Loss of alkoxide ion yields a carboxylic acid which is deprotonated to give a carboxylate anion. Isotope-labelling studies confirm this mechanism. Acidic hydrolysis can occur by more than one mechanism. i. The usual route is by the reverse of Fischer esterification. Aminolysis. a. Esters can be converted to amides by heating with ammonia/amines, but it's easier to start with an acid chloride. Reduction. a. LiAlFLt reduces esters to primary alcohols by a route similar to that described for acid chlorides. at -78 °C is used, reduction yields an aldehyde. b If DIB Reaction with Grignard reagents. a. Esters react twice with Grignard reagents to produce tertiary alcohols containing ii. b. 4. 5 . AH . 6 . two identical substituents. E. Chemistry of amides (Section 21.7). Amides are prepared by the reaction of acid chlorides with ammonia/amines. 1 2. Hydrolysis. a. Hydrolysis occurs under more severe conditions than needed for hydrolysis of . other acid derivatives. b . c. 3 . Acid hydrolysis occurs by addition of water to a protonated amide, followed by loss of ammonia or an amine. Basic hydrolysis occurs by attack of HO~, followed by loss of ~NH2- Reduction. a. LiAlH4 reduces amides to amines. Carboxylic Acid Derivatives: Nucleophilic Acyl Substitution Reactions 547 F. Thiol esters and acyl phosphates (Section 21.8). 1 Nature uses thiol esters and acyl phosphates in nucleophilic acyl substitution reactions because they are intermediate in reactivity between acid anhydrides and . esters. Acetyl CoA is used as an acylating agent Polyamides and polyesters (Section 21.9). A. Formation of polyesters and polyamides. When a diamine and a diacid chloride react, a polyamide is formed. 1 2 When a diacid and a diol react, a polyester is formed. 3 These polymers are called step-growth polymers because each bond 2 III. . . . . B is formed independently of the others. Types of polymers. . 1 . 2 . Nylons are the most common polyamides. The most common polyester, Dacron, is formed from dimethylterephthalate and ethylene glycol. Biodegradable polymers are usually polyesters of naturally-occurring hydroxy carboxylic acids. IV. Spectroscopy of carboxylic acid derivatives and nitriles (Section 21.10). 3 . A. Infrared spectroscopy. All of these compounds have characteristic carbonyl absorptions that help identify 1 them; these are listed in Table 21.3. B spectroscopy is of limited usefulness in distinguishing carboxylic acid . . NMR derivatives. 1 . Hydrogens next to carbonyl groups absorb at around but this absorption can't be used to distinguish 2. among ! H NMR spectrum, carboxylic acid derivatives. 2. 1 6 in a Carbonyl carbons absorb in the range 160-180 5, but, again, be used to distinguish among carboxylic acid derivatives. this absorption can't Solutions to Problems 21.1 lists the suffixes for naming carboxylic acid derivatives. the functional group is part of a ring are in parentheses. Table 21.1 when (b) (a) The suffixes used (c) P 4-Methylpentanoyl Cyclohexylacetamide chloride Benzoic anhydride Isopropyl cyclopentanecarboxylate Isopropyl 2-methylpropanoate Cyclopentyl 2-methylpropanoate 548 Chapter 21 (h) (g) (i) O SCH 2 CHg HgC^ HoC. H 2 C=CHCH 2 CH 2 CNHCH 3 / HO N-Methyl-4-pentenamide 21.2 (a) O O ^ As OP03 C V 2- C=C \ / H (/?)-2-Hydroxypropanoyl phosphate (b) Ethyl 2,3-dimethyl2-butenethioate (C) CH 3 CH 2 CH 2 CNCH 2 CH 3 O CH 3 CH 3 CHCH2 CHCCI CH- Phenyl benzoate Af-Ethyl-/V-methylbutanamide 2,4-Dimethylpentanoyl chloride (d) (e) CH- OCH. O O II II CHgCH 2 CCH 2 COCH 2 CH 3 Methyl 1-methylcyclo- SCH. Ethyl-3-oxopentanoate hexanecarboxylate Methyl /7-bromobenzenethioate (h) (g) O O II II h /^LrfCOBr HC^ ^CCH 2CHg H Formic propanoic cw-2-Methylcyclopentanecarbonyl bromide anhydride 21.3 :OCH 3 c :o: :o: II c CI rocH 3 OCH3 + addition of methoxide form a tetrahedral to intermediate elimination of Cl~ cr Carboxylic Acid Derivatives: Nucleophilic Acyl Substitution Reactions 21.4 Use Figure 2 1 .2 if you need help. Most reactive ( a) 549 Least reactive O ? II > CH 3 CCI (b) CH3COCH3 O > CH3CNH2 o O II CH3 COCH(CF3 The most reactive 2 ) is to be the leaving group. Identify the nucleophile (boxed) group by the nucleophile (a) CH 3 COCH 2 CH 3 > acyl derivatives contain strongly electron-withdrawing groups in the part of the structure that 21.5 CH 3 COCH 2 CCI 3 > and the leaving group (circled), O O + Na !"OH II HOCH3 c. h3c (b) 0"Na H3 C .;och 3 ; o O 2 NHo' ; NH 4+ I HoC (c) O /''0 II (d) / \ II + Na CI" O !_"OCH3j 1? II CH 3 /' \0 HoC and replace the leaving in the product. HoC OCH- + Na ~ O O HCI II .C. II .C^ HOCHpCHoCHoCHo + OH H3 C 1-Butanol Acetic acid + OCH2CH2CH2CH3 H3 C H2 Butyl acetate (b) HCI ?\ + HOCH3 OH CH3CH2CH2 ft 5== (C> H V/ ff CH3 H S/ H HCI 2-Propanol Cyclopentane- H2 Methyl butanoate Methanol Butanoic acid + ^ OCH3 CH3CH2CH2 CH 3 Isopropyl cyclopentanecarboxylate carboxylic acid 21.8 Under Fischer esterification conditions, many hydroxycarboxylic acids can form intramolecular esters (lactones). B 8 \ 2 C C ^OH HCI^ H2°^ C I C C /\ /\ HH HH 5-Hydroxypentanoic acid 21.9 I + HoO HpC^ ^.CH20H H2C,. a lactone Pyridine neutralizes the HCI byproduct by forming pyridinium chloride. This neutralization removes from the product mixture acid that might cause side reactions. As mentioned previously, positioning the reacting groups so that they face each other makes it easier to predict the products. < a) O O Pyridine II + CH3CH2 CH3CH2 CI Propanoyl chloride II HOCH3 Methanol OCH3 Methyl propanoate Carboxylic Acid Derivatives: Nucleophilic Acyl Substitution Reactions 551 (b) H3C ./Cn H3C CI Acetyl chloride (c) Pyridine HOCH 2 CH 3 Ethanol Ethyl acetate O Pyridine II HOCH 2 CH 3 cr CI Benzoyl chloride 21 1 . OCH2CH3 OCH 2 CH 3 Ethyl benzoate Ethanol As explained in the text, only simple, low boiling alcohols are convenient to use in the Fischer esterification reaction. Thus, reaction of cyclohexanol with benzoyl chloride is the preferred method for preparing cyclohexyl benzoate. 'V O Pyridine CI HO + Cyclohexanol Benzoyl chloride Cyclohexyl benzoate 21.11 :OH nucleophilic addition of morpholine OCH- deprotonation by hydroxide O ch3 °yyH' OCH,J Trimetozine + 1 NaCI elimination or chloride OCH- + HoO Chapter 21 552 21 12 . An extra equivalent of base and reactions. In (a) (a) (b), must be added to neutralize the acid produced in these two equivalents of the amine may be used in place of NaOH. O CH 3 CH 2 f >a NaOH H 2 NCH 3 + (b) NaCI O O HN(CH 2 CH 3 NaOH ) II 2 a^N(CH CH CI 2 + Benzoyl chloride (c) + N-Methylpropanamide Methylamine Propanoyl chloride H2 + NHCH 3 CH3 CH 2 CI H2 + 3) 2 NaCI AfAf-Diethylbenzamide Diethylamine O II /C^ CH 3 CH 2 2 + n NH 3 CH 3 CH 2 CI orNH4+^CI NH 2 Propanamide Propanoyl chloride 21.13 Two combinations + of acid chloride and organocopper reagent are possible, (a) ^r\.C^ + [(CH 3 ) 2 CH] 2 CuLi c. CH(CH 3 ) 2 of /C^ • CH(CH 3 Cr ) 2 (b) B H 2 C=CH /C^ CI + (CH 3 CH 2 CH 2 2 CuLi ) _ — CH or H2 C O (H 2 C=CH) 2 CuLi ft ^C^ + CI CH 2 CH 2 CH 3 /C^ CH 2 CH 2 CH 3 Carboxylic Acid Derivatives: Nucleophilic Acyl Substitution Reactions 553 21.14 Acetaminophen Step 1: Nucleophilic addition of /7-hydroxy aniline. Step 2: Deprotonation by hydroxide. Step 3: Loss of acetate ion. 21.15 Phthalic anhydride The second half of the anhydride becomes a carboxylic 21.16 acid. Acidic hydrolysis of an ester is a reversible reaction because the products are an alcohol and a carboxylic acid. Basic hydrolysis of an ester is irreversible because its products are an alcohol and a carboxylate anion, which has a negative charge and does not react with nucleophiles. 554 Chapter 21 21.17 J . 2. UAIH4 H3 A (a) 1 DIBAH 2. H3 + O + 21.18 Lithium aluminum hydride 9 CH 2 OH reduces an ester to form two alcohols, H HoC 3f ff CH 3 CH 2 CH 2 CHCOCH3 1 . UAIH4 H3Q + 1 CH3CH2CH2CHCH2OH Methanol 2-Methyl- 1 -pentanol 1 2. H3 HO^^/n. CH 2 OH UAIH4 CH 3 OH + + Benzyl alcohol Phenol 21.19 Remember that Grignard reagents can only be used with esters to form a tertiary alcohol two substituents, which come from the Grignard reagent, and work backward to select the ester (the alkyl group of the ester is unimportant). two that has identical substituents. Identify these Grignard Reagent Tertiary Alcohol + Ester (a) OR 2 CH 3 MgBr O H3 C MgBr OR O (c) » x CH 3 CH2 XyCh^Ch^ i 2 CH 3 CH2CH2CH2 OH CH 3 CH 2 MgBr + CH 3 CH 2 CH 2 CH 2 OR Carboxylic Acid Derivatives: Nucleophilic Acyl Substitution Reactions 555 21.20 O (a) Cv NHCH 2 CH 3 j^^J^ H3 CH 2 OH + heat Af-Ethylbenzamide (c) 1 . 2. UAIH4 H2 aCH 2 NHCH 2 CH 3 21.21 MgBr 1 . 2- CQ 2 H3 , ether + a OH SOCIc CH 2 N(CH 3 ) J. 2. 2 LiAIH 4 H2 N(CH 3 2 ) (CH 3 2 NH CI ) NaOH The product is a ./V-disubstituted amine, which can be formed by reduction of an amide. The amide results from treatment of an acid chloride with the appropriate amine. The acid chloride is the product of the reaction of SOCI2 with a carboxylic acid that is formed by carboxylation of the Grignard reagent synthesized from the starting material. Width: 612 Height: 792 556 Chapter 21 21.22 Even though the entire molecule in this problem only with the of coenzyme A is biologically important, we are concerned group. The remainder of the structure is represented here -SH as "R". Ho: O \ii ii HoC f O ( : O O". II O I — Adenosine H3 C O" O / s 1. O I — Adenosine o- N R RS— H^lBase J*- t 1? n S H3C Acetyl o O l_ — Adenosine CoA 1: Nucleophilic addition of the -SR group of adenylate to form a tetrahedral intermediate. Step CoA (after deprotonation) to acetyl Step 2: Loss of adenosine monophosphate. 2 1 .23 In each example, if n molecules of one component react with n molecules of the other component, a polymer with n repeating units is formed, and 2n small molecules are formed as byproducts; these are shown in each reaction. (a) BrCH 2 CH 2 CH 2 Br + HOCH 2 CH 2 CH 2 OH / \ -^-CH 2 CH 2 CH 2 OCH 2 CH 2 CH 2 0-^- Base + < b> 2n HBr HOCH 2 CH 2 OH + H0 2 C(CH 2 6 C02 H II f -V- ) \ catalyst ) o / H 2 N(CH 2 6 NH 2 ) + CIC(CH 2 4 CCI ) m 2n H 2 (c) II II OCH 2 CH 2 0— C(CH 2 6 C-/+ II 0\ O / H 2 S0 4 o -V-HNC(CH 2 6 NH ) + \ — C(CH2 4C—h II II [ 2n ) HCI Carboxylic Acid Derivatives: Nucleophilic Acyl Substitution Reactions 557 21.24 o O II HOC \_J^ C0H + 1,4-Benzenedicarboxylic acid 21.25 Use Table 21.3 if you need 21.26 cm JR 1735 1,4-Benzenediamine Functional group present Saturated ester or 6-membered ring lactone Saturated acid chloride 2500 - 3300 cm" 1 and 1710 (d) 1715 | ^/ help. Absorption -l (a) 1735 cm -l (b) 1810 cm (c) H2N -1 cm -l -l Carboxylic acid Saturated ketone or 6-membered ring ketone cm-1 corresponds to a saturated ester. and twelve hydrogens can be arranged in a number of ways to produce a structure for this compound. For example: (a) The remaining five carbons CH3CH2COCH2CH2CH3 The structural m o formula indicates that this or compound 11 CH 3 CN(CH 3 (c) ) 2 O O II CH 3 CH = CHCCI II or HCOCH2CH2CH2CH2CH3 H 2 C=C(CH 3 )CCI can't be a lactone. Chapter 21 558 Visualizing Chemistry 21.27 (a) (b) H H HH fH 3 V C CH, II H 3 CH iV O J\ II HCH3 HH O 3-Methylbutyl benzoate Af,Af-Dimethyl-3-methylbutanamide 21.28 (a) CI * pyridine O oBromobenzoic \ 2-Propanol acid =\\ m CH 3 CHCHc^ / / SOCIc CHr Br OH OCHCHo / * O // Isopropyl o-bromobenzoate This compound can also be synthesized by Fischer esterification of o-bromobenzoic acid with 2-propanol and an acid catalyst. (b) O // 1 . socio *- CH 2 C \ 2. OH 2NHod Cyclopentylacetic acid I y— ch c 2 NH 2 Cyclopentylacetamide 21.29 O O O" I + 2 NH- H 2 C=CHCHCH 2 H 2 C=CHCHCH^ CI | CH 3 NH 3 \ CI H 2 C=CHCHCH 2 CH 3 CHo + starting material is 3-methyl-4-pentenoyl chloride, as indicated tetrahedral intermediate. which eliminates Cl~ by the -CI Ammonia adds to give the observed tetrahedral to yield the NH 4+ cr 3-Methyl-4-pentenamide 3-Methyl-4-pentenoyl chloride The NH 2 above amide. in the intermediate, Carboxylic Acid Derivatives: Nucleophilic Acyl Substitution Reactions 559 21.30 According to the electrostatic potential maps, the carbonyl carbon of acetyl azide is more electron-poor (less red) and therefore more reactive in nucleophilic acyl substitution reactions. Resonance donation of nitrogen lone-pair electrons amide than in an acyl azide. to the carbonyl group is greater in an :o: :o: ^ II ^ NH 2 H 3C I NH 2 H3C Acetamide Additional Problems Naming Carboxylic Acid Derivatives 21.31 (a) (b) (c) . Cyclohexyl cyclohexanecarboxylate Ethyl 2-cyclobutenecarboxylate Succinic anhydride can be drawn for each part of this problem, structures (a) O OCH 2 CH 3 o 21.33 Many . CH3CH2 .0 // O ^CHg // N C \ / c=c\ C=0 H CI / H?C=C CHoC 2 \ CI / CH2CH3 CI Cyclopentanecarbonyl chloride (£)-2-Methyl-2pentenoyl chloride 3-Ethyl-3-butenoyl chloride (b) ,0 ff CHgCh^Ch^C^z CCH2C h^C^- CHCH^- CHC^ cr 1 -Cyclohexenecarboxamide N(CH 3 ) 2 3-Heptynamide N,N-Dimethyl2 ,4-pentadienamide Carboxylic Acid Derivatives: Nucleophilic Acyl Substitution Reactions Nucleophilic Acyl Substitution Reactions 21.34 (a) O II CH 3 CH 2 CCI (b) CH3CH ether O II CH 3 CH 2 CCI (C) <^!i 1 UAIH4 2. H3 + 1 2 CH 3 CH 2 CH 2 OH O II OH . CH 3 MgBr^ I ^ ™ CH3CH2CCI CHgCh^C^CHg^ (d) II CH 3 CH 2 CCI ® H3 + II * CH 3 CH 2 COH OO O II CH 3 CH 2 CCI CHoC0 2"Na + — ^ - + I' *~ I' CH 3 CH 2 C— OCCH 3 HCI 561 562 Chapter 21 21.35 The reagents in parts and (a), (e), (g) don't react with methyl propanoate. (b) LLiAIH, II ~ - CH3CH2COCH3 3 (c) CH 3 CH 2 CH 2OH q+ O + CH3OH OH 1.2 CHoMgBr " CH 3 CH 2 COCH 3 + » CHgCHgCCCH^g + CH3OH 3 (d) o o + —— H3 II II - CH 3 CH 2 COCH 3 CH 3 CH 2 COH + CH3OH m O CH 3 CH 2 COCH 3 21.36 The reagents in parts CH3CH2C H2N + (a), (e), (f), and (g) don't react with (b) CH 3 CH2 CNH 2 (c) g H^ 4 * * CH3CH2CH2NH2 O O II CH 3 CH 2 CNH 2 1 . — CHoMgBr " CH 3 CH 2 CNH 2 + CH 4 3 (d) O " CH 3 CH 2 CNH 2 O — H3 - H + II *~ CH 3 CH 2 COH + NH 4 + propanamide. Carboxylic Acid Derivatives: Nucleophilic Acyl Substitution Reactions 21.37 Dimethyl carbonate is a diester. Use your knowledge of the Grignard way through this problem. reaction to work your Triphenylmethanol The overall reaction consists of three additions of eliminations of methoxide and one protonation. 563 phenylmagnesium bromide, two 564 Chapter 21 21.38 (a) BH 1 CH3CH2CH2CO2H * CH3CH2CH2CH2OH + 3 * ^ CH3CH2CH2CH2OH Periodinane CH 3 CH 2 CH 2 CHO CH2C 2 ' from (a) (C) PBr3 CHgCh^Ch^Ch^OH from NaCN K+ ~OC(CH 3 )3^ _ Ch^Ch^CH^- CH2 (c) W H 3Q CH3CH2CH2CH2CN from CH3CH2CH2CH2CN (c) CHgCh^Ch^Ch^Br from Ch^Ch^Ch^Ch^Br (a) ^nunununuD CH 3 CH2CH 2 CH2Br from , + CH3CH2CH2CH2CO2H (d) | CH 3 NH 2 CH3CH2CH2CH2CONHCH3 * SOCI 2 CH3CH2CH2CH2COCI (g) CH3CH2CH2CH2CN 1 • 2 from(d) (h) CH 3 MgBr ^ Ch^Ch^ChkCh^CCH; ' ^ socio CH3CH2CH2CO2H CH 3 CH 2 CH 2 COCI C6 H 6 I AICI3I aCH2CH 2 CH 2 CH3 ^ ^ 2 NHo SOCIo CH3CH2CH2CO2H || CCH 2 CH 2 CH 3 CH 3 CH 2 CH 2 COCI CH3CH2CH2CONH2 I S0 2 + 2 HCI SOCI 2 + CH 3 CH 2 CH 2 CN Carboxylic Acid Derivatives: Nucleophilic Acyl Substitution Reactions 21.39 (a) HC OCH 2 CH 3 1 . 2. (6) CH 3 ' + O II H 2 C=CHCHCH 2 COCH 3 CH 2 CH 3 , CH 2 CH 3 CH 3 CH 2 MgBr H3 \ + CH 3 CH 2 OH CH 3 LLiAIH, " 3 ' H 2 C= CHCHCH 2 CH 2 OH + CH 3 OH 565 Width: 612 Height: 792 566 Chapter 21 21.40 The reactivity in saponification reactions is influenced by steric factors. Branching and alkyl portions of an ester hinders attack of the hydroxide nucleophile. This effect is less dramatic in the alkyl portion of the ester than in the acyl portion because alkyl branching is one atom farther away from the site of attack, but it is still significant. of esters in both the acyl Most reactive O Least reactive O O CH 3 O CH3COCH3 > CH 3 COCH 2 CH 3 > CH3COCHCH3 > CH3COCCH3 CH, 21.41 2,4,6-Trimethylbenzoic acid 2,4,6-Trimethylbenzoic acid has two methyl groups ortho to the carboxylic acid functional group. These bulky methyl groups block the approach of the alcohol and prevent esterification from occurring under Fischer esterification conditions. possible route to the A methyl ester: 0H 1 . 2. NaOH OCH< CH3I This route succeeds because reaction occurs farther away from the site of steric hindrance. It is also possible to form the acid chloride of 2,4,6-trimethylbenzoic acid and react it with methanol and pyridine. Carboxylic Acid Derivatives: Nucleophilic Acyl Substitution Reactions Reaction of an ester with Grignard reagent produces a tertiary alcohol, not a ketone. (d) O CN .CCH, 1 2. CH 3 MgBr H3 + 567 Chapter 21 43 Step 1: The carboxylic Step 2: The carboxylate oxygen adds Step 3: The amine acid protonates to DCC. DCC to form a reactive intermediate. nitrogen adds to the carbonyl group to yield a tetrahedral intermediate. Step 4: The intermediate loses dicyclohexylurea to produce the lactam. Carboxylic Acid Derivatives: Nucleophilic Acyl Substitution Reactions Step 569 1: Protonation. Step 2: Addition of methanol. Step 3: Proton transfer. Step 4: Loss of ethanol. Step 5: Deprotonation. In acidic methanol, the ethyl ester reacts by a nucleophilic acyl substitution mechanism to yield a methyl ester. The equilibrium favors the methyl ester because of the large excess of methanol present. 21.45 2D f? C (CH 3 3 CO (CH 3 ) ) 3 CO' (CH 3 3 CO^ ) addition elimination of azide of chloride + ^N3 ci~ This reaction is a typical nucleophilic acyl substitution reaction, with azide as the nucleophile and chloride as the leaving group. 570 Chapter 21 Step-Growth Polymers 21.46 Step 1: Water opens the caprolactam ring H form the amino acid intermediate. to .. NH' addition L proton rmg of water shift opening C II o Step 2: Reaction of the intermediate with a second molecule of caprolactam forms a dimer. O .0 H 2 N(CH 2 5 C +N(CH2) 5C ) OH addition of OH amino group to carbonyl group OH II proton 1to H H o ^N^H^sC o // I shift H 2 N(CH 2 5 C-N(CH 2 ) 5 C OH ) nng OH opening and beyond: Reaction of the dimer with caprolactam. This process repeats itself many, many times until the polymer stops growing. Remember that each new bond is formed in a discrete step. Heat forces the equilibrium in the direction of polymer formation. Steps 3 O HN(CH 2 ) 5 O C— N(CH 2 5 CH) nylon 6 n I H 21.47 Look for the O monomer units, which are difunctional compounds, in the polymer. O HOC(CH 2 6 COH ) -^-C(CH 2 ) 6 + H2N C— NH ( CH, } — CH — 2 NH, ( J — NH-}- Qiana + 2n H 2 Carboxylic Acid Derivatives: Nucleophilic Acyl Substitution Reactions 21.48 Hydroxide opens the lactone ring, 571 and the resulting anion can add to a second lactone molecule to produce a polyester. HOCCH0CH0O: | | •• i o o II ° <& HOCCH 2 CH 2 0^\_p II HOCCH 2 CH 2 0— CCH 2 CH 2 0~ repeat many . times O O CCH 2 CH 2 0— CCH 2 CH 2 021.49 The polyimide pictured is a step-growth polymer of a benzene tetracarboxylic acid and an aromatic diamine. O o II II HOC HOC YT COH HoN COH II II o o 1 NHc ,4-Benzenediamine 9 ^ a ^c 1,2,4,5-Benzenetetracarboxylic acid % if o o a polyimide + 2n H 2 Spectroscopy 2 1 .50 In some of these others, either ! pairs, IR spectroscopy alone can differentiate between the isomers. For H NMR or a combination of H NMR and IR data is necessary. (a) O O II CH 3 CH 2 CNHCH 3 Af-Methylpropanarnide IR: 1680 cm" (Af-substituted ] H NMR: 1 amide) one methyl group one ethyl group CH 3 CN(CH 3 2 ) AfiV-Dimethylacetamide 1650 cm" 1 (Af,Af-disubstituted amide) three methyl groups 572 Chapter 21 if (b) HOCH 2 CH 2 CH 2 CH 2 C= N CNH 2 Cyclobutanecarboxamide 5-Hydroxypentanenitrile 3300-3400 cm" 1 IR: 1690 (hydroxyl) cm" 1 (amide) 2250 cm- 1 (nitrile) (c) O O II II CICH 2 CH 2 CH 2 COH CH 3 OCH 2 CH 2 CCI 4-Chloropentanoic acid 3-Methoxypropanoyl chloride 2500-3300 cm" 1 IR: 1810 (hydroxyl) 1710 cm" cm" 1 (carboxylic acid chloride) 1 (carboxylic acid) (d) O O II CH3CH 2 COCH 2 CHg CHgCOCh^Ch^CHg Ethyl propanoate ] H NMR: Propyl acetate two triplets two quartets one singlet one triplet one quartet one multiplet 21.51 The IR spectrum CI indicates that this O II I CH3CHCOCH3 a b c compound has a= 1.69 6 b = 3.79 6 c = 4.41 6 a carbonyl group. 21.52 (a) (b) O I? CHgCH 2 CH 2 CCI a b II N=CCH 2 COCH 2 CH 3 b c c a= 1.00 5 a= 1.32 6 b = 1.75 6 b = 3.51 6 c = 2.86 6 c= 4.27 5 a Carboxylic Acid Derivatives: Nucleophilic Acyl Substitution Reactions 573 General Problems 2 1 .53 A negatively charged tetrahedral intermediate is formed when the nucleophile ~OH attacks the carbonyl carbon of an ester. An electron-withdrawing substituent can stabilize this negatively charged tetrahedral intermediate and increase the rate of reaction. (Contrast this effect with substituent effects in electrophilic aromatic substitution, in which positive charge developed in the intermediate is stabilized by electron-donating substituents.) Substituents that are deactivating in electrophilic aromatic substitution are activating in ester The substituents strongly electron-donating. hydrolysis, as the observed reactivity order shows. electron- withdrawing; -NH2 is Most reactive Y = -N0 2 > -CN and -CHO are Least reactive -ON >-CHO > -Br > -H > -CH 3 > -OCH 3 > -NH 2 21.54 :o: CH 2 0"^ \" R / SCoA CHOH CH 2 OH .C^ " CoAS R -H + — I CHOH ? I j CH 2 0* W CHOH C> R + "SCoA 2. 2- 2- CH 2 OP0 3 : I CH 2 OP0 3 2- CH 2 OP03 ' ; Glycerol 3-phosphate 1-Acylglycerol 3-phosphate Addition of -OH to the fatty acyl CoA (Step 1), followed by loss of -SCoA from the tetrahedral intermediate (Step 2), produces 1-acylglycerol 3-phosphate. 21.55 :o: OH I I FT^rOH H2 HoO: The tetrahedral intermediate : OH HO* T T can eliminate any one of the three -OH groups to reform either the original carboxylic acid or labeled carboxylic acid. Further reaction of water with mono-labeled carboxylic acid leads to the doubly labeled product. 574 Chapter 21 21.56 18 O 18 CHoC d — OH 1. 2. BH H3 18 V** + CHoCHoOH d CH 3 CH 2 COCI tL Pyridine I? 18 CH 3 CH 2 C— OCH 2 CH 3 + H2 Ethyl propanoate 1 Remember that the Q O label appears in both oxygens of the acetic acid starting material. 21.57 :o: F3 C" addition of carboxylic x O :o: - H+ «+ R 0C0CF3 loss •*r^> F3 C of proton II y<" ^°COCF3 R elimination of acid trifluoroacetate FoC (b) The electron-withdrawing ?\ if "OCOCF 3 o fluorine atoms polarize the carbonyl group, R making it more reactive toward nucleophiles. (c) Because trifluoroacetate is a better leaving group than other carboxylate anions, the reaction proceeds as indicated. Carboxylic Acid Derivatives: Nucleophilic Acyl Substitution Reactions 21.58 Formation 575 of the dipeptide: I HoN R H H . ISL + . CO, C H3 N 6 H R + i r H Step 1: r H The carboxylate group from one amino acid adds to DCC to form a reactive intermediate. Step 2: The amino group of the second amino acid adds to the carbonyl group to yield a tetrahedral intermediate. Step 3: The intermediate loses dicyclohexylurea to produce the amide. Proton transfers occur in steps 1 and 3. Width: 612 Height: 792 576 Chapter 21 Formation of the 2,5-diketopiperazine: Addition of carboxylate to DCC. Step 1: Step 2: Intramolecular nucleophilic attack acylating agent. Step 3: Loss of dicyclohexylurea. Proton transfers occur in steps 1 and 2. of the amino terminal end of the amide on Carboxylic Acid Derivatives: Nucleophilic Acyl Substitution Reactions 577 21.59 1 HoC" C\ 2 ;o: 7 I H2 HoC*" C\ C H2 1. :0 a ?+ H"T NH U C O II II H 2 C-" C v ,° 1 O H 2C 2. II H 2 C-*" C\ O O + HoC-^' 3. 4 . HoC" c H 2 C^ C £)NH 2 HO: ho: VNH^3 OH ^NH 2 :OH + 3 + H 0^ 3 .. + + OH H2 by OH HO: GoH ho: :oh ; \ / H 2 C~" Cx HoC C H2 C H2 NH 9. H2 C ^NH NH HoC 8. I H 2 C^ / 7. C s o O H 2 C- C s \. : C II fi o o ^OH HoC + NH, 6. I .NH 2 HoC A summary of steps: Step 1: Protonation Steps 3,5,7,9: Proton transfers Step 6: Nucleophilic addition of -NH2 Step Step Step Nucleophilic addition of 2: This reaction requires high temperatures because the intermediate amide nucleophile and the carboxylic acid carbonyl group is unreactive. 21.60 NH3 Ring opening 8: Loss of H2O 4: is a poor This synthesis requires a nucleophilic aromatic substitution reaction, explained in Section 16.7. NHCOCH 3 NHc (CH 3 ) 3 CO- 1.SnCI 2 2. Nucleophilic aromatic OC(CH 3 , H 3 Q^ * I NaOH "OH Reduction ) 3 of nitro substitution CH 3 COC group Aminolysis 0(CH 3) 3 0(CH 3 ) 3 Butacetin The amide can be formed by the reaction of acetyl chloride with the appropriate amine, which is produced by reduction of the nitro group of the starting material. A nucleophilic aromatic substitution of -F by -OC(CH3)3 can take place because the ring has an electronwithdrawing nitro group para to the to acetylate the amine. site of substitution. Acetic anhydride can also be used 578 Chapter 21 21.61 soci. Formation of 1 acid chloride . SnCI 2 H 3 + , 2. "OH Reduction of nitro group Phenyl 4-aminosalicylate 21.62 HoC HoC 1 %y I C0 2 H HoC MgBr - CQ 2 + III 2. H3 Grignard SOCI 2 carboxylation formation of acid chloride O COCI HN(CH 2 CH 3 ) 2 NaOH Af,Af-Diethyl-m-toluamide 'XX formation of amide Grignard carboxylation yields m-methylbenzoic acid, which can be converted to an acid chloride and treated with diethylamine to produce the amide. 21.63 C0 2 H H3 + C0 2 H NBS excess NHo* (PhC0 2 ); CH< C0 2 H CH 2 Br H CH 2 NH 2 Tranexamic acid Using a rhodium catalyst, the aromatic ring is hydrogenated to form the cis-substituted cyclohexane, which is converted to the trans isomer by heating to 300°. The nitrile starting material is hydrolyzed to form a carboxylic acid, and the methyl group is brominated and treated with ammonia to form the amine. 579 Carboxylic Acid Derivatives: Nucleophilic Acyl Substitution Reactions 21.64(a) + H 2 C— n=n: *- H 2 C=N=N: Resonance forms show that the carbon of diazomethane can occur to form a methyldiazonium ion. M ^C^..^H ^..- + basic, f? h 2 c— n=n: C£) R is + R ch 3 and reaction with an acid — n=n: O: (b) O If R' II /C^ CHo-j-N=N: 3 U "of An Sn2 reaction takes place in which to form the methyl 21.65 Both + OCH 3 R the carboxylate ion displaces N2 as the leaving group ester. steps involve nucleophilic acyl substitutions. Formation of acyl phosphate: :0: CoAS Acyl phosphate Step Step 1: 2: Reaction of the phosphate oxygen with the carbonyl carbon of succinyl Co A. Loss of ~SCoA from the tetrahedral intermediate, yielding acyl phosphate. Conversion of acyl phosphate to succinate: -q 9 o- =.97 :0 — POPO— Guanosine I IT \fl POPO- Guanosine II "OPOPOPO- Guanosine I 0"0" GDP I I 0"0" 0~0"0" GTP 1: Reaction of the diphosphate oxygen of GDP with the phosphorus of the acyl phosphate to produce an intermediate similar to the intermediates formed in nucleophilic acyl substitutions of carboxylic acid derivatives. Step 2: Loss of phosphate to form GTP and succinate. Step 580 Chapter 21 2 1 .66 In all of these reactions, a nucleophile adds to either carbon or phosphorus to form an intermediate that expels a leaving group to give the desired product. Formation of 1,3-bisphosphoglycerate: :|5 / II \^0— ADP 9f /\^_0-ADP S.°" / 'o- -.or o. + H— C— OH I " CH 2 OP0 32- CH 2 OP0 32 CH 2 OP03 2 1,3 Route enzyme-bound to Bisphosphoglycerate thioester: Enz O Base vwv est; P04 3" Enz .TV? H o~ _.\(\ H— C— OH / + O" O^ I 2- CH 2 OP0 3 2 CH 2 OP03 CH 2 OP0 3 ': -.S^-Enz H— C— OH H— C— OH I Enzyme-bound Reduction OADP H— C— OH H— C— OH " thioester to glyceraldehyde 3-phosphate: NAD j ")/^S-t-Enz :o S-t-E nz H o. + "S-t-Enz + H — C— OH I CH 2 OP0 3 2 H— C— OH " H— C— OH " CH 2 OP0 3 2 I CH 2 OP0 32 NAD + " Glyceraldehyde 3-phosphate Carboxylic Acid Derivatives: Nucleophilic Acyl Substitution Reactions 581 21.67 (a) ^°^ uF C °2 Enz Enz-Nu :B '•Aw /"V, r (b) N O O A— H \ (c) I P°2 H N— N ( Nu Enz H3 C \0 S=0 ^N. H o CO. In (c), the imine rearranges to an a,(3-unsaturated ester, to which the nucleophile adds give the trapped p-lactamate. 21.68 :o: :o: Go: / Ph Ph Benzil Ph nucleophihc addition of hydroxide c— a \ y\phOH \ HoO 2 * , , phenyl ,C-C Ph^/ Ph shift HO + ^C— Ph^/ Ph o // o // \ HoO + N OH , proton transfer HO \ O // .0— c OH protonation Benzylic acid Ph^7 Ph * O to 582 Chapter 21 21.69 OH r HOCH 2 CHCH 2 OH Dimethyl terephthalate O . C o . —% II Glycerol o — COCH CHCH OC II (\ II /) / 2 O ff \ COCH 2 CHCH 2 0- 2 ? o=c OCH 2 CHCH 2 OC COCHoCHCHoOC 2 2 o II II O O The product of the reaction of dimethyl terephthalate with cross-linking and is more rigid than Dacron. glycerol has a high degree of 21.70 (a) (b) O O II II CH 3 COCH(CH 3 b ) CH=CHCOCH 2 CH 3 2 e a c b c a = 1.22 5 a= 1.32 5 b = 2.01 6 b = 4.24 5 c = 4.99 6 c = 6.41 6 d = 7.36, 7.49 6 e = 7.68 5 21.71 (a) (b) O II CICH 2 CH 2 COCH 2 CH 3 c a b a= da Q Q ii n CH 3 CH 2 OCCH 2 COCH 2 CH 3 a c b 1.26 5 a= 1.27 6 b = 2.77 6 b = 3.34 6 c= 3.76 5 c = 4.20 6 d = 4.19 6 c a Carboxylic Acid Derivatives: Nucleophilic Acyl Substitution Reactions 21.72 Addition 21.73 This is 583 of the triamine causes formation of cross-links between prepolymer chains. a nucleophilic acyl substitution reaction whose mechanism is similar to others we have studied. o :o: II HCL .a Fry R' addition OH R elimination OH R O" acid-base reaction I3C: can act as a leaving group because the electron- withdrawing iodine atoms stabilize the carbanion. Review Unit 8: Carbonyl Compounds 1. Reaction at the Carbonyl Group Major Topics Covered (with vocabulary); Aldehydes and ketones: -carbaldehyde acyl group acetyl group formyl group benzoyl group hydrate Reactions of aldehydes and ketones: nucleophilic addition reaction gem diol cyanohydrin imine enamine carbinolamine 2,4-dinitrophenylhydrazone Wolff-Kishner reaction acetal hemiacetal Wittig reaction betaine Cannizzaro reaction conjugate addition Carboxylic acids and their derivatives: carboxylation carboxylic acid derivative a,p-unsaturated carbonyl acid halide acid anhydride ylide compound amide ester nitrile -carbon itrile Reactions of carboxylic acids and their derivatives: nucleophilic acyl substitution hydrolysis alcoholysis aminolysis Fischer esterification reaction lactone saponification DIB lactam thiol ester acyl phosphate polyamide polyester step-growth polymer chain-growth polymer nylon AH Types of Problems: After studying these chapters you should be able to: - Name and draw aldehydes, ketones, carboxylic acids and their derivatives. - Prepare all of these compounds. - - Explain the reactivity difference between aldehydes and ketones and between carboxylic acids and all their derivatives. Calculate dissociation constants of carboxylic acids, and predict the relative acidities of substituted carboxylic acids. Formulate mechanisms for reactions related to the reactions we have studied. Predict the products of the reactions for all functional groups we have studied. Use spectroscopic techniques to identify these compounds. Draw representative segments of step-growth polymers. Points to * Remember: In all of these reactions, a nucleophile adds to a positively polarized carbonyl carbon to form a tetrahedral intermediate. There are three possible fates for the tetrahedral intermediate: (1) The intermediate can be protonated, as occurs in Grignard reactions, reductions, and cyanohydrin The intermediate can lose water (or "OH), as happens in imine and enamine The intermediate can lose a leaving group, as occurs in most reactions of formation. (2) formation. (3) carboxylic acid derivatives. * Many of the reactions in these three chapters require acid or base catalysis. An acid catalyst, protonates the carbonyl oxygen, making the carbonyl carbon more reactive toward nucleophiles, and/or protonates the tetrahedral intermediate, making loss of a leaving group easier. base catalyst deprotonates the nucleophile, making it more nucleophilic. The pH A optimum for these reactions is a compromise between the two needs. Review Unit 8 585 Here are a few reminders for drawing the mechanisms of nucleophilic addition and substitution reactions. (1) When a reaction is acid-catalyzed, none of the intermediates are negatively charged, although, occasionally, a few may be neutral. Check your mechanisms for charge balance. (2) Make sure you have drawn arrows correctly. The point of the arrow shows the new location of the electron pair at the base of the arrow. (3) In a polar reaction, two arrows never point at each other. If you find two arrows pointing at each other, redraw the mechanism. Reactions of acyl halides are almost always carried out with an equivalent of base present. The base is used to scavenge the protons produced when a nucleophile adds to an acyl halide. If base were not present, hydrogen ions would protonate the nucleophile and make it unreactive. The products of acidic cleavage of an amide are a carboxylic acid and a protonated amine. products of basic cleavage of an amide are a carboxylate anion and an amine. The In some of the mechanisms shown in the answers, a series of protonations and deprotonations occur. These steps convert the initial tetrahedral intermediate into an intermediate that more These deprotonations may be brought about by the solvent, by the conjugate base of the catalyst, by other molecules of the carbonyl compound or may occur intramolecularly. When a "proton transfer" is shown as part of a mechanism, the base that removes the proton has often not been shown. However, it is implied that the proton transfer is assisted by a base: the proton doesn't fly off the intermediate unassisted. easily loses a leaving group. The most useful spectroscopic information for identifying carbonyl compounds comes from IR spectroscopy and 13 C spectroscopy. Carbonyl groups have distinctive identifying absorptions in their infrared spectra. 13 C is also useful for identifying aldehydes, ketones, and nitriles, although other groups are harder to distinguish. The *H absorptions of aldehydes and carboxylic acids are also significant. Look at mass spectra for McLafferty rearrangements and alpha-cleavage reactions of aldehydes and ketones. NMR NMR NMR Self-Test: A with: LiAlH4, then H30+ (b) CgHsMgBr, then H 3 0+; (c) (CH3 ) 2NH, H 3 0+; (d) CH3 OH, H+ catalyst (e) (C 6H5 ) 3P+-CH2-; (f) 1 equiv. CH 3 CH2NH2, H30+ How would you reduce A to yield a saturated hydrocarbon? Where would you expect the carbonyl absorption of A to occur in its IR spectrum? Predict the products of the reaction of (a) ; . Predict the products of B with the reagents (a) - (d) above. What product(s) would be formed if B was treated with Br2, FeBr 3 ? Where do the carbonyl absorptions occur in the IR spectrum of B? Describe the 13 C NMR spectrum of B. Width: 612 Height: 792 Review Unit 8 586 COCH 3 CH 3 CH 2

CH3CHCCHOH C 6 H 5 CH 2 CH 2 — N OH C NHCO(pHCH 2 CH 3 O D Kethoxal I OCH3 CH 3 Julocrotine (antiviral) E Xanthoxylin Kethoxal (C) exists in solution as an equilibrium mixture. With what compound Why does the equilibrium lie on the side of kethoxal? is it in equilibrium. Identify the carboxylic acid derivatives present in D. with (a) -OH, H2 Name E. (b) LiAlH 4 then , Show the products of treatment of D H 20. Describe the IR spectrum and l H NMR spectrum of E. Multiple Choice: 1 . In which of the following nucleophilic addition reactions does the equilibrium of the products? (a) Propanal + HCN (b) Acetone + H 2 (d) 2,2,4,4-Tetramethyl-3-pentanone + 2 . 3 . (c) Acetaldehyde + lie on the side HBr HCN alcohol can be formed by three different combinations of carbonyl compound + Grignard reagent? (a) 2-Butanol (b) 3-Methyl-3-hexanol (c) Triphenylmethanol (d) 1-Phenylethanol Which A nitrile can be converted to all of the following except: (a) an aldehyde (b) an amide (c) an amine (d) A nitrile can be converted to all of the above compounds. 4. Which of the following p(a) 5 . CH COC 6 H5 C0 2 H 3 substituted benzoic acids (b) C^OC^CO^ is (c) the least acidic? BrC^CC^H A carboxylic acid can be reduced by all of the following except: (a) UA1H4, then H 3 0+ (b) BH 3 THF, then H 3 0+ (c) NaBH 4 , , (d) then NCC6 H5 C0 2 H H 30+ (d) All of these reagents can reduce a carboxylic acid. 6. Which of the following carboxylic acids can be formed by both Grignard carboxylation and hydrolysis? (a) Phenylacetic acid (b) Benzoic acid (c) Trimethylacetic acid (d) 3-Butynoic acid by 7 . nitrile Acid anhydrides are used mainly (a) synthesizing (d) 8. carboxylic acids for: (b) forming alcohols (c) forming aldehydes A ketone is formed from an acid halide by reaction with: (a) DIB AH (b) L1AIH4 (c) RMgBr (d) (CH 3 CH 2 )CuLi introducing acetyl groups Review Unit 8 9 . From which carboxylic acid derivative can you form a ketone as the product of a Grignard reaction? (a) acid chloride 10. An (b) ester infrared absorption at 1650 aromatic acid chloride (d) aromatic ester (a) (c) nitrile cm-1 (d) amide indicates the presence of: (b) Af,7V-disubstituted amide (c) a,p-unsaturated ketone 587 Chapter 22 - Carbonyl Alpha-Substitution Reactions Chapter Outline I. Keto-enol tautomerism (Section 22.1). A. Nature of tautomerism. 1 Carbonyl compounds with hydrogens bonded their corresponding enols. . 2. This rapid equilibration is called tautomerism, to their a carbons equilibrate with and the individual isomers are tautomers. 3 . 4 . Unlike resonance forms, tautomers are isomers. Despite the fact that very little of the enol isomer is present enols are very important because they are reactive. at room temperature, B Mechanism of tautomerism. . 1 2 form an . In acid-catalyzed enolization, the carbonyl carbon is protonated to . intermediate that can lose a hydrogen from its a carbon to yield a neutral enol. In base-catalyzed enol formation, an acid-base reaction occurs between a base and an a hydrogen. a. b II. . The resultant enolate ion is protonated to yield an enol. Protonation can occur either on carbon or on oxygen. c. Only hydrogens on the a positions of carbonyl compounds are acidic. Enols (Sections 22.2-22.4). A. Reactivity of enols (Section 22.2). 1 The electron-rich double bonds of enols cause them to behave as nucleophiles. a. The electron-donating enol -OH groups make enols more reactive than alkenes. 2 When an enol reacts with an electrophile, the initial adduct loses -H from oxygen to give an a-substituted carbonyl compound. B. Reactions of enols (Sections 22.3-22.4). Alpha halogenation of aldehydes and ketones (Section 22.3). 1 a. Aldehydes and ketones can be halogenated at their a positions by reaction of X2 . . . in acidic solution. b . c. d 2 . . The reaction proceeds by acid-catalyzed formation of an enol intermediate. Halogen isn't involved in the rate-limiting step: the rate doesn't depend on the + identity of the halogen, but only on [carbonyl] and [H ]. a-Bromo ketones are useful in syntheses because they can be dehydrobrominated by base treatment to form a,/3-unsaturated ketones. Alpha-bromination of carboxylic acids (Section 22.4). In the Hell-Volhard-Zelinskii (HVZ) reaction, a mixture of Br2 and PBr 3 can a. be used to brominate carboxylic acids in the a position. b The initially formed acid bromide reacts with Br2 to form an a-bromo acid bromide, which is hydrolyzed by water to give the a-bromo carboxylic acid. The reaction proceeds through an acid bromide enol. c . . HI. Enolates (Sections 22.5-22.7). A. Enolate ion formation (Section 22.5). 1 Hydrogens alpha to a carbonyl group are weakly acidic. a. This acidity is due to overlap of a filled p orbital with the carbonyl group p orbitals, allowing the carbonyl group to stabilize the negative charge by resonance. b. The two resonance forms aren't equivalent: the form with the negative charge on oxygen is of lower energy. . Carbonyl Alpha-Substitution Reactions 589 Strong bases are needed for enolate ion formation. a. Alkoxide ions are often too weak to use in enolate formation. b Lithium diisopropylamide can be used to form the enolates of many different carbonyl compounds. 3 When a hydrogen is flanked by two carbonyl groups, it is much more acidic, a. Both carbonyl groups can stabilize the negative charge. Reactivity of enolate ions (Section 22.6). Enolates are more useful than enols for two reasons: 1 a. Unlike enols, stable solutions of enolates are easily prepared. b Enolates are more reactive than enols because they are more nucleophilic. 2 Enolates can react either at carbon or at oxygen. 2. . . B . . . . Reaction at carbon yields an a-substituted carbonyl compound. Reaction at oxygen yields an enol derivative. C. Reactions of enolate ions (Sections 22.6-22.7). a. b 1 . . Base-promoted a-halogenation. a. Base-promoted halogenation of aldehydes and ketones proceeds readily because each halogen added makes the carbonyl compound more reactive. b Consequently, polyhalogenated compounds are usually produced. c. This reaction is only useful with methyl ketones, which form HCX3 when . d. reacted with halogens. This reaction is known as the haloform reaction. i. ii. 2. The HCX3 is a solid that can be identified. The last step of the reaction involves a carbanion leaving group. Alkylation reactions of enolates (Section 22.7). a. General features. i. ii. b . Alkylations are useful because they form a new C-C bond. Alkylations have the same limitations as Sn2 reactions; the alkyl groups must be methyl, primary, allylic or benzylic. The malonic ester synthesis. The malonic ester synthesis i. used for preparing a carboxylic acid from a by two carbon atoms. useful because its enolate is easily prepared by reaction is halide while lengthening the chain ii. iii. Diethyl malonate is with sodium ethoxide. Since diethyl malonate has two acidic hydrogens, two alkylations can take place. Heating in aqueous HC1 causes hydrolysis and decarboxylation of the alkylated malonate to yield a substituted monocarboxylic acid.. (a). Decarboxylations are common only to /3-keto acids and malonic acids. v. Cycloalkanecarboxylic acids can also be prepared. iv. c. The acetoacetic ester synthesis. i. The acetoacetic ester synthesis ii. iii. is used for converting an alkyl halide to a methyl ketone, while lengthening the carbon chain by 3 atoms. As with malonic ester, acetoacetic ester has two acidic hydrogens which are flanked by a ketone and an ester, and two alkylations can take place. Heating in aqueous HC1 hydrolyzes the ester and decarboxylates the acid to yield the ketone. Most /3-keto esters can undergo this type of reaction. Direct alkylation of ketones, esters, and nitriles. i. in a nonprotic solvent can be used to convert the above iv. d . LDA compounds to their enolates. ii. Alkylation of an unsymmetrical ketone leads to a mixture of products, but the major product is alkylated at the less hindered position. 590 Chapter 22 Solutions to Problems 22.1-22.2 Acidic hydrogens in the keto form of each of these compounds are bold. these hydrogens is removed by base when an enolate is formed. One of Number of Keto Form (a) Enol Form O (b) r ft SCHg SCHg H H H (c) ft H OCH 2 CH 3 / \ H OCH 2 CH 3 C H H (d) H ft HoC. HoC. C H / \ H In (d) and C H I H (f), cis H and trans enolates are possible. Acidic Hydrogens Carbonyl Alpha-Substitution Reactions 591 22.3 The first two monoenols equivalent; equivalent; more less stable are stable more stable because the enol double bond conjugated with is the carbonyl group. 22.4 T deuteration loss of proton h h of carbonyl HO 2 HOD + enol at alpha position oxygen 0+ * D- OD 2 + : ff {? C D C CH3 deuteration enol CH< / \ - H H H of enol double bond 22.5 D3 + H loss of deuterium on carbonyl oxygen Alpha-bromination, followed by dehydration using pyridine, yields the enone below. o CH3CH2 II II II C ^ CH 3 /\ H H Br2 CH3CO0H * CH0CH0 C ^CH3 /\ H Br pyridine ' heat ^ , CH0CH0 C I H l-Penten-3-one H 592 Chapter 22 22.6 OH O I PBr< H, C OH C H Br Br / \ / \ CH(CH 3 )CH 2 CH 3 H CH(CH 3 )CH 2 CH 3 CH(CH 3 )CH 2 CH 3 enolization formation of acid bromide 1 alphasubstitution Br, O f CH 3 OH Br^ C OCH- CH(CH 3 )CH 2 CH 3 with methanol L Methyl 2-bromo-3-methylpentanoate The mechanism of the ester-forming step is A H CH(CH Br reaction / \ H II Br. 3 )CH 2 CH 3 a nucleophilic acyl substitution, which was described in Chapter 21. (To: :0: v ll Br -c^ Br. Br. C Br /\ H 2 CH 3 addition v Base C" of methanol 22.7 Hydrogens a to one carbonyl group (or carbonyl groups are protons. (a) O much more loss of elimination proton of bromide nitrile) are weakly acidic. + Br" Hydrogens a to two acidic, but they are not as acidic as carboxylic acid (b) (c) O II ft II (CH 3 3 C H / \ H weakly acidic o c ) / \ H-S J C H H OCH- /\ PCH 3 H H weakly acidic (e) CH 3 CH 2 most H ac idic weakly acidic m ft /j-CSN H H weakly acidic * V H I H CH 3 weakly acidic Carbonyl Alpha-Substitution Reactions 22.8 weakly acidic because the bonds of the nitrile group. Nitriles are the jz H 2 C— C=n: 22.9 Halogenation -« anion can be stabilized by resonance involving H 2 C=C=n: *- in acid nitrile medium is acid-catalyzed because hydrogen ions are regenerated: O O H II x2 C H 593 + W' H + 1 Na 2. CH 3 CH 2 CH 2 Br CH 2 (C0 2 Et) 2 ~OEt CH 3 CH 2 CH 2 — CH(C02 + 1 Na . HoO CHC0 2 H CH 3 CH 2 CH 2 CH 3CH 2 CH 2 heat "OEt CH 3 Br 1 2. + NaBr + Et) 2 — C(C02 CH 3 Et) NaBr + 2 CHo 2-Methylpentanoic acid + C0 2 + 2 EtOH (c) fromhaUde((CH 3 2 CHCH^H;CH 2 C02 H ) from malonic ) CH 2 (C0 2 Et) 2 Na+ 1 0Et ^ : 2. (CH 3 ) 2 CHCH 2 Br (CH 3 ) 2 CHCH 2 — CH(C0 H3 | (CH 3 ) 2 CHCH 2 ester + , + 2 Et) 2 NaBr heat — CH2 C02 H + C0 2 + 2 EtOH 4-Methylpentanoic acid 22.11 Since malonic ester has only two acidic hydrogen atoms, it can be alkylated only two times. Formation of trialkylated acetic acids is thus not possible. 22.12 from malonic ester C0 2 H from halides + 1 CH 2 (C0 2 Et); 2. Na ~OEt CH 3 CHCH 2 Br CH 3 CH 3 CHoCHCHo— CHCOoH I CH 3 2,4-Dimethylpentanoic acid + C0 2 + 2 EtOH + 1 . 1 2. heat CH 3 — CH(C02 Na Et) + 2 NaBr "OEt | + - HqO I CH 3 CHCH 2 CH 3 CHCH 2 CH 3 CH 3 Br — C(C0 2 ^^ 3 Et) 2 + NaBr Carbonyl Alpha-Substitution Reactions 595 22.13 As in the malonic ester synthesis, you should identify the structural fragments of the target compound. The acetoacetic ester synthesis converts an alkyl halide to a methyl ketone ("substituted acetone"). The methyl ketone component comes from acetoacetic ester; the other component comes from a halide. from halide N ../T% ( (CH 3 )2CHCH 2 t\CH 2 CCH 3 / from acetoacetic ester n 1 CHd2 CCHo6 Na . (CHo) dV2 CHCH d2 (CH 3 2 CHCH 2 Br 2. \ ^ "OEt — CHCCh \ ) C02 Et C0 2 Et | H3 + , heat O (CH 3 2 CHCH 2 ) — CH2 CCH 3 C0 2 + EtOH + 5 -Methy 1-2-hexanone (b) II .' from halide \ (c 6 H 5 CH 2 CH 2 t\CH 2 CCH 3 / from acetoacetic ester O f? 1 CHoCCHo 2 6 . Na+ "OEt " 2.C 6 H 5 CH 2 CH 2 Br I CHCCHo6 + CfiHcCHpCHo— b 5 2 2 NaBr | C0 2 Et C0 2 Et | H3 + , heat O II C 6 H 5 CH 2 CH 2 — CH2 CCH3 + C0 2 EtOH + 5-Phenyl-2-pentanone 22.14 The acetoacetic ester synthesis can only be used for certain products: (1) (2) (3) Three carbons must originate from acetoacetic ester. In other words, compounds of the type RCOCH3 can't be synthesized by the reaction of RX with acetoacetic ester. Alkyl halides must be primary or methyl. The acetoacetic ester synthesis can't be used to prepare compounds that are trisubstituted at the a position. (b) (a) H (c) H \ / r^V Phenylacetone (a) CH3 i^V Cv Acetophenone H3 ch3 V H 3C c^ OH 3 3,3-Dimethylbutan-2-one Phenylacetone can't be produced by an acetoacetic ester synthesis because bromobenzene, the necessary halide, does not enter into Sn2 reactions. [See above.] ch 3 (2) Width: 612 Height: 792 596 Chapter 22 (b) Acetophenone (c) 3,3-Dimethyl-2-butanone can 't be prepared because can't be produced by an acetoacetic ester synthesis. [See it is (1) above.] trisubstituted at the a position. [See (3) above.] 22.15 f? CH 2 CCH 3 + 1.2 Na ~OEt BrCH 2 CH 2 CH 2 CH 2 Br 2. C0 2 Et ' H c + 2 NaBr C0 2 Et H3 + , heat J C0 2 + + EtOH HoC 22.16 Direct alkylation is used to introduce substituents a to an ester, the target molecule to identify these substituents. Alkylation starting material with (a) LDA, is ketone or nitrile. Look at achieved by treating the followed by a primary halide. o O 1. II CH2 CCH 3 2. LDA II \ CH3I CHCCHo3 # I CH 3 3-Phenyl-2-butanone Alkylation occurs at the carbon next to the phenyl group because the phenyl group can help stabilize the enolate anion intermediate. (b) _ CH 3 CH 2 CH 2 CH 2 C= N 1 . 2. LDA CH 3 CH 2 I CH 3 CH 2 CH 2 CHC= N CH 2 CH 3 2-Ethylpentanenitrile (c) 1. O LDA 2.ICH 2 CH=CH 2 nr CH 2 CH — CH 2 v. O 2-Allylcyclohexanone (d) H3 Q . excess LDA 2. excess CH 3 I 1 tf Cl / CH< 2,2,6,6-Tetramethylcyclohexanone Carbonyl Alpha-Substitution Reactions 597 (e) ff CH 2 CH 3 1. 2. LDA CH 3I CHCHo3 a" I CHo Isopropyl phenyl ketone (f) CH 3 O O CHo3 II I CH3CHCH0COCH3 1. LDA 2. CH 3 CH 2 I CH3CHCHCOCH3 CH2CH3 Methyl 2-ethyl-3-methylbutanoate Visualizing Chemistry 22.17 Check to is a methyl ketone or a substituted carboxylic acid. a methyl ketone, and the reaction is an acetoacetic ester synthesis.) Next, identify the halide or halides that react with acetoacetic ester. (The halide is 1-bromo3-methyl-2-butene.) Formulate the reaction, remembering to include a decarboxylation (a) see if the target molecule (The target molecule is step. ^CH 3 from halide ^ ) 2 + f? 1 CH2CCH3 . Na 11 C= CHCH 2,'K( CH2CCHy • "OEt from acetoacetic (CH 3 2 C= 2.(CH 3 ) 2 C=CHCH 2 Br CHCH 2 ) ester — CHCCH C0 2 Et 3 + NaBr C0 2 Et HoO I + , heat 9 \ co 2 EtOH + (CH 3 2 C=CHCH 2 + ) — CH 2CCH 3 6-Methyl-5-hepten-2-one (b) This product is formed from the reaction of malonic ester with both benzyl bromomethane. t C 6 H 5 CH2rCHC0 2 H J from malonic ester H>' fromhafides '^9. + 1 CH 2 (C0 2 Et) 2 2. Na "OEt C 6 H 5 CH 2 Br C 6 H 5 CH 2 — CH(C02 1 , CfiHcCHo 2 6 5 CHCOoH 2 | HoP + heat C 6 H 5 CH 2 CH 3 2-Methyl-3-phenylpropanoic acid . 2. ( Na + Et) 2 CH 3 Br — C(C0 2 C0 2 NaBr "OEt Et) 2 CH 3 + + + 2 EtOH + NaBr bromide and 598 Chapter 22 22.18 H H O HO. Ordinarily, p-diketones are acidic because they can form enolates that can be stabilized by derealization over both carbonyl groups. In this case, loss of the proton at the bridgehead carbon doesn't occur because the strained ring system doesn't allow formation of the bridgehead double bond. Instead, enolization takes place in the opposite direction, and the diketone resembles acetone, rather than a /3-diketone, in it pKa and degree of dissociation. 22.19 O Enolization can occur on only one side of the carbonyl group because of the two methyl groups on the other side. The circled axial hydrogen is more acidic because the p orbital that remains after its removal is aligned for optimum overlap with the % electrons of the carbonyl oxygen. Additional Problems Acidity of Carbonyl 22.20 Compounds Acidic hydrogens are bold. The most acidic hydrogens are the two between the carbonyl groups in (b) and the hydroxy 1 hydrogen in (c). The hydrogens in (c) that are bonded to the methyl group are acidic (draw resonance forms to prove it). (b) (a) (c) H H CH3CH2CHCCH; CH 3 H (d) (e) H (f) Carbonyl Alpha-Substitution Reactions 22.21 Check your answer by using the i? (CH 3 CH 2 2 NH < CH3CCH3 < ) 22.22 (a) :o: :o: • C<^ in Table 22. 1 »- Least Acidic HoC pKa s "y C\ c I H 599 Most Acidic !?!?{?{? CH 3 CH 2 OH < CH 3 CCH 2 CCH 3 < CH 3 CH 2 COH < CCI3COH 600 Chapter 22 ;0: .0: rc \ OCH:0: 22.23 Enolization at the y position produces a conjugated enolate anion that derealization of the negative charge over the n system of five atoms. 22.24 The illustrated compound, l-phenyl-2-propenone, doesn't with base because the hydrogen on the a carbon is i l-Phenyl-2-propenone H stabilized yield an anion when by treated vinylic and isn't acidic (check Table 22.1 for acidity constants). | is Carbonyl Alpha-Substitution Reactions a-Substitution Reactions 22.25 (a) C0 2 H C0 2 H heat j^^^ C0 2 + OH 3 (b) O + 1 Na . CH 3 I 2. (c) ~OEt PBro CH 3 CH 2 CH 2 C0 2 H CHoCHoCHCOBr 2 3 Br2 2 CH 3 CH 2 CHC0 2 H \ | A Br (d) —H B Br O II a"CH "OH, H 2 Q 3 O" + HCI3 h 22.26 + (a) CH 2 (C0 2 Et) 2 1 2. Na ~OEt CH 3 CH 2 CH 2 Br CH 3 CH 2 CH 2 | EtOH CH 3 CH 2 CH 2 CH 2 C0 2 Et H Ethyl pentanoate ^ CH (C0 ™ 2 2 Et) 2 2. + H3 , (CH 3 2 CHBr + + 2 (CH 3) 2 CH— CH(C0 2 Et) 2 EtOH + ) EtOH ) H + NaBr heat C0 2 H3 + , heat (CH 3 2 CHCH 2 C0 2 H ) catalyst + Ethyl 3-methylbutanoate Some elimination product will 2 CH 3 CH 2 CH 2 CH 2 C0 2 H Na+ "OEt (CH 3 2 CHCH 2 C0 2 Et Et) catalyst + 1 . + — CH(C02 also be formed. C0 2 + 2 EtOH NaBr 601 602 Chapter 22 (c) Na 1 CH 2 (C0 2 Et) 2 + ~OEt CH 3 CH 2 CH(C0 2 Et) 2 1 2. Na+ ~OEt 1 CH 3 CH 2 Br 2. NaBr + CH 3 Br CHo3 I CH 3 CH 2 C(C0 2 Et) 2 NaBr EtOH I CH 3 CH 2 CHC0 2 Et H + H3 | CH< CHo3 CH 3 CH 2 CHC0 2 H C0 2 + + 2 + , heat EtOH catalyst Ethyl 2-methylbutanoate The malonic (d) acid synthesis can't be used to synthesize carboxylic acids that are trisubstituted at the alpha position. 22.27 Look back to Problem 22.14, which describes compounds that can be prepared by an acetoacetic ester synthesis. Neither (a) or (c) are products of an acetoacetic ester synthesis because the halide component that would be needed for each synthesis doesn't undergo Sn2 reactions. Compound (b) can be prepared by the reaction of acetoacetic ester with 1 ,5dibromopentane. 22.28 Two alkylations are needed two the target molecule has if alkyl groups «to the carbonyl group, (a) + f? ^ 1 CH 2 CCH 3 2. 2 Na 2 O H3 Q+ f? OEt CH 3 CH 2 Br (CH 3 CH 2 2 CCCH 3 ) C0 2 Et II (CH 3 CH 2 2 CHCCH 3 ) heat C0 2 Et 3-Ethyl-2-pentanone + + 2 NaBr C0 2 (b) P + — Na II 1 CHoCCHo d 6 2. | p " ~OEt CH 3 CH 2 CH 2 Br CHoCHoCHoCHCCHo 6 d 6 * 11 Na . HoC 3 I Q CH 3 CH 2 CH 2 CH — CCH 3 3-Methyl-2-hexanone + HoC 3 II EtOH * HoO^ ^ + ~OEt CH 3 Br 2. C0 2 NaBr C02 Et C0 2 Et + + \ I O II CH 3 CH 2 CH 2 C— CCH 3 C0 2 Et + NaBr + EtOH Carbonyl Alpha-Substitution Reactions 22.29 Use a malonic derivative. ester synthesis if the product Use an you want is 603 an a-substituted carboxylic acid or you want is an a-substituted acetoacetic acid synthesis if the product methyl ketone. (a) 1.2 Na+ ~OEt CH 2 (C0 2 Et) 2 2. 2 CH 3 C(C0 2 Et) 2 CH 3 Br + 2 NaBr CHo (b) O II + 1.2 Na f? CH 2 CCH 3 CCH 3 "OEt BrCH 2 (CH 2 4 CH 2 Br 2. + NaBr + C0 2 C0 2 Et ) C0 2 Et H3 + , heat O II CCH 3 EtOH + H (c) + 1.2 Na CH 2 (C0 2 Et) 2 C0 2 Et ~OEt BrCH 2 CH 2 CH 2 Br 2. 0<:C0 | H3 + 2 NaBr 2 Et + , heat /\.C02 H + C0 2 + 2 EtOH (d) + f? 1 CH 2 CCH 3 . 2. Na "OEt H 2 C=CHCH 2 Br H 2 C=CHCH 2 CHCCH 3 C0 2 Et + NaBr C0 2 Et H3 + , heat J H 2 C— CHCH 2 CH 2 CCH 3 + qq + 22.30 The haloform reaction (Problem 22.25d) is an alpha-substitution reaction by which a is CH 3 CCH 3 (b) C 6 H 5 CCH 3 is Negative haloform reaction: Positive haloform reaction: (a) in trihalogenated at the alpha position, and the trihalomethyl group -OH. It is a test for methyl ketones. methyl ketone displaced EtOH (c) CH 3 CH 2 CHO (d) CH 3 C0 2 H (e) CH 3 CN 604 22.31 Chapter 22 with PBr 3 to form First, treat geraniol (CH 3 )2C=CHCH2CH2C(CH3)=CHCH 2 Br (geranyl bromide). (a) CH 3 C0 2 Et " 2 Geranyl* C= CHCH2CH2C(CH 3)= CHCH 2CH 2C0 2 Et (CH 3) 2 b ro m d e Ethyl geranylacetate i Alternatively: + 1 CH 2 (C0 2 Et) 2 Na . ~OEt — -r-z c. oeranyi . (CH 3 2 C=CHCH 2 CH 2 C(CH 3 )=CHCH 2 CH 2 (C0 2 Et) 2 ) bromide C0 2 + 2 I H3 + , heat EtOH + (CH 3 2 C=CHCH 2 CH 2 C(CH 3 )=CHCH 2 CH 2 C0 2 H ) SOCI 2 EtOH, pyridine (CH 3 2 C= CHCH 2 CH 2 C(CH 3 )= CHCH 2 CH 2 C0 2 Et ) Ethyl geranylacetate (b) O U CH 2 CCH 3 g C0 2 Et Qerany?^ (CH 3 ) 2 C= CHCH2CH2C(CH3)= CHCH 2CHCCH3 bromide I H + , C0 2 Et heat 1 t C0 2 EtOH + + (CH 3 2 C=CHCH 2 CH2 C(CH 3 )=CHCH 2 CH 2 CCH 3 ) Geranylacetone 22.32 Dialkylation of diethylmalonate: Et0 2 C^^C02 Et .C0 2 Et Et0 2 C. + 1 . 2. Na "OEt H 2 C=CHCH 2 Br Et0 2 C. + \ 1 \ . Na .C0 2 Et "OEt 2.(CH 3 2 CHBr ) V ^ Carbonyl Alpha-Substitution Reactions 605 Nucleophilic acyl substitution: This series of steps is repeated to form the 6-membered ring. General Problems hydrogen atoms is treated with NaOD in D2O, all by deuterium atoms. For each proton (atomic weight 1) lost, a deuteron (atomic weight 2) is added. Since the molecular weight of cyclohexanone increases by four after NaOD/D20 treatment (from 98 to 102), cyclohexanone contains four acidic hydrogen atoms. 22.33 When a compound containing acidic acidic hydrogens are gradually replaced 22.34 Reaction of (/?)-2-methylcyclohexanone with aqueous base is shown below. Reaction with aqueous acid proceeds by a related mechanism through an enol, rather than an enolate ion, intermediate. (K)-2-Methylcyclohexanone its chirality when the enolate ion double bond is formed. Protonation occurs with equal probability from either side of the planar s^-hybridized carbon 2, resulting in a racemic product. Carbon 2 loses Width: 612 Height: 792 606 Chapter 22 22.35 (S)-3-Methylcyclohexanone (5)-3-Methylcyclohexanone isn't racemized by base because involved in the enolization reaction. 22.36 The Hell-Volhard-Zelinskii its chirality center is not reaction involves formation of an intermediate acid bromide enol, with loss of stereochemical configuration at the chirality center. Bromination of (R)- 2-phenylpropanoic acid can occur from either face of the enol double bond, producing racemic 2-bromo-2-phenylpropanoic acid. If the molecule had a chirality center that didn't take part in enolization (Problem 22.35), the product would be optically active. 22.37 (a) Na + ~OCH 3 , then CH3 dimethylcyclohexanone I; (b) may H3 + , heat; (c) LDA, then CH 3 I (some 2,2- also be formed). 22.38 carbonyl oxygen protonation at y position of a proton loss of proton on oxygen Carbonyl Alpha-Substitution Reactions 607 22.39 :Q: :o: :0: :o: H H H-[OH I abstraction + protonation H H H of a proton The "OH aty position enolate of 3-cyclohexenone can be protonated at three different positions. Protonation at the y position yields the a,6-unsaturated ketone. 22.40 O :o: CH< H H Vu nr* abstraction H ho: H of Y proton H '3 is used for oxidizing alcohols to carbonyl compounds, ecgonine has the structure shown above. Again, the stereochemistry is unspecified. reaction with hydroxide. The complete reaction sequence: Cocaine Ecgonine Carbonyl Alpha-Substitution Reactions 22.54 Laurene 613 differs in stereochemical configuration from the observed product at the carbon a methylene group. Since this position is a to the carbonyl group in the precursor to laurene, enolization and isomerization must have occurred during the reaction. Isomerization of the ketone precursor is brought about by a reversible reaction with the basic Wittig reagent, which yields an equilibrium mixture of two diastereomeric ketones. One of the ketone isomers then reacts preferentially with the Wittig reagent to give only the observed product. to the 22.55 The key step is an intramolecular alkylation reaction of the ketone a-carbon, with the tosylate in the second ring serving as the leaving group. 614 Chapter 22 22.56 O H if I O Na + ~QEt_ r. II CH 3 CNHCC0 2 Et CH 3 CNHCC0 2 Et formation of enolate C0 2 Et C0 2 Et J CH 3 I H 2 NCHCOH + H 63 C ll| \ O II > CH 3 C-|-NHC— C-| OEt '"fy ou 2 t decarboxylation C0 2 CH 3 C0 2 H + Acid hydrolyzes both shown heat amide hydrolysis Alanine EtOH , ester hydrolysis CH 3 + 2 O + H3 f? alkylation in Figure 21.8 ester bonds, as well as the amide bond, by mechanisms that were and Section 21.6. Decarboxylation of the /S-keto acid produces alanine. 22.57 1 CH 2 (C0 2 Et) 2 . 2. Na+ "OEt (CH 3 2 CHCH 2 CH(C0 2 Et) 2 (CH 3 2 CHCH 2 Br ) ) | Br 1 (CH 3 2 CHCH 2 CHC02 H ) * 1 . 2. Br d 9 PBro , H2 ^ H3 + , heat (CH 3 2 CHCH 2 CH 2 C02 H ) + C0 2 + 2 EtOH NH. (CH 3 2 CHCH 2 CHC02 H Leucine ) A malonic ester synthesis is used to form 4-methylpentanoic acid. Hell-Volhard-Zelinskii bromination of the acid, followed by reaction with ammonia, yields leucine. The reaction is an Sn2 displacement of bromide by ammonia. last Carbonyl Alpha-Substitution Reactions 615 22.58 Step Step 1: Protonation. 3: Deprotonation. 22.59 This sequence resembles Step 2: Hydride shift. Step 4: Enolization to form the aromatic the one shown in Problem 22.32. Sodium Pentothal The series of steps is repeated to form the 6-membered ring. ring. Width: 612 Height: 792 Chapter 23 - Carbonyl Condensation Reactions Chapter Outline I. The aldol reaction (Sections 23.1-23.6). A. Characteristics of the aldol reaction (Sections 23.1-23.2). 1 . The aldol condensation is a base-catalyzed dimerization of two aldehydes or ketones. 2 . 3 . The reaction can occur between two components that have a hydrogens. One component (the nucleophilic donor) is converted to its enolate and undergoes an a-substitution reaction. 4. 5 . 6 . The other component (the electrophilic acceptor) undergoes nucleophilic addition. For simple aldehydes, the equilibrium favors the products, but for other aldehydes and ketones, the equilibrium favors the reactants. Carbonyl condensation reactions require only a catalytic amount of base (Section 23.2). Alpha-substitution reactions, on the other hand, use one equivalent of base. a. B. Dehydration of aldol products (Section 23.3). Aldol products are easily dehydrated to yield a,/?-unsaturated aldehydes and ketones. a. Dehydration is catalyzed by both acid and base. b. Reaction conditions for dehydration are only slightly more severe than for condensation. Often, dehydration products are isolated directly from condensation reactions. c. Conjugated enones are more stable than nonconjugated enones. 2 3 Removal of the water byproduct drives the aldol equilibrium towards product formation. C. Aldol products (Sections 23.4-23.5). 1 Using aldol reactions in synthesis (Section 23.4). a. Obvious aldol products are: 1 . . . . i. a,/S-Unsaturated aldehydes/ketones. ^-Hydroxy aldehydes/ketones. it's possible to work backwards from a compound that doesn't seem resemble an aldol product and recognize aldol components. ii. b 2 . . Often, Mixed a. b . to aldol reactions (23.5). two similar aldehydes/ketones react under aldol conditions, 4 products may be formed - two self-condensation products and two mixed products. A single product can be formed from two different components i. If one carbonyl component has no a-hydrogens. ii. If one carbonyl compound is much more acidic than the other. If : D. Intramolecular aldol condensations (Section 23.6). 1 Treatment of certain dicarbonyl compounds with base can lead to cyclic products. 2 A mixture of cyclic products may result, but the more strain-free ring usually . . predominates. II. The Claisen condensation (Sections 23.7-23.9). A. Features of the Claisen condensation (Section 23.7). Treatment of an ester with 1 equivalent of base yields a /3-keto ester. 1 2 The reaction is reversible and has a mechanism similar to that of the aldol reaction. . . Carbonyl Condensation Reactions 3 . 4 . 5 . 617 A major difference from the aldol condensation is the expulsion of an alkoxide ion from the tetrahedral intermediate of the initial Claisen adduct. Because the product is often acidic, one equivalent of base is needed; addition of this amount of base drives the reaction to completion. Addition of acid yields the final product. Claisen condensations (Section 23.8). Mixed Claisen condensations of two different esters can succeed B Mixed 1 . 2. if one component has no a hydrogens. Mixed Claisen condensations between a ketone and an ester with no a hydrogens are also successful. C. Intramolecular Claisen condensations: the Dieckmann cyclization (Section 23.9). The Dieckmann cyclization is used to form cyclic /3-keto esters. 1 1,6-Diesters form 5-membered rings. a. b 1 ,7-Diesters form 6-membered rings. 2. The mechanism is similar to the Claisen condensation mechanism. 3 The product /3-keto esters can be further alkylated. a. This is a good route to 2-substituted cyclopentanones and cyclohexanones. Other carbonyl condensation reactions (Sections 23.10-23.13). A. The Michael reaction (Section 23.10). The Michael reaction is the conjugate addition of an enolate to an a,/3-unsaturated 1 carbonyl compound. a. The highest-yielding reactions occur between stable enolates and unhindered a,/3-unsaturated carbonyl compounds. 2. Stable enolates are Michael donors, and «,y5-unsaturated compounds are Michael . . . III. . acceptors. B. The Stork 1 2 . . reaction (Section 23.1 1). ketone that has been converted to an enamine can act as a Michael donor in a A reaction known as the Stork reaction. The sequence of reactions in the Stork reaction: a. Enamine formation from a ketone. b . c. Michael-type addition to an a,/3-unsaturated carbonyl compound. Enamine hydrolysis back to a ketone. This sequence is equivalent to the Michael addition of a ketone to an a,/3-unsaturated carbonyl compound and yields a 1,5 diketone product.. C. The Robinson annulation reaction (Section 23.12). The Robinson annulation reaction combines a Michael reaction with an 1 intramolecular aldol condensation to synthesize substituted ring systems. 3 . . The components are a nucleophilic donor, such as a /3-keto ester, and an a,punsaturated ketone acceptor. 3 The intermediate 1 ,5-diketone undergoes an intramolecular aldol condensation to yield a cyclohexenone. D. Biological carbonyl condensation reactions (Section 23.13). 1 Many biomolecules are synthesized by carbonyl condensation reactions. 2 The enzyme aldolase catalyzes the addition of a ketone enolate to an aldehyde, a. This mixed aldol reaction is successful because of the selectivity of enzyme 2 . . . . catalysis. 3 . Acetyl Co A is the major building block for the synthesis of biomolecules. Acetyl Co can act as an electrophilic acceptor by being attacked at its carbonyl a. group. A b . Acetyl CoA can act as a nucleophilic donor by loss of its acidic a hydrogen. 618 Chapter 23 Solutions to Problems 23.1 ( 1) Form the enolate of one molecule of the carbonyl compound. f? CH 3 CH 2 CgCH + :OH : CH3CCH3 + 1 (3) Reprotonate the enolate anion. O HO" + CH2CCH3 Problem 23.1. 619 Chapter 23 As in Problem 23.1, align the two carbonyl compounds so that the location of the new bond is apparent. After drawing the addition product, form the conjugated enone product by dehydration. In parts (b) and (c), a mixture of E,Z isomers may be formed. heat |J (C) O OH M (CH 3 ) 2 CHCH 2 CH II + CH 2 CH CH(CH 3) 2 NaOH, « * l| I (CH 3 2 CHCH 2 ) Et0H C— CHCH H CH(CH 3 ) 2 heat ti O = CCH II (CH 3 ) 2 CHCH 2 CH + CH(CH 3) 2 H2 Carbonyl Condensation Reactions 23.4 621 Including double bond isomers, 4 products can be formed. The major product is formed by reaction of the enolate formed by abstraction of a proton at position "a" because position "b" has more steric hindrance. major (less hindered) minor (more hindered) 23.5 (a) OH O I CH3CH2CH2C II CH 2-Hydroxy-2-methylpentanal CHa This is not an aldol product. The hydroxyl group in an aldol product must be fi, not the carbonyl group. (b) H3C Jj) CH 3 CH 2 C= C— CCH 2 CH 3 5-Ethyl-4-methyl-4-hepten-3-one CH2CH3 This product results from the aldol self-condensation of 3-pentanone, followed by dehydration. a, to Chapter 23 O O II 2 CH 3 CH NaOH, EtOH 1 2. heat NaBH 4 + H ^+ 1. II = CHCH CHoCH 6 n , 2. CH 3 CH=CHCH 2 OH , 3 |h 2 Pd/C , CH 3 CH2CH 2 CH 2 OH 1-Butanol O // H— H— NaOH N EtOH H c— O-C\ H H H H H OH \ / H heat .0 Cyclopropylacetaldehyde H— C— I H H (a) OH O II O II I CH • NaOH, Or-? CHCH 2 CCH 3 heat = CH0CCH03 + HoO EtOH [ 4-Phenyl-3-buten-2-one This mixed aldol will succeed because one of the components, benzaldehyde, is a good acceptor of nucleophiles, yet has no a-hydrogen atoms. Although it is possible for acetone to undergo self-condensation, the mixed aldol reaction (b) o a CHo II is much more O O II I II aC=CHCCH CCH3 3 1 NaOH, EtOH 2. heat favorable. ) + CHq6 O I O II CH3CCH3 + ^nn (CH 3 2 C=CHC Vs O U II .C=CHC. (CH 3 2 C=CHCCH 3 + ) h,o ^s. kJ Four products result from the aldol condensation of acetone and acetophenone. The two upper compounds are mixed aldol products, and the bottom two are self-condensation products. Carbonyl Condensation Reactions (c) HoC 1 NaOH, EtOH 2. heat CH 3 CH 2 CHO 623 CHO CH 3 CH 2 CH=CCHO CHo As in (b), a mixture of products is formed because both carbonyl partners contain ahydrogen atoms. The upper two products result from mixed aldol condensations; the lower two are self-condensation products. 23.9 O o it- ii CH3CCHCCH3 A NaOH O o // CH3CCH2CCH3 O NaOH O O II II." | CH3CCH2CCH2 is in I C=CH HoC B 2,4-Pentanedione H 2 C— equilibrium with two enolate ions after treatment with base. Enolate A is stable and unreactive, while enolate B can undergo internal aldol condensation to form a cyclobutenone product. But, because the aldol reaction is reversible and the cyclobutenone product is highly strained, there is little of this product present when equilibrium is reached. At equilibrium, only the stable, diketone enolate ion A is present. 23.10 This intramolecular aldol condensation gives a product with a seven-membered ring fused to a flve-membered To ring. O NaOH, EtOH EtOH H2 + heat 624 Chapter 23 23.11 As in the aldol condensation, writing the makes it two Claisen components in the correct orientation easier to predict the product. (a) I? ff (CH 3 2 CHCH 2 COEt CH 2 COEt + ) 1 . Na+ "OEt H^ Q 2 + ff I? (CH 3 2 CHCH 2 C— CHC( CHCOEt ) I CH(CH 3 ) 2 CH(CH 3 EtOH + ) 2 O (b) •I II CH 2 COEt + CH 2 COEt Na+ "OEt» - + 2 H 3° 1. ff ff CH 2 C— CHCOEt - I EtOH + (c) + ff 1 CH 2 COEt + CH 2 COEt I . d. Na "OEt u H 3 n+ U ff ff CH 2 C— CHCOEt + EtOH 23.12 O c I CH 3 3. C0 2 CH 3 OCH 3 Step 1: Conjugate addition of Step 3: Loss of methoxide. H3C . Step 2: Claisen condensation. Carbonyl Condensation Reactions 23.62 C0 2 Et C0 2 Et C02 Et OEt 1. C0 2 Et EtOH "OEt + o Step Is Enolate formation. Step 2: Intramolecular aldol condensation. Step 3: Retro-aldol condensation. Step 4: Protonation. 23.63 Step 1: Deprotonation and retro aldol reaction. Step 2: Equilibration between two enolates. Step 3: Internal aldol condensation. Step 4: Protonation. 651 652 Chapter 23 23.64 H-A (a) ej)H2 H3C H !. 3 Z - £ HN(CH 3 ) 3 HN(CH 3 3 ) - + H3 c^Sr h C n(CH : 2 3 )2 4. 2 ti HoO + CH3 CH= N(CH 3 ) 2 iminium ion : |j CH=N(CH 3 2 ) L. \J 5 - Aminopy rimidine 24.4 are less basic than hydroxide but more basic than amides. The pKa values of the conjugate acids of the amines in (c) are shown. The larger the pKa the stronger the base. Amines , More Basic Less Basic (a) CH 3 CH 2 NH 2 CH 3 CH 2 CONH 2 (b) NaOH CH 3 NH 2 CH3NHCH3 pKa = 10.73 pyridine (C) pKa = 5.25 24.5 aCH pKa = 2 NrV CH 3 CH 2 CH 2 NH 3 + pKa = 9.33 stronger acid (smaller aCH pKb = 14-9.33 = pKa) 10.71 weaker acid (larger p#a) 2 NH 2 CH 3 CH 2 CH 2 NH 2 pKb = 14-10.71 = 4.67 weaker base 3.29 stronger base The stronger base (propylamine) holds a proton more tightly than the weaker base (benzylamine). Thus, the propylammonium ion is less acidic (larger p^a ) than the benzylammonium ion (smaller pKa). To calculate p#b Ka -Kh = : 10~ 14 , pKa + pKb = 14 and pKb = 14 - pKa . 660 Chapter 24 24.6 The basicity order of substituted arylamines is the same as their reactivity order in electrophilic aromatic substitution reactions because, in both cases, electron- withdrawing make substituents 24.7 Use the expressions [RNH2 log [RNH 3 = j [RNH3 24.8 = 1 pH = 7.3, shown more electron-poor and destabilize a positive charge. in Section 24.5. pH-pK a = 7.3-1.3 = 6.0 ] [RNH2 At the site of reaction antilog (6.0) = 10 6 [RNH 2 ] = 10 6 [RNH 3 : + ] ] virtually 100% of the pyrimidine molecules are in the neutral form. Amide reduction can be used to synthesize most amines, but nitrile reduction can be used compounds in (b) and (d) can be synthesized only by amide reduction. to synthesize only primary amines. Thus, the Amine Nitrile Precursor Amide Precursor (a) CH 3 CH 2 CH 2 NH 2 CH 3 CH 2 C= N CH 3 CH 2 CNH 2 (b) CH 3 CH2 CNHCH 2 CH2 CH 3 (CH 3 CH 2 CH2)2NH (c) O C=N CH2 NH 2 (d) o The compounds "12 o II NHCH 2 CH 3 in parts (b) NHCCH 3 and (d) can't be prepared by reduction of a nitrile. Amines and Heterocycles 661 24.9 Step 1: Addition of hydroxide. Step 2: Ring opening. Step 3: Proton transfer. Step 4: Addition of hydroxide. Step 5: Elimination of amine. Step 6: Proton transfer. 24.10 The upper reaction is the azide synthesis, and the lower reaction OH is the Gabriel synthesis. 662 Chapter 24 24.11 Look at the target molecule to find the groups bonded to nitrogen. One group comes from the aldehyde/ketone precursor, and the other group comes from the amine precursor. In most cases, two combinations of amine and aldehyde/ketone Amine Amine Precursor are possible. Carbonyl Precursor O CH 3 (a) II CH 3 CH 2 NHCHCH 3 CH 3 CCH 3 CH 3 CH 2 NH. or CHo3 I CH 3 CHO H 2 NCHCH 3 — NHCH CH , Pt »» HoN- 1 J CH 3 CNH-^^-S02 -NH-<^ N (CH 3 CO) 2 HgN-/""^— S0 2 - NH— ^ jj N 4. Sulfathiazole j) Amines and Heterocycles 24.17 665 nitrated and the nitro group is ultimately reduced, but important in arriving at the correct product. In (a), nitrobenzene is immediately reduced and alkylated. In (c), chlorination occurs before reduction so that chlorine can be introduced in the m-position. In (b) and (d), nitrobenzene is reduced and then acetylated in order to overcome amine basicity and to control reactivity. In both cases, the acetyl group is removed in the last step. In all of these reactions, benzene the timing of the reduction step is is Either method of nitro group reduction (SnCl2, H2) can be used in problem; both methods are shown. Mono- and trialkylated anilines are also formed. all parts of this Width: 612 Height: 792 666 Chapter 24 24.18 Br Br + " N 2 HS0 4 /7-Bromobenzoic acid The shown above one of several ways to synthesize p- bromobenzoic acid and is definitely ngj the simplest way. (The simplest route is Friedel-Crafts alkylation -» bromination -* oxidation). The illustrated synthesis shows the use of the diazonium replacement reaction that substitutes bromine for a nitro group. Oxidation of the methyl group yields the substituted benzoic acid. route is m-Bromobenzoic acid Again, this isn't the easiest route to this compound. In this case, nitration is followed by bromination, then by diazotization, treatment with CuCN, and hydrolysis of the nitrile. Amines and Heterocycles from (b) 667 ra-Bromochlorobenzene /?-(Af,Af-Dimethylamino)azobenzene Coupling takes place between Af,N-dimethylaniline and a benzenediazonium the desired product. salt to yield 668 Chapter 24 24.20 Thiazole Thiazole contains six ^electrons. Each carbon contributes one electron, nitrogen two electrons contributes one electron, and sulfur contributes sulfur and nitrogen have lone electron pairs in sp 2 to the ring orbitals that he n system. Both in the plane of the ring. 24.21 log [RNH 2| [RNH 3 [RNH 2 [RNH 3 [RNH 3 ] pH-pK a = 7.37-6.00 = 1.37 ] ] = + = antilog(1.37) 23.4: [RNH 2 ] = 23.4[RNH 3 ] ] + 23.4 [RNH 3 [RNH 3 = ] + ] = 24.4 [RNH 3 ] = 100% =100% -24.4 =4.1% [RNH 2 ] = 100% - 4.1 = 95.9% 4.1% of histidine molecules have in the protonated form at the imidazole nitrogen physiological pH. 24.22 Attack at C2: unfavorable J Amines and Heterocycles E H E H E 669 H unfavorable Reaction at C3 is favored over reaction at C2 or C4. The positive charge of the cationic intermediate of reaction at C3 is delocahzed over three carbon atoms, rather than over carbons and the electronegative pyridine nitrogen as occurs in reaction at C2 or C4.. two 24.23 CH 2 CH 2 N(CH 3 ) 2 Af,Af-Dimethyltryptarnine atom of Af,Af-dimethyltryptarnine is more basic than the ring lone electron pair is more available for donation to a Lewis The aromatic nitrogen electron lone pair is part of the ring k electron system. The side chain nitrogen nitrogen atom because its acid. 670 Chapter 24 Attack at C3: Positive charge can be stabilized by the nitrogen lone-pair electrons in reaction at both C2 and C3. In reaction at C2, however, stabilization by nitrogen destroys the aromaticity of the fused benzene ring. Reaction at C3 is therefore favored, even though the cationic intermediate has fewer resonance forms, because the aromaticity of the six-membered-ring is preserved. 24.25 NH 2 NHi (CH 3 3 CCCH 3 ) NaBH 3 CN A (CH 3 ) 3 CCHCH 3 B -1 The IR spectrum (pair of bands at around 3300 cm ) shows that B is a primary amine, and the *H NMR spectrum shows a 9-proton singlet, a one-proton quartet, and a 3-proton doublet. An absorption due to the amine protons is not visible. Visualizing Chemistry 24.26 (b) (a) H 63 C (c) H \ / HoC 3 =v / N CH(CH 3 HoN I H A^-Methylisopropylamine secondary amine trans-(2-Methy\cyclopentyl)amine N-Isopropylaniline primary amine secondary amine ) 2 Amines and Heterocycles 671 24.27 (1 S,2S)-( 1 ,2-Diphenylpropy l)amine Hofmann be 180° elimination apart. is (Z)- 1 ,2-Diphenyl- 1 -propene an E2 elimination, in The product that results from which the two groups to be eliminated must geometry is the Z isomer. this elimination 24.29 most basic / H The indicated nitrogen is Lewis acid. it is more electron-rich. The electrons of the n system and are less available for donation to a most basic because other nitrogens are part of the fused-ring 672 Chapter 24 Additional Problems Naming Amines 24.30 (b) (a) Br (c) [^^— CH2CH2 NH 2 v 1^^— NHCH2CH 3 Br 2,4-Dibromoaniline (2-Cyclopentylethyl)amine (d) A^Ethylcyclopentylamine (e) (f) CH, N— CH2 CH2 CH 3 N H 2 NCH 2 CH 2 CH 2 CN CH 3 A/,Af-Dimethylcyclopentylamine 4-Aminobutanenitiile Af-Propylpyrrolidine 24.31 (a) (b) N(CH 3 ) NHCH 3 CH 2 NH 2 3 (Cyclohexylmethyl)amine Af-Dimethylaniline « (c) CH 3 ( A^-Methylcyclohexylamine e> (H 3 C) 2 NCH 2 CH 2 C0 2 H NH 2 (2-Methylcyclohexyl)amine 3-(N, Af-dimethylaminopropanoic acid 24.32 (a) fi — tertiary secondary amine (b) amine NHCH 3 ^ *\ || secondary amine N — •* H \ H Lysergic acid diethylamide secondary amine Amines and Heterocycles Amine 673 Basicity 24.33 The pyrrole anion, C4H4N:", is as the cyclopentadienyl anion. a 6 n electron species that has the same electronic structure Both of these anions possess the aromatic stability of 6 x electron systems. 24.34 The The "a" nitrogen is most basic because "c" nitrogen is the least basic part of the ring its electron pair is most available to Lewis acids. because the lone-pair electrons of the pyrrole nitrogen are k electron system. 24.35 inductive effect of the electron-withdrawing nitro group makes the amine nitrogens of both m-nitroaniline and p-nitroaniline less electron-rich and less basic than aniline. The When nitro the nitro group is para to the amino group, conjugation of the amino group with the group can also occur. /7-Nitroaniline is thus even less basic than m-nitroariiline. 674 Chapter 24 Synthesis of Amines 24.36 (a) NaN iai \^ 3 PBr<, rpi3 CH2CH2CH2CH2OH i Ch^Ch^Ch^Ch^Br ^ CH2CH2CH2CH2N3 1 . 2. UAIH4 H2 CH2CH2CH2CH2NH2 Butylamine (b) CH2CH2CH2CH2OH ^ Cr0 3 H3 SOCIc CH2CH2CH2CO2H CH 2 CH 2 CH 2 COCI CH2CH2CH2CH2NH2 from (a) NaOH ? CH3CH2CH2CH2NHCH2CH2CH2CH3 ^ 1 . " L1AIH4 2 |_| CH3CH2CH2CNHCH2CH2CH2CH3 Q Dibutylamine (c) CrQ 3 H 3 Q , CH2CH2CH2CH2OH Propylamine ^ CH2CH2CH2CO2H 1 — — CH2CH2CH2CONH2-*— Br2 j CH2CH2CH2NH2'' + , NaOH 777; I -v 2U \ H 2 0, S0C 2 ' 2 NHo JSJV CH2CH 2 CH 2 CON 3 heat CH 2 CH 2 CH 2 COCI (d) CH2CH2CH2CH2OH PBr 3 ^ ^ NaCN_ ^ CH2CH2CH2CH2Br CH2CH2CH2CH2CN 1 . 2. UAIH4 H2 CH2CH2CH2CH2CH2NH2 Pentylamine (e ) ^ Periodinane CH2CH2CH2CH2OH CH » C| NaBhU CH 2 CH 2 CH 2 CHO CH 2 CH2CH2CH 2 N(CH 3 )2 MA^-Dimethylbutylamine W excess CH 2 CH 2 CH2NH 2 from ( c ) QH 3 + * CH 2 CH 2 CH 2 N(CH 3 )r 1 . Ag P 0, H P yf^j ' " CH 3 CH=CH 2 Propene + (CH 3 ) 3 N Amines and Heterocycles 675 24.37 (a) CH3CH2CH2CH2CO2H CH3CH2CH2CH2COCI 2 NHt ** CH3CH2CH2CH2CONH2 Pentanamide (b) from CH3CH2CH2CH2NH2 H2 (a) ^ 1 Butylamine LiAIH 4 . CH3CH2CH2CH2CONH2 from NaOH Br ?) CH3CH2CH2CH2CONH2 CH3CH2CH2CH2CH2NH2 2 (a) Pentylamine 1 Br2 PBr3 2. H2 , CH3CH2CH2CH2CO2H **" CH3CH2CH2CHCO2H 2-Bromopentanoic acid (e) LBH3 ~ /-^+^ CH3CH2CH2CH2CO2H 2. ChtaChtaChtaChtaChtaOH H3O I NaCN CH3CH2CH2CH2CH2CN PBr 3 CH3CH2CH2CH2CH2Br Hexanenitrile (f) 1 CH3CH2CH2CH2CH2CN ~ . 2. from UAIH4 H2 ^ CH3CH2CH2CH2CH2CH2NH2 Hexylamine (e) 24.38 (a) N0 2 HNOi 1 , H 2 S0 4 (b) 2. CONH- Hair SnCI 2 H 3 Q+ NaOH,.h 2 o Y T NH 2 NHc Br2 , NaOH H2 (c) COCI CONH- NHc Width: 612 Height: 792 676 Chapter 24 24.39 First, shown synthesize aniline from benzene, as in Problem 24.38 (a). " HS0 4 N2 H 2 NH 2 CuCN HNO 2_ H 2 S0 4 1 2. LiAH H2 24.40 (a) CH 3 CH 2 CH 2 CH 2 CONH 2 1 UAIH4 2. H2 CHgCH 2 CH 2 CH 2 CH 2 NHj (b) 1 . UAIH4 CHgCH 2 CH 2 CH2 CN ~ ~ ~ * (c) CH0CH0CHi=r.M. — CHo2 32 1 . 2. BHo, CH3CH 2 CH 2 CH 2 CH 2 NH 2 THF PBi> CH0CH0CH0CH0OH 2 3 H 2 2 rOH 2 2 CH3CH 2 CH 2 CH 2 Br NaCN | CH3CH 2 CH 2 CH 2 CH 2 NH 2 (d) CH 3 CH 2 CH 2 CH 2 CH 2 CONH 2 Br 2 , J . UAIH4 — — H~~0 Cri3CH 2 CH 2 CH 2 CN NaOH CHgCH 2 CH 2 CH 2 CH 2 NHj H2 (e) PBrv CH3CH 2 CH 2 CH 2OH CH 3 CH 2 CH 2 CH 2 Br NaCN CH 3 CH 2 CH 2 CH 2 CN 1 I . \ 2. UAIH4 H2 CHgCH 2 CH 2 CH 2 CH 2 NH 2 (f) CHoCH 2 CH 2 CH 2 CH — CHCH 2 CH 2 CH 2 CHg CH3CH 2 CH 2 CH 2 CHO NH- NaBK 1. -~ 0< ~ 2. Zn, H3O + 2 CH3CH 2 CH 2 CH 2 CH 2 NH 2 CHgCH 2 CH 2 CH 2 CHO ^ ' Amines and Heterocycles (g) SOCI^ CH 3 CH2CH 2 CH 2 C02H CH 3 CH2CH 2 CH 2 COCI 1 2 1 CH 3 CH 2 CH 2 CH 2 CH 2 NH 2 ^ . LiAIH 4 - -4 NH 3 CH 3 CH 2 CH 2 CH 2 CONH 2 24.41 OH OH O II I I CH— CCH 3 + H 2 NCH 3 I CH— CHCH3 NaBH 4 Ephedrine CH 3 OH Reactions of Amines 24.42 (b) 24.43 NHCHo3 677 678 Chapter 24 In (e), the amine reacts with the Grignard reagent and inactivates it. Amines and Heterocycles 24.45 Hydrogens that can be eliminated are starred. In cases where more than one alkene can form, the alkene with the less substituted double bond is the major product.. 1 . CH 3I Ag 2 Q, H 2 Q 3. heat Amine (a) excess 2. Alkene Amine * r> (b) O NHCH 3 H 2C CHo3 N(CH 3 3 ) — CHCH2 CH2CH 2CH3 N(CH 3 2 ) I major HNCHCH 2 CH2 CH 2 CH 3 CH 3 CH^— CHCH 2 CH 2 CH 3 minor (c) CHo3 CHo3 r I _ CH 3 CHCH — CHCH 2 CH3 CH3 CHCHCH 2 CH 2 CH 3 N(CH 3 3 ) major NHo CHo3 r CH 3 C — CHCH 2 CH 2 CH 3 minor 24.46 (a) HNO' H 2 Pt H 2 S0 4 EtOH* , (b) CH 2 NH, HNO, H 2 S0 4 CH 3 from (a) 1 LiAIH 4 2. H2 679 680 Chapter 24 Amines and Heterocycles (d) HoC — CHo H2C NaOH CH, Br HoO Br equiv + 2 H2 + 2 NaBr N I NH 2 1 681 CH- I CHo Spectroscopy 24.48 The HNMRofthe amine shows ! 5 peaks. at 3.40 5 is Two two are due electronegative element (oxygen), due to an ethyl group bonded to an 4 aromatic ring hydrogens, and the peak are to due to 2 amine hydrogens. HCOCH 3 "OH HoO OCH 2 CH 3 OCH0CH3 + " CH 3 C0 2 CgH^NO C 10 H 13 NO 2 Phenacetin /7-Ethoxyaniline C 6 H 7 NO /7-Aminophenol 24.49 (b) (a) HOCH2CH2CH2NH2 e d a c b a= 1.68 6 b = 2.69 5 c= 2.89 5 d = 3.72 5 (CH 3 0)2CHCH 2 NH2 c d b a a= 1.29 6 b = 2.78 6 c= 3.39 6 d = 4.31 6 682 Chapter 24 General Problems 24.50 CHCHg CH 3 a b, A c aCHCH NCH CH_CH2 2 £ 3 ^j^ NH (a) 3 NaBH4 (b) excess CH 3 I; (c) Ag 2 0, H 2 0, heat; (d) RCO3H (e) (CH 3) 2 NH. Step (e) is an Sn2 ring opening of the epoxide by nucleophilic substitution of the amine at the primary carbon. , ; 24.51 NO (J Oxazole is an aromatic 6 electron are in p jt electron heterocycle. orbitals that are part from each carbon. Oxazole Two oxygen electrons of the n electron system of the and one nitrogen along with one ring, 2 An oxygen lone pair and a nitrogen lone pair are in sp orbitals that lie in the plane of the ring. Since the nitrogen lone pair is available for donation to acids, oxazole is more basic than pyrrole. electron 24.52 H :o' :o: II R' A^-Protonation (no resonance stabilization) I R' R' NH 2 O-Protonation (resonance stabilization) Protonation occurs on oxygen because an O-protonated amide is stabilized by resonance. Amines and Heterocycles 683 24.53 HpC HpC \ / C— R— Cw R' CHp CHp HpC R— C ^C— R' Ik \ i // O o ,:0: ^ H— nh 3 Ho 3. tl HpC — CH HoC— CHp HpC R "9 C rC— R* :0: ~, R' \ I r-<5 O :kH H HoO + ~xP O H ? 0> NH 2 L.NH2 «« HoC— CH R— C^ C— /II HoC— CH 2 W / ( R' / Qokt:nh 2 R w- H 2 C— CH w ;c H.q: c R R- * + CX H 2 (X^ . V C-R" IJJ H H H 9. * H rU C— CH HC— CH w w // R— C— N N tl R— C. R' + 10. .. ' Steps Steps Steps Steps Steps R' N I H .C— H + H 2° 1,6: Protonation. 2,7: Nucleophilic addition. 3,8: Proton transfer. 4,9: Loss of water. 5,10: Loss of proton. The mechanism consists of the nucleophilic addition of ammonia, first to one of the ketones, and then to the other, with loss of two equivalents of water. . 684 Chapter 24 24.54 HQ CHo H2 qp/CH3 c/i. CCH2. 3. V. NHoOH H 2C NHOH HoC \ /CCHg ,CCHo CCH- o o o 4. tl CH 3 CH< CH 3 ^:b CHo3 H I N HoC ^ + CH< HO: HoC \ \p:OH OH / f HoC u H v NOH 5. H 2 C' \N0H CCHo6 .CCHo // CH 3 O + H2 tl 10. 9. HoC CH 3 CHo CH< /% HC I \\ C O H 2 COCH 3 r H2 + I CH 2 CHCH 3 + Icr03 H 3 CHo3 | OH H 2 C— CHCH 3 3 NaBH„ Cyclopentamine / [ — CH CCH 2 3 The last step of the synthesis is a reductive amination of a ketone that is formed by oxidation of the corresponding alcohol. The alcohol results from the Grignard reaction between cyclopentylmagnesium bromide and propylene oxide. 24.60 (a) COCH 2 CH 2 N(CH 3 CH 3 (CH 2 3 NH ) CH 3 CH 2 CH 2 CHO ); NaBH, O COCH 2 CH 2 N(CH 3 2 OoN ) 1 (b) . 2. H 2> Pt e^qh II » H 2N COCH 2 CH 2 N(CH 3 SOCI 2 (c) HOCH 2 CH 2 N(CH 3 ) 2 )i Benzene CH 3 CI , Pyridine AICI 3 O II OoN\\ The / COH-« KMn0 ± 4 H2 HNO' OoN d H 2 S0 4 \ CH< // synthesis in (a) is achieved by a reductive amination reaction. Reactions in (b) include formation of an acid chloride, esteriflcation, and reduction of the nitro group. 688 Chapter 24 24.61 Atropine Tropidene Tropine ch 2OH q H0 2CCHC 6 H 5 CH 2 OH Tropic acid We know the location of the -OH group of tropine because optically inactive alcohol. This hydroxyl group results that is composed of tropine and it is stated that tropine is an from basic hydrolysis of the ester tropic acid. 24.62 1 . CH 3 I 1 . CH3I 2. Ag 2 Q, H 2 Q 2. Ag 2 Q, H 2Q 3. heat 3. heat Tropidene Tropilidene results from two cycles of 24.63 The formula C9H17N N(CH 3 Tropilidene Hofmann elimination on tropidene. indicates two degrees of unsaturation in the product. Both are probably due to rings since the product results from catalytic reduction. Step 1: Reduction of nitrile Step 2: Nucleophilic addition. Step 3: Dehydration. Step 4: Reduction of double bond. ) 3 Amines and Heterocycles 689 24.64 The molecular formula indicates that coniine has one double bond or ring, and the Hofmann elimination product shows that the nitrogen atom is part of a ring. 1 . excess CH 3 I 2. Ag 2 Q, H 2 Q 3. heat (CH 3 2 N ) 5-(N, A^Dimethylamino)- 1 -octene Coniine 24.65 C0 2 Et HC0 2 Et CH 2 CH0CH0CH9C 3 2 2jf O + 1 CHo2 . 2. II CHCN Na "OEt, + H3 EtOH CH 2 CHoCHoCHoC 3 2 ZJj | CH 2 CN O t. H3 H H H H 2 ,Pt CH 2 4. O H2 H Step 1: Michael addition. Step 2: Ester hydrolysis; decarboxylation. Step 3: Reduction. Step 4: Cyclization. Step 5: Reduction. 3. CH 3 CH 2 CH 2 C O CH 2 N-^ Coniine , heat H \ / \ / CHoCHpCHoC + + h" C0 2 CH 2 CH 2 CN + EtOH 690 Chapter 24 24.66 OH OH OH OH Tyramine When you see -CH2NH2, substitution of a benzylic think of the reduction of a nitrile. The nitrile comes from bromide by ~CN. 24.67 The reactive intermediate is benzyne, which undergoes a Diels-Alder reaction with cyclopentadiene to yield the observed product. Amines and Heterocycles 691 24.68 CH 3 I 1 2. Ag 2 Q, H 2 Q 2. Ag 2 Q, H 2 Q 3. heat 3. heat 1 I N j . N(CH 3 Two successive cycles of Hofmann CH3I . N(CH 3 ) 3 Cyclooctatriene ) 2 elimination lead to formation of cyclooctatriene. NBS EtOH Cyclooctatetraene Allylic bromination followed by elimination yield cyclooctatetraene. 24.69 H HC) -V H f? C "nh2 .. c H fX .^c + + Br2 , NH 3 NaOH H2 Hofmann rearrangement (the mechanism is shown in Section 24.6) of an a-hydroxy amide produces a carbinolamine intermediate that expels ammonia to give an aldehyde. 692 Chapter 24 24.70 OCH 3 eeri- ^rj:NH2 CH3 I- er NHCH 3 /\ O HH 3. I - .. :o: H+ :OCH 3 :0: OCHo r-X. NH-CH3 N- CH 3 5. O Step Step Step Step Step 1: + 4. o CH3OH Conjugate addition of amine. 2: Proton transfer. 3: Nucleophilic addition of amine. 4: Proton transfer. 5: Elimination of methanol. 24.71 Br C0 2 CH 3 CO0CH0 HoC 2 1. OH Et 3 N tl Y ' Br 2. : CO0CH3 I" N + C0 2 CH 3 "* + OH 3. C0 2CH 3 - Step 1: Sn2 displacement of Br by amine. Step 2: Deprotonation. Step 3: Conjugate addition of alcohol. Step 4: Proton transfer. °H + Et 3 NH n Amines and Heterocycles 24.72 (5)-Norcoclaurine Step 1: Nucleophilic addition of the amine to the protonated aldehyde. Step 2: Proton transfer. Step 3: Loss of water. Step 4: Electrophilic aromatic substitution. Step 5: Loss of proton. 693 694 24.73 Chapter 24 Amines and Heterocycles Steps 1-3: El Steps 4-6: Sn2 elimination (protonation, loss of HOCH3, deprotonation). substitution (protonation, substitution, deprotonation). Step 7: El elimination of carbamate. Steps 8-9: conjugate addition of DNA (addition, deprotonation). Notice that five of the nine steps are either protonations or deprotonations. 695 Width: 612 Height: 792 696 Chapter 24 24.74 (a) N— CH 2 N(CH 3 2 HoC ) - b Ch^CHg b dore a a= 2.25 6 a= 1.14 6 b = 2.89 6 b = 3.40 6 c = 6.66 6, 7.03 6 c = 4.47 6 d = 6.65 6 d ore e = 7.24 6 24.75 \|| I . -NHg R H R NH 2 I ^C R ^C0 2 H H :OH :0: Go: ^ CC-NH 3 H3 + *4. NH 2 \ H H NHo Cnh 2 I «- R CCN H A .C^ C 3. R :CN H Step 1: Addition of NH 3 Step 2: Elimination of ~OH. Step 3: Addition of ~CN. Step 4: Hydrolysis of nitrile. . The mechanism of acid-catalyzed nitrile hydrolysis is shown in Problem 20.51. Amines and Heterocycles 24.76 Step 1: Conjugate addition of hydrazine. Step 2: Proton Step transfer. 3: Nucleophilic acyl substitution, Step 4: Proton transfer. forming the cyclic amide. 697 Review Unit 9: Carbonyl Compounds Reaction at the II a Carbon; Amines Major Topics Covered (with vocabulary): Carbonyl a-substitution reactions: tautomerism tautomer enolate ion Hell-Volhard-Zelinskii reaction (3-diketone p-keto eater malonic ester synthesis acetoacetic eater synthesis LDA a-substitution reaction Carbonyl condensation reactions: carbonyl condensation reactions aldol reaction enone mixed aldol reaction Claisen condensation reaction Dieckmann cyclization Michael reaction Michael acceptor Michael donor Stork enamine reaction Robinson annulation reaction Amines: primary, secondary, tertiary amine quaternary ammonium salt arylamine heterocyclic amine pyramidal inversion K\> azide synthesis Gabriel amine synthesis reductive amination Hofmann rearrangement Curtius rearrangement Hofmann elimination reaction arenediazonium salt diazotization Sandmeyer reaction azo compound diazonium coupling reaction pyrrole thiophene furan pyridine fused-ring heterocycle pyrimidine purine nitrogen rule Types of Problems: After studying these chapters, you should be able to: - Draw keto-enol tautomers of carbonyl compounds, identify acidic hydrogens, and draw the resonance forms of enolates. Formulate the mechanisms of acid- and base-catalyzed enolization and of other a-substitution reactions. - Predict the products of a-substitution reactions. - Use - Predict the products of carbonyl condensation reactions. Formulate the mechanisms of carbonyl condensation reactions. - Name - Predict the basicity of alkylamines, arylamines and heterocyclic amines. Synthesize alkylamines and arylamines by several routes. - - a-substitution reactions in synthesis. Use carbonyl condensation reactions in synthesis. and draw amines, and classify amines as primary, secondary, tertiary arylamines, or heterocyclic amines. , quaternary, Predict the products of reactions involving alkylamines and arylamines. Use diazonium involving arylamines, including diazo coupling reactions. and explain their acid-base properties. Explain orientation and reactivity in heterocyclic reactions, and predict the products of reactions involving heterocycles. Propose mechanisms for reactions involving alkylamines, arylamines, and heterocycles. Identify amines by spectroscopic techniques. Draw salts in reactions orbital pictures of heterocycles Review Unit 9 Points to * 699 Remember: unusual to think of a carbonyl compound as an acid, but the protons a to a carbonyl group can be removed by a strong base. Protons a to two carbonyl groups are even more acidic: in some cases, acidity approaches that of phenols. This acidity is the basis for a-substitution reactions of compounds having carbonyl groups. Abstraction by base of an a proton produces a resonance-stabilized enolate anion that can be used in alkylations involving alkyl hahdes and It is tosylates. * Alkylation of an unsymmetrical LDA-generated enolate generally occurs at the less hindered carbon. * When you need to synthesize a (3-hydroxy a ketone or aldehyde or an a,p-unsaturated ketone or aldehyde, use an aldol reaction. When you need to synthesize a p-diketone or p-keto ester, use a Claisen reaction. When you need to synthesize a 1,5-dicarbonyl compound, use a Michael reaction. The Robinson annulation is used to synthesize polycyclic molecules by a combination of a Michael reaction with an aldol condensation. many of the mechanisms in this group of chapters, the steps involving proton transfer are not explicitly shown. The proton transfers occur between the proton and the conjugate base with the most favorable pK of those present in the solution. These steps have been omitted at times to simplify the mechanisms. * In * In the Claisen condensation, the enolate of the p-dicarbonyl compound is treated with H3O" to yield the neutral product. * For an amine, the larger the value of pKa of its ammonium ion, the stronger the base. The smaller the value of pA'b of the amine, the stronger the base. * The Sandmeyer reaction allows the synthesis of substituted benzenes that can't be formed by These reactions succeed because N2 is a very good electrophilic aromatic substitution reactions. leaving group. Self-Test: O o CH3CH2 H (CH 3 )2CHCH 2 CH2 || A into how they? B Pentymal Dypnone (a sedative) (sunscreen) The six-membered What are O ring in A is formed by the cyclization of two difunctional compounds. What type of reaction occurs to form the ring? The two alkyl groups are introduced to cyclization. What type of reaction is occurring, and type of reaction occurs in the formation of Dypnone (B)? Why might B one of the difunctional compounds prior is it carried out? What be effective as a sunscreen? Review Unit 9 700 o CHoCHNCH. CH ; C Benzphetamine Butralin (an herbicide) (an appetite suppressant) What type of amine is C? Do you expect it to be more or less basic than ammonia? Than What product do you expect from Hofmann elimination of C? What significant absorptions might be seen in the IR spectrum of C? What information can be obtained from the aniline? mass spectrum? Plan a synthesis of D from benzene. Multiple Choice: 1 . Which of the following compounds has (a) 2 . 2-Pentanone (b) 3-Pentanone (c) four acidic hydrogens? Acetophenone (d) Phenylacetone In which of the following reactions is an enol, rather than an enolate, the reacting species? alkylation (d) (a) acetoacetic acid synthesis (b) malonic ester synthesis (c) Hell-Volhard-Zelinskii reaction LDA 3 4 . . Cyclobutanecarboxylic acid is probably the product of a: (a) malonic ester synthesis (b) acetoacetic ester synthesis (d) Hell-Volhard-Zelinskii reaction An LDA (a) 5 . 6 . alkylation can be used to alkylate aldehydes (b) ketones (c) esters all (c) LDA alkylation of the following, except: (d) nitriles you want to carry out a carbonyl condensation, and you don't want to form a-substitution product, you should: (a) lower the temperature (b) use one equivalent of base (c) use a catalytic amount of base (d) use a polar aprotic solvent If Which reaction forms a cyclohexenone? (a) Dieckmann cyclization (b) Michael reaction (c) Claisen condensation (d) intramolecular aldol condensation 7 . All of the following molecules are good Michael donors except: (a) Ethyl acetoacetate (b) Nitroethylene (c) Malonic ester (d) Ethyl 2-oxocyclohexanecarboxylate 8 . The ammonium ion of which of the following amines has (a) 9 . 10. Methylamine (b) Trimethylamine (c) Aniline (d) the smallest value of pKa ? p-Bromoaniline All of the following methods of amine synthesis are limited to primary amines, except: (a) Curtius rearrangement (b) reductive animation (c) Hofmann rearrangement (d) azide synthesis To form an azo compound, an aryldiazonium (a) CuCN (b) benzene (c) nitrobenzene (d) should react with: phenol salt Chapter 25 - Bio molecules: Carbohydrates Chapter Outline I. Classification of carbohydrates (Section 25.1). A. Simple vs. complex: Simple carbohydrates (monosaccharides) can't be hydrolyzed to smaller units. 1 2. Complex carbohydrates are made up of two or more simple sugars linked together. . a. b . A disaccharide is composed of two monosaccharides. A polysaccharide is composed of three or more monosaccharides. B. Aldoses vs. ketoses: 1 A monosaccharide with an aldehyde carbonyl group is an aldose. 2 A monosaccharide with a ketone carbonyl group is a ketose. C. Tri-, tetr- pent-, etc. indicate the number of carbons in the monosaccharide. Monosaccharides (Sections 25.2-25.7). A. Configurations of monosaccharides (Section 25.2-25.4). . . , II. 1 . Fischer projections (Section 25.2). a. Each chirality center of a monosaccharide is represented by a pair of crossed lines. The horizontal line represents bonds coming out of the page. The vertical line represents bonds going into the page. Allowed manipulations of Fischer projections: i. ii. b . i. A Fischer projection can be rotated on the page by 1 80°, but not by 90° or 270°. ii. Holding one group steady, the other three groups can be rotated clockwise or counterclockwise. c. d. Rules for assigning R,S configurations. i. Assign priorities to the substituents in the usual way (Section 5.5). ii. Perform one of the two allowed motions to place the lowest priority group at the top of the Fischer projection. iii. Determine the direction of rotation of the arrow that travels from group 1 to group 2 to group 3, and assign R or S configuration. Carbohydrates with more than one chirality center are shown by stacking the centers on top of each other. i The carbonyl carbon is placed at or near the top of the Fischer projection. . 2. D,L sugars (Section 25.3). a. (/?)-Glyceraldehyde is also b . In D sugars, the known as D-glyceraldehyde. -OH group farthest from the carbonyl group points to the right in a Fischer projection. i. 3 . Most naturally-occurring sugars are D sugars. L sugars, the -OH group farthest from the carbonyl group points c. In d a Fischer projection. D,L designations refer only to the configuration farthest from the carbonyl . to the left in carbon and are unrelated to the direction of rotation of plane-polarized Configurations of aldoses (Section 25.4). a. There are 4 aldotetroses - D and L erythrose and threose. b. c. light. There are 4 D,L pairs of aldopentoses: ribose, arabinose, xylose and lyxose. There are 8 D,L pairs of aldohexoses allose, altrose, glucose, mannose, gulose, idose, galactose, and talose. : 702 Chapter 25 d . A scheme for drawing and memorizing the D-aldohexoses: i . ii. Draw Draw all the -OH groups at C5 pointing to the right. first four -OH groups at C4 pointing to the right and the second four pointing to the left. Alternate -OH groups at C3: two right, two left, two right, two left. iv. Alternate -OH groups at C2: right, left, etc. v Use the mnemonic " All .altruists .gladly make .gym in .gallon Xanks " to assign hi. . names. B. Cyclic structures of monosaccharides (Section 25.5). Hemiacetal formation. 1 a. Monosaccharides are in equilibrium with their internal hemiacetals. i. Glucose exists primarily as a six-membered pyranose ring, formed between the -OH group at C5 and the aldehyde group at CI Fructose exists primarily as a five-membered furanose ring. ii. b Structure of pyranose rings. Pyranose rings have a chair-like geometry. i. . . ii. iii. iv. The hemiacetal oxygen is at the right rear for D-sugars. group on the right in a Fischer projection is on the bottom face pyranose ring, and an -OH group on the left is on the top face. For D sugars, the -CH2OH group is on the top. An -OH Mutarotation. a. When a monosaccharide cyclizes, a new chirality center The two diastereomers are anomers. i. 2. is in a generated. The form with the anomeric -OH group trans to the -CH2OH group is the a anomer (minor anomer). iii. The form with the anomeric -OH group cis to the -CH2OH group is the p anomer (major anomer). When a solution of either pure anomer is dissolved in water, the optical rotation ii. b . of the solution reaches a constant value. This process is called mutarotation. i. ii. Mutarotation is due to the reversible opening and recyclizing of the hemiacetal ring and is catalyzed by both acid and base. C. Reactions of monosaccharides (Section 25.6). 1 . Ester and ether formation. by treatment with an acid anhydride or acid chloride. Ethers are formed by treatment with methyl iodide and Ag20. c Ester and ether derivatives are crystalline and easy to purify. Glycoside formation. a. Treatment of a hemiacetal with an alcohol and an acid catalyst yields an acetal. Acetals aren't in equilibrium with an open-chain form and do exhibit i. mutarotation. Aqueous acid reconverts the acetal to a monosaccharide. ii. b. These acetals, called glycosides, occur in nature. c. Glycosides are named by first citing the alkyl group and then replacing the -ose suffix of the sugar with -oside. d. The laboratory synthesis of glycosides is achieved by the Koenigs-Knorr a. b Esterification occurs . . 2 . reaction. i. ii. iii. Treatment of the acetylpyranose with HBr, followed by treatment with the appropriate alcohol and Ag20, gives the acetylglycoside. Both anomers give the same product. The reaction involves neighboring-group participation by acetate. Biomolecules: Carbohydrates 3 . 703 Phosphorylation. Monosaccharides can be phosphorylated by ATP to form a glycosyl monophosphate. b The resulting glycosyl monophosphate can react with a second nucleoside triphosphate to produce a glycosyl diphosphate. This product can react with a lipid or a protein to form a glycoconjugate. c. 4. Reduction of monosaccharides. a. Reaction of a monosaccharide with NaBFLt yields an alditol (a poly alcohol). Oxidation of monosaccharides. 5 Several mild reagents can oxidize the carbonyl group to a carboxylic acid a. a. . . (aldonic acid). Br2 i. In the laboratory, aqueous ii. Historically, Tollens reagent, Fehling's reagent is used to oxidize aldoses (not ketoses). and Benedict's reagent have served as tests for reducing sugars. All aldoses and some ketoses are reducing sugars, but glycosides are nonreducing. b. The more powerful oxidizing agent, dilute HNO3, oxidizes aldoses to dicarboxylic acids (aldaric acids). Enzymes can oxidize the -CH2OH of a monosaccharide (with oxidizing the c. aldehyde) to form a uronic acid. 6. Chain-lengthening: the Kiliani-Fischer synthesis. a. In the Kiliani-Fischer synthesis, an aldehyde group becomes C2 of a chainlengthened monosaccharide and the added carbon is the new CI. b The reaction involves cyanohydrin formation, reduction and hydrolysis. The products are two diastereomeric aldoses that differ in configuration at C2. c. 7 Chain-shortening: the Wohl degradation. a. The Won! degradation shortens an aldose by one carbon. b The reaction involves treatment of the aldose with hydroxylamine, dehydration and loss of from the resulting cyanohydrin. D. Eight essential monosaccharides (Section 25.7). 1 Glucose, galactose, mannose and xylose are aldoses. 2 Fucose is a deoxy sugar. 3 N-Acetylglucosamine and ^-acetylgalactosamine are amino sugars. 4. A^-Acetylneuraminic acid is the parent compound of the sialic acids. All of the essential monosaccharides arise from glucose. 5 HI. Other carbohydrates (Sections 25.8-25.1 1). A. Disaccharides (Section 25.8). 1 Cellobiose and maltose. ii. . . . HCN . . . . . Cellobiose and maltose contain a l-» 4-glycosidic acetal bond between two glucose monosaccharide units. a. Maltose consists of two glucopyranose units joined by a l-» 4-a-glycosidic bond. c. Cellobiose consists of two glucopyranose units joined by a l-» 4-(3-glycosidic bond. d. Both maltose and cellobiose are reducing sugars and exhibit mutarotation. Humans can't digest cellobiose but can digest maltose. e. b 2 . . Lactose. a. Lactose consists of a unit of galactose joined by a (3-glycosidic bond between CI and C4 of a glucose b. Lactose is unit. a reducing sugar found in milk. 704 Chapter 25 Sucrose. Sucrose is a disaccharide that yields glucose and fructose on hydrolysis. a. Sucrose is called "invert sugar" because the sign of rotation changes when i. sucrose is hydrolyzed. ii. Sucrose is one of the most abundant pure organic chemicals in the world. b. The two monosaccharides are joined by a glycosidic link between CI of glucose and C2 of fructose. c. Sucrose isn't a reducing sugar and doesn't exhibit mutarotation. B. Polysaccharides and their synthesis (Section 25.9). Polysaccharides have a reducing end and undergo mutarotation, but aren't 1 considered to be reducing sugars because of their size. 2 Important polysaccharides. 3. . . a. b. Cellulose. i. Cellulose consists of thousands of D-glucose units linked by l-> 4-p- ii. glycosidic bonds. In nature, cellulose is used as structural material. Starch. i. Starch consists of thousands of D-glucose units linked by 1-* 4-a-glycosidic bonds. ii. Starch can be separated into amylose (water-soluble) and amylopectin (water-insoluble) fractions. (a). iii. Amylopectin contains l-» 6-a-glycosidic branches. is digested in the mouth by glycosidase enzymes, which only cleave Starch a-glycosidic bonds. c. 3 . An a. b . c. Glycogen. Glycogen i. is an energy-storage polysaccharide. Glycogen contains both 1^-4- and l-> 6-links. ii. outline of the glycan assembly method of polysaccharide synthesis. A glycal (a monosaccharide with a C1-C2 double bond) is protected at C6 by formation of a silyl ether and at C3-C4 by formation of a cyclic carbonate ester. The protected glycal is epoxidized. Treatment of the glycal epoxide (in the presence of ZnCl2) with a second glycal having a free C6 hydroxy 1 group forms a disaccharide. The process can be repeated. d Other important carbohydrates (Section 25.10). C. Deoxy sugars have an -OH group missing and are components of nucleic 1 2 In amino sugars, an -OH is replaced by a -NH2. Amino sugars are found in chitin and in antibiotics. a. surface carbohydrates and influenza viruses (Section 25.11). D. Cell . . acids. . 1 . Polysaccharides are involved in cell surface recognition. a. Polysaccharide markers on the surface of influenza viruses are variants of two types of glycoproteins -hemagglutinin (H - Type 5 or Type 1), and neuraminidase (N - Type b . Infection occurs by c. when 1). a virus binds to a receptor on a target cell and is engulfed the cell. New viral particles are produced, pass out of the cell, and are held to surface receptors. d . A neuraminidase enzyme cleaves the receptor-virus bond, allowing the virus to invade a 2 . new cell. Antiviral vaccines block the neuraminidase enzyme, limiting the spread of the virus. Biomolecules: Carbohydrates 705 Solutions to Problems 25.1 (b) (a) (d) (c) ? r° H^C-*OH HO^Cj^H H^C-OH r° HO*- C-" H I CH 2 OH I H^C—H H^C-OH I H»-C-OH I Hy>0 CH 2 OH CH 2 OH HO"- I H I H^ I H^C- OH CH 2 OH CH 2 OH I CH 2 OH Threose an 25.2 Ribulose 2-Deoxyribose Tagatose a ketopentose aldotetrose an aldopentose a ketohexose Horizontal bonds of Fischer projections point out of the page, and vertical bonds point into the page. (a) C0 2 H (b) C0 2 H 90 2 H H H2 N -H H 2 N- NH, HoC CH< CHc CHO CHO OH CHO R i ' i H OH H- CH 3 i (c) CH< CH 3 CH< CH 3 decide .it CH2CHg :ho CH CH 2 CHg two Fischer projections if CH- H j CH 2 CH3 To HO '^CHO H -CHO H" 25.3 OH are identical, use the superimpose two groups of each projection. after rotation, the projections represent the If the two allowable rotations to remaining groups are also superimposed same enantiomer. in both A and B, keep it steady, and rotate the other groups are superimposed, the two projections are identical. If only two groups are superimposed, the projections are enantiomers. Thus, A (a) Since -H is in three groups. is If, the same position after rotation, all identical to B. A HO B CHO r I Ml OH H = HOCH 2 1 steady CH 2 OH CHO H Width: 612 Height: 792 706 Chapter 25 B OH HOCH H r H CHO D CH 2 OH HO- CHO steady B OH CH 2OH 180° CHO OHC- » OH HOCH, H CHO CH 2 OH B Projections A, 25.4 + OH and C are identical, and D is their enantiomer. Rotate the structure 1 80° around the horizontal axis to arrive at a drawing having the hydrogen at the rear. Assign the R,S configuration as usual, and draw the Fischer projection H CI T HOCH 2 — C~CH HOCHo*^ 3 ' ^ CH< R H 25.5 CH< HOCHc CI the skeleton of the Fischer projection and add the -CHO and -CH2OH groups to the top and bottom, respectively. Look at each carbon from the direction in which the -H and -OH point out of the page, and draw what you see on the Fischer projection. Draw View C3 from -OH this side; on the is right. \ CHO HO H M 1 .a HO^ A CHO H*-l— OH i\ HOH H H I CH 2 OH t View C2 from -OH is on the H»-j— OH CHO R OH R OH this side; right. CH 2OH Hi onio ecu Its: I 25.6 The hydroxy! group bonded the right in a D S HO- . H H CH 2 OH (c) — R C=0 H ) CH 2 OH sugar. — S HO- L CHO R OH (b) HO- R OH H- CH 2 OH OH CH 2 OH D-Xylose L-Erythrose D-Xylulose 25.7 CHO R OH H- HO- H HO- H L-(+)-Arabinose CH 2 OH 25.8 CHO (a) HO- HO- H H- CHO (b) CHO (c) H HO- OH H- OH HO- H H- OH HO- H HO- HO- CH 2 OH HO- CH 2OH CH 2 OH L-Xylose 25.9 An L-Galactose aldoheptose has 5 chirality centers. Thus, there are 2 L-Allose 5 = 32 aldoheptoses aldoheptoses and 16 L aldoheptoses. 25.10 See Problem 25.5 View C3 from -OH is for the method of solution. this side; on the right. CHO OH OH — R OH — R OH — R OH CH 2 OH CH 2 OH 1 Hdta OHC 2 .a C 3 C H OH H OH t View C2,C4 from -OH is CH 2 OH l\ l\ t 4 on the this side; right. 707 carbon farthest from the carbonyl group points to sugar, and points to the left in an CHO (a) to the chiral Carbohydrates -OH D-Ribose - 16 D 708 Chapter 25 25.11 The steps for drawing a furanose are similar to the steps for drawing a pyranose. Ring formation occurs between the -OH group at C4 and the carbonyl carbon. groups on the right in a Fischer projection is on the bottom face of the ring. A CHO R OH R OH R OH — — — CH 2 OH HOCH O OH D-Ribose OH (Furanose form) of fructose results from ring formation between the -OH group at C5 and the C2. In the a anomer, the anomeric -OH group is trans to the C6 -CH2OH group, and in the /j anomer the two groups are cis. In the pyranose form, cyclization occurs between the -OH group at C6 and the ketone. The more stable chair conformations are 25.12 The furanose ketone at shown. a-D-Fructopyranose a-D-Fructofuranose /3-D-Fructopyranose /3-D-Fructofuranose Biomolecules: Carbohydrates 709 25.13 There are two ways to draw these anomers: (1) Following the steps in Worked Example draw the Fischer projection, lay it on its side, form the pyranose ring, and convert it to a chair, remembering that the anomeric -OH group is cis to the C6 group; (2) Draw p-Dglucopyranose, and exchange the hydroxyl groups that differ between glucose and the other two hexoses. 25.3, a /3-D-Mannopyranose /3-D-Galactopyranose p-D-Galactopyranose and (3-D-mannopyranose each have one hydroxyl group position and are therefore of similar stability. 25.14 In the previous problem we drew /?-D-galactopyranose. In this problem, invert the D enantiomer and perform a ring-flip to configuration at each chirality center of the the structure of the in the axial arrive at L enantiomer. /3-D-Galactopyranose /3-L-Galactopyranose All substituents, except for the -OH at C4, are equatorial in the more stable conformation of /3-L-galactopyranose. 25.15 From the model, we can form of a D-hexose. It is an a-anomer because the anomeric hydroxyl group is trans to the group at C6. Comparing the model with a-D-glucopyranose, we see that all groups have the same axial/equatorial relationship, except for the hydroxyl group at C3, which is axial in the model and equatorial in a-D-glucopyranose. The monosaccharide is a-D-allopyranose. Use see that the monosaccharide Figure 25.3 as a reference. OH HOCH 2 OH OH OH OH a-D-Allopyranose OH CH 2 OH D-Allose is the pyranose 710 Chapter 25 25.16 CH 3 OCH 2 CH 3I,Ag 2 (a) OCH, HOCH OCH 3 OH OCH3 OH /?-D-Ribofuranose \ (b) (c^CO^O, pyridine CHoCO 3 H 25.17 CHO H- OH H- H LNaBH^ H- OH 2.H 2 H- OH HO- OH OH H- -OH H- H HO- H 1. NaBH, HO- -H H- OH HO- H 2. H2 HO- -H H- OH H- HO- CH 2 OH CH 2 OH D-Glucose D-Glucitol Reaction of D-galactose with CH 2 OH CHO CH 2 OH NaBH4 yields OH CH 2 OH CH 2 OH D-Galactose an alditol that -OH H- Galactitol has a plane of symmetry and is a meso compound. 25.18 CHO H- HO- 9H 2 OH OH H- H 1 H- OH 2.H 2 H- OH CH 2 OH D-Glucose . NaBH 4 HO- -OH HO- H -H HO- H H- -OH H- H- -OH HO- CH 2 OH D-Glucitol CHO CH 2 OH 1 . HO- H NaBH 4 HO- H OH 2.H 2 H CH 2 OH H- HO- OH H CH 2 OH L-Gulose Reaction of an aldose with NaBH4 produces a polyol (alditol). Because an alditol has the same functional group at both ends, two different aldoses can yield the same alditol. Here, L-gulose and D-glucose form the same alditol (rotate the Fischer projection of L-gulitol 180° to see the identity). Biomolecules: Carbohydrates 711 25.19 CHO C0 2 H OH H- HO- H H- OH H- OH -OH H- OH -H H- OH H- -OH H- OH H- -OH H- OH H- HO- HNO, dil. heat CH 2 OH C0 2 H D-Glucose Glucaric acid 25.20 dil. HNO; H- OH H- OH H- OH H- OH heat CH 2 OH C0 2 H D-Allose Allaric acid symmetry and no symmetry plane. Allaric acid has a plane of acid has C0 2 H CHO is an optically inactive meso compound. Glucaric D-Allose and D-galactose yield meso aldaric acids. All other D-hexoses produce optically active aldaric acids on oxidation because they lack a plane of symmetry. 25.21 The products of Kiliani-Fischer reaction of D-ribose have the same configuration at C3, C4 and C5 Use Figure 25.3 as D-ribose. as a reference. CHO CHO 1. HCN OH 2. H 2 Pd H- OH 3. H3 H- OH H- , catalyst + CHO H- OH HO- H- OH H- OH H- OH H- OH H- OH H- OH H CH 2 OH CH 2 OH D-Ribose D-Allose CH 2 OH D-Altrose 25.22 The aldopentose, L-xylose has the same configuration as the configuration C5 of L-idose and L-gulose. Use Figure 25.3 as a reference. CHO CHO H1 HOH- HO- . H OH H CH 2 OH L-Xylose CHO OH HO- H H HO- H HCN 2. H 2 Pd 3. H3 , + HOcatalyst H- HO- OH H CH 2 OH L-Idose H- HO- OH H CH 2 OH L-Gulose at C3, C4 and 712 Chapter 25 25.23 The aldopentoses have the CHO HO- H 1 H or OH H" C3 and C4 as D-threose. H 2 NOH . HOH- CH 2 OH H OH 3. CH 3 C0 2"Na+ + Na ~OCH 3 CH 2 OH HO- H OH H- CH 2OH D-Lyxose D-Xylose CHO (CH^CO)pO, 2. HO at CHO OH H- same configurations D-Threose 25.24 COc c=o I HoC — > Base COc " C=0 C0 2 c=o C=0 CH2 H- " C0 2 H .C "^ O: CH 2 H- -OH CH3CONH CH3CONH- H CH3CONH- -H HO HO- H HO- -H CH 2 OH A^-Acetylmannosamine H- OH H- -OH H- OH H- -OH CH 2 OH CH 2 OH N-Acetyl-D-neuraminic acid Hi onio ecu Its: I Carbohydrates 713 25.25 CH 2 OH CH 2 OH 2.H 2 OH CH 2 OH HO Cellobiose (b) CH3COCI CH 2OH CH 2 OH ^AjtOH O o H HoO* OH A cO (c) Ac0 pyridine OAc OAc OAc Ac = CH 3 C Visualizing Chemistry 25.26 Convert the model to a Fischer projection, remembering that the aldehyde group is on and that the groups bonded to the carbons below point out of the page. The model represents a D-aldose because the -OH group at the chiral carbon farthest from the aldehyde points to the right. (a) top, pointing into the page, CHO HO H HO^Jc^CHO^ A\ HO- A\ H- HO H H H H OH CH 2 OH D-Threose Break the hemiacetal bond and uncoil the aldohexose. Notice that all hydroxyl groups point to the right in the Fischer projection. The model represents the (3 anomer of D(b) allopyranose. CHO CH 2OH -OH -OH OH H -OH /3-D-Allopyranose -OH CH 2 OH 714 Chapter 25 25.27 The hints in the previous problem also apply ™ _ (a) u HOH ^ here. Molecular models are helpful, (b) CHO HO^ C HO- CHO i\ ohcVg H CH 2OH H H CHO HO H is X^oh ,'k OH OH ,'k HOH HH CH 2 OH L-Glyceraldehyde D-Erythrose structure represents an a anomer because the anomeric -OH group and the -CH2OH group are trans. The compound is a-L-mannopyranose because the -OH group at C2 is the only non-anomeric axial hydroxyl group. 25.28 The trans *OH OH a-L-Mannopyranose 25.29 (a) HOH HO H OHC, C ,'k H CHO CHO CHO Vf vr C .c x OH ,'k OH HO H H- -OH HO- H H- H- -OH HO- H HO- c" H H OH H HO- -H H- OH H- OH HO- -H H- OH H- OH CH 2 OH L-Mannose The model CH 2 OH CH 2 OH D-Mannose D-Glucose (enantiomer) (diastereomer) represents an L-aldohexose because the hydroxyl group on the chiral carbon from the aldehyde group points to the left. (c) This is tricky! The furanose ring of an aldohexose is formed by connecting the -OH group at C4 to the aldehyde carbon. The best way to draw the anomer is to lie L-mannose on its side and form the ring. All substituents point down in the furanose, and the anomeric -OH and the -CH(OH)CH2 OH group are cis. (b) farthest HOCH p-L-Mannofuranose Hi onio ecu Its: I Carbohydrates Additional Problems Carbohydrate Structure 25.30 (b) (a) (c) (pH 2 OH CHO CH 2 OH r° CH OH OH H- C=0 2 H— a ketotriose OH H- HO- -OH H- CH 2 OH HO- a ketopentose H OH H OH H" CH 2 OH an aldoheptose 25.31 (a) (b) CH 2 OH (c) CHoOH 2 (d) CHO CHO I C=0 C=0 -OH H— CH 2 OH HO— H- -OH -H H- H H- NH 2 H- OH H- OH H H- OH HO- CH 2 OH OH H- CH 2OH CH 2 OH a ketopentose a ketotetrose 25.32 D-Ribose and L-xylose a deoxyaldohexose a five-carbon amino sugar are diastereomers and differ in all physical properties (or if they have identical physical properties in any category, it is a coincidence). 25.33-25.34 Ascorbic acid has an L configuration because the hydroxyl group center points to the HO. HO C=0 H- HO- R left. o -H CH 2 OH L- Ascorbic acid OH ^ ^ H Mannose c \\~\ HO— C r ~^~H— OH HO— H^jw 25.64 O- C0 2 H H- HO- OH H H- OH H- OH HOCH 2 H OH H" HO- H O OH OH H- enohzation H- lactone CH 2 OH CH 2 OH formation D-Gluconic acid + H2 -H HO- -H H- -OH H- -OH enediol OH enohzation pyridine jj HOCH 2 OH O HO HO CH 2 OH H formation D-Mannonic acid double bond. v OH lactone Isomerization at pyridine \l tt HOCH 2 HO'^V^----' C0 2 H HO- O CH 2 OH + H2 C2 occurs because the enediol can be reprotonated on either side of the 728 Chapter 25 25.65 There are eight diastereomeric cyclitols. H HQ HO HO OH HO HO HO^\ ^- ^*\^OH OH , -i OH OH OH I OH HO QH OhJ OH HO HO y^^\^OH HO^^-*^\^ OH OHJ HO^^*^^X OH 1 OH OH ^4 OH OH 25.66 CHO C0 2 H OH H- dil. D-Ribose OH H- H- -OH H- -OH H- -OH HNO< heat OH Hh C0 2 H CH 2 OH A 1. HCN 2. H 2 Pd 3. -H H- "OH H- -OH -OH C0 2 H E , H3 CHO H H OH H- OH H OH H- OH H OH H- OH H OH HOdil. HN0 3 heat catalyst + CHO C0 2 H HO- B CH 2 OH C D-Altrose CH 2 OH D C0 2 H -OH dil. HNO< heat H- -OH H- -OH -OH C0 2 H F D-Allose Because A is oxidized to an optically inactive aldaric acid, the possible structures are Dribose and D-xylose. Chain extension of D-xylose, however, produces two hexoses that, when oxidized, yield optically active aldaric acids. Biomolecules: Carbohydrates 729 25.67 .0 (a) HN ^NNHPh C CH 2 OH C C=0 OH or H HOH- HO- HO- H C= NNHPh H 2 H 2 NNHPh OH H- OH OH H- OH H- OH -OH CH 2 OH CH 2OH D-Fructose D-Mannose CH 2 OH An osazone (c) ^N— NHPh H. . NHt- NHPh NH + I HO- -OH H- C H- -H HO- H- CH 2 OH H^ H OH D-Glucose (b) HOor OH C=0 o-^h H HO- H HO- H- OH H- OH H- OH H- OH (d) H ^NH PhNH — NH 2 0=0 HO- H I -H -OH H- CH 2OH enol CH 2 OH phenylhydrazone H 2 NPh -OH CH 2 OH keto imine H H I H ,q PhNHNmJ-c— NH, i PhNHN— C— NHo2 .. I PhNHN=C I ' C=0 H HO- - C= c=o H HO- + NHo -H HO- H- OH H- OH H- OH H- -OH H- OH H- OH H- OH H- -OH CH 2 OH CH 2 OH CH 2 OH V V PhNHN=C PhNHN=C CH 2 OH H H PhNHN=C PhNH— Kh^C=0: PhNHNH 2 -C-0:~ H HO- H HO- H- OH H- OH H- OH H- CH 2 OH I PhNHNH— C— OH HO + PhNHN=C —— | HoO PhNHN=C _^ HO- H -OH H- OH OH -OH H- OH CH 2 OH CH 2 OH H- CH 2 OH two nucleophilic addition reactions take place to yield imine products. The mechanism has been worked out in greater detail in Section 19.8, but the essential In these last steps, steps are additions of phenylhydrazine, first to the imine, then to the ketone. Proton transfers are followed by eliminations, first of ammonia, then of H2O. 730 Chapter 25 25.68 (a) more less stable stable p-D-Idopyranose OH OH more less stable stable a-D-Idopyranose (b) is more stable than (3-D-idopyranose because only one group is axial more stable chair conformation, whereas f3-D-idopyranose has two axial groups in more stable conformation. a-D-Idopyranose in its its (c) 1,6-Anhydro-D-idopyranose is formed from the p anomer because the axial hydroxyl groups on carbons 1 and 6 are close enough for the five-membered ring to form. (d) The hydroxyl groups stable conformation at carbons and 1 and 6 of D-glucopyranose are equatorial are too far apart for a ring to form. 25.69 HO D-Ribofuranose is OH the sugar present in acetyl CoA. in the most Hi onio ecu Its: I 25.70 Cleavage of fructose Carbohydrates 731 1,6-bisphosphate occurs by a retro aldol reaction. CH 2 OP0 3 2 CH 2 OP0 3 2- " CH 2OP03 2 C— 6:^ " HO— C— H LH "jjA J C=Q — C=0 11 Ho{-C— H + — C— O — H ,D H " I CH 2 OH Dihydroxyacetone 3-phosphate — C= O H I H— C— OH CH 2 OP0 3 2 Fructose 25.71 1 I H— C— OH " ^~ I CH 2 OP0 3 2 ,6-bisphosphate Glyceraldehyde 3-phosphate + Oxidation by NADP elimination, and conjugate reduction by NADPH give the + observed product. Notice that there is no net consumption of NADP The mechanism of + NADP oxidations and reductions has been shown many times in this book and also appears in part (c). (a) , . HOCH 2 OH HO HO<. NADP + ADPH/H O HO. + O OGDP CH 2 OH Q OGDP HO GDP-D-Mannose HoC NADP + OGDP NADPH/H" i CH 2 OH O HO O OGDP Chapter 25 (b) Two epimerizations, both a to the carbonyl group, cause a change in stereochemistry. HO (c) Reduction at C4 by NADPH forms GDP-L-fucose. GDP-L-Fucose Chapter 26 - Biomolecules: Amino Acids, Peptides and Proteins Chapter Outline I. Amino acids (Sections 26.1-26.3). A. Structure of amino acids (Section 1 . 26.1). Amino acids exist in solution as a. zwitterions. Zwitterions are internal salts and have many of the properties associated with salts. i. ii. hi. b . Zwitterions can act either as acids or as bases. i. The -CO2 group acts as a base. The ammonium group ii. 2 . 3 . 4 . 5 . They have large dipole moments. They are soluble in water. They are crystalline and high-melting. All but one (proline) of the 20 common amino acids are primary amines. All of the amino acids are represented by both a three-letter code and a one-letter code. See Table 26. 1 All a. b . amino acids except glycine are chiral. Only one enantiomer (L) of each pair is naturally-occurring. In Fischer projections, the carboxylic acid is at the top, and the amino group points to the c 6 . acts an acid. All natural amino acids are a-amino acids: the amino group and the carboxylic acid group are bonded to the same carbon. . left. a- Amino acids are referred to as L-amino acids. Side chains can be neutral, acidic, or basic. Fifteen of the 20 amino acids are neutral. b. Two (aspartic acid and glutamic acid) are acidic. i. At pH = 7.3, their side chains exist as carboxylate ions. a. c. Three At i. (lysine, arginine pH = and histidine) are basic. 7.3, the side chains of lysine and arginine exist as ammonium ions. not quite basic enough to be protonated at pH = 7.3. hi. The double-bonded nitrogen in the histidine ring is basic. d. Cysteine and tyrosine are weakly acidic but are classified as neutral. 7 Humans are able to synthesize only 1 1 of the 20 amino acids. a. These are nonessential amino acids. b The 9 essential amino acids must be supplied in the diet. B. The Henderson-Hasselbalch equation and isoelectric points (Section 26.2). Histidine ii. is . . 1 . The Henderson-Hasselbalch a. b . equation. and pKa we can calculate the percentages of protonated, neutral and deprotonated forms of an amino acid. If we do these calculations at several pH values, we can construct a titration If we know the values of pH , curve for each amino acid. 2 . The isoelectric point (pi) is the pH at which an amino acid exists as a neutral, dipolar zwitterion. a. pi i. ii. hi. is related to side chain structure. The 15 amino acids that are neutral have pi near neutrality. The two acidic amino acids have pi at a lower pH. The 3 basic amino acids have pi at a higher pH. 734 Chapter 26 For neutral amino acids, pi is the average of the two pKa values. For acidic amino acids, pi is the average of the two lowest pKa values. d For basic amino acids, pi is the average of the two highest pKa values. e. Proteins have an overall pi. 3 Electrophoresis allows the separation of amino acids by differences in their pi. a. A buffered solution of amino acids is placed on a paper or gel. b. Electrodes are connected to the solution, and current is applied. Negatively charged amino acids migrate to the positive electrode, and positively c. charged amino acids migrate to the negative electrode. d. Amino acids can be separated because the extent of migration depends on pi. C. Synthesis of a-amino acids (Section 26.3). 1 The Hell-Volhard-Zelinskii method and the phthalimide method. An a-bromo acid is produced from a carboxylic acid by a-bromination. a. b Displacement of -Br by ammonia gives the a-amino acid. 2. The amidomalonate synthesis. b. c. . . . . a. An alkyl halide reacts with the anion of diethyl amidomalonate. Hydrolysis of the adduct yields the a-amino acid. Reductive animation. a. Reductive animation of an a-keto carboxylic acid gives an a-amino acid. b This method is related to the biosynthetic pathway for synthesis of amino acids. 4 All of the methods listed above produce a racemic mixture of amino acids. D. Enantioselective synthesis of amino acids. Resolution of racemic mixtures: 1 a. The mixture can react with a chiral reagent, followed by separation of the diastereomers and reconversion to amino acids. b Enzymes selectively catalyze reactions that form one of the enantiomers, but not b 3 . . . . . . 2 . the other. Enantioselective synthesis. a. Enantioselective hydrogenation of Z-enamido acids produces chiral a-amino b The most effective acids. . catalysts are complexes of rhodium (I), cyclooctadiene and a chiral diphosphine. II. Peptides (Sections 26.4-26.8). A. Peptide structure (Section 26.4). 1 Peptide bonds. a. A peptide is an amino acid polymer in which the amine group of one amino acid forms an amide bond with the carboxylic acid group of a second amino acid. b The sequence of -N-CH-CO- is known as the backbone of the peptide or . . protein. Rotation about the amide bond is restricted. The N-terminal amino acid of the polypeptide is always drawn on the left. The C-terrninal amino acid of the polypeptide is always drawn on the right. Peptide structure is described by using three-letter codes, or one-letter codes, for the individual amino acids, starting with the N-terminal amino acid on the left. c. 2 . 3 . 4. 5 . Disulfide bonds. a. b . Two cysteines can form a disulfide bond Disulfide bonds can link polypeptide chain. ( -S-S-). two polypeptides or introduce a loop within a Biomolecules: B . Amino Acids, Peptides and Proteins 735 Structure determination of peptides (Sections 26.5-26.6). 1 Amino acid analysis (Section 26.5). . a. b. c. Amino acid analysis provides the identity and amount of each amino acid present in a protein or peptide. First, all disulfide bonds are reduced and all peptide bonds are hydrolyzed. The mixture is placed on a chromatography column, and the residues are eluted. i. As each amino acid elutes, it undergoes reaction with ninhydrin, which produces a purple color that is detected and measured spectrophotometrically d. 2 . Alternatively, the mixture can be analyzed Amino by HPLC. reproducible on properly maintained equipment; residues always elute at the same time, and only small sample sizes are needed. The Edman degradation (peptide sequencing) (Section 26.6). a. The Edman degradation removes one amino acid at a time from the -NH2 end of a peptide. i. The peptide is treated with phenylisothiocyanate (PITC), which reacts with the arnmo-terrninal residue. The PITC derivative is split from the peptide. ii. iii. The residue undergoes acid-catalyzed rearrangement to a PTH, which is identified chromatographically. iv. The shortened chain undergoes another round of Edman degradation. b Since the Edman degradation can only be used on peptides containing fewer than 50 amino acids, a protein must be cleaved into smaller fragments. Partial acid hydrolysis is unselective and therefore is of limited usefulness. i. The enzyme trypsin cleaves proteins at the carboxyl side of Arg and Lys ii. e. acid analysis is . residues. iii. c. The enzyme chymotrypsin cleaves proteins at the carboxyl side of Phe, Tyr and Trp residues. The complete amino acid sequence of a protein results from determining the individual sequences of peptides and overlapping them. C. Synthesis of peptides (Sections 26.7-26.8). Laboratory synthesis of peptides (Section 26.7). 1 a. Groups that are not involved in peptide bond formation are protected. Carboxyl groups are often protected as methyl or benzyl esters. i. ii. Amino groups are protected as Boc or Fmoc derivatives. b The peptide bond is formed by coupling with DCC (dicyclohexylcarbodiimide). c. The protecting groups are removed. i. Boc groups are removed by brief treatment with trifluoroacetic acid. Fmoc groups are removed by treatment with base. ii. iii. Esters are removed by mild hydrolysis or by hydrogenolysis (benzyl). 2 Automated peptide synthesis - Merrifield solid-phase method (Section 26.8). a. The carboxyl group of a Boc-protected amino acid is attached to a polystyrene resin by formation of an ester bond. b The resin is washed with trifluoroacetic acid, and the Boc group is removed. A second Boc-protected amino acid is coupled to the first, and the resin is c. . . . . washed. d. The cycle (deprotecting, coupling, washing) is repeated as many times as needed. e. Finally, treatment with anhydrous HF removes the final Boc group and frees the polypeptide. f . Robotic peptide synthesizers have improved yield and preparation time. Width: 612 Height: 792 736 Chapter 26 HI. Proteins (Section 26.9). A. Classification of proteins. Fibrous proteins consist of long, filamentous polypeptide chains. 1 Globular proteins are compact and roughly spherical. 2 Protein structure. 1 Levels of protein structure. a. Primary structure refers to the amino acid sequence of a protein. b Secondary structure refers to the organization of segments of the peptide backbone into a regular pattern, such as a helix or sheet. c. Tertiary structure describes the overall three-dimensional shape of a protein. d Quaternary structure describes how protein subunits aggregate into a larger . . B . . . . structure. 2. Examples of structural a. features. a-Helix. i. An a-helix is a right-handed coil; each turn of the coil contains 3.6 amino acids. ii. b. structure is stabilized and C=0 groups four residues by hydrogen bonds between amide N-H groups away. /3-Pleated sheet. i. ii. c. The In a /^-pleated sheet, hydrogen bonds occur between residues in adjacent chains. In a /3-pleated sheet, the peptide chain is extended, rather than coiled. Tertiary structure. i. The nonpolar amino acid side chains congregate in the center of a protein to avoid water. ii. hi. The polar side chain residues are on the surface, where they can take part hydrogen bonding and salt bridge formation. Other important features of tertiary structure are disulfide bridges and hydrogen bonds between amino acid side chains. in Denaturation of proteins. a. Modest changes in temperature and pH can disrupt a protein's tertiary structure. i. This process is known as denaturation. Denaturation doesn't affect protein primary structure. ii. b. Denaturation affects both physical and catalytic properties of proteins. c. Occasionally, spontaneous renaturation can occur. C. Enzymes (Sections 26.10-26.11). 1 Description of enzymes and cofactors (Section 26. 10). a. An enzyme is a substance (usually protein) that catalyzes a biochemical reaction. b. An enzyme is specific and usually catalyzes the reaction of only one substrate, i. Some enzymes, such as papain, can operate on a range of substrates. 3 . . c. How enzymes function. i. Enzymes form an enzyme-substrate complex, within which the conversion to product takes place. ii. Enzymes accelerate the rate of reaction by lowering the energy of the transition state. The rate constant for the conversion of E S to E + P is the turnover number. Enzymes are grouped into 6 classes according to the reactions they catalyze. hi. d. i. ii. iii. iv. Oxidoreductases catalyze oxidations and reductions. Transferases catalyze the transfer of a group from one substrate to another. Hydrolases catalyze hydrolysis reactions. Lyases catalyze the addition or loss of a small molecule to or from a substrate. v. Isomerases catalyze isomerizations. Amino Biomolecules: Acids, Peptides and Proteins 737 Ligases catalyze bond formation between two molecules, often coupled with hydrolysis of ATP The name of an enzyme has two parts, ending with -ase. vi. e. The first part identifies the substrate. The second part identifies the enzyme's class. Most enzymes are globular proteins, and many consist of a protein portion i. ii. f . (apoenzyme) and a cofactor. Cofactors may be small organic molecules (coenzymes) or inorganic ii. Many coenzymes are derived from vitamins. i. 2 . How enzymes work - citrate a. . c. d e. 1). Citrate synthase catalyzes the aldol-like addition of acetyl produce b synthase (Section 26. 1 Co A to oxaloacetate to citrate. Functional groups in a cleft of the enzyme bind oxaloacetate. Functional groups in a second cleft bind acetyl CoA. The two reactants are now in close proximity. i. Two enzyme amino acid residues generate the enol of acetyl CoA. The enol undergoes nucleophilic addition to the ketone carbonyl group of oxaloacetate. f Two enzyme amino . g . acid residues deprotonate the enol and protonate the carbonyl oxygen. Water hydrolyzes the thiol ester, releasing citrate and CoA. Solutions to Problems 26.1 Amino Acids with aromatic rings: Phe, Tyr, Trp, His. Amino acids containing sulfur: Cys, Met. Amino acids that are alcohols: Ser, Thr (Tyr is a phenol.) Amino acids having hydrocarbon side chains: Ala, He, Leu, Val, Phe. 26.2 C0 2 H H2N H H H0 2 C NH 2 R R A Fischer projection of the a-carbon of an L-amino acid is pictured above. For most L-amino acids: Group ions. Priority -NH 2 -C02 H For cysteine: Group Priority 2 -NH 2 -CH 2 SH -R 3 -CO?H 3 -H 4 -H 4 1 1 2 4 R 2 738 Chapter 26 26.3 — —c zo 2 ( c — R NH — R OH s •1 J 2 R s ) i CH 3 Diastereomers of Threonine L-Threonine 26.4 CH 3 3H 3 ( On the low pH net positive charge at 26.5 and on the high pH hemoglobin (pi = 6.8) has a (acidic) side of pi, a protein has a net positive charge, (basic) side of pi, a protein has a net negative charge. Thus, pH = 5.3 and a net negative charge at pH = 7.3. This method of amino acid synthesis is simple and uses methods we have already studied. The phthalimide synthesis can also be used to introduce the amino group. Remember that only racemic amino acids are produced by this method. (a) NH 2 Br Br2 PBr3 1. C 6 H 5 CH 2 CH 2 C02H , CgHgCH 2 CHC0 2 H H Q ,,. NH 3 I I C 6 H 5 CH 2 CHC0 2 •xcess* 3-Phenylpropanoic acid Phenylalanine (b) NH 2 Br 1 (CH 3 2 CHCH 2 C0 2 H ) PBro J Br?, . (CH 3 ) 2 CHCHC0 2 H H q 2 Nhk I ' I (CH 3 excess 3-Methylbutanoic acid ) 2 CHCHC02 ~ Valine 26.6 O^O-B C02 Et H + 1 — C — C02 Et ^N^ C ~OEt RX ^CH 3 | N heat >V CH3 * II + H qO — C-r COoEt H- o + 2 EtOH I R 2. H Na OH I + C0 2 + CH 3 C0 2 H H II O In the amidomalonate synthesis, RCH(NH3 +)C02H. Choose an shown above, an alkyl halide Amino Acid Halide NH 3 + I " (CH 3 2 CHCH 2 CHC0 2 ) Leucine RX is converted to alkyl halide that completes the structure of the target acid. (a) I R— C— u H— N— (CH 3 2 CHCH 2 Br ) amino Amino Biomolecules: Amino Acid Acids, Peptides and Proteins 739 Halide (b) N N " CH 2 CHC0 2 CH 2 Br H H Histidine (C) NHo3 + I ~ CH 2 Br CH 2 CHC0 2 Tryptophan (d) NHo + 1 CH3SCH 2 CH 2 CHC0 2 CH3SCH 2 CH 2 Br Methionine 26.7 The precursor to an amino acid prepared by enantioselective hydrogenation has bond conjugated with a carboxylic acid carbonyl group. O HoC OH ^ NHCOCH3 (3 H3C H [Rh(DiPAMP)(COD)] 1. hfe, 2 NaOH, H 2 . " BF4 \F II HoC, C C H NH 2 H 3C H Leucine 26.8 Val-Tyr-Gly (VYG) Val-Gly-Tyr (VGY) 26.9 The N-terminal group Tyr-Gly-Val Tyr-Val-Gly is on the H C ,2 ~CH 2 u r 2 right, c %.^iJk^^ 9 \ 9 N-terminal $, "1 + II O ^C*"*Q " HoN. 3 ' M \ H;. C0 2 V M C-terminal H "CH(CH3) 2 CH3SCH 2 CH 2 H Met amide bonds — ProP is HH w II I Gly-Val-Tyr (GVY) Gly-Tyr-Val (GYV) and the C-terminal group O H V (YGV) (YVG) Val Gly v— G Z double O H H + a on the left. O 740 Chapter 26 26.10 C0 2H H 3 NCHCH 2 SH The cysteine H 3 NCHCH 2 S— CH 2 C0 2 H ICH 2 C02 H sulfur is a good nucleophile, and iodide is + T~ a good leaving group. 26.11 One product of the reaction of an amino acid with ninhydrin is the extensively conjugated purple ninhydrin product. The other major product is the aldehyde derived from the side chain of the amino acid. When valine reacts, the resulting aldehyde is 2-methylpropanal. The other products are carbon dioxide and water. The identity of the aldehyde is determined by the amino acid side chain. T3 OH + (CH 3 2 CHCHC0 2 ) OH O O O II (CH 3 2 CHCH + ) 26.12 C0 2 + 3 H2 Trypsin cleaves peptide bonds at the carboxyl (right) side of lysine and arginine. Chymotrypsin cleaves peptide bonds at the carboxyl side of phenylalanine, tyrosine and tryptophan. — Trypsin Asp-Arg *- + Val-Tyr-Ile-His-Pro-Phe Asp-Arg-Val-Tyr-Ile-His-Pro-Phe Chymotrypsin Asp-Arg-Val-Tyr + 26.13 The part of the PTH Ile-His-Pro-Phe derivative that lies to the right of the indicated dotted lines comes from Complete the structure to identify the amino acid, which in this the N-terminal residue. problem is methionine. O C 6 H 5, (I / "0 CH0CH9SCH3 t N 2 ^H CCH— CH 2CH 2SCH 3 + H + C 6 H 5 N=C=S NH 3 Methionine 26.14 The N-terminal residue of angiotensin II is C«H 6 n 5. aspartic acid. -CH2CO2H to arrive O derivative in Figure 26.4 with N— X / c ,.CH 2 C0 2 H H H Replace the -R group of the at the correct structure. PTH Amino Biomolecules: 26.15 Line up the (a) Acids, Peptides and Proteins 741 fragments so that the amino acids overlap. Arg-Pro Pro-Leu-Gly (b) V-M-W W-N-V V-L Gly-Ile-Val The complete sequence: The complete sequence: V-M-W-N-V-L Arg-Pro-Leu-Gly-Ile-Val 26.16 :<£) \\y (CH 3 ) 3 :o: :o: ii CO— C— O— C— OC(CH3 3 5=* ) addition of H 2 NCHRC0 2 H (CH 3 ) 3 CO— C— C02 amino acid nitrogen NHCHRC0 2 ~ . Et 3 N:v^ ) loss of x ) deprotonation H :o: r :6: (CH 3 3 CO- C- O 3- C-r OC(CH 3 3 -* HOC(CH 3) 3 . + .. II C- O- C- OC(CH 3 3 HI^HRC0 2 H nucleophilic ( + (CH 3 3 CO- ) :o: .. I ) y HNCHRC0 H i + 2 C02and Et 3 fer?-butoxide 26.17 CH 2 CH(CH 3 + . Protect the R = CH 2 CH(CH 3) 2 R 2 +1 Leu = H 3 NCHC0 2~ 1 ) 1 f " H 3 NCHC0 2 = amino group of leucine. o o O (CH 3 ) 3 COCOCOC(CH 3 3 + ) H 2 NCHC0 2 EtoN " II ~ (CH 3 3 COCNHCHC0 2 + 2. Protect the carboxylic acid group of alanine. + Ala + CH 3 OH H CaaySs * I ) Leu H 2 NCHC0 2 H R H 2 NCHC0 2 CH 3 C0 2 + HOC(CH 3 ) 3 NH +- 742 Chapter 26 3 . Couple the protected amino acids with DCC. 9 O II II (CH 3 3 COCNHCHC0 2 ~ + N=C=N H 2 NCHCOCH 3 ) R CHo O I II O (CH 3 ) 3 ff f COCNHCHC— NHCHCOCH3 CH 3 R 4. Remove the leucine protecting group. {? (CH 3 ) 3 NH-C-NH ff CF 3 CQ2 H f? COCNHCHC— NHCHCOCH3 ff CH 3 R R (CH 3 )2C 5 . CH< — CH2 ff f? R 1 NaOH, H 2 (3 2. H3 + H 3 NCHC — NHCHCOH (b) (c) + CH 3 OH CH 3 (CH 3 )2CHCH2 CH< Leu(a) COc + Remove the alanine protecting group. H 3 NCHC— NHCHCOCH 3 26.18 ff H 3 NCHC— NHCHCOCH 3 -Ala Pyruvate decarboxylase is a lyase. Chymotrypsin is a hydrolase. Alcohol dehydrogenase is an oxidoreductase. Visualizing Chemistry 26.19 (a) (b) - H H (c) ^C02 H 3 N+ H 3 N+ H A "OoC H3N CO. j H N Isoleucine 26.20 c / II N-terminal H 3 N. N • H HCHoSH Cys- c— Lys- -K — C a\ || n u CH 2 C0 2 II I C ~ O H H I N \ H CHo3 H Ala - A- Asp -D CO. C-terminal H C \ II H Glutamine Histidine H O H O Biomolecules: Amino Acids, Peptides and Proteins 743 26.21 H H w .a H 3 C^ s C cho H H3 N s + 26.22 It's C0 2 possible to identify this representation of valine as the configuration at the chirality center. The configuration is D enantiomer by noting the R, and thus the structure is D- valine. ~0 2 C^R^ CH(CH 3 2 ) D-Valine (/?)-Valine H3N H 26.23 After identifying the amino acid residues, notice that the tetrapeptide has been drawn with the amino terminal residue on the right. To name the sequence correctly, the amino terminal residue must be cited first. Thus, the tetrapeptide should be named Ser-Leu-Phe-Ala. Additional Problems Amino Acid 26.24 Both Structures and Chirality (/?)-serine on the and (/?)-alanine are D-amino acids. In a right. COc H- -NH3+ H- -NH 3+ CH 2 OH (tf)-Serine (R)- Alanine D-amino acid, the -NH2 group is 744 Chapter 26 26.25 + H HoN- NHo "OoC + R CH 2 Br L-Bromoalanine (/?)-Bromoalanine This L "amino acid" also has an R configuration because the in priority than the -CO2H group. -CH2Br "side chain" 26.26 + H H 2 N- (5)-Proline 7 H 2C CH 2 26.27 (a) CH 2 CHC0 2 - (b) (c) CHo3 I " CH 3 CH 2 CHCHC0 2 + Tryptophan (Trp) Histidine (His) NHo Isoleucine (lie) HSCH0CHCO0" 2 2, NHo Cysteine (Cys) is higher Amino Biomolecules: Acids, Peptides and Proteins 745 26.28 C— O" C— O" -H+ K,a2 H 2 A+ deprotonated protonated At pH = 2.50: log [h 2 a +] = p H - pKa] = 2.50 - 1.99 At pH = 2.50, approximately three times form as exist in the protonated form. = = Q.51;J^h + H 2A as many 3. 24 ] proline molecules exist in the neutral At pH = 9.70: [A"] log [HA] At 26 29 . (a) (b) = pH - pH = 9.70, p/ra 2 = 9.70 - 10.60 = -0.90;J^r = 0.126 [HA] the ratio of deprotonated proline to neutral proline Val-Leu-Ser V-L-S Ser-Val-Leu S-V-L Val-Ser-Leu V-S-L Leu-Val-Ser L-V-S Ser-Leu-Val S-L-V Leu-Ser-Val L-S-V is approximately Ser-Leu-Leu-Pro S-L-L-P Leu-Leu-Ser-Pro L-L-S-P Ser-Leu-Pro-Leu S-L-P-L Leu-Leu-Pro-Ser L-L-P-S Ser-Pro-Leu-Leu S-P-L-L Leu-Ser-Leu-Pro L-S-L-P Pro-Leu-Leu-Ser P-L-L-S Leu-Ser-Pro-Leu L-S-P-L Pro-Leu-Ser-Leu P-L-S-L Leu-Pro-Leu-Ser L-P-L-S Pro-Ser-Leu-Leu P-S-L-L Leu-Pro-Ser-Leu L-P-S-L 26.30 Aldehydes and ketones can undergo 1:8. nucleophilic addition reactions. In particular, aldehydes and ketones can react with amines to form imines and enamines, reactions that might compete with formation of amide bonds between amino acids. Because of this reactivity, aldehydes and ketones are unlikely to be found in amino acid side chains. Width: 612 Height: 792 746 Chapter 26 Amino Acid Synthesis and Reactions 26.31 The diethylamidomalonate ethoxide. anion is formed by treating diethylamidomalonate with sodium Choose the appropriate halide based on the amino acid side chain. C0 2 Et (a) C0 2 Et — (CHo)oCHCHoBr ":C— C02 Et .N I (CH 3 2 CHCH 2 ) C— C02 Et CH3 H II O o HoO + CH 3 C0 2 H + C0 2 + EtOH 2 (CH 3 2 CHCH 2 CHC0 2 H + + (b) Leucine ) NHo C0 2 Et COoEt 2 I CH 2 -C— C0 2 Et CH 2 Br ~:C— C02 Et ,CH3 II O CH 2 CHC0 2 H CH 3 C0 2 H + C0 2 + 2 EtOH + nrv N 26.32 (a) I? CH 3 SCH 2 CH 2 CC0 2 H NH 2 NH- NaBH, CH 3 SCH 2 CH 2 CHC0 2 H Methionine W HoC NH 2 3 HoC O 3 | II CH 3 CH 2 CHCC0 2 H NH. NaBH, | I CH 3 CH 2 CHCHC0 2 H Isoleucine NH 3 Tryptophan u H Amino Biomolecules: Acids, Peptides and Proteins 747 26.33 (a) O H C— OH n \. 1 NCOCH3 (b) y 2 . H2, [Rh(DiPAMP)(COD)] a ~ O" BF4 NH NaOH, H 2 Proline O CHo HoC 3 . + II C y C\ CH 3 1 OH . H2, [Rh(DiPAMP)(COD)] 2. I + V/ ~ BF4 H3C NaOH, H 2 fj> y ^\ y O C NHCOCH3 _ Valine H NH 2 26.34 9 . H Cv ( H C02 Et C0 2 Et I :c— C02 Et "OCH 2 — C— C0 2 CH 3 N H 3 0! Et HOCH 2 CHC0 2 H + NH 3 Serine x CHd y ff I? O O + + CH 3 C0 2 H C0 2 + 2 EtOH 26.35 (a) ~ (CH 3 2 CHCHC0 2 ) + CH0CH0OH H + ** ( CH 3)2 CH HC02 CH 2 CH 3 p catalyst NH 3 NH< Valine (b) O O II (CH 3 2 CHCHC0 2 ) + NH II (CH 3 3 COCOCOC(CH 3 ) Et3 N ) 3 (CH 3 2 CHCHC0 2~ ) (CH 3 2 CHCHC0 2 ~ ) NH, HOC(CH 3 NHCOC(CH 3 3 ) O (c) + KOH, HoO ^— (CH3 ) 2 CHCHC02"K NH' + + H2 ) + C0 2 3 748 Chapter 26 ( d) (CH 3 2 CHCHC0 2" 1 - . CHoCOCI, - ) + NHo3 pyridine ** ~ (CH 3 2 CHCHC0 2 ) g " NHCCHo3 „ O 26.36 O + H2 Step 1: Dehydration. Step 2: Nucleophilic addition of the amino group of the amino Step 3: Proton transfer. Step 4: Loss of water. (b) Decarboxylation produces a different imine. o acid. R Biomolecules: Amino Acids, Peptides and Proteins Step 1: Addition of water. Step 2: Proton transfer. Step 3: Bond cleavage to yield an aldehyde and an amine, (d) Step 1: Addition of the amine to a carbonyl carbon of a second ninny drin molecule. Step 2: Proton shift. Step 3: Loss of water to form the purple anion. Notice that the amino nitrogen is all that remains of the original amino acid. 749 750 Chapter 26 It is also possible to draw many other resonance forms aromatic 6-membered rings. that involve the n electrons of the Biomolecules: Peptides and Amino Acids, Peptides and Proteins 751 Enzymes 26.38 (a) N— CH 2 CH 2 SCHg HsN C >- °V - HSCH 2 H N °f ^ f co f OH - CHgCH^COg" Cys- Met Glu- His- C— H— •E — M (b) H H 1 y P V O . ft ^ H 2CH 2 C02 j» " / H ~ CH 2 C0 2 H 3 C OH H CH 2 CH 2 C0 2 6 E Pro- - H N + N /\ H H 3^ / CH 2 CH3 N\ /C^ X CV /N^ /C^ H H " 2 Glu — E Pro- Thr Asp- lie 26.39 The tripeptide Leu is cyclic. Leu / \ Ala Phe —\ / Phe Ala Glu 752 Chapter 26 26.40 Step 1: Valine II is protected as its Boc + O —^ Et 3 N II (CH3) 3 COCOCOC(CH 3 )3 derivative. Val II (CH 3 3 COC— Val— ) OH (Boc— Val-OH) Step 2: Boc-Val bonds to the polymer in an S N2 reaction. Boc — Val— OH Step + CICH 2 The polymer 3: Base ^Polymer) is first washed, then is » Boc , treated with — Val— OCH 2 —( Polymer CF3C02H to cleave the Boc group. — Val— OCH2 —/Polymer) ^/ ""Sl Boc 1 . f Step 4: 2. S* wash "X, Val— OCH 2 —f Polymer! CFgC0 2 H A Boc-protected Ala is coupled to the polymer-bound valine by reaction with DCC. The polymer is washed. Boc— Ala + /* Val— OCH 2 —(Polymer) Step 5: The polymer is treated with S 6: . DCC Boc- Ala— Val— OCH 2 — (^Polymer CF3 C02H to remove Boc. —^—^ \ CFoCOoH * Boc-Ala— Val— OCH 2 — (Polymer) Step 1 - Ala— Val— OCH 2 ~( Polymer A Boc-protected Phe is coupled to the polymer by reaction with DCC. The polymer is Boc-Phe washed. + X Ala— Val— OCH 2 —f Polymer 1 . 2. _ DCC wash Boc— Phe -Ala— Val— OCH 2— (Polymer Step 7: Treatment with anhydrous HF removes the Boc group and cleaves the ester the peptide and the polymer. bond between Boc— Phe—Ala— Val— OCH 2— Phe-Ala-Val + f (CH 3 2 C=CH 2 ) "\ HF (Polymer) + C0 2 + HOCH 2 — (Polymer Amino Biomolecules: Acids, Peptides and Proteins 753 26.41 PITC Peptide (a) Phenylthiohydantoin C6H5 S Ile-Leu-Pro-Phe — P— I— L Shortened Peptide /0 H N-C 9 3 \ ^--CHCHoCHo / n S * "> N Leu-Pro-Phe L — P— H I H (b) O H 5, C6 n fi Asp-Thr-Ser-Gly-Ala — T— S— G— D N— Thr-Ser-Gly-Ala V / ,-CH 2 C0 2 H c T— S— G— H H 26.42 Phe|-Leu— Met— Lys-j-Tyr|-Asp— Gly— Gly— Arg-j-Val— lie— Pro— Tyr Cleaved by trypsin = Cleaved by chymotrypsin = ++++ 26.43 (a) (b) (c) Hydrolases catalyze the cleavage of bonds by addition of water (hydrolysis). Lyases catalyze the elimination of a small molecule (H2O, CO2) from a molecule. Transferases catalyze the transfer of a functional group between substrates. 26.44 Amino acids with polar side chains are likely to be found on the outside of a globular where they can form hydrogen bonds with water and with each other. Amino acids with nonpolar side chains are found on the inside of a globular protein, where they can avoid water. Thus, aspartic acid (b) and lysine (d) are found on the outside of a protein, globular protein, and valine (a) and phenylalanine (c) are likely to be found on the inside. Refer to Table 26.1. 754 Chapter 26 26.45 Leuprolide Glu E His- — H — Trp- Ser Tyr Leu- w- S Y— L The N-terminal glutamic (a) — acid is a cyclic lactam. Leu-L The C-terminal Pro-NHEt P NHEt Arg- R — proline is an Af-ethyl amide. (b) One of the (c) See above. leucines (indicated above) has (d) The charge on a peptide side chain that is neutral charged is due D stereochemistry. According to Table 26. 1 the only Thus, leuprolide has a charge of +1 at to the side chains. at neutral pH is arginine. , pH. General Problems 26.46 A proline residue in a polypeptide chain interrupts a-helix formation because the amide nitrogen of proline has no hydrogen that can contribute to the hydrogen-bonded structure of an a-helix. 26.47 Formation of cation: H 3 C— O-1 CH 2 — SnCI 4 CI Electrophilic aromatic substitution: rv V H 2 C=OCH 3 - CH 2 OCH 3 " SnCI 5 Protonation of the ether oxygen, followed by displacement of methanol by Cl~. h— a 9 \ CH20CH2 V • // \ CH 2 CI CHpT.OCHo H :ci: + HOCH3 Biomolecules: Amino Acids, Peptides and Proteins 755 Step 1: NaOH brings about ehmination of the carboxylated peptide. Step 2: Loss of C0 2 . The Fmoc group which The is first is acidic because the resonance-stabilized and step is is Fmoc anion is similar to the cyclopentadienyl anion, aromatic. a substitution similar to the nucleophilic acyl substitution reactions that studied in Chapter 21 we Width: 612 Height: 792 756 Chapter 26 O (b) f NHCHCH — C-^NHCHC Ik I -|-NHCH— C=NHCHC-|- -|- I p H2 C R' 2 X \0\J H2C H 2 C^.. ^S-C=N: + R' H 3 C— S— C=N: CHo Sn2 displacement of sulfide results in formation of a 5-membered ring containing an iminium group. Internal (c) H 2or> -4- o ; 4 NHCH — C— NHCHC NHCH— C= NHCHC-f- I H 2 C^ of water H2C NHCH— C H2C R R' proton transfer O r :OH ^1 * lis -4- NHCH— C— NHpCHC-4 s_ ., H 2 NCHC- O N I O H2 C O .. I H 2 C^ addition R' ,>+ O OH; H 2 Cv 1 breaking of peptide bond H 2C O R' H2 C lactone bond In this sequence of steps, water adds to the imine double bond, and the peptide is cleaved. (d) H2 I b:^h on^OH r£OH —5* NHCH HoO + .. —C \k > —J- \ „UH ^OH NHCH— C ^ -4- \>^OH _t -S-NHCH— C NHCH— C * H 2C \ H2C / H2 C O addition of water \ H 2C / + HO:-) O proton transfer \ OH \ rin H 2C \ I HoC *OH H2 C opemng tt O + 'I CH 2 OH > H 3 NCHC-^- + O -|-NHCH — C^ I H2 C R from Water opens the lactone ring to give the product shown. \ (c) OH CH 2 OH Amino Biomolecules: Acids, Peptides and Proteins 757 26.50 Qo: :0: C x o: :0: ii ,-C^/R R C0 2 4 H « N R N R' OH 1 N R + / \ l H(OH + I H H C0 2 + "OH R' 26.51 ^ II NHR H2N + H+ NH NH, NH, NHR H2 N The protonated guanidino group can be 26.52 NHR H2N stabilized 100 g of cytochrome c contains 0.43 g ™Ql ^ 0.43 g Fe x i 55.8 g Fe Assuming that NH, NH, H2 N NHR H2 N + NHR by resonance. iron, or 0.0077 mol Fe: = 0.0077 mol Fe each mole of protein contains 1 mol Fe, then mol Fe = mol protein. 100 g Cytochrome c _ 13,000 g Cytochrome c " 0.0077 mol Fe 1 mol Fe Cytochrome c has a minimum molecular weight of 13,000 26.53 ! g/ mol. H NMR shows that the two methyl groups of Af,Af-dimethylformamide are nonequivalent room temperature. If rotation around the CO-N bond were unrestricted, the methyl ! absorptions would coalesce into a groups would be interconvertible, and their H at NMR single signal. (a) O H3C c (b) H d3 C (b) rotate H 3C H \ » ,0 // N— C H3C (a) H The presence of two methyl absorptions shows that there is a barrier to rotation around the CO-N bond. This barrier is due to the partial double-bond character of the CO-N bond, indicated by the two resonance forms below. Rotation to interconvert the two methyl slow at room temperature, but heating to 180° supplies enough energy rapid rotation and to cause the two absorptions to merge. groups is NMR HcC qfy HoC / / \ N=C N-*-C H3 C H :o: \+ H3 C H to allow as 758 Chapter 26 26.54 Gly Gly-Asp-Phe-Pro Phe-Pro-Val Val-Pro-Leu The complete sequence: Gly-Gly-Asp-Phe-Pro-Val-Pro-Leu 26.55 Cjly He yai ? lu f I i I I Gm^ys-Cys-Tbj-Ser-ne-Cys-Ser-Leu-Tyr|Gln-Leu-Glu-Asn-T S > — rps-Leu^ys^ly-Ser-ms-Leu-Val^lu-Ala-Ixu-Tyr|Leu-Val-^ys ' Glu Gly r i ^ sn Glu w phe Jal Arg s i s s Tm*]-Lys-Pro-TliiiTyiiphetphe-Gly Cleaved by Trypsin = Cleaved by Chymotrypsin = 26.56 Ser-Ile-Arg-Val-Val-Pro-Tyr-Leu-Arg 26.57 Reduced oxytocin: Cys-Tyr-Ile-Gln-Asn-Cys-Pro-Leu-Gly-NH 2 Oxidized oxytocin: Cys-Tyr-Ile-Gln-Asn-Cys-Pro-Leu-Gly-NH 2 S S The C-terminal end of oxytocin information given. is actually an amide, but this can't be determined from the Amino Biomolecules: Acids, Peptides and Proteins 26.58 (b) (a) O O .C^ H 2 N^ j N C CH 2 ^ o C .OCHo3 HofM O M. > \ H H \ ^ OCHg I CH 2H o co 2 Aspartame (nonzwitterionic form) 7.3, ^ ^ ~ C0 2 H At pH = C Aspartame at aspartame exists in the zwitterionic form, as pH = does it 5.9, 7.3 at pH = 5.9. 26.59 ^ S-C* H 2 0:-n / VV c r H r C6 H 5 H C 6 H5 H I OH /^OH H 3 : H C H A c-. :s H r H R 4. C6 H5 2 H I H 2b:^ H \:N-CrOH QPH ?5o„ I C6 H5 H H 4 . ^ fi \ C6 \ 2. C6 H 5 H C 6 H5 >>+ V,oh' :S— 1. ? H2 . + //+ HoO + H 5. C6 n .0 ft :N— \ / 7. H 1 H H I H Step 1: Protonation Step 3: Proton transfer. Step 5: Bond rotation, addition of amine. Step 7: Loss of water ^ H H Step 2: Addition of water Step 4: Ring opening. Step 6: Proton transfer. Step 8: Deprotonation. 759 760 Chapter 26 26.60 26.61 4-Methylideneimidazol-5-one (MIO) Step 1: Nucleophilic addition of the amino group of the amino Step 2: Proton transfer. Step 3: Elimination. Step 4: Elimination. acid. Biomolecules: Step 1: Step 2: Amino Acids, Peptides and Proteins Nucleophilic addition of the amine to a-ketoglutarate. Loss of water. Review Unit Carbohydrates, 10: Bio molecules I Amino - Acids, Peptides Major Topics Covered (with vocabulary): Monosaccharides: carbohydrate monosaccharide furanose aldose ketose anomer anomeric center a anomer Fischer projection D,L sugars pyranose p anomer mutarotation glycoside Koenigs- Knorr reaction aldonic acid alditol reducing sugar aldaric acid Kiliani-Fischer synthesis degradation rucose glucosamine galactosamine neuraminic acid Wohl Other sugars: disaccharide 1,4' cellobiose link maltose lactose sucrose polysaccharide cellulose amylose amylopectin glycogen glycal assembly method deoxy sugar amino sugar cell-surface carbohydrate hemagglutinin neuraminidase Amino acids: amino acid zwitterion amphoteric electrophoresis resolution a-amino acid side chain isoelectric point (pi) amidomalonate synthesis reductive amination Henderson-Hasselbalch equation enantioselective synthesis Peptides: residue backbone Af-terminal amino acid C-terminal amino acid disulfide link amino acid analysis Edman degradation phenylthiohydantoin trypsin chymotrypsin peptide synthesis protection Boc derivative Fmoc derivative Merrifield solid-phase technique DCC Proteins: simple protein conjugated protein quaternary structure a-helix primary structure p-pleated sheet apoenzyme holoenzyme coenzyme vitamin oxidoreductase transferase secondary structure tertiary structure bridge prosthetic group isomerase hydrolase ligase salt enzyme cofactor lyase denaturation Types of Problems: After studying these chapters, you should be able to: - Classify carbohydrates as aldoses, ketoses, D or L sugars, monosaccharides, or polysaccharides. - Draw monosaccharides - Identify the - as Fischer projections or chair conformations. Predict the products of reactions of monosaccharides and disaccharides. Deduce the structures of monosaccharides and disaccharides. Formulate mechanisms for reactions involving carbohydrates. common amino acids and draw them with correct stereochemistry in dipolar form. Explain the acid-base behavior of amino acids. Synthesize amino acids. Draw the structure of simple peptides. Deduce the structure of peptides and proteins. Outline the synthesis of peptides. Explain the classification of proteins and the levels of structure of proteins. Draw structures of reaction products of amino acids and peptides. Review Unit 10 Remember: Points to Aldohexoses, ketohexoses and aldopentoses can forms. * 763 all exist in both pyranose forms and furanose A reaction that produces the same functional group at both ends of a monosaccharide halves the number of possible stereoisomers of the monosaccharide. * The * At physiological pH, * reaction conditions that form a glycoside are different from those even though both reactions, technically, form -OR bonds. that form a polyether, the side chains of the amino acids aspartic acid and glutamic acid exist as anions, and the side chains of the amino acids lysine and arginine exist as cations. The imidazole ring of histidine exists as a mixture of protonated and neutral forms. Since the amide backbone of a protein is neutral and uncharged, the isoelectric point of a is determined by the relative numbers of acidic and basic amino acid residues present in the peptide. protein or peptide * In the Merrifield technique of protein synthesis, a protecting group carboxyl group because it is isn't needed for the attached to the polymer support. Self-Test: OH CH O NHo3 + I OH CH 3 H3NCH2CH2CH2CHCO2" OH A C Digitalose Ornithine (hydrolysis product of digitoxigenin, a heart medication) Digitalose (A) is related to which D-aldohexose? Provide a name for A, including the configuration at the anomeric carbon. Predict the products of the reaction of + catalyst; (b) I, Ag 0. H CH 3 Vicianose (B) of B with CH3I and is (a) CH3OH, 2 a disaccharide associated with a natural product found in seeds. Treatment followed by hydrolysis, gives 2,3,4-tri-O-methyl-D-glucose and Ag 2 0, 2,3,4-tri-O-methyl-L-arabinose. Ornithine (C) A with: is What is the structure of B? Is B a reducing sugar? a nonstandard amino acid that occurs in metabolic processes. Which amino acid does it most closely resemble? Estimate pKa values and pi for ornithine, and draw the major form present at pH = 2, pH = 6, and pH = 1 1. If ornithine were a component of proteins, how would it affect the tertiary structure of a protein? Review Unit 10 764 Tyr-Gly-Gly-Phe-Leu-Arg-Arg-Ile-Arg-Pro-Lys-Leu-Lys-Trp-Asp-Asn-Gln Porcine Dynorphin (D) Dynorphin (D)is a neuropeptide. Indicate the /^-terminal end and the C-terminal end. Show the products of cleavage with: (a) trypsin; (b) chymotrypsin. Show the Af-phenylthiohydantoin that results from treatment of with phenyl isothiocyanate. Do you expect to be an acidic, a neutral D D or a basic peptide? Kallidin (E) is a decapeptide that serves as a vasodilator. The composition of E is Arg2 Gly Lys Phe2 Pro3 Ser. The C-terminal residue is Arg. Partial acid hydrolysis yields the following fragments: What is Pro-Gly-Phe, Lys-Arg-Pro, Pro-Phe-Arg, Pro-Pro-Gly, Phe-Ser-Pro the structure of E. Multiple choice: 1 . The enantiomer of a-D-glucopyranose (a) 2 . 3 . p-D-Glucopyranose (b) is: a-L-Glucopyranose (c) p-L-Glucopyranose (d) none of these All of the following reagents convert an aldose to an aldonic acid except: (a) dilute HNO3 (b) Fehling's reagent (c) Benedict's reagent (d) aqueous Br2 Which two aldoses yield D-lyxose after Wohl degradation? D-Glucose and D-Mannose (b) D-Erythrose and D-Threose D- Altrose (d) D-Galactose and D-Talose (a) (c) D-Galactose and 4. All of the following disaccharides are reducing sugars except: (a) Cellobiose (b) Sucrose (c) Maltose (d) Lactose 5 . Which of the following polysaccharides contains p-glycosidic bonds? (a) Amylose (b) Amylopectin (c) Cellulose (d) Glycogen 6 . To find the p/ of an acidic average of the (a) find the pKa values (c) find the amino acid: two lowest pKa values average of all pKa values (b) find the average of the (d) use the value of the two highest pKa of the side chain. 7 . Which of the following techniques can synthesize a single enantiomer of an amino acid? Hell-Volhard-Zelinskii reaction (b) reductive amination (c) amidomalonate synthesis (d) hydrogenation of a Z enamido acid (a) 8 . 9 . The purple product that results from the reaction of ninhydrin with an amino acid contains which group of the amino acid? (a) the amino group (b) the amino nitrogen (c) the carboxylic acid group (d) the side chain Which of the following reagents is not used in peptide synthesis? Phenylthiohydantoin (b) Di-terf-butyl dicarbonate (c) Benzyl alcohol (d) Dicyclohexylcarbodiimide (a) 10. Which element is not present in myoglobin? group (b) regions of a-helix (c) hydrophobic regions structural (a) a prosthetic structure (d) quaternary Chapter 27 - Bio molecules: Lipids Chapter Outline I. Esters (Sections 27.1-27.3). A. Waxes, 1 . and oils (Section 27.1). are esters of long-chain carboxylic acids with long-chain alcohols. fats Waxes 2. Fats and oils are triacylglycerols. Hydrolysis of a fat yields glycerol and three fatty acids. a. b The fatty acids need not be the same. 3 Fatty acids. a. Fatty acids are even-numbered, unbranched long-chain (C12-C20) carboxylic . . acids. b. The most abundant saturated fatty acids are palmitic (Ci6) and stearic (Ci8) acids. c. The most abundant unsaturated d Cig). Linoleic and arachidonic acids are polyunsaturated fatty acids. i. Unsaturated fatty acids are lower-melting than saturated fatty acids because the double bonds keep molecules from packing closely. The C=C bonds can be catalytically hydrogenated to produce higher-melting . e. fatty acids are oleic and linoleic acids (both fats. Occasionally, cis-trans bond isomerization takes place. i. (Section 27.2). Soap is a mixture of the sodium and potassium salts of fatty acids produced by hydrolysis (saponification) of animal fat. Soap acts as a cleanser because the two ends of a soap molecule are different. a. The hydrophilic carboxylate end dissolves in water. b. The hydrophobic hydrocarbon tails solubilize greasy dirt. c. In water, the hydrocarbon tails aggregate into spherical clusters (micelles), in which greasy dirt can accumulate in the interior. 2+ 2+ or Ca cations. Soaps can form scum when a fatty acid anion encounters a. This problem is circumvented by detergents, which don't form insoluble metal B Soap . 1 . 2 . 3 . Mg salts. C. Phospholipids (Section 27.3). Glycerophospholipids. a. Glycerophospholipids consist of glycerol, two fatty acids (at CI and C2 of glycerol), and a phosphate group bonded to an amino alcohol at C3 of glycerol. 2. Sphingomyelins. a. Sphingomyelins have sphingosine or a related dihydroxy amine as their backbone. b They are abundant in brain and nerve tissue. Phospholipids comprise the major lipids in cell membranes. 3 a. The phospholipid molecules are organized into a lipid bilayer, which has polar groups on the inside and outside, and nonpolar tails in the middle. Prostaglandins and other eicosanoids (Section 27.4). 1 . . . II. A. Prostaglandins. 1 . 2 . 3 . Prostaglandins are C20 lipids that contain a C5 ring and two side chains. Prostaglandins are present in small amounts in all body tissues and fluids. Prostaglandins have many effects: they lower blood pressure, affect blood platelet aggregation, affect kidney function and stimulate uterine contractions. Width: 612 Height: 792 766 Chapter 27 B. Eicosanoids. 1 . Prostaglandins, thromboxanes, and leukotrienes make up the eicosanoid class of compounds. 2 3 . . Eicosanoids are named by their ring system, substitution pattern and number of double bonds. Eicosanoids are biosynthesized from arachidonic acid, which is synthesized from linoleic acid. a. The transformation from arachidonic b. (COX) enzyme. One form of the acid is catalyzed by the cyclooxygenase COX enzyme catalyzes the usual functions, and a second form produces additional prostaglandin as a result of inflammation. IE. Terpenoids (Section 27.5). A. Facts about terpenoids. Terpenoids occur as essential oils in lipid extractions of plants. 1 2. Terpenoids are small organic molecules with diverse structures. . 3 . All terpenoids are structurally related. a. Terpenoids arise from head-to-tail bonding of isopentenyl diphosphate units. b Carbon 1 is the head, and carbon 4 is the tail. 4. Terpenoids are classified by the number of five-carbon multiples they contain. Monoterpenoids are synthesized from two five-carbon units. a. b. Sesquiterpenoids are synthesized from three five-carbon units. c Larger terpenoids occur in both animals and plants. Biosynthesis of terpenoids. 1 Nature uses the isoprene equivalent isopentenyl diphosphate (IPP) to synthesize . . B . . terpenoids. a. IPP is biosynthesized by two routes that depend on the organism and the structure of the terpenoid. i. The mevalonate pathway produces sesquiterpenoids and most animals and ii. triterpenoids in plants. The 1-deoxy xylulose 5-phosphate pathway gives monoterpenoids, diterpenoids, and tetraterpenoids. 2. The mevalonate pathway. CoA undergoes Claisen condensation to form acetoacetyl CoA. Another acetyl CoA undergoes an aldol-like addition to acetoacetyl CoA Acetyl a. b . to give (35)-3-hydroxy-3-methylglutaryl CoA (HMG-CoA). CoA is reduced by NADPH, yielding (7?)-mevalonate. c. d. Phosphorylation and decarboxylation convert (/?)-mevalonate to IPP. Conversion of IPP to terpenoids. a. IPP is isomerized to dimethylallyl diphosphate (DMAPP) by a carbocation HMG 3 . pathway. b . c. The C=C bond of IPP displaces the PPO~ group of dimethallyl diphosphate, to form geranyl diphosphate (GPP), the precursor to all monoterpenoids. Geranyl diphosphate reacts with IPP to yield farnesyl diphosphate (FPP), the precursor to sesquiterpenoids. GPP is isomerized and cyclizes on the way to yielding many monoterpenoids. IV. Steroids (Sections 27.6-27.7). A. Steroids are derived from the triterpenoid lanosterol. 1 Steroids have a tetracyclic fused ring system, whose rings are designated A, B, C, and D. 2. The three six-membered rings adopt chair geometry and do not undergo ring-flips. B Stereochemistry of steroids (Section 27.6). Two cyclohexane rings can be joined either cis or trans. 1 a. In a trans-fused ring, the groups at the ring junction are trans. b. In cis-fused rings, the groups at the ring junction are cis. d. . . . Biomolecules: Lipids A Cis ring fusions usually occur between rings and B. In both kinds of ring fusions, the angular methyl groups usually protrude above the c. 2 . 3 . 4 . rings. A-B trans fusions are more common. Substituents can be either axial or equatorial. Steroids with a. Equatorial substituents are more favorable for steric reasons. C. Types of steroid hormones. 1 Sex hormones. a. Androgens (testosterone, androsterone) are male sex hormones. b Estrogens (estrone, estradiol) and progestins are female sex hormones. 2. Adrenocortical hormones. + + Mineralocorticoids (aldosterone) regulate cellular Na and K balance. a. b Glucocorticoids (hydrocortisone) regulate glucose metabolism and control . . . inflammation. Synthetic steroids. a. Oral contraceptives and anabolic steroids are examples of synthetic steroids. D. Biosynthesis of steroids (Section 27.7). All steroids are biosynthesized from lanosterol. 1 Lanosterol is formed from squalene, which is the product of dimerization of 2 farnesyl diphosphate (FPP). 3 Squalene is first epoxidized to form 2,3-oxidosqualene. 3 . . . 4. Nine additional 5 The first several steps are cyclization reactions. b The last steps are hydride and methyl shifts involving Other enzymes convert lanosterol to cholesterol. needed steps are to form lanosterol. a. carbocations. . . Solutions to Problems 27.1 CH 3 (CH 2 from ) Carnaubawax 18 C^ C20 acid 0(CH 2 from ) 3 i CH 3 C 32 alcohol 27.2 O O II II CH 2 OC(CH 2 ) 14 CH 3 CH 2 OC(CH 2 ) 7 CH= CH(CH 2 7CH 3 ) (cis) O O II II CHOC(CH 2 ) CHOC(CH 2 7 CH= CH(CH 2 7 CH 3 14 CH 3 ) ) (cis) O I 767 ff CH 2 OC(CH 2 II ) CH 2 OC(CH 2 7 CH = CH(CH 2 7 CH 3 14 CH 3 ) Glyceryl tripalmitate Glyceryl tripalmitate is ) Glyceryl trioleate higher melting because it is saturated. (cis) 768 Chapter 27 27.3 CH 3 (CH2)7CH=CH(CH 2 )7CO" Mg 2+ "OC(CH 2 )7CH=CH(CH 2 )7CH3 Magnesium oleate The double bonds are cis. 27.4 O IP CH 2 OC(CH 2 ) CH 2 OH 14 CH 3 + Na ~OC(CH 2 O ) 14 CH 3 Sodium palmitate NaOH HoO II CHOC(CH 2 ) 7 CH= CH(CH2 7 CH 3 ) O + CHOH + 2 Na f CH 2 OC(CH 2 7 CH= CH(CH 2 7 CH 3 ) CH 2 OH ) Glyceryl dioleate monopalmitate (cis double bonds) "OC(CH 2 7 CH ) = CH(CH 2 7 CH 3 Sodium oleate ) cis Glycerol 27.5 C0 2 H Prostaglandin E2 h 27.6 6h h H The pro-S hydrogen OH (blue) ends up cis to the methyl group, and the pro-R hydrogen (red) ends up trans. O pro-R pro -S i 9 H 3 C O— H ,CH 2 OPP ^C. .C. C ^C^ ^CH 2 OPP C A \ H T pro -S A P t \ H H ?H3 / \ | H pro -R H H H Biomolecules: Lipids 27.7 769 As described in Worked Example 27. 1 draw the diphosphate precursor so that it resembles the product. Often, the precursor is linalyl diphosphate, which results from isomerization of geranyl diphosphate (the mechanism is shown in Figure 27.10). In (a), it's not easy to see the relationship, but once you've arrived at the product, rotate the , structure. 770 Chapter 27 27.8 Both ring systems are trans-fused, and both hydrogens at the ring junctions are axial. Refer back to Chapter 4 if you have trouble remembering the relationships of substituents on a cyclohexane ring. equatorial axial 27.9 Draw the three-dimensional structure and note the relationship of the hydroxyl group to groups whose orientation OH-*— equatorial is known. Biomolecules: Lipids 771 27.10 1 /\ H HgC CH3 Cholesterol Lanosterol 1 . Two methyl groups 2. One methyl group 3 4. C5-C6 C8-C9 5 Double bond . . Cholesterol at at 1 . Two 2 . One hydrogen C4. CI 4. single bond. double bond. at at C4. C 1 4. 4. C5-C6 double bond C8-C9 single bond. 5 Saturated side chain. 3. in side chain hydrogens . Visualizing Chemistry 27.11 C0 2 H Cholic acid equatorial Cholic acid have a is an A-B cis steroid because the groups at the fusion of ring A and ring B cis relationship. 27.12 : Draw Base Helminthogermacrene farnesyl diphosphate in the configuration that resembles the product, then draw its isomer (the mechanism for the formation of the isomer is shown in Problem 27.7). allylic In this reaction, a cyclization, followed by loss of a proton to form the double bond, gives helminthogermacrene. 772 Chapter 27 26.13 C0 2 H Linoleic acid A polyunsaturated fat such as linoleic acid is more likely to be found in peanut oil. Additional Problems Fats, Oils, and Related Lipids 27.14 C0 2 H Eicosa-5,8,1 1,14,17-pentaenoic acid (all cis) 27.15 CH 2 OC(CH 2 )i6CH3 CH 2 OC(CH 2 ) 16 CH 3 o I II I I pHOC(CH 2 ) 16 CH 3 f? CHOC(CH 2 or O ) 7 CH 2 OC(CH 2 ) ) ) (cis) p CH 2 OC(CH 2 7 CH= CH(CH 2 7 CH 3 CH= CH(CH2 7CH 3 ) 16 CH 3 (cis) optically active 1 . 2. CH 2 OH optically inactive "OH, H 2 + H3 O •" II II I I CHOH + HOC(CH 2 7 CH=CH(CH 2 7 CH 3 ) ) (cis) + 2 HOC(CH 2 ) 16 CH 3 I Oleic acid CH 2 OH Four bonded different groups are Stearic acid to the central glycerol fat. 27.16 CH 3 (CH 2 ) 14 C Cetyl palmitate OCH 2 (CH 2 ) -| 4 CH 3 carbon atom in the optically active Biomolecules: Lipids 27.17 O II CH 2OC(CH2)7CH=CH(CH 2 )7CH3 (cis) O II CHOC(CH 2 I ) 7 CH= CH(CH2 7 CH 3 (cis) ) {? CH 2 OC(CH 2 7 CH= CH(CH 2 7 CH 3 ) (cis) ) Glyceryl trioleate (a) O CH 2OC(CH 2 I Glyceryl Brc trioleate CH 2 CI 2 I ) 7 CH(Br)CH(Br)(CH 2 ) 7 CH 3 Q II CHOC(CH 2 7 CH(Br)CH(Br)(CH 2 ) 7 CH 3 ) O II CH 2 OC(CH 2 (b) ) 7 CH(Br)CH(Br)(CH 2 ) 7 CH 3 O II CH 2 OC(CH 2 H 2 /Pd Glyceryl I ) 16 CH 3 ff I CH 2 OC(CH 2 ) (c) Glyceryl 16 CH 3 CHoOH 2 NaOH I CHOH HoO trioleate 16 CH 3 n CHOC(CH 2 trioleate ) + 3 + "0 C(CH 2 2 Na ) 7 CH= CH(CH 2 7CH 3 ) CH 2OH (d) o O CH 2 OC(CH 2 7 CH ) o Glyceryl 1. trioleate 2. Zn, o o CH 3 C0 2 H CHOC(CH 2 7 CH ) O O + 3 HC(CH 2 7 CH 3 ) O CH 2 OC(CH 2 7CH ) (e) CH 2 OH Glyceryl trioleate 1. LiAIH 4 2. H3 + CHOH I CH 2 OH + 3 HOCH 2 (CH 2 7CH=CH(CH 2 7 CH 3 ) ) 773 774 Chapter 27 ® CH 2OH B£ SoSte ^ CH 3 3 HOC(CH 2 7 CH=CH(CH 2 7 CH 3 + 3 ? H0H ) CH 2OH ) CH 3 27.18 CH 3 (CH 2 ) 7 CH= CH(CH 2 7 C02 H (cis) ) Oleic acid (a) ^ Oleic acid . CH 3*OH . , CH 3 (CH 2 HCI ^ Methyl from 2 /Pd - i.o 3 Oleic acid 2. ) ) Methyl (c) CH= CH(CH2 7 C02CH 3 CH 3 (CH 2 (a) stearate OHC(CH 2 7 C0 2 H + ) ) n ^ -a 9-Oxononanoic acid '[ XT Nonanal Cr0 3 9-Oxononanoic acid C0 2 CH 3 16 CHo(CH 3V 2 7 CHO z Zn,CH 3 C0 c2 H (d) from 7 Methyl oleate —H oleate ) H0 2 C(CH 2 7 C02 H ) Nonanedioic acid (c) (e) Bf2 Oleic acid » CH 2 CI 2 CH 3 (CH 2 7 CH(Br)CH(Br)(CH 2 7C02 H 1 3 NaNH 2 NH 3 ) ) . , \ H3 + 2. CH 3 (CH 2 7 C= C(CH 2 7 C0 2 H ) ) StearoUc acid Three equivalents of the base are needed because one of them is neutralized by the CH 3 (CH 2 CHC0 2 H carboxylic acid. (f) H 2 /Pd Oleic acid 1 CH 3 (CH 2 ) 15 CH 2 C0 2 H . — Br2 PBr 3 *^ , 2 H2<~ ' ) 15 > Stearic acid (g) + 1 2 CH 3 (CH 2 ) 16 . C0 2 CH 3 2. from Na H3 2-Bromostearic acid " O "OCH-: CH 3 (CH 2 + ) 16 CCH(CH 2) 15 CH 3 + 'I HOCH 3 C0 2 CH 3 (b) H3 + , heat O CH 3 (CH 2 1 ) 16 CCH 2 (CH 2 ) 15 CH 3 + C0 2 + HOCH 3 8-Pentatriacontanone This synthesis uses a Claisen condensation, followed by a /3-keto ester decarboxylation. Biomolecules: Lipids 27.19 775 Fats and plasmalogens are both esters of a glycerol molecule that has carboxylic acid ester groups at C2 and C3. The third group bonded to glycerol, however, differs with the type of lipid: a fat has a carboxylic acid ester at CI, and a plasmalogen has a vinyl ether in that location. 27.20 O ? CH 2 OH + O + I HOCR' P NaOH CHOCR' * + Na II "OCR" yet o HoO + if CHOH CH 2 OH CH 2 OCR" HOCR" + CHOH H2 O + + Na "OCR' II CH 2 OH + CH 2 OCH=CHR HCCH 2 R CH 2 OCH=CHR Basic hydrolysis cleaves the carboxylic acid ester bonds but doesn't affect the ether bond. Acidic hydrolysis cleaves all three groups bonded to glycerol and produces an aldehyde from the vinyl ether group. 27.21 L RC-|OCH H CH 2 0|-CR O 2 I ll | ? if? ,M HOCH 2 CHCH 2 OH + 2 Na3 pQ 4 3 I l R'C^-OCH I ll n CHO"|-CR" NaOH O o I ^ l -|U s II s I CH 2 0|-P-|-0CH 2 CHCH 2 0|-P^-0CH 2 OH O" O II II + RCO" Na+ + + + R"CO~ Na + R"'CO~ Na I + + R'CO" Na H 2° (j> 0~ jj) a cardiolipin Saponification of a cardiohpin yields 4 different carboxylates, 3 equivalents of glycerol and two equivalents of phosphate. 27.22 CH 3 (CH 2 7 C=C(CH 2 7 C0 2 H ) ) Stearohc acid Stearolic acid contains a triple acids. ^ CH 3 (CH 2 7 C0 2 H ) Nonanoic acid + H02 C(CH 2 7C0 2 H ) Nonanedioic acid bond because the products of ozonolysis are carboxylic Width: 612 Height: 792 776 Chapter 27 27.23 CH 3 (CH 2 7 ) NaNHc C= CH CH 3 (CH 2 7 C=C: > Na +J I I-r " _ NaCN CH 3 (CH 2 7 C= C(CH 2 6 CH 2 CN ) CH 3 (CH 2 H3 ) ) 7 — CI _ C(CH CH — CI C= 2 6 2 ) + _ 7 C_ C(CH 2 6 CH 2 C0 2 H ) CH2 (CH 2 5 CH 2 CH 3 (CH 2 ) | u NH 3 + ) NHo ) Stearolic acid - rather than CI - displaced is a better leaving group than chloride. I , by , is acetylide because iodide Terpenoids and Steroids 27.24-27.26 Remember that a compound with n chirality centers can have a maximum of 2 n stereoisomers. Not all the possible stereoisomers of these compounds are found in nature or can be synthesized. Some stereoisomers have highly strained ring fusions; others contain 1 ,3-diaxial interactions. Sabinene Guaiol (8 possible stereoisomers) If carbon 1 (4 possible stereoisomers) Cedrene (16 possible stereoisomers) of each diphosphate were isotopically labeled, the labels would appear circled positions of the terpenoids. at the Biomolecules: Lipids 777 27.27 ff ^C^ CoAS Co AS — ^C^ CHo * CH 3 * *~ CoAS CH 2 * CH 3 * HSCoA + ff H2 HoC 3 \ VOH .C. .C. H 3C Q HoC 3 t OH / Q A "O CH 2 * CH 2 * dimethallyl diphosphate IPP CH 2 * is * isomerized to is CH 3 * CH 2 * + HSCoA SCoA IPP ^C. H 2 C^* . CH 2 CHoOPP + COo * DMAPP. CHo I I ^C. * CH 2 * converted to isopentenyl diphosphate (IPP) and CHo H 2C II (DMAPP). .CHoOPP CH 2 * .c' "O Q .C. PH H3 C ft * + "SCoA mevalonate 5-diphosphate First, H "O * (#)-Mevalonate 27.28 2 NADPH/H -< .CH 2 OH SCoA f . CH 2 CHoOPP .CHoOPP .C^ H3C * * CH DMAPP * DMAPP and IPP couple to give geranyl diphosphate (GPP). GPP * OPP + ' * * OPP Qpp A second molecule of IPP adds to GPP to give farnesyl diphosphate, the precursor to acadinene. 778 Chapter 27 Notice that the C labels are located at two different positions: -OPP was bonded; Now, (2) at the (1) at the carbon to which carbon bonded to the methyl group. arrange farnesyl diphosphate to resemble the skeleton of a-cadinene. The first step sequence is formation of the allylic isomer of FPP; the mechanism was in the reaction shown 27.29 in Problem 27.7. Farnesyl diphosphate (from the previous problem) dimerizes to form squalene. OPP Farnesyl diphosphate Biomolecules: Lipids 27.30 Squalene is 779 converted to lanosterol by the series of steps pictured in Figure 27.14. 27.31 Draw farnesyl diphosphate in the correct orientation in order to make this problem much displacement of ~OPP by the electrons of one double bond is followed by attack of the electrons of the second double bond on the resulting carbocation. Loss of a proton from the carbon next to the resulting carbocation produces the double bond. easier. Internal 780 Chapter 27 General Problems 27.32 (^OPP Farnesyl diphosphate f ^ OPP Isopentenyl diphosphate OPP Geranylgeranyl diphosphate The precursor to flexibilene is formed from the reaction of farnesyl diphosphate and isopentenyl diphosphate. + The precursor cyclizes by the now-familiar OPP Flexibilene mechanism to produce flexibilene. 27.33 O i^-Ionone (3-Ionone + H3 + Acid protonates a double bond, and the electrons of a second double bond attack the carbocation. Deprotonation yields p-ionone. Biomolecules: Lipids 27.34 The two hydrocarbon substituents are equatorial in the most stable chair conformation. 27.35 OH As H H OH H always, use the stereochemistry of the groups at the ring junction to label the other and then esterify the appropriate -OH group. substituents as equatorial or axial 781 782 Chapter 27 27.37 Linalyl diphosphate Isoborneol The initial addition is followed by a carbocation rearrangement to produce a secondary carbocation, which reacts with water to yield the secondary alcohol. 27.38 HoO 2. i. Isoborneol 3. H2 p:^ 4. CHr. Camphene Step 1: Protonation. Step 3: Carbocation rearrangement. The key step ring bonds. is Step 2: Loss of water. Step 4: Loss of proton. the carbocation rearrangement, which occurs by the migration of one of the Biomolecules: Lipids 783 27.39 O Digitoxigenin The hydroxy 1 group in ring A is axial, and the hydroxy C and axial to ring D. fusion and a C-D cis ring fusion. equatorial to ring 1 group at the ring C-D fusion is Notice that digitoxigenin has both an A-B cis ring 27.40 O Lithium aluminum hydride reduces the lactone ring to a hydroxyl group because the second group is tertiary. diol. Periodinane oxidizes only one 784 Chapter 27 27.41 CH 3 (CH 2 5 CHO ) v /C H ' 1 =cx Heptanal 2. Zn, H CH 3 C0 2 H + OHC(CH 2 9 C0 2 H Vaccenic acid ) 1 CH2I2 1-Oxoundecanoic acid CH 2 (CH 2 8 C0 2 H CH 3 (CH 2 4 CH 2 ) ) (and enantiomer) Zn/Cu a r q Lactobacillic acid 27.42 r r H CH0CH0CH0CH0 3 2 2 2 C C 1 C 1 1 CH 2 (CH2) 6 C0 2 H H H (9Z,1 1£,13£)-9,1 1,13-Octadecatrienoic acid (Eleostearic acid) 1- 3 1 2. Zn, I CH 3 CH 2 CH 2 CH 2 CHO + OHC— CHO CH 3 C0 2 H + OHC— CHO The stereochemistry of the double bonds 27.43 can't + OHC(CH 2 7 C0 2 H ) be determined from the information given. This mechanism also appears in Problem 27.32 Biomolecules: Lipids 785 27.44 CHc -OH --H ^ HO Estradiol H3 C Diethylstilbestrol Estradiol and diethylstilbestrol resemble each other in having similar carbon skeletons, in having a phenolic ring, and in being diols. 27.45 1. NaH 2. CH 3 Br CH 3 CH3CH2COCI AICI3 CH3O XT" I 1 . 2. MgBr J Br NaBH, H3 + I ^^Syy, CHCH2CH3 /' S^*****S*S OH I OHCH2CH3 CHCH2CH3 PBr, C I CH2CH3 CH3O Diethylstilbestrol The key reaction is a Grignard reaction between two molecules that are both synthesized from phenol. Phenol is first converted to anisole, in order to avoid problems with acidic hydrogens interfering with the Grignard reaction. Next, anisole undergoes Friedel-Crafts acylation with propanoyl chloride.The resulting ketone is one of the Grignard components. The other component is prepared by reduction, bromination and treatment with magnesium of a quantity of the ketone. After the Grignard reaction, HI serves to both dehydrate the alcohol and cleave the methyl ether groups. Width: 612 Height: 792 786 27.46 Chapter 27 Biomolecules: Lipids 787 27.47 One equivalent of H2 hydrogenates the least substituted double bond. Dihydrocembrene has no ultraviolet absorption because it is not conjugated. 27.48 a-Fenchone The mechanism follows the usual path: cyclization of linalyl diphosphate, followed by attack of the k electrons of the second double bond, produces an intermediate carbocation. A carbocation rearrangement occurs, and the resulting carbocation reacts with water to form an alcohol that is oxidized to give a-fenchone. 788 Chapter 27 27.49 r=0: 11/ CH 3 C— S— Protein CHoC—S— Protein 3 1. | CHC— S— Protein f :CHC— S— Protein I II o2c o I ~0 C 2 2. + O :0:~ C0 2 :ov / ii CH 3 C = CHC — S — P HA O O ii II I CHoCy CHC— S— Protein rote n i O*Cf.. 3. O 4. O" - + - S— Protein- 1 O CH 3 CCH 2 C— S— Protein 3-Ketobutyryl-protein Step 1: Attack of malonyl-protein anion on acetyl-protein (Claisen condensation). Step 2: Loss of S-protein. Step 3: Decarboxylation. Step 4: Protonation and tautomerization. 27.50 PPO In this series of steps, dissociation of diphosphate ion allows bond isomerization to take making it possible for ring formation to occur. This mechanism is very similar to the place, mechanism shown in Figure 27.10. Biomolecules: Lipids Chapter 28 - Biomolecules: Nucleic Acids Chapter Outline I. Nucleic acids (Sections 28.1-28.2). A. Nucleotides (Section 28.1). 1 Nucleotides are composed of a heterocyclic purine or pyrimidine base, an aldopentose, and a phosphate group. a. In RNA, the purines are adenine and guanine, the pyrimidines are uracil and cytosine, and the sugar is ribose. b. In DNA, thymine replaces uracil, and the sugar is 2'-deoxyribose. 2 Positions on the base receive non-prime superscripts, and positions on the sugar receive prime superscripts. 3 The heterocyclic base is bonded to C 1' of the sugar. 4 is vastly larger than RNA and is found in the cell nucleus. B Nucleic acids. 1 Nucleic acids are composed of nucleotides connected by a phosphodiester bond between the 5' ester of one nucleotide and the 3' hydroxy 1 group of another. a. One end of the nucleic acid polymer has a free hydroxy 1 group and is called the . . . . DNA . . 3' end. The other end has a free phosphate group and is called the 5' end. The structure of a nucleic acid depends on the order of bases. The sequence of bases is described by starting at the 5' end and listing b. 2 . 3 . the bases by their one-letter abbreviations in order of occurrence. in DNA (Section 28.2). DNA consists of two polynucleotide strands coiled in a double helix. C. Base-pairing 1 . Adenine and thymine hydrogen-bond with each other, and cytosine and guanine hydrogen-bond with each other. Because the two DNA strands are complementary, the amount of A equals the amount of T, and the amount of C equals the amount of G. The double helix is 20 A wide, there are 10 bases in each turn, and each turn is 34 a. 2. 3 . 4 . A in height. The double helix has a major groove and a minor groove into which polycyclic aromatic molecules can intercalate. D. The "central dogma" of molecular genetics. The function of is to store genetic information and to pass it on to RNA, 1 which, in turn, uses it to make proteins. 2. Replication, transcription and translation are the three processes that are responsible for carrying out the central dogma. transfer of genetic information (Sections 28.3-28.5). A. Replication of (Section 28.3). Replication is the enzyme-catalyzed process whereby makes a copy of itself. 1 2. Replication is semiconservative: each new strand of consists of one old strand and one newly synthesized strand. 3 How replication occurs: DNA . II. The DNA DNA DNA . . a. The i. b . DNA helix partially unwinds. is catalyzed by the enzyme helicase. nucleotides form base pairs with their complementary partners. This process New Biomolecules: Nucleic Acids c. Formation of new bonds 5' catalyzed by DNA polymerase and takes place in the -* 3' direction. Bond formation occurs by i. d. is 791 attack of the 3' hydroxy 1 group on the 5' triphosphate, with loss of a diphosphate leaving group. in the 5' -» 3' direction. Both new chains are synthesized i. One ii. The other chain synthesized continuously (the leading strand). is synthesized in small pieces, which are later joined by ligase enzymes (the lagging strand). (Section 28.4). B. Transcription - synthesis of 1 There are 3 main types of RNA: a. Messenger (mRNA) carries genetic information to ribosomes when protein synthesis takes place. (rRNA), complexed with protein, comprises the physical b. Ribosomal makeup of the ribosomes. c. Transfer (tRNA) brings amino acids to the ribosomes, where they are joined to make proteins. d. There are also small RNAs, which carry out a variety of cellular functions. 2. contains "promoter sites", which indicate where synthesis is to begin, and base sequences that indicate where synthesis stops. polymerase binds to the promoter sequence. a. 3 is synthesized in the nucleus by transcription of DNA. a. The partially unwinds, forming a "bubble". bases. b Ribonucleotides form base pairs with their complementary is strand DNA RNA . RNA RNA RNA DNA mRNA mRNA . RNA mRNA DNA DNA . c. d . e. 4. Bond formation occurs in the 5' -» 3' direction. Only one strand of (the antisense, or noncoding, strand) is transcribed. Thus, the synthesized is a copy of the sense (coding) strand with DNA mRNA U replacing T. Synthesis of a. b. mRNA is not necessarily continuous. DNA DNA Often, synthesis begins in a region of called an exon and is interrupted by seemingly noncoding region of called an intron. a In the final mRNA, the noncoding sections have been removed and the remaining pieces have been spliced together by specific enzymes. Translation (Section 28.5). C. 1 Translation is the process in which proteins are synthesized at the ribosomes by using as a template. 2. The message delivered by is contained in "codons" - 3-base groupings that . mRNA mRNA amino acid. Amino acids are coded by 61 of the possible 64 codons. a. b. The other 3 codons are "stop" codons. Each tRNA is responsible for bringing an amino acid to the growing protein chain. a. A tRNA has a cloverleaf- shaped secondary structure and consists of 70-100 are specific for an 3 . ribonucleotides. b Each tRNA contains an anticodon complementary to the mRNA codon. 4. The protein chain is synthesized by enzyme-catalyzed peptide bond formation. 5 A 3-base "stop" codon on mRNA signals when synthesis is complete. . . IE. DNA technology (Sections 28.6-28.8). A. DNA sequencing (Section 28.6). Before sequencing, the DNA chain is cleaved at specific sites by restriction 1 . endonucleases. The restriction endonuclease recognizes both a sequence on the sense strand and complement on the antisense strand. b. The DNA strand is cleaved by several different restriction endonucleases, to produce fragments that overlap those from a different cleavage. a. its 792 Chapter 28 2. 3 . Maxam-Gilbert DNA sequencing. a. This method uses chemical techniques. Sanger dideoxy DNA sequencing. The following mixture is assembled: i. The restriction fragment to be sequenced. A primer (a small piece of DNA whose sequence ii. on the 3' end of the fragment). iii. The 4 DNA nucleoside triphosphates. a. iv. complementary to that Small amounts of the four dideoxynucleotide triphosphates, each of which is b. is labeled with a different fluorescent dye. DNA polymerase is added to the mixture, and a strand begins to grow from the end of the primer. B. c. Whenever a dideoxynucleotide d. When reaction is complete, e. Because fragments of all possible lengths are represented, the sequence can be read by noting the color of fluorescence of each fragment. is incorporated, chain growth stops. by gel electrophoresis. the fragments are separated DNA synthesis (Section 28.7). DNA synthesis is based on principles similar to those for peptide synthesis. 1 . 2 . The following steps are needed: The nucleosides are protected and bound a. i. ii. to a silica support. Adenine and cytosine bases are protected by benzoyl groups. Guanine is protected by an isobutyryl group. Thymine isn't protected. The 5' -OH group is protected as a DMT ether. The DMT group is removed. The polymer-bound nucleoside is coupled with a protected nucleoside iii. iv. b . c. containing a phosphoramidite group. d . e. f . i. One of the phosphoramidite oxygens ii. Tetrazole catalyzes the coupling. is protected as a /3-cyano ether. The phosphite is oxidized to a phosphate with I2. Steps b - d are repeated until the desired chain is synthesized. All protecting groups are removed and the bond to the support treatment with aqueous ammonia. is cleaved by C. The polymerase chain reaction (Section 28.8). The polymerase chain reaction (PCR) can produce vast quantities of a 1 . DNA fragment. 2 . 3. The key to PCR is Taq DNA polymerase, a heat-stable enzyme. a. Newer heat-stable DNA polymerase enzymes have become available. Steps in PCR: a. The following mixture is heated to 95 °C (a temperature at which DNA becomes single-stranded); i. Taq polymerase. ii. Mg2+ ion. iii. iv. b c . . d. The 4 deoxynucleotide triphosphates. A large excess of two oligonucleotide primers, each of which is complementary to the ends of the fragment to be synthesized. The temperature is lowered to 37 °C - 50 °C, causing the primers to hydrogenbond to the single-stranded DNA. After raising the temperature to 72 °C, Taq catalyzes the addition of further nucleotides, yielding two copies of the original DNA. The process is repeated until the desired quantity of DNA is produced. Biomolecules: Nucleic Acids 793 Solutions to Problems 28.1 5' end 2'-Deoxyadenosine 5'-phosphate (A) H N NH 2 2'-Deoxyguanosine 5 '-phosphate (G) 3' end OH 28.2 5* end H O" N* I "O— P=0 I Uridine 5'-phosphate (U) I OCH "O— P=0 Adenosine 5'-phosphate (A) 3' 28.3 end OH OH DNA (5' end) GGCTAATCCGT (3' end) is complementary to DNA (3' end) CCGATTAGGCA (5* end) Remember that the complementary strand has the 3' end on the left and the 5' end on the right. The complementary sequence can DNA used (5' (3' end) to 5', also be written as: ACGGATTAGCC (3' end). Be sure that you know which format is being or 5' to 3'). Chapter 28 794 28.4 Adenine O H 28.5 28.6 DNA RNA RNA DNA 28.7-28 (5' (3' end) end) end) end) (5' (3' GATTACCGTA (3' CUAAUGGCAU (5' UUCGCAGAGU (3' AAGCGTCTCA (5' end) end) end) end) is complementary to antisense (noncoding) strand Several different codons can code for the same amino acid (Table 28. 1). The 8 corresponding anticodon follows the slash mark after each codon. The codons are written with the 5' end on the left and the 3' end on the right, and the tRNA anticodons have the 3' end on . mRNA the left and the Amino 5' end on the Ala acid: Codon sequence/ tRNA right. anticodon: Leu Phe UUU/AAA UUC/AAG GCU/CGA GCC/CGG GCA/CGU GCG/CGC UUA/AAU UUG/AAC CUU/GAA CUC/GAG CUA/GAU CUG/GAC Tyr UAU/AUA UAC/AUG 28.9-28.10 The mRNA base sequence: The amino The (5' end) DNA sequence: (3* end) (5' end) Leu— Met— Ala— Trp— Pro-(stop) acid sequence: (antisense strand) CUU-AUG-GCU-UGG-CCC-UAA (3' end) GAA-TAC-CGA-ACC-GGG-ATT Biomolecules: Nucleic Acids 28.11 OCH. OCH- °-Q-HJ O O CH 2 Ohv H j Polymer)- CB CO / + CH 2 OH Polymer- CO H-rOH 2 + O Base H HoO O Base H OCH< CH 3 OH O u .11 polymer)- CCX^vA HoO CHoOH v O H H-CH20H ft Jolymej)+ HoO + O N Base H Cleavage of DMT ethers proceeds by an cation is CO^vX Sjsjl mechanism and is N Base rapid because the DMT unusually stable. 28.12 ^-:nh 3 r ..^7 0= P— O— CH2 This is nitrile H r-i CHCN 0=P— O" + H 2 C=CHCN + NH 4H an E2 elimination reaction, which proceeds easily because the hydrogen a to the group is acidic. 795 Width: 612 Height: 792 796 Chapter 28 Visualizing Chemistry 28.13 H Guanine (G) DNA RNA All three bases are found in Uracil (U) RNA RNA, but only Cytosine (C) DNA RNA guanine and cytosine are found in DNA. 28.14 The triphosphate made from 2',3'-dideoxythymidine 5' phosphate is labeled with a fluorescent dye and used in the Sanger method of DNA sequencing. Along with the restriction fragment to be sequenced, a DNA primer, and a mixture of the four dNTPs, small quantities of the four labeled dideoxyribonucleotide triphosphates are mixed together. DNA polymerase is added, and a strand of DNA complementary to the restriction fragment is synthesized. Whenever a dideoxyribonucleotide is incorporated into the DNA chain, chain growth stops. The fragments are separated by electrophoresis, and each terminal dideoxynucleotide can be identified by the color of its fluorescence. By identifying these terminal dideoxy nucleotides, the sequence of the restriction fragment can be read. 28.15 According map, the nitrogen at the 7 position of 9methylguanine is more electron-rich (red) and should be more nucleophilic. Thus 9methylguanine should be the better nucleophile. to the electrostatic potential 9-Methylguanine 9-Methyladenine Biomolecules: Nucleic Acids 797 Additional Problems DNA that codes for natriuretic peptide (32 amino acids) consists of 99 bases; 3 bases code for each of the 32 amino acids in the chain (96 bases), and a 3 -base "stop" codon is 28.16 The also needed. 28.17 Position 9: Horse amino acid = Gly mRNA codons (5' GGU GGC GGA GGG Human amino acid = — > 3'): Ser UCU UCC UCA UCG AGU AGC DNA bases (antisense strand 3' — > 5'): CCC AGA AGG AGT AGC T£A TCG CCA CCG CCT The underlined horse DNA base underlined) by only one base. triplets differ from their human counterparts (also Position 30: Horse amino acid = Ala mRNA codons Human amino acid = Thr (5' —> GCU GCC GCA GCG 3'): ACU ACC ACA ACG DNA bases (antisense strand 3' — > 5'): CGA CGG CGT CGC TGA TGG TGT TGC Each of the above groups of DNA bases from horse insulin has a counterpart in human insulin that differs from it by only one base. It is possible that horse insulin DNA differs from human insulin DNA by only two bases out of 159! 28.18 The percent of A always equals the percent of T, since A and T are complementary. The G equals the percent C for the same reason. Thus, sea urchin DNA contains about 32% each of A and T, and about 18% each of G and C. percent 28.19 Even though doesn't stop. DNA UAA shown contains the stretch of in sequence, protein synthesis are read as 3-base individual units from start to end, and, in this The codons mRNA sequence, the unit UAA is read as part of two codons, not as a single codon. 28.20 Restriction endonucleases cleave DNA base sequences that are palindromes, meaning that complement when both are read in the (5') to (3') sequence in (c), CTCGAG is recognized. The sequence in (a), GAATTC, is also a palindrome and is recognized by a restriction endonuclease. The sequence in (b) is not a palindrome and is not recognized. the sequence reads the same as the direction. Thus, the 28 21—28 23 mRNA codon Amino DNA tRNA The : (5'->3') acid: sequence: anticodon: (3'->5*) (3'->5') (a) AAU (b) GAG (c) UCC (d) CAU Asn Glu Ser His TTA CTC GTA UUA CUC AGG AGG DNA sequence of the antisense (noncoding) strand is shown. GUA 798 Chapter 28 UAC is a codon for tyrosine. DNA chain. 28.24-28.25 strand of a It was transcribed from ATG of the antisense mRNA codon 5' DNA end 3' end O— P=0 O u N CH 2 J f O > N 0=P— O" I I NH' o £-0 CH, T O V H' I ^CHoO=P— 0" I o ISL . N o CH, M > ? 0=P— O" I O 3' 28.26 Tyr— UAC UAU — end 5' Gly Gly Phe GGU GGC GGA GGG GGU GGC GGA GGG UUU A total of2x4x4x2x 1 x3 = metenkephalin! o -Met AUG UUC 194 different (stop) is coded by UAA UAG UGA mRNA sequences can code for end 799 Biomolecules: Nucleic Acids 28.27 Angiotensin II: Asp Arg Tyr Val lie His Phe Pro (stop) mRNA sequence: GAU CGU GUU UAU AUU CAU CCU UUU UAA (5'->3*) GAC CGC GUC UAC AUC CAC CCC UUC UAG AUA UGA CGA GUA CCA CGG GUG CCG AGA AGG As in the previous problem, many mRNA sequences (13,824) can code for angiotensin 28.28 DNA coding strand mRNA (5'->3'): (5'->3'): CTT- CGA-CCA- GAC — AGC — TTT CUU-CGA-CCA-GAC-AGC-UUU Amino acid sequence: The mRNA sequence is Arg Pro Asp Phe Ser complement of the DNA noncoding (antisense) Leu the DNA coding (sense) strand. Thus, the the complement of the of the coding (sense) strand, with T replaced by U. is strand, which mRNA sequence is a copy DNA 28.29 mRNA sequence (5'->3'): CUA-GAC-CGU-UCC-AAG-UGA Amino Leu Acid: 28.30 DNA coding strand (5'->3'): mRNA sequence (5'->3'): Asp Arg Ser Lys (stop) Original Sequence Miscopied Sequence -CAA-CCG-GAT-CAA-CCG-GAU- -CGA-CCG-GAT-CGA-CCG-GAU- — — Amino — — acid sequence: -Gln Pro Asp-Arg Pro Aspgene sequence were miscopied in the indicated way, a glutamine in the original protein would be replaced by an arginine in the mutated protein. If this 28.31 1 . First, protect the nucleosides. (a) II. Bases are protected by amide formation. NH 2 C 6 H 5 COQ Adenine pyridine C 6 H 5 COCI Cytosine pyridine (CH 3 2 CHCOCI ) Guanine pyridine NHCOCH(CH 3 Thymine does not need to be protected. ); 800 Chapter 28 The (b) hydroxyl group 5' is protected as Base its p-dimethoxytrityl (DMT) Base DMTOCH' HOCHc 1 . ether. Base i 2. DMTBr OCCH 2 CH 2 CO O OCCHoCHoCO 2 2 O 2. Attach a protected 2-deoxycytidine nucleoside to the II H o o polymer support. Cytosine DMTOCH 2 OCCH 2 CH 2 CNH(CH 2 OCCHoCHoCO 2 2 II H o o Silica) — 3. O — CCH CH CNH(CH — 3 Si — 2 ) 3 Si 2 (^Silica O Support DMT ether. Cytosine Cytosine DMTOCHc HOCHc CHCI 2 C0 2 H CH 2 CI 2 O ^Silica {? 2 Cleave the ) Si(CH 2 ) 3 NH 2 ?\ Let Cytosine DMTOCH2 (Support O (Support Biomolecules: Nucleic Acids 4. 801 Couple protected 2'-deoxythymidine to the polymer-2'-deoxycytidine. (The nucleosides have a phosphoramidite group at the 3' position.) Thymine DMTOCH, Thymine DMTOCH' Tetrazole O o P— N(/- Pro)2 NCCH 2 CH 2 P-OCH2 Cytosine NCCH 2 CH 2 O C Support HOCH 5 . 6. Oxidize the phosphite product to a phosphate triester, using iodine. Repeat steps 3-5 with protected 2'-deoxyadenosine and protected 2'-deoxyguanosine. 802 Chapter 28 7 . Cleave all protecting groups with aqueous yield the desired sequence. Guanine Guanine DMTOCH, ammonia to HOCH 2 o ? 0=P— O" I 0=P— OCH 2 CH2 CN Adenine Adenine OCH 2 OCH, HoO O 0=P— o~ I 0=P— OCHoCHoCN Thymine I OCH 2 o ? 0=P— O" I 0=P — OCH2 CH 2CN Cytosine O Thymine I OCH 2 (Support Cytosine OCHc OH Biomolecules: Nucleic Acids 28.32 Both of these cleavages occur by the now-familiar nucleophilic acyl substitution route. nucleophile adds to the carbonyl group, a proton shifts location, and a second group is eliminated. Only the reacting parts of the structures are sown. Deprotection at 1 HoN Deprotection at HoN 2: OH :0. O. C / \ V^r (P. mi N H3 t :orNH2 803 A 804 Chapter 28 28.34 This reaction involves addition of a thiol residue of the enzyme to malonic semialdehyde, + yielding a hemithioacetal (Step 1). Oxidation by (step 2), followed by nucleophilic acyl substitution by Co A (Step 3), gives malonyl Co A. NAD Malonic semialdehyde O SCoA Malonyl 28.35 The steps: (1) CoA phosphorylation by ATP; (2) cyclization; (3) loss of phosphate; (4) tautomerization. ADPO— P0 3^ H ADP 1 I. 5-Phosphoribose Formylglycinamidine ribonucleotide i :b H P; :B H /A 4. NH NH, 5-Phospho- 5-Phospho- 5-Phospho- ribose ribose ribose Aminoimidazole ribonucleotide Biomolecules: Nucleic Acids 28.36 The Addition of water; (2) Proton steps: (1) shift; (3) Elimination of NH 3 805 . ..-r< Guanine H + NH 3 Xanthine 28.37 Both steps are nucleophilic acyl substitutions, (a) "0 C. 2 o2c, Pi, - ..H 2 N HoN '» H+ °2 C H 2N r-H HoN' " C0 2 2-, °="°) H H °* O N I r-H C0 2 H Carbamoyl aspartate " Width: 612 Height: 792 806 Chapter 28 28.38 (a), (b) The -CH2- group (d) C2 of deoxyribose The missing atoms ring, the (e) at is missing from ganciclovir. are part of the relatively inflexible deoxyribose ring. Without the DNA chain is floppy and base pairing to form a double helix can't occur. As mentioned in (d), the inability to form base pairs stops the replication of DNA. Chapter 29 - The Organic Chemistry of Metabolic Pathways Chapter Outline I. Overview of metabolism and biochemical energy (Section 29.1). A. Metabolism. The reactions that take place in the cells of organisms 1 . are collectively called metabolism. The reactions that produce smaller molecules from larger molecules are called catabolism and produce energy. b The reactions that build larger molecules from smaller molecules are called anabolism and consume energy. Catabolism can be divided into four stages: a. In digestion, bonds in food are hydrolyzed to yield monosaccharides, fats, and a. . 2 . amino b . c. acids. These small molecules are degraded In the citric acid cycle, acetyl to acetyl CoA. CoA is catabolized to CO2, and energy is produced. d. Energy from the is citric acid cycle enters the electron transport chain, where ATP synthesized. B Biochemical energy. . 1 2 . ATP, a phosphoric acid anhydride, is the storehouse for biochemical energy. The breaking of a P-O bond of ATP can be coupled with an energetically unfavorable reaction, so that the overall energy change is favorable. The resulting phosphates are much more reactive than the original compounds. Lipid metabolism (Sections 29.2-29.4). A. Catabolism of fats (Section 29.2-29.3). Triacylglycerols are first hydrolyzed in the stomach and small intestine to yield 1 3 II. . . . glycerol plus fatty acids (Section 29.2). a. The reaction is catalyzed by a lipase. i. Aspartic acid, serine and histidine residues in the enzyme bring about reaction. b . Glycerol i . ii. 2. 3 . is phosphorylated and oxidized and enters glycolysis. The mechanism of oxidation involves a hydride The addition to NAD + is stereospecific. ^-Oxidation transfer to NAD+ . mitochondria) (Section 29.3). by /3-oxidation, a 4-step spiral that results in the cleavage of an n-carbon fatty acid into nil molecules of acetyl CoA. (in the a. Fatty acids are degraded b. Before entering /^-oxidation, a fatty acid is first converted to its fatty-acyl CoA. Steps of p oxidation. Introduction of a double bond conjugated with the carbonyl group. a. i. The reaction is catalyzed by acyl CoA dehydrogenase. ii. The enzyme cofactor FAD is also involved and is reduced. iii. The mechanism involves abstraction of the pro-R a and p hydrogens, resulting in formation of a trans double bond. b . c. Conjugate addition of water to form an alcohol. i. The reaction is catalyzed by enoyl CoA hydratase. Alcohol oxidation. i. The reaction is catalyzed by L-3-hydroxyacyl CoA dehydrogenase. + + ii. The cofactor NAD is reduced to NADH/H at the same time. iii. Histidine deprotonates the hydroxyl group. 808 Chapter 29 d . Cleavage of acetyl i. The reaction, CoA from the chain. which is catalyzed by /3-keto thiolase, is a retro-Claisen reaction. Nucleophilic addition of coenzyme A to the keto group is followed by loss of acetyl CoA enolate, leaving behind a chain-shortened fatty-acyl CoA. ^-carbon fatty acid yields nil molecules of acetyl CoA after (rc/2-1) passages of ii. 3 . An /^-oxidation. a. Since most fatty acids have an even number of carbons, no carbons are left over after p- oxidation. B . b. Those with an odd number of carbons require further steps for degradation. Biosynthesis of fatty acids (Section 29.4). 1 . General principles. a. b . c. 2 . most cases, the pathway of synthesis isn't the exact reverse of degradation. i. If AG° is negative for one route, it must be positive for the exact reverse, which is thus energetically unfavorable. ii. The metabolic strategy is for one pathway to be related to its reverse but not to be identical. All common fatty acids have an even number of carbons because they are synthesized from acetyl CoA. In vertebrates, a large multienzyme synthase complex catalyzes all steps in the pathway. In Synthetic pathway. Steps 1-2: Acyl transfers convert acetyl CoA to a. Acetyl CoA is converted to acetyl ACP. i. ii. b . The acetyl group of acetyl more reactive species. ACP is transferred to the synthase enzyme. Steps 3-4: Carboxylation and acyl transfer. i. Acetyl CoA reacts with bicarbonate to yield malonyl (a). The coenzyme biotin, a CoA CO2 carrier, transfers CO2 in and ADP. a nucleophilic acyl substitution reaction. ii. At CoA is converted to malonyl ACP. both acetyl groups and malonyl groups are bound to the synthase enzyme. Step 5: Condensation. i. Claisen condensation forms acetoacetyl CoA from acetyl synthase and iii. c. Malonyl this point, A malonyl ACP. The reaction proceeds by an initial decarboxylation of malonyl an enolate that adds to acetyl synthase to form acetoacetyl CoA. Steps 6-8: Reduction and dehydrogenation. The ketone group of acetoacetyl CoA is reduced by NADPH. i. ii. d. ACP to give The ^-hydroxy thiol ester is dehydrated. The resulting double bond is hydrogenated by NADPH to yield butyryl ACP. The steps are repeated with butyryl synthase and malonyl ACP to give a sixii. iii. e. carbon unit. up to palmitic acid (16 carbon atoms) are synthesized by this route, Elongation of palmitic acid and larger acids occurs with acetyl CoA units as the two-carbon donor, rather than ACP. HI. Carbohydrate metabolism (Sections 29.5-29.8). A. Catabolism of carbohydrates (Sections 29.5-29.7). 1. Glycolysis (Section 29.5). Glycolysis is a 10-step series of reactions that converts glucose to pyruvate. a. f . Fatty acids i. The Organic Chemistry b. of Metabolic Pathways 809 Steps 1-2: Phosphorylation and isomerization. Glucose i. is phosphorylated at the 6-position The enzyme hexokinase by reaction with ATP. involved. ii. Glucose 6-P is isomerized to fructose 6-P by glucose-6-P isomerase. Step 3: Fructose 6-P is phosphorylated to yield fructose 1,6-bisphosphate. (a). c. is (a). ATP and phosphofructokinase are involved. Step 4: Cleavage. Fructose 1,6-bisphosphate is cleaved to glyceraldehyde 3-phosphate and i. dihydroxyacetone phosphate. (a). The reaction is a reverse aldol reaction catalyzed by aldolase. Step 5: Isomerization. e. Dihydroxyacetone phosphate is isomerized to glyceraldehyde 3-phosphate. i. ii. The net result is production of two glyceraldehyde 3 -phosphates, both of which pass through the rest of the pathway. Steps 6-7: Oxidation, phosphorylation, and dephosphorylation. f Glyceraldehyde 3-phosphate is both oxidized and phosphorylated to give i. 1 ,3-bisphosphoglycerate. + (a). Oxidation by occurs via a hemithioacetal to yield a product that forms the mixed anhydride. ii. The mixed anhydride reacts with ADP to form ATP and 3 -phosphogly cerate (a). The enzyme phosphogly cerate kinase is involved. g Step 8: Isomerization. 3 -Phosphogly cerate is isomerized to 2-phosphogly cerate by 1 phosphoglycerate mutase. h. Steps 9-10: Dehydration and dephosphorylation. 2-Phosphoglycerate is dehydrated by enolase to give phosphoenolpyruvate. i. Pyruvate kinase catalyzes the transfer of a phosphate group to ADP, with ii. formation of pyruvate. The conversion of pyruvate to acetyl Co A (Section 29.6). d. . NAD . The conversion pyruvate a. -» acetyl Co A is catalyzed by an enzyme complex called pyruvate dehydrogenase complex. b . Step i. 1 : Addition of thiamin. A nucleophilic ylide group on thiamin diphosphate adds to the carbonyl group of pyruvate to yield a tetrahedral intermediate. Decarboxylation. d 3: Reaction with lipoamide. The enamine product of decarboxylation reacts with lipoamide, displacing i. sulfur and opening the lipoamide ring. Step 4: Elimination of thiamin diphosphate ylide. e. f Step 5: Acyl transfer. i. Acetyl dihydrolipoamide reacts with coenzyme to give acetyl Co A. ii. The resulting dihydrolipoamide is reoxidized to lipoamide by FAD. + iii. FADH 2 is reoxidized to FAD by g Other fates of pyruvate. In the absence of oxygen, pyruvate is reduced to lactate. i. ii. In bacteria, pyruvate is fermented to ethanol. The citric acid cycle (conversion of acetyl Co to CO2) (Section 29.7). a. Characteristics of the citric acid cycle. The citric acid cycle is a closed loop of eight reactions. i ii. The intermediates are constantly regenerated. and FADH2 are available, which means iii. The cycle operates as long as c. . Step Step 2: . A NAD . A . NAD that oxygen must also be available. . 810 Chapter 29 b. Steps 1-2: Addition to oxaloacetate. Acetyl CoA adds to oxaloacetate to form citryl i. Co A, which is hydrolyzed to citrate. c. The is catalyzed by citrate synthase. isomerized to isocitrate by aconitase. (a). The reaction is an Elcb dehydration, followed by conjugate addition of water. Steps 3-4: Oxidative decarboxylations. i. Isocitrate is oxidized by isocitrate dehydrogenase to give a ketone that loses (a). Citrate ii. reaction is CO2 to give d. e . a-ketoglutarate. is transformed to succinyl CoA in a reaction catalyzed by a multienzyme dehydrogenase complex. Steps 5-6: Hydrolysis and dehydrogenation of succinyl CoA. Succinyl CoA is converted to an acyl phosphate, which transfers a i. phosphate group to GDP in a reaction catalyzed by succinyl CoA synthase. ii. Succinate is dehydrogenated by FAD and succinate dehydrogenase to give a-Ketoglutarate ii. fumarate; the reaction is stereospecific. Steps 7-8: Regeneration of oxaloacetate. i. Fumarase catalyzes the addition of water to fumarate to produce (5)-malate. + ii. (5)-malate is oxidized by and malate dehydrogenase to complete the NAD cycle. C. Carbohydrate biosynthesis: gluconeogenesis (Section 29.8). 1 . 2 . 3 . Step 1 Carboxylation. a. Pyruvate is carboxylated to yield oxaloacetate in a reaction that uses biotin and : ATP. 4. Step 2: Decarboxylation and phosphorylation. a. Concurrent decarboxylation and phosphorylation of oxaloacetate produce phosphoenolpyruvate. Steps 3-4: Hydration and isomerization. Conjugate addition of water gives 2-phosphogly cerate. a. b. Isomerization produces 3 -phosphogly cerate. Steps 5-7: Phosphorylation, reduction and tautomerization. Reaction of 3 -phosphogly cerate with ATP yields an acyl phosphate. a. + b The acyl phosphate is reduced by NADPH/H to an aldehyde. c. The aldehyde tautomerizes to dihydroxy acetone phosphate. Step 8: Aldol reaction. a. Dihydroxy acetone phosphate and glyceraldehyde 3-phosphate join to form . 5 . fructose 1,6-bisphosphate. This reaction involves the imine of dihydroxyacetone phosphate, which forms an enamine that takes part in the condensation. Steps 9-11: Hydrolysis and isomerization. a. Fructose 1,6-bisphosphate is hydrolyzed to fructose 6-phosphate. b Fructose 6-phosphate isomerizes to glucose 6-phosphate. c. Glucose 6-phosphate is hydrolyzed to glucose. Several of these steps are the reverse of steps of glycolysis. b 6 . . . 7 . The Organic Chemistry of Metabolic Pathways 811 IV. Protein metabolism (Section 29.9). Catabolism of proteins: Deamination. 1 . The pathway to amino acid catabolism: a. The amino group is removed as ammonia by transamination. b The ammonia is converted to urea. c. What remains is converted to a compound that enters the citric i. Each carbon skeleton is degraded in a unique pathway. . 2. acid cycle, Transamination. The -NH2 group of an amino acid adds to the aldehyde group of pyridoxal phosphate to form an imine (Schiff base). b The imine tautomerizes to a different imine. c. The second imine is hydrolyzed to give an a-keto acid and an amino derivative of pyridoxal phosphate. d The pyridoxal derivative transfers its amino group to a-ketoglutarate, to regenerate pyridoxal phosphate and form glutamate. 3 Deamination. a. The glutamate from transamination undergoes oxidative deamination to yield ammonium ion and a-ketoglutarate. conclusions about biological chemistry (Section 29.10). 1 The mechanisms of biochemical reactions are almost identical to the mechanisms of a. . . . V. Some . laboratory reactions. 2 . Most metabolic pathways are linear. a. Linear pathways make sense when a multifunctional molecule b. transformation. Cyclic pathways is undergoing may be more energetically feasible when a molecule Solutions to Problems 29.1 This reaction is a substitution at phosphorus, with ADP as the leaving group. is small. 812 Chapter 29 29.2 O II CH 3 CH2 - CH 2 CH 2 - CH 2 CH 2 - CH 2 CSCoA Caprylyl CoA Of | (passage 4) O II II CH 3 CH 2 -CH 2 CH 2 -CH 2 CSCoA Hexanoyl CoA | + CH3CSC0A (passage 5) O CH 3 CH 2 -CH 2 CSCoA CoA Butanoyl O 1 + CH3CSC0A r. . (passage 6) O I T II II CH3CSC0A 29.3 + CH3CSC0A A fatty acid with n carbons yields nil acetyl CoA molecules after {nl2 - 1) passages of the p-oxidation pathway. (a) \ \ \ \ \ \ \ CH 3 CH 2 — CH 2 CH 2 — CH 2 CH 2 — CH 2 CH 2 — CH 2 CH 2 — CH2 CH 2 — CH 2 CH 2 — CH 2 C0 2 H IP oxidation O II 8 CH3CSC0A Seven passages of the /^-oxidation pathway are needed. (b) O CH 3 CH 2 — (CH CH — CH C0 H 2 2) 8 2 j / oxidation^ 2 1 Q CH3 q SCoA Nine passages of the ^-oxidation pathway are needed. 29.4 p-Hydroxybutyryl ACP resembles the ^-hydroxy ketones that were described in Chapter that dehydrate readily by an ElcB mechanism. 23 and SACP :0^ ^SACP ^SACP c H-C-H^ H— C— OH " B I h-c; H— C-r OH CH II CH I CH 3 CH 3 + HoO The Organic Chemistry 29.5 of Metabolic Pathways 813 A fatty acid synthesized from 13CH3CC>2H has an alternating labeled and unlabeled carbon The carboxylic chain. acid carbon is unlabeled. CH3CH2CH2CH2CH2CH2CH2CH2CH2CH2CH2CH2CH2CH2CH2CO2H 29.6 The face in front of the plane of the page is the it occurs at the Si face. Re face. Since addition occurs from behind the plane of the page, — 29.7 ATP is produced 29.8 Step 1 is a nucleophilic acyl substitution at phosphorus (phosphate transfer) by the group at C6 of glucose, with ADP as the leaving group. in step 7 (1,3-bisphosphogly cerate > 3-phosphoglycerate) and in step 10 (phosphoenolpyruvate > pyruvate). Refer to Figure 29.7. — -OH Step 2 is an isomerization, in which the pyranose ring of glucose 6-phosphate opens, tautomerism causes isomerization to fructose 6-phosphate, and a furanose ring is formed. Step 3 is a substitution, similar to the one in step 1 , involving the -OH group at C 1 of fructose 6-phosphate (phosphate transfer). Step 4 is a retro-aldol reaction that cleaves fructose 1,6-bisphosphate to glyceraldehyde 3-phosphate and dihydroxyacetone phosphate. Step 5 is an isomerization of dihydroxyacetone phosphate by keto-enol tautomerization. to glyceraldehyde 3-phosphate that occurs Step 6 begins with a nucleophilic addition reaction to the aldehyde group of glyceraldehyde 3-phosphate by a thiol group of an enzyme to form a hemithioacetal, which + is oxidized by NAD to an acyl thioester. Nucleophilic acyl substitution by phosphate yields the product 1,3-bisphosphoglycerate. Step 7 is a nucleophilic acyl substitution reaction at phosphorus, in with 1,3-diphosphoglycerate, yielding ATP and 3-phosphoglycerate Step 8 is an isomerization of 3-phosphoglycerate Step 9 is an ElcB elimination of H2O to to ADP reacts (phosphate transfer). 2-phosphogly cerate. form phosphoenolpyruvate. is a substitution reaction at phosphorus that forms tautomerizes to pyruvate (phosphate transfer). Step 10 which ATP and enolpyruvate, which 814 Chapter 29 29.9 1 CHO 1 —2«C— OH 2C=0 HO— — H H— C— OH H-C-OH H 1 HO—3 C— J H-^C— OH Dihydroxyphosphate 2.5 (pHOH 1.6 CH 2 OP03 2- 2.5 1 - Glyceraldehyde 3 -phosphate and I CHO 2- I 2.5 dehydrogenase 6CH 3 C=0 1.6 complex 2 C0 2 CH 3 Acetyl -CH3 + I Pyruvate 1 and 6 of glucose end up as glucose end up as CO2. CHO SCoA Pyruvate C=0 Carbons 3 Glyceraldehyde 3-phosphate COo" 2 I CHOH 6CH 2 OP03 Fructose 1,6-bisphosphate 3.4 2 CoA groups of acetyl CoA. and carbons 3 and 4 of Refer to Figure 29.12. isocitrate are tricarboxylic acids. 29.11 hO/rC0 2 OH A— H^ ' OoC "OoC COc H H HO— cw-Aconitate 29.12 The pro-R hydrogen is removed during dehydration, and the reaction occurs with anti pro-S Citrate OH (2^,35")-Isocitrate geometry. pro-R 2- H— C— OH + ' CHO Citrate 4 CH 2 OP0 3 acetone I 29.10 3CH 2 OH Aldolase 6CH 2 OP0 3 CH 2 OH > -phosphate^ 2- Glucose 3.4 CH 2 OP0 3 2~ Tnose isomerase I 6 1 2C=0 (j) H—JC— OH 2- CH 2OP0 3 cw-Aconitate The Organic Chemistry 29.13 of Metabolic Pathways 815 1,3-bisphosphoglycerate reacts with a cysteine residue of the enzyme in a nucleophilic acyl substitution reaction, with loss of phosphate. Then, reduction by in a second nucleophilic acyl substitution reaction yields glyceraldehyde 3-phosphate. First, NADH 29.14 CH 3 CH 3 PMP a-Ketoglutarate a-Keto acid imine tautomer Nucleophilic acyl substitution, followed by loss of water, forms the imine tautomer. H 2 N-frEnzi 2- 3 PO. \/ O" H* H C0 2 " I rr I N n N C0 2 H H H 3 N-f(En£j + 2 "OoPO. 6 > .c. H CO, COc CHr. CH. a-Keto acid imine tautomer a-Keto acid imine A lysine residue deprotonates the carbon next to the ring, leading to tautomerization. Width: 612 Height: 792 816 Chapter 29 PLP-glutamate imine a-Keto acid imine Enzymatic protonation of the keto acid imine yields CH 3 PLP imine PLP glutamate imine. CH 3 Glutamate Addition of the enzyme, followed by loss of glutamate, regenerates PLP imine. The Organic Chemistry 29.15 of Metabolic Pathways 817 Position leucine and a-ketoglutarate so that the groups to be exchanged are aligned. This arrangement makes it easy to predict the products of transamination reactions. " CH 3 C0 2 2.0 mol of acetyl CoA fatty acid with n carbons yields n/2 moles of acetyl palmitic acid (C15H31CO2H), 1.0 acid x 8 1 (c) is is its that also has a negative AG°'. One mole of glucose is catabolized mole of acetyl CoA. Thus, 1.0 energetically unfavorable. Since exact reverse has a positive AG°; synthesized by gluconeogenesis, an favorable reaction energetically favorable (negative AG°'), energetically unfavorable. Instead, glucose alternate 29.34 is mol of acetyl CoA mol of palmitic acid CoA per mole of fatty acid. For — > 8.0 mol of acetyl CoA Maltose is a disaccharide that yields two moles of glucose on hydrolysis. Since each mole of glucose yields two moles of acetyl CoA, 1.0 mol of maltose — > 2.0 mol of glucose — > 4.0 mol of acetyl CoA The Organic Chemistry 29.35 Glucose (a) (b) Palmitic acid amu 256.4 of Metabolic Pathways (c) amu Maltose amu Molecular weight 180.2 Moles 0.5549 mol 0.3900 mol 0.2921 mol 2 x 0.5549 mol 8 x 0.3900 mol = 3.120 mol 4x0.2921 mol = 1.168 mol 2526 g 945.6 g in 342.3 100.0 g Moles of acetyl CoA =1.1 10 mol produced Grams acetyl 898.6 g CoA produced Palmitic acid is the most efficient precursor of acetyl CoA on a weight basis. 29.36 Amino acid a-Keto acid (a) " " CH 3 CHCHC0 2 CH 3 CHCC0 2 OH OH (b) " ~ CH 2 CHC0 2 ^ O II NH 3 + I " H 2 NCCH 2 CHC0 2 CH 2 CC0 2 P P II II " H 2 NCCH 2 CC0 2 821 822 Chapter 29 in Section 29.1, formation of glucose 6-phosphate from glucose and ATP is ') The reverse reaction, transfer of a phosphate group energetically favorable (negative 29.37 As we saw AG ADP from glucose 6-phosphate, is energetically unfavorable and doesn't occur spontaneously. Phosphate transfers to ADP from either 3-phosphoglyceroyl phosphate or to phosphoenolpyruvate have negative AG°' values and are energetically favorable reactions. In chemical terms, the leaving groups in the reactions of 3-phosphoglyceroyl phosphate (carboxylate) and phosphoenolpyruvate (enolate) are more stable anions than the leaving group in the reaction of glucose (alkoxide), so these reactions are more favorable. ... RO— P>^ RO— FL— O: :Nu' + Y :o— P- Nu I :0:~ :0: Glucose :0:' Phosphoenolpyruvate 3-Phosphoglyceroyl phosphate CHO RO" = ro: COo" 2 1 COo" 2 I I H-•C — OH H— C— OH 1 C— I HO-C — H-C — OH CH 2 OP0 3 O" II 2- CH 2 1 1 H-•C — OH 1 CH 2 0" 29.38 H^ CH0H ;3 Q B: .0— h^:b H, .0 rL 0H -OH OH H OH -OH OH H- OH -OH H— o 2- CH 2OP0 3 ' CH 2OP0 3 Ribulose 5-phosphate The isomerization of ribulose 5-phosphate intermediate enolate. 2- 2- CH 2 OP0 3 ' ' Ribose 5-phosphate to ribose 5-phosphate occurs by way of an The Organic Chemistry 29.39 This is a reverse aldol reaction, similar to step c. — < ^ u —o H-C-OH + C « II I H+ H— C— O— H— C— OH I CH 2 OP0 3 823 4 of glycolysis. p4r4enz5 " Hv. £enz4 NH P of Metabolic Pathways H2 H— C— OH H— C=0 H— C— OH H— J O— H— C— OH I 2- CH 2OP0 3 I 2- CH 2 OP0 3 2I |H 2 (enz)| NH 2 v° CH 2OH steps in the conversion of a-ketoglutarate to succinyl CoA are similar to steps in the conversion of pyruvate to acetyl CoA shown in Figure 29.1 1, and the same coenzymes are + involved: lipoamide, thiamin diphosphate, acetyl CoA and 29.40 The NAD . 824 Chapter 29 Step 1: Nucleophilic addition of thiamin diphosphate ylid. Step 2: Decarboxylation. Step 3: Addition of double bond to lipoamide, with ring opening. Step 4: Elimination of thiamin diphosphate. Step 5: Nucleophilic addition of acetyl Co A to succinyl lipoamide and elimination of dihydrolipoamide to give succinyl CoA. Step 6: Reoxidation of dihydrolipoamide to lipoamide. 29.41 Addition of -OH: For carbon face to give an Addition of R H+ face to give an : 2, the top face configuration at carbon 2. For carbon 3, the S configuration at top face carbon 3. is the face, and H + adds -OH adds from this from the bottom, or Re, reaction occurs with anti geometry. is Si, The Re and The Organic Chemistry of Metabolic Pathways 825 General Problems 29.42 sn-Glycerol 3-phosphate pro-S CONH, CONH 2 Dihydroxyacetone phosphate pro-R NAD" The face above the plane of the ring -H takes place from the Re face, is problem states that addition of hydrogen comes from ^-glycerol. The added the Si face. Since the the circled hydrogen has pro-R stereochemistry. 29.43 2 Acetyl O HSCOA HSp P CH3CSC0A O H2 HSCoA CH 3 CCH 2 CSC0A CoA Acetoacetyl o o II II CH 3 CCH 2 CO~ CoA CO- V NADH/H + Acetoacetate NAD + O OH IT I CH 3 CHCH 2 CO CH3CCH3 Acetone 3-Hydroxybutyrate ketone bodies 29.44 :0: \|| — Q QT + » H 3 C^ ^SCoA H 2 C^ ^SCoA 29.45 o O II II CH 3 CCH 2 CSCoA HsC^/n^CHoCSCoA d CoAS-«J Addition occurs from the Si face to form the O + "SCoA R enantiomer. Re H 3 < OH vr _r ,CH 2 COSACP HoC fl 11 C H H Si "h:^ O SACP Width: 612 Height: 792 826 Chapter 29 29 .4 6 If dehydration removes the pro-R hydrogen and the resulting double bond must have taken place. is trans, as indicated, anti elimination 29.47 Butyryl The reduction 29.48 is a syn addition. (a) The first sequence of steps in this mechanism involves formation of the imine (Schiff base) of sedoheptulose 7-phosphate, followed by retro-aldol cleavage to form erythrose 4-phosphate and the enamine of dihydroxyacetone. CH 2 OH (enz^NH2 C=^U> HO— C— T H— C — OH H— C — OH H— C — OH CH 2 OH CH 2 OH C-NH^enz; )= Nh&enz 3 HoO ^ f-*L I H-C7O7H B4enz I CH 2OP0 3 H CHO H— C— OH H— C— OH I H— C— OH 2- + I H— C— OH I CH 2 OP03 HO 2- CH 2 OP0 3 Erythrose 4-phosphate 2- The Organic Chemistry of Metabolic Pathways 827 The enamine of dihydroxyacetone adds to glyceraldehyde 3-phosphate to yield fructose 6-phosphate. This reaction is almost identical to the reaction pictured for Step 8 of gluconeogenesis in Section 29.8. (b) CH 2 OH CH 2OH CH 2 OH C=0 acetyl Co A is the formation of acetoacetyl Co A. This reaction also occurs as the first step in fatty acid catabolism. Although we haven't studied the mechanism, it involves formation of a mixed anhydride. HSCoA, ATP XX The final step AMP, PPj O . is SCoA a retro-Claisen reaction, whose mechanism is pictured in Section 29.3 as Step 4 of ^-oxidation of fatty acids. :oT) O £0: O :dp II -4i MO in the -fy- Ground ground Since all orbitals are occupied in Excited state HOMO making n>\ the and and the other occupies y>2 j/n the excited state, there is no state, In the excited state, one electron occupies HOMO. state , Orbitals and Organic Chemistry: Pericyclic Reactions 839 For 1,3-butadiene: ^4* LUMO A HOMO hv s- D C Four 2p atomic 4- ^2 orbitals *-* Ground In the ground state, HOMO, and y4* is the HOMO, and LUMO. ip2 is the V3 is the LUMO. Excited state state In the excited state, ^3 is the 30.2 conrotatory — CH 3 CH 3 CH3 cis product octatriene H C CH 3 H trans product HOMO (not formed) The symmetry of the octatriene HOMO predicts that ring closure occurs by a disrotatory path in the thermal reaction and that only cis product 30.3 Note: rra/is-3,4-dimethylcyclobutene argument. is chiral; is formed. the S,S enantiomer will be used for this Path A: H H E,E CH< Conrotatory ring opening of rra/2s-3,4-dimethylcyclobutene can occur in either a clockwise or a counterclockwise manner. Clockwise opening (path A) yields the E,E isomer; counterclockwise opening (path B) yields the Z,Z isomer. Production of (2Z,4Z)-2,4hexadiene is disfavored because of steric strain between the methyl groups in the transition state leading to ring-opened product. 840 Chapter 30 30.4 conrotato CH *" H H*l H (2£,4Z,6£> trans-5 ,6-Dimethyl- 2,4,6-Octatriene Ground HOMO state Excited state HOMO 1 ,3-cyclohexadiene conrotatory h^7v ch3 h (y ch 3 HoC (2E,4Z,6Z)cis-5 ,6-Dimethyl- 2,4,6-Octatriene Ground state HOMO Excited state HOMO 1 ,3-cyclohexadiene Photochemical electrocyclic reactions of 6 n electron systems always occur in a conrotatory manner. 30.5 (2£,4£)-2,4-Hexadiene (2£,4Z)-2,4-Hexadiene Diene LUMO Alkene HOMO H HoC ^ H CH<: XIX HoC , H The Diels-Alder reaction is a thermal [4 + 2] cycloaddition, which occurs with geometry. The stereochemistry of the diene is maintained in the product. H CH< suprafacial Orbitals and Organic Chemistry: Pericyclic Reactions 841 30.6 reaction of cyclopentadiene and cycloheptatrienone is a [6 + 4] cycloaddition. This thermal cycloaddition proceeds with suprafacial geometry since five electron pairs are The involved in the concerted process. The % electrons of the carbonyl group do not take part in the reaction. 30.7 This [1 ,7] sigmatropic reaction proceeds with antarafacial geometry because four electron pairs are involved in the rearrangement. 842 Chapter 30 30.8 Scrambling of the deuterium label of 1 -deuterioindene occurs by a series of [ 1 ,5] sigmatropic rearrangements. This thermal reaction involves three electron pairs - one pair of jz electrons from the six-membered ring, the n electrons from the five-membered ring, and two electrons from a carbon-deuterium (or hydrogen) single bond - and proceeds with suprafacial geometry. [1,5] D 30.9 The Claisen rearrangement of an unsubstituted allyl phenyl ether is a [3,3] sigmatropic rearrangement in which the allyl group usually ends up in the position ortho to oxygen. In this problem both ortho positions are occupied by methyl groups. The Claisen intermediate undergoes a second [3,3] rearrangement, and the final product is /7-allyl phenol. Orbitals and Organic Chemistry: Pericyclic Reactions CH. HoC, ' 843 [3,3,1 shift Number of 30.10 Type of reaction electron pairs (a) Thermal electrocyclic (b) Photochemical electrocyclic Photochemical cycloaddition (c) (d) (e) Thermal cycloaddition Photochemical sigmatropic four four four four four Stereochemistry conrotatory disrotatory suprafacial antarafacial suprafacial rearrangement Visualizing Chemistry 30.11 [3,33 *shift CH= CH 2 This reaction product. is CH— CH 2 a [3,3] sigmatropic rearrangement that yields 1 ,5-cyclodecadiene as a 30.12 [3,3 ] "shift" The 13 C NMR spectrum of homotropilidene would show five peaks if rearrangement were NMR slow. In fact, rearrangement occurs at a rate that is too fast for to detect. The C spectrum taken at room temperature is an average of the two equilibrating forms, in NMR which positions 1 and 5 are equivalent, as are positions 2 and 4. Thus, only three distinct 13 types of carbons are visible in the spectrum of homotropilidene. C NMR 844 Chapter 30 Additional Problems Electrocyclic Reactions 30.13 Rotation of the orbitals in the 6 According to the rules in ;r electron system occurs in a disrotatory fashion. Table 30. 1, the reaction should be carried out under thermal conditions. Ground state HOMO Excited state For the hydrogens to be trans This can happen only if the in the product, rotation HOMO must occur in a conrotatory manner. HOMO has the symmetry pictured. For a 6 n electron system, this HOMO must arise from photochemical excitation of a m electron. To obtain a product having the correct stereochemistry, the reaction must be carried out under photochemical conditions. Orbitals and Organic Chemistry: Pericyclic Reactions 30.14 The diene can cyclize by either of two conrotatory paths to form cyclobutenes A and B. 845 Width: 612 Height: 792 846 30.15 Chapter 30 A photochemical electrocyclic reaction involving two electron pairs proceeds in a disrotatory manner (Table Ground 30.1). HOMO state Excited state HOMO ti H H The two hydrogen atoms in the four-membered ring are cis to ' > each other in the cyclobutene product. 30.16 The cyclononatriene thermal conditions. is a 6 k electron system that cyclizes by a disrotatory route under The two hydrogens at the ring junction have a cis relationship. heat 30.17 heat ^ } H3C CH conrotatory' HoC CH^ HoC CH< HoC CH<: hv disrotatory (2£,4Z,6Z,8£)2,4,6,8-Decatetraene Four electron pairs undergo reorganization in this electrocyclic reaction. The thermal reaction occurs with conrotatory motion to yield a pair of enantiomeric fra/2s-7,8-dimethyl1,3,5-cyclooctatrienes. The photochemical cyclization occurs with disrotatory motion to yield the cz's-7,8-dimethyl isomer. Orbitals and Organic Chemistry: Pericyclic Reactions 847 30.18 30.19 Two electrocyclic reactions, involving three electron pairs each, occur in this The thermal reaction is a disrotatory process that yields two ds-fused sixrings. membered The photochemical reaction yields the trans-fused isomer. The two pairs of n electrons in the eight-membered ring do not take part in the electrocyclic reaction. isomerization. Cycloaddition Reactions 30.20 is a reverse [4 + 2] cycloaddition. The reacting orbitals have the correct for the reaction to take place by a favorable suprafacial process. This reaction symmetry 848 Chapter 30 H H This [2 + 2] reverse cycloaddition antarafacial is not likely to occur as a concerted process because the geometry required for the thermal reaction is not possible for a four ^-electron system. Formation of the bicyclic ring system occurs by a suprafacial [4 + 2] Diels-Alder cycloaddition process. Only one pair of n electrons from the alkyne is involved in the reaction; the carbonyl k electrons are not involved. Loss of CO2 is a reverse Diels-Alder [4 + 2] cycloaddition reaction. Orbitals and Organic Chemistry: Pericyclic Reactions 849 30.22 The first reaction is a Diels-Alder [4 + 2] cycloaddition, which proceeds with suprafacial geometry. The second reaction is a reverse Diels-Alder [4 + 2] cycloaddition. Sigmatropic Rearrangements 30.23 This thermal sigmatropic rearrangement involved in the reaction. is a suprafacial process since five electron pairs are 850 Chapter 30 30.24 The product of this [3,3] sigmatropic rearrangement is an enol that tautomerizes to a ketone. 2 OH 30.25 [1,3] heat 1 This reaction is 2 Cyclopentene Vinylcyclopropane a [1,3] sigmatropic rearrangement. 30.26 CHo HoC-C^) [3,3] 10 1 ^C^ 2C shift 3 ^ H 3 C— C= C= CH— CH 2 CCH 3 2 1 heat CH 2 3 3 2 3 III' 3C I H An allene is formed by a [3,3] sigmatropic rearrangement. :OH CH< HgC C — C — CH H 3 C— C= C= CH-^CH^ CCH 3 CH2CCH3 A—H^ CHo3 — H tlT O II I H 3 C— C= C— CH= CHCCH 3 O H Acid catalyzes isomerization of the allene to a conjugated dienone via an intermediate enol. 30.27 H3 HoC [3,3] shift heat Karahanaenone Karahanaenone is formed by a [3,3] sigmatropic rearrangement (Claisen rearrangement). Orbitals and Organic Chemistry: Pericyclic Reactions 851 General Problems 30.28 Tables 30.1-30.3 may be helpful. The first step is always to find the number of electron pairs involved in the reaction. Number of Type of reaction Stereochemistry electron pairs Photochemical [1,5] sigmatropic rearrangement (b) Thermal [4 + 6] cycloaddition antarafacial (a) (c) Thermal (d) Photochemical [2 suprafacial [1,7] sigmatropic rearrangement + antarafacial suprafacial 6] cycloaddition 30.29 H HA D Ad HoC D HoC D D h-7 H Each of the two D electrocyclic reactions involves two pairs of electrons D and proceeds in a conrotatory manner. 30.30 Ring opening of Dewar benzene is a process involving two electron pairs and, according to Table 30.1, should occur by a conrotatory pathway. However, if you look back to other ring openings of cis-fused cyclobutenes, you will see that conrotatory ring opening produces a diene in which one of the double bonds is trans. Since a trans double bond in a six-membered ring is not likely to be formed, ring opening occurs by a different, higher energy, nonconcerted pathway. 30.31 Ring opening of the /rarcs-cyclobutene isomer proceeds by the expected conrotatory route to form the observed product. For the ds-cyclobutene isomer, the observed product can be formed by a four-electron pericyclic process only if the four-membered ring geometry is trans. Ring opening of the cis isomer by a concerted process would form a severely strained six-membered ring containing a trans double bond. Reaction of the cis isomer to yield the observed product occurs instead by a higher energy, nonconcerted path. Both reactions are [2 + 2] photochemical electrocyclic reactions, which occur with disrotatory motion. 30.33 [3,3] shift [3,3] shift etc. Bullvalene Bullvalene can undergo [3,3] sigmatropic rearrangements in all directions. At 100 °C, the rate of rearrangement is fast enough to make all hydrogen atoms equivalent, and only one l spectrum. signal is seen in the U NMR Orbitals and Organic Chemistry: Pericyclic Reactions 853 30.34 The observed products A B and result from a [1,5] sigmatropic hydrogen shift with suprafacial geometry, and they confirm the predictions of orbital symmetry. and C D are not formed. 30.35 This [2,3] sigmatropic suprafacial geometry. rearrangement involves three electron pairs and should occur with 854 Chapter 30 30.36 Concerted thermal ring opening of a cis-fused cyclobutene is conrotatory and yields a product having one cis and one trans double bond. The ten-membered ring product of reaction 2 is large enough to accommodate a trans double bond, but a seven-membered ring containing a trans double bond is highly strained. Opening of the cyclobutene ring in reaction 1 occurs by a higher energy nonconcerted process to yield a seven-membered ring having two cis double bonds. 30.37 Thermal ring opening of the methylcyclobutene ring can occur by allowed conrotatory paths to yield the observed product mixture. either of two symmetry- Orbitals and Organic Chemistry: Pericyclic Reactions 30.38 The first reaction is an electrocyclic opening of a cyclobutene Formation of estrone methyl ether occurs by a Diels-Alder [4 ring. + 2] cycloaddition. 30.39 Reaction 1: Reverse Diels-Alder [4 + 2] cycloaddition; Reaction 2: Conrotatory electrocyclic opening of a cyclobutene ring; Reaction 3: Diels-Alder [4 + 2] cycloaddition. 855 Width: 612 Height: 792 856 Chapter 30 H ^Ch^CHg H H ^CH^CHg H3 ^Ch^CHg + OHC H0 2 C Coronafacic acid Treatment with base enolizes the ketone and changes the ring junction from trans to cis. A cis ring fusion is less strained when a six-membered ring is fused to a five-membered ring. 30.40 CHo HoC CH 2 P,3] 3^ CcH HoC^ NH(CH 3 shift ) 2 *heat CH2CH=CH2 2 3 CH2CH=CH2 H 30.41(a) (b) The rearrangement product absorbs at a longer wavelength because extensive system of conjugated double bonds. it has a more Chapter 31 - Synthetic Polymers Chapter Outline I. Chain-growth polymers (Sections 31.1-31.3). A. General features of chain-growth polymerization reactions (Section 31.1). 1 . How polymerization occurs. a. An initiator adds to b The reactive intermediate adds The process is repeated. . c. 2 . a carbon-carbon double bond of a vinyl monomer. to a second molecule of monomer. Types of polymerization. a. A radical initiator leads to radical polymerization. b An . acid causes cationic polymerization. Acid-catalyzed polymerization is effective only if the vinyl monomers contain electron-donating groups. Anionic polymerization can be brought about by anionic catalysts. i. Vinyl monomers in anionic catalysis must have electron-withdrawing groups. Polymerization occurs by conjugate nucleophilic addition to the monomer. ii. iii. Acrylonitrile, styrene and methyl methacrylate can be polymerized i. c. anionically. "Super glue" is an example of an anionic polymer. Stereochemistry of polymerization (Section 3 1 .2). There are three possible stereochemical outcomes of polymerization of a substituted 1 iv. B . . vinyl a. monomer. If the substituents all lie on the same side of the polymer backbone, the polymer is isotactic. along the backbone, the polymer is syndiotactic. randomly oriented, the polymer is atactic. The three types of polymers have different properties. Although polymerization using radical initiators can't be control stereochemically, Ziegler-Natta catalysts can yield polymers of desired stereochemical orientation. b 2 . 3 . . If the substituents alternate c. If the substituents are a. Ziegler-Natta catalysts are organometallic-transition metal complexes. i. They are usually formed by treatment of an alkylaluminum with titanium tetrachloride. Ziegler-Natta polymers have very little chain-branching. c. Ziegler-Natta catalysts are stereochemically controllable. d Polymerization occurs by coordination of the alkene monomer to the complex, followed by insertion into the polymer chain. 4 Common Ziegler-Natta polymers. a. Polyethylene produced by the Ziegler-Natta process (high-density polyethylene) is linear, dense, strong, and heat-resistant. b Other high-molecular- weight polyethylenes have specialty uses. Copolymers (Section 31.3). b . . . . C 1 . 2 . 3 . Copolymers are formed when two different monomers polymerize together. The properties of copolymers are different from those of the corresponding monomers. Types of copolymers. a. Random copolymers. b. Alternating copolymers. B lock copolymers c . 858 Chapter 31 Block copolymers are formed when an excess of a second monomer is added to a still-active mix. Graft copolymers i. Graft copolymers are made by gamma irradiation of a completed homopolymer to generate a new radical initiation site for further growth of a i. d . chain. II. Step-growth polymers (Section 31.4). A. Step-growth polymer are formed by reactions in which each bond is formed independently of the others. B Most step-growth polymers result from reaction of two difunctional compounds. 1 Step-growth polymers can also result from polymerization of a single difunctional . . compound. C. Types of step-growth polymers. Polyamides and polyesters. 1 2 Polycarbonates (formed from carbonates and alcohols or phenols). . . 3 . Polyurethanes. a. urethane has a carbonyl group bonded to both an -NR2 group and an -OR group. b Most polyurethanes are formed from the reaction of a diisocyanate and a diol. Polyurethanes are used as spandex fibers and insulating foam. c. Foaming occurs when a small amount of water is added during i. polymerization, producing bubbles of CO2. Polyurethane foams often use a polyol, to increase the amount of crossii. A . linking. HI. Olefin metathesis polymerization (Section 31.5). A. General features. In an olefin metathesis reaction, two olefins (alkenes) exchange substituents. 1 2 The catalysts contain a carbon-metal (usually ruthenium) double bond. a. They react reversibly with an alkene to form a 4-membered metallacyte. b The metallacyte opens to give a different catalyst and a different alkene. 3 The reaction is compatible with many olefin functional groups. 4 The double bonds allow for further manipulations. B Ring-opening metathesis polymerization (ROMP). 1 The monomer is a moderately strained cycloalkene. 2 The resulting polymer has double bonds spaced regularly along the chain. . . . . . . . . C Acyclic diene metathesis (ADMET). 1 The monomer is a long-chain dialkene with double bonds at the end of the chain. 2 As the reaction progresses, gaseous ethylene escapes, driving the reaction toward product. IV. Polymer structure and physical properties (Section 31.6). A. Physical properties of polymers. 1 Because of their large size, polymers experience large van der Waals forces, a. These forces are strongest in linear polymers. 2 Many polymers have regions held together by van der Waals forces; these regions . . . . . 3 . are known a. Polymer b Tm is . as crystallites. Some polymers have a. by the substituents on the chains. which the crystalline regions of a polymer melt. crystallinity is affected the temperature at little ordering but are hard at room temperature. soft at a temperature T (glass transition temperature). g These polymers become Synthetic Polymers B . 859 Polymers can be classified by physical behavior. 1 . Thermoplastics. a. Thermoplastics have a high Tg and are hard at room temperature. b Because they become soft at higher temperatures, they can be molded. c. Plasticizers such as dialkyl phthalates are often added to thermoplastics to keep them from becoming brittle at room temperature. . 2. Fibers. a. b. Fibers are produced by extrusion of a molten polymer. On cooling and drawing out, the crystallite regions orient along the axis of the fiber to 3 . add tensile strength. Elastomers. a. Elastomers are amorphous polymers that can stretch and return to their original shape. b. These polymers have a low Tg and a small amount of cross-linking. c. The randomly coiled chains straighten out in the direction of the pull, but they return to their random orientation when stretching is done. Natural rubber is an elastomer, but gutta percha is highly crystalline. Thermosetting resins. Thermosetting resins become highly cross-linked and solidify when heated. a. b Bakelite, a phenolic resin formed from phenol and formaldehyde, is the most familiar example. d. 4. . Solutions to Problems Most reactive **» H 2 C=CHC 6 H 5 > H 2 C=CHCH 3 > H 2C=CHCI Least reactive > H 2 C=CHC0 2 CH 3 The alkenes most reactive to cationic polymerization contain electron-donating functional groups that can stabilize the carbocation intermediate. The reactivity order of substituents cationic polymerization is similar to the reactivity order of substituted benzenes in electrophilic aromatic substitution reactions. 31.2 Most reactive ^- Least reactive H 2 C=CHC=N > H 2 C=CHC 6 H 5 > H 2 C=CHCH 3 Anionic polymerization occurs most readily with alkenes having electron-withdrawing substituents. 31.3 The intermediate anion can be stabilized by resonance involving the phenyl ring. in 860 Chapter 31 31.4 n * -^CH 2 -CCI 2 H 2C=CCI 2 ' 'n Vinylidene chloride doesn't polymerize in isotactic, syndiotactic or atactic forms because no asymmetric centers are formed during polymerization. 31.5 None of the polypropylenes rotate plane-polarized light. If an optically inactive reagent and an achiral compound react, the product must be optically inactive. For every chirality center generated, an enantiomeric chirality center is also generated, and the resulting polymer mixture is optically inactive. 31.6 CHo CHo I n H 2 C=CCH=CH 2 + H 2 C=C n CHo / / -4- i_i+ I CHo I I CH 2 C= CHCH 2 CH 2 C V In L. CHf *~ CHg 2-Methylpropene 2-Methyl- 1 ,3butadiene 31.7 -^CH2CH = CHCH 2 -^ —»* -^-CH 2 CH=CHCH-^ | m H 2 C=CHC 6 H 5 CH 2 CH = CHCH polybutadiene chain C 6 H 5 CH -j/ Irradiation homolytically cleaves an allyhc energy. The polystyrene C-H bond because it has the lowest bond resulting radical adds to styrene to produce a polystyrene graft. 31.8 O n cham m HOCH 2 CH 2 OH + n , — HOC />|v» N^N H2N Melamine, .NHCH 2 NH f repeat times Melmac NH 2 .NH2 N N CH2 many YT >y n NH2 Synthetic Polymers 869 31.29 •o-J V \ H 2 C— CHCH 2 CI CICH 2 CH— CH 2 :o: CI— f — CHoCHCHo\_/— H 2 C— \\ / \ v CHCH 2 repeat several times A H C CHCH 2 A OH 2 O-^x ^-OCH 2 CHCHp ^~9~-C-^ CH2 CHCH 2 -tO-^v I OH V y Cross-linking occurs prepolymer. CH 3 ^— OCH CHCH2|-0-^v =/ ^ when CH3 2 OH - n ' S_ CH3 the triamine opens epoxide rings on two OCH2CHCH2NCH2CH2 ^ OH different chains of the 4 870 31.30 Chapter 31 (a) The diamine is formed by an electrophilic aromatic substitution reaction of formaldehyde with two equivalents of aniline. (b) The diamine reacts with two equivalents of phosgene. Synthetic Polymers 871 31.31 O O CNH — 7-CH (' // \ II NHC— OCH2 CH 2 0- 31.32 NH 2 H2N + O O II II CH 2 k r 31.33 Step 1: Polystyrene and the phthalimide combine in an electrophilic aromatic substitution reaction -(CH 2 -CH^ -(CH 2 -CH^ -(- C H 2 -Ch\ -(CH2 -CH^ CF3SO3H CHo2 — OH v y° I CH 2 -rOH 2 I N o o k| CH 2? I °yy° o Chapter 31 Step 2: The phthalimide is cleaved in a series of steps that involve nucleophilic acyl substitution reactions. — CH^n -^CH 2 Let = R OH; R HgN-NHg R r R H 2 N— NH 2 H. H ,N N o ~ NH 2 O: ^ // attack of / nucleophile x L % A proton shift ring tt opening + NHR NH 2 NH RNH H 2 N-NH RNH 2 HN- NH ' O attack of nucleophile ti HN-NH 0==( + RNH 2 = "(cH 2 — CH-^ n 7=0 CH 2 NH 2 \\ v / "' Synthetic Polymers 873 31.34 2 1 CH 3 CH 2 CH 2 CHO . NaOH, EtOH : 2. heat ' CH 3 CH 2 CH 2 CH= CCHO I I 7 or KMn04 from aldehydes by oxidation from alkyl halides by conversion into Grignard reagents followed by (Sec. 8.8) (Sec. 16.9) (Sec. 19.3) (Sec. 20.5) reaction with (Sec. 20.5, 20.7) (Sec. 21.4) (Sec. 21.5) (Sec. 21.6) (Sec. 21.7) Cyanohydrins, alkyl halides CO2 from nitriles by acid or base hydrolysis from acid chlorides by reaction with aqueous base from acid anhydrides by reaction with aqueous base from esters by hydrolysis with aqueous base from amides by hydrolysis with aqueous base RCH(OH)CN from aldehydes and ketones by reaction with (Sec. 19.6) HCN Cycloalkanes from alkenes by addition of dichlorocarbene from alkenes by reaction with CH2I2 and Zn/Cu (Simmons-Smith (Sec. 8.9) (Sec. 8.9) reaction) from arenes by hydrogenation (Sec. 16.10) Disulfides, RS-SR' from (Sec. 18.8) Enamines, thiols by oxidation with bromine RCH=CRNR 2 from ketones or aldehydes by reaction with secondary amines (Sec. 19.8) O Epoxides, R 2C— CR 2 (Sec. 8.9, 18.5) (Sec. 18.5) Esters, from alkenes by treatment with a peroxyacid from halohydrins by treatment with base RC0 2 R' (Sec. 21.3) (Sec. 21.3) from carboxylic acid salts by Sn2 reaction with primary alkyl halides from carboxylic acids by acid-catalyzed reaction with an alcohol (Fischer esteriflcation) (Sec. 2 1 .4) (Sec. 21.5) (Sec. 22.7) (Sec. 22.7) from acid chlorides by base-induced reaction with an alcohol from acid anhydrides by base-induced reaction with an alcohol from alkyl halides by alkylation with diethyl malonate from esters by treatment of their enolate ions with alkyl halides Functional-Group Synthesis 880 Ethers, R-O-R' (Sec. 16.8) from activated haloarenes by reaction with alkoxide ions from unactivated haloarenes by reaction with alkoxide ions via benzyne (Sec. 18.2) from primary alkyl halides by Sn2 reaction with alkoxide ions (William- (Sec. 18.2) from alkenes by alkoxymercuration/demercuration from alkenes by epoxidation with peroxyacids (Sec. 16.7) intermediates son ether synthesis) (Sec. 18.5) Halides, alkyl, R 3 C -X from alkenes by electrophilic addition of HX from alkenes by addition of halogen from alkenes by electrophilic addition of hypohalous acid (HOX) (Sec. 7.7) (Sec. 8.2) (Sec. 8.3) (Sec. 9.3) (Sec. 9.3) to yield halohydrins from alkynes by addition of halogen from alkynes by addition of from alkenes by ally lie bromination with Af-bromosuccinimide (NBS) from alcohols by reaction with from alcohols by reaction with SOCI2 from alcohols by reaction with PBr3 from alkyl tosylates by Sn2 reaction with halide ions from arenes by benzylic bromination with Af-bromosuccinimide (NBS) from ethers by cleavage with either HX (Sec. 10.3) HX (Sec. 10.5) (Sec. 10.5) (Sec. 10.5) (Sec. 11.2, 11.3) (Sec. 16.9) HX (Sec. 18.3) from ketones by a-halogenation with bromine from carboxylic acids by a-halogenation with phosphorus and PBr3 (Sec. 22.3) (Sec. 22.4) (Hell-Volhard-Zelinskii reaction) Ar-X Halides, aryl, (Sec. 16.1, 16.2) (Sec. 24.8) from arenes by electrophilic aromatic substitution with halogen from arenediazonium salts by reaction with cuprous halides (Sandmeyer reaction) Halohydrins, R 2 CXC(OH)R 2 (Sec. 8.3) (Sec. 18.6) Imines. R 2 C=NR' (Sec. 19.8) Ketones, from alkenes by electrophilic addition of hypohalous acid (HOX) from epoxides by acid-induced ring opening with HX from ketones or aldehydes by reaction with primary amines R2 C = (Sec. 8.8) (Sec. 8.7) (Sec. 9.4) (Sec. 9.4) (Sec. 16.3) (Sec. 17.7, 19.2) (Sec. 19.2, 21.4) from alkenes by ozonolysis from 1,2-diols by cleavage reaction with sodium periodate from alkynes by mercuric-ion-catalyzed hydration from alkynes by hydroboration/oxidation from arenes by Friedel-Crafts acylation reaction with an acid chloride from secondary alcohols by oxidation from acid chlorides by reaction with lithium diorganocopper (Gilman) reagents (Sec. 19.13) (Sec. 20.7) (Sec. 22.7) (Sec. 22.7) from conjugated enones by addition of lithium diorganocopper reagents from nitriles by reaction with Grignard reagents from primary alkyl halides by alkylation with ethyl acetoacetate from ketones by alkylation of their enolate ions with primary alkyl halides Functional-Group Synthesis 881 R-C-N Nitriles, (Sec. 1 from primary alkyl halides by Sn2 reaction with cyanide ion from primary amides by dehydration with SOCI2 from nitriles by alkylation of their a-anions with primary alkyl halides from arenediazonium ions by treatment with CuCN 20.7) 1.3, (Sec. 20.7) (Sec. 22.7) (Sec. 24.8) Nitroarenes, Ar-N(>2 from arenes by (Sec. 16.2) Organometallics, (Sec. 10.6) electrophilic aromatic substitution with nitric/sulfuric acids R-M formation of Grignard reagents from organohalides by treatment with magnesium formation of organolithium reagents from organohalides by treatment with (Sec. 10.7) lithium (Sec. 10.7) formation of lithium diorganocopper reagents (Gilman reagents) from organolithium reagents by treatment with cuprous halides Ar-OH Phenols, from arenediazonium salts by reaction with CU2O and Cu(NC>3)2 from aryl halides by nucleophilic aromatic substitution with hydroxide ion (Sec. 24.8) (Sec. 16.7) Quinones (Sec. 17.10) Sulfides, from [(KS0 3 ) 2NO] thiols by Sn2 reaction of thiolate ions with primary alkyl halides R-S0 2 -R' from (Sec. 18.8) Sulfoxides, sulfides or sulfoxides by oxidation with peroxyacids R-SO-R' (Sec. 18.8) Thiols, salt R-S-R' (Sec. 18.8) Sulfones, from phenols by oxidation with Fremy's from sulfides by oxidation with H2O2 R-SH (Sec. 11.3) (Sec. 18.8) from primary alkyl halides by Sn2 reaction with hydrosulfide anion from primary alkyl halides by Sn2 reaction with thiourea, followed by hydrolysis Functional-Group Reactions The following groups are table summarizes the reactions of important functional groups. The functional followed by a reference to the appropriate text section. listed alphabetically, Acetal 1 . Hydrolysis to yield a ketone or aldehyde plus alcohol (Sec. 19. 10) Acid anhydride 1 . 2 . 3 . 4 . Hydrolysis to yield a carboxylic acid (Sec. 21.5) Alcoholysis to yield an ester (Sec. 2 1 .5) Aminolysis to yield an amide (Sec. 2 1 .5) Reduction to yield a primary alcohol (Sec. 2 1 .5) Acid chloride 1 . 2. 3 . 4 . 5 6 7 . . . compound to yield an aryl ketone (Sec. 16.3) Hydrolysis to yield a carboxylic acid (Sec. 21 .4) Alcoholysis to yield an ester (Sec. 2 1 .4) Aminolysis to yield an amide (Sec. 2 1 .4) Reduction to yield a primary alcohol (Sec. 2 1 .4) Grignard reaction to yield a tertiary alcohol (Sec. 21 .4) Reaction with a lithium diorganocopper reagent to yield a ketone (Sec. 21.4) Friedel-Crafts reaction with an aromatic Alcohol 1 . 2. Acidity (Sec. 17.2) Oxidation (Sec. 17.7) a. Reaction of a primary alcohol to yield an aldehyde or acid b Reaction of a secondary alcohol to yield a ketone Reaction with a carboxylic acid to yield an ester (Sec. 21 .3) Reaction with an acid chloride to yield an ester (Sec. 21.4) Reaction with an acid anhydride to yield an ester (Sec. 21.5) Dehydration to yield an alkene (Sec. 17.6) Reaction with a primary alkyl halide to yield an ether (Sec. 18.2) Conversion into an alkyl halide (Sec. 17.6) a. Reaction of a tertiary alcohol with b. Reaction of a primary or secondary alcohol with SOCI2 Reaction of a primary or secondary alcohol with PBr3 c. . 3 . 4. 5 . 6. 7 . 8. HX Aldehyde Oxidation to yield a carboxylic acid (Sec. 19.3) 2. Nucleophilic addition reactions a. Reduction to yield a primary alcohol (Sees. 17.4, 19.7) b. Reaction with a Grignard reagent to yield a secondary alcohol (Sees. 17.5, 19.7) Grignard reaction of formaldehyde to yield a primary alcohol (Sec. 17.5) c. d. Reaction with HCN to yield a cyanohydrin (Sec. 19.6) Wolff-Kishner reaction with hydrazine to yield an alkane (Sec. 19.9) e. Reaction with an alcohol to yield an acetal (Sec. 19. 10) f g Wittig reaction to yield an alkene (Sec. 19. 1 1) h Reaction with an amine to yield an imine or enamine (Sec. 19.8) 3 Aldol reaction to yield a (3-hydroxy aldehyde (Sec. 23. 1) 4. Alpha bromination of an aldehyde (Sec. 22.3) 1 . . . . . Functional-Group Reactions Alkane 1 Radical halogenation to yield an alkyl halide (Sees. 6.3, 10.3) . Alkene 1 Electrophilic addition of . HX to yield an alkyl halide (Sees. 7.7-7.1 1) Markovnikov regiochemistry 2 . 3 . 4. 5 . 6. 7 . 8 . 9. 10. 1 1 . 12. is observed. Electrophilic addition of halogen to yield a 1 ,2-dihalide (Sec. 8.2) Oxymercuration/demercuration to yield an alcohol (Sec. 8.4) Markovnikov regiochemistry is observed, yielding the more highly substituted alcohol. Hydroboration/oxidation to yield an alcohol (Section 8.5) Hydrogenation to yield an alkane (Sec. 8.6) Hydroxylation to yield a 1,2-diol (Sec. 8.7) Oxidative cleavage to yield carbonyl compounds (Sec. 8.8) Simmons-Smith reaction with CH2I2 to yield a cyclopropane (Sec. 8.9) Reaction with dichlorocarbene to yield a dichlorocyclopropane (Sec. 8.9) Allylic bromination with NBS (Sec. 10.4) Alkoxymercuration to yield an ether (Sec. 1 8.2) Reaction with a peroxyacid to yield an epoxide (Sees. 8.7, 18.5) Alkyne 1 Electrophilic addition of . 3 . Hydroboration/oxidation to yield an aldehyde (Sec. 9.4) Alkylation of an alkyne anion (Sec. 9.8) 4. 5 HX to yield a vinylic halide (Sec. 9.3) Electrophilic addition of halogen to yield a dihalide (Sec. 9.3) Mercuric-sulfate-catalyzed hydration to yield a methyl ketone (Sec. 9.4) 2. . Reduction (Sec. 9.5) Hydrogenation over Lindlar catalyst to yield a a. b. Reduction with Li/NH3 to yield a trans alkene 6. cis alkene Amide 1 Hydrolysis to yield a carboxylic acid (Sec. 21.7) Reduction with LiAlH4 to yield an amine (Sec. 21.7) Dehydration to yield a nitrile (Section 20.7) . 2. 3 . Amine 1. 2. 3 . Basicity (Sec. 24.3) Sn2 alkylation of an alkyl halide to yield an amine (Sec. 24.6) Nucleophilic acyl substitution reactions Reaction with an acid chloride to yield an amide (Sec. 21.4) b. Reaction with an acid anhydride to yield an amide (Sec. 21 .5) Hofmann elimination to yield an alkene (Sec. 24.7) Formation of an arenediazonium salt (Sec. 24.8) a. 4. 5. Arene 1 2 . . 3. 4. Oxidation of an alkylbenzene side chain to yield a benzoic acid (Sec. 16.9) Catalytic reduction to yield a cyclohexane (Sec. 16. 10) Reduction of an aryl alkyl ketone to yield an arene (Sec. 16. 10) Electrophilic aromatic substitution (Sees. 16.1-16.3) a. b. c. d. e. Halogenation (Sees. 16.1-16.2) Nitration (Sec. 16.2) Sulfonation (Sec. 16.2) Friedel-Crafts alkylation (Sec. 16.3) Aromatic ring must be at least as reactive as a halobenzene Friedel-Crafts acylation (Sec. 16.3) 883 Functional-Group Reactions 884 Arenediazonium 1 . 2 . 3. 4. 5 . 6. Conversion Conversion Conversion Conversion Conversion Conversion salt into an aryl chloride (Sec. 24.8) bromide (Sec. 24.8) an aryl iodide (Sec. 24.8) an aryl cyanide (Sec. 24.8) a phenol (Sec. 24.8) an arene (Sec. 24.8) into an aryl into into into into Carboxylic acid 1 . 2. Acidity (Sees. 20.2-20.4) Reduction to yield a primary alcohol (Sees. 17.4, 21.3) a. Reduction with LiAlrLt b. Reduction with BH3 Nucleophilic acyl substitution reactions (Sec. 21.3) a. Conversion into an acid chloride b Conversion into an acid anhydride Conversion into an ester c. (1) Fischer esterification (2) Sn2 reaction with an alkyl halide 4. Alpha bromination (Hell-Volhard-Zelinskii reaction) (Sec. 22.4) 3. . Diene 1 . 2. Conjugate addition of HX and X2 (Sec. 14.2) Diels-Alder reaction (Sees. 14.4, 14.5, 30.5) Epoxide 1. 2. HX Acid-catalyzed ring opening with to yield a halohydrin (Sec. 18.6) Ring opening with aqueous acid to yield a 1,2-diol (Sec. 18.6) Ester 1. 2. 3. 4. 5. 6 . Hydrolysis to yield a carboxylic acid (Sec. 21.6) Aminolysis to yield an amide (Sec. 21.6) Reduction to yield a primary alcohol (Sees. 17.4, 21.6) Partial reduction with DIB to yield an aldehyde (Sec. 21.6) Grignard reaction to yield a tertiary alcohol (Sees. 17.5, 21.6) Claisen condensation to yield a p-keto ester (Sec. 23.7) AH Ether 1. 2. Acid-induced cleavage to yield an alcohol and an alkyl halide (Sec. 18.3) Claisen rearrangement of an allyl aryl ether to yield an o-allyl phenol (Sees. 18.4, 30.8) Halide, alkyl 1 . 2. 3 . 4. 5 . 6 . Reaction with magnesium to form a Grignard reagent (Sec. 10.6) Reduction to yield an alkane (Sec. 10.6) Coupling with a diorganocopper reagent to yield an alkane (Sec. 10.7) Reaction with an alcohol to yield an ether (Sec. 18.2) Nucleophilic substitution (Sn 1 or Sn2) (Sees. 11.1-11.5) Dehydrohalogenation to yield an alkene (El or E2) (Sees. 1 1 .7-1 1 10) . Halohydrin 1. Conversion into an epoxide (Sec. 18.5) Functional-Group Reactions 885 Ketone 1 . Nucleophilic addition reactions a. Reduction to yield a secondary alcohol (Sees. 17.4, 19.7) b. Reaction with a Grignard reagent to yield a tertiary alcohol (Sees. 17.5, 19.7) Wolff-Kishner reaction with hydrazine to yield an alkane (Sec. 19.9) c. to yield a cyanohydrin (Sec. 19.6) d. Reaction with e. Reaction with an alcohol to yield an acetal (Sec. 19.10) f. Wittig reaction to yield an alkene (Sec. 19. 1 1) g. Reaction with an amine to yield an imine or enamine (Sec. 19.8) HCN 2 . 3 . Aldol reaction to yield a (3-hydroxy ketone (Sec. 23. 1) Alpha bromination (Sec. 22.3) Nitrile Hydrolysis to yield a carboxylic acid (Sees. 20.5, 20.7) 1. Reduction to yield a primary amine (Sec. 20.7) 2. 3. Reaction with a Grignard reagent to yield a ketone (Sec. 20.7) Nitroarene 1 . Reduction to yield an arylamine (Sees. 16.2, 24.6) Organometallic reagent 1 . 2. 3 . 4. 5 . 6. Reduction by treatment with acid to yield an alkane (Sec. 10.6) Nucleophilic addition to a carbonyl compound to yield an alcohol (Sees. 17.5, 19.7) Conjugate addition of a lithium diorganocopper to an a,p-unsaturated ketone (Sec. 19. 13) Coupling reaction of a lithium diorganocopper reagent with an alkyl halide to yield an alkane (Sec. 10.7) Coupling reaction of a lithium diorganocopper with an acid chloride to yield a ketone (Sec. 21.4) Reaction with carbon dioxide to yield a carboxylic acid (Sec. 20.5) Phenol 1 . 2. 3. 4. Acidity (Sec. 17.2) Reaction with an acid chloride to yield an ester (Sec. 21.4) Reaction with an alkyl halide to yield an ether (Sec. 18.2) Oxidation to yield a quinone (Sec. 17. 10) Quinone 1 . Reduction to yield a hydroquinone (Sec. 17.10) Sulfide 1 2. 3 . Reaction with an alkyl halide to yield a sulfonium Oxidation to yield a sulfoxide (Sec. 18.8) Oxidation to yield a sulfone (Sec. 18.8) salt (Sec. 18.8) Thiol 1. 2. Reaction with an alkyl halide to yield a sulfide (Sec. 18.8) Oxidation to yield a disulfide (Sec. 18.8) Width: 612 Height: 792 Reagents in Organic Chemistry table summarizes the uses of some important reagents in organic chemistry. The reagents are listed alphabetically, followed by a brief description of the uses of each and references to the appropriate text sections. The following Acetic acid, CH3CO2H: Used as a solvent for the reduction of ozonides with zinc (Section 8.8) and the a-bromination of ketones and aldehydes with Br2 (Section 22.3). Acetic anhydride, (CH3CO)20: Reacts with alcohols to yield acetate esters (Sections 21.5 and 25.6) and with amines to yield acetamides (Section 21.5). Aluminum chloride, AICI3: acylation reactions of aromatic Acts as a Lewis acid catalyst compounds (Section in Friedel-Crafts alkylation and 16.3). Ammonia, NH3: Used - as a solvent for the reduction of alkynes by lithium metal to yield trans alkenes (Section 9.5). Reacts with acid chlorides and acid anhydrides to yield amides (Sections 21.4 and 21.5). Borane, BH3: Adds - to alkenes, giving alkylboranes that can be oxidized with alkaline H2O2 to yield alcohols (Section 8.5). Adds to alkynes, giving vinylic organoboranes that can be oxidized with aldehydes (Section 9.4). - Reduces H2O2 to yield carboxylic acids to yield primary alcohols (Section 21.3). Bromine, Br2: Adds to alkenes, yielding 1 ,2-dibromides (Sections 8.2, 14.2). - Adds to alkynes yielding either 1 ,2-dibromoalkenes or 1,1,2,2-tetrabromoalkanes (Section 9.3). - Reacts with arenes in the presence of FeBr3 catalyst to yield bromoarenes (Section 16. 1). - Reacts with ketones in acetic acid solvent to yield a-bromo ketones (Section 22.3). carboxylic acids in the presence of PBr3 to yield a-bromo carboxylic acids (Hell-Volhard-Zelinskii reaction; Section 22.4). Oxidizes aldoses to yield aldonic acids (Section 25.6). - Reacts with - N-Bromosuccinimide (NBS), (Ct^CO^NBr: Reacts with alkenes in the presence of aqueous dimethylsulfoxide to yield bromohydrins (Section 8.3). - Reacts with alkenes in the presence of light to yield allylic bromides (Section 10.3). - Reacts with alkylbenzenes in the presence of light to yield benzylic bromides; (Section 16.9). Di-terf-butoxy dicarbonate, protected amino acids suitable Butyllithium, anions, CH3CH2CH2CH2Li: A which can be alkylated (Section - Reacts with dialkylamines - (I-BuOCOhO: strong base; reacts with alkynes to yield acetylide 9.8). to yield lithium dialkylamide bases such as amide] (Section 22.5). Reacts with alkyltriphenylphosphonium (Section 19.11). Reacts with amino acids to give r-Boc for use in peptide synthesis (Section 26.7). salts to yield LDA [lithium diisopropyl- alkylidenephosphoranes (Wittig reagents Reagents Carbon dioxide, CO2: Reacts with Grignard 887 reagents to yield carboxylic acids (Section 20.5). Chlorine, CI2: Adds to alkenes to yield 1,2-dichlorides (Sections 8.2 and 14.2). - Reacts with alkanes in the presence of light to yield chloroalkanes by a radical chain reaction pathway (Section - Reacts with arenes 10.2). in the presence of FeCb catalyst to yield chloroarenes (Section 16.2). wi-Chloroperoxybenzoic acid, m-ClC6H4CC>3H: Reacts with alkenes to yield epoxides (Sections 8.7, 18.5). Chlorotrimethylsilane, (CH3)3SiCl: Reacts with alcohols to add the trimethylsilyl protecting group (Section 17.8). Chromium trioxide, CrC^: Oxidizes alcohols in aqueous acid to yield carbonyl-containing products. Primary alcohols yield carboxylic acids, and secondary alcohols yield ketones (Sections 17.7 and 19.3). Cuprous bromide, CuBr: Reacts with arenediazonium salts to yield bromoarenes (Sandmeyer chloride, CuCl: Reacts with arenediazonium salts to yield chloroarenes (Sandmeyer reaction; Section 24.8). Cuprous reaction; Section 24.8). Cuprous cyanide, CuCN: Reacts with arenediazonium (Sandmeyer salts to yield substituted benzonitriles reaction; Section 24.8). Cuprous iodide, Cul: Reacts with organolithiums to yield lithium diorganocopper reagents (Gilman reagents; Section 10.7). Cuprous oxide, CU2O: Reacts with arenediazonium salts to yield phenols (Section 24.8). Dess-Martin periodinane, C7H4l02(OAc)4: Oxidizes primary alcohols to aldehydes (Sections 17.7 and 19.2) Dichloroacetic acid, CI2CHCO2H: Cleaves DMT protecting groups in DNA synthesis (Section 28.7). Dicyclohexylcarbodiimide (DCC), C2, Heat ft RCHp-CHoCCHo + C0 2 + CHoOH H 3 + heat , a catalyst used for the hydrogenation of carbon-carbon Aldol condensation reaction (Section 23.1): the nucleophilic addition of an enol or enolate ion to a ketone or aldehyde, yielding a p-hydroxy ketone. ft 2 NaOH I R-C-(pH H ft * ? I | r_g_^_g-H R Amidomalonate amino acid synthesis to the (Section 26.3): a multistep reaction sequence, similar malonic ester synthesis, for converting a primary alkyl halide into an amino acid. — RCH 2 X + -:C(C0 2 Et) 2 2. I NHAc Cannizzaro reaction (Section ? 2 is C02 + 2 EtOH NH 2 test for aldehydes, involving treatment with cupric ion 19.12): the disproportionation reaction that occurs when a treated with base. 1.HO- RoCCH 3 + heat Benedict's test (Section 25.6): a chemical in aqueous sodium citrate. nonenolizable aldehyde RCH 2-CHCOH *~ HoO+ 3 ff RoCCOH + R0CCH0OH 3 3 2 2.H 3 0+ Claisen condensation reaction (Section 23.7): a nucleophilic acyl substitution reaction that occurs when an ester enolate ion attacks the carbonyl group of a second ester molecule. The product is a (3-keto ester. ff 2 R-CH 2-C-OCH 3 1.HO- - fP f? R_CH 2-C-(j)H-C-OCH3 R + CH 3 OH 894 Name Reactions Claisen rearrangement (Sections 18.4 and 30.8): the thermal of an allyl vinyl ether or an allyl phenyl ether. [3.3] sigmatropic rearrangement H Cope rearrangement (Section to a new 1,5-diene. 30.8): the thermal [3.3] sigmatropic rearrangement of a 1,5-diene heat Curtius rearrangement (Section 24.6): the thermal rearrangement of an acyl azide isocyanate, followed by hydrolysis to yield an amine. to an O II + - R—C— N=N=N 1. heat 2. H2 RNH 2 Diazonium coupling reaction diazonium salt 0~ and a phenol or n+=n Dieckmann reaction C02 + N2 (Section 24.8): the coupling reaction between an aromatic aniline. 0^ + + h —* O~ ^O~ n=n 0h (Section 23.9): the intramolecular Claisen condensation reaction of a 1,6- or 1,7-diester, yielding a cyclic (3-keto ester. C0 2 Et C0 2 Et 1. NaOEt 2- H3 + + EtOH Diels-Alder cycloaddition reaction (Sections 14.4-14.5 and diene and a dienophile to yield a cyclohexene ring. heat 30.5): the reaction between a Name Reactions 895 Edman degradation (Section 26.6): a method for cleaving the N-terminal amino acid from a peptide by treatment of the peptide with Af-phenylisothiocyanate. O 'I Ph-N=C=S + H 2 N-CH-C-NH-£ R Fehling's test (Section 25.6): a chemical test for aldehydes, involving treatment with cupric ion in aqueous sodium tartrate. Fischer esterification reaction (Section 21.3): the acid-catalyzed reaction carboxylic acid and an alcohol, yielding the ester. O H+ II R-C-OH O , heat II R-C-OR' + H 2 R'-OH + between a Friedel-Crafts reaction (Section 16.3): the alkylation or acylation of an aromatic ring treatment with an alkyl- or acyl chloride in the presence of a Lewis-acid catalyst. by R-CI, AICI 3 Alkylation O 11 R-C-CI, AICI3 R Acylation Gabriel amine synthesis (Section 24.6): a multistep sequence for converting a primary alkyl halide into a primary amine by alkylation with potassium phthalimide, followed by hydrolysis. NTK + + RCH 2X 1. mix 2. NaOH 2 NH 2 Gilman reagent (Section 10.7): a lithium dialkylcopper reagent, R^CuLi, prepared by treatment of a cuprous salt with an alkyllithium. Gilman reagents undergo a coupling reaction with alkyl halides, a 1,4-addition reaction with a,|3-unsaturated ketones, chlorides to yield ketones. and a coupling reaction with acid Width: 612 Height: 792 Name 896 Reactions Glycal assembly method (Section 25.9): a method of polysaccharide synthesis in which a glycal is converted into its epoxide, which is then opened by reaction with an alcohol. Grignard reaction (Section 19.7): the nucleophilic addition reaction of an alkylmagnesium halide to a ketone, aldehyde, or ester carbonyl group. OH O R-Mg-X + 1 IT mix I 10.6): an organomagnesium halide, RMgX, prepared by reaction between an organohalide and magnesium metal. Grignard reagents add to carbonyl compounds Grignard reagent (Section to yield alcohols. Hell-Volhard-Zelinskii reaction (Section 22.4): the a-bromination of treatment with bromine and phosphorus tribromide. HO III Br — C— C— OH 1 . , 2. I Hofmann amine — Br 2 PBr 3 H2 a carboxylic acid by O — C— C— OH I II | elimination (Section 24.7): a method for effecting the elimination reaction of an The amine is first treated with excess iodomethane, and the resultant to yield an alkene. quaternary R2N ammonium salt is heated H V 1.CHJ C— C ^\ " 2. Ag 2 0, H 2 with silver oxide. \ / C=C / \ + R2 NCH 3 Name Hofmann rearrangement Reactions 897 (Section 24.6): the rearrangement of an TV-bromoamide to a primary amine by treatment with aqueous base. O Br, I II R— C— NH. R—C— NHBr NaOH RNH 2 + C0 2 Kiliani-Fischer synthesis (Section 25.6): a multistep sequence for chain-lengthening an aldose into the next higher homolog. CHO CHO i.HCN R 2. H2 3. H3 + , I CH(OH) Pd, BaS0 4 | R Knowles amino acid synthesis synthesis (Section 26.3): an enantioselective method of amino acid chiral hydrogenation catalyst. by hydrogenation of a Z enamido acid with a C0 2 H C=C NHCOCH3 H2) [Rh(DiPAMP)(COD)] 1 2. + C0 2 H " BF4 NaOH, H 2 H2 N H Koenigs-Knorr reaction (Section 25.6): a method for synthesizing a glycoside by reaction of a pyranosyl bromide with an alcohol and Ag20. CH 2 OAc ;h 2 ROH, Ag 2 AcO Ac 2. OAcI oh H0 ~OH, H 2 Br Malonic ester synthesis (Section f? R-CH 2-X + 1 . :pH-C-OCH 3 2. C0 2 CH 3 sequence for converting an alkyl halide two carbon atoms to the chain. 22.7): a multistep into a carboxylic acid with the addition of heat H3O+ P heat RCH 2-CH 2 COCH 3 + C0 2 + CH 3 OH 898 Name Reactions McLafferty rearrangement (Section 19.14): a mass spectral fragmentation pathway for carbonyl compounds having a hydrogen three carbon atoms away from the carbonyl carbon. Meisenheimer complex (Section 16.7): an intermediate formed in the nucleophilic aryl substitution reaction of a base with a nitro-substituted aromatic ring. Merrifield solid-phase peptide synthesis (Section 26.8): a rapid and efficient means of peptide synthesis in which the growing peptide chain is attached to an insoluble polymer support. Michael reaction (Section 23.10): the 1,4-addition reaction of a stabilized enolate anion such that from a 1,3-diketone to an a,p-unsaturated carbonyl compound. — O O c \ -c* 2 H 29 C=CHCR Base II CH 29 / as + 9 CH-CH 29 CH 29 CR \ — C \\ o II / \\ o Robinson annulation reaction (Section 23.12): a multistep sequence for building a new initial Michael reaction of the cyclohexenone ring onto a ketone. The sequence involves an ketone followed by an internal aldol cyclization. Sandmeyer reaction halide (Section 24.8): a method for converting an aryldiazonium salt into an aryl by treatment with a cuprous 0~ N+=N + CuBr Sanger dideoxy method (Section halide. — Br " 28.6): an O"' (or CI, enzymatic method for I) DNA sequencing. Name Reactions 899 Simmons-Smith reaction (Section 8.9): a method for preparing a cyclopropane by treating an alkene with diiodomethane and zinc-copper. CH 2 2 + I Stork enamine reaction (Section 23.11): a multistep sequence whereby a ketone is converted into an enamine by treatment with a secondary amine, and the enamine is then used in Michael reactions. ^— O NFL2 HNR2 Suzuki-Miyaura reaction HpC=CHCOR' 1. I O O A^\A II || R , (Section 10.7): an organometallic coupling of an aromatic or vinyl substituted boronic acid with an aryl or vinyl substituted organohalide in the presence of a base and a palladium catalyst. + Ar-B(OH) 2 I— Ar' (or vinyl) Pd(Pho) 4 Ar-Ar' CaC0 3 THF Tollens' test (Section 25.6): a chemical test for detecting aldehydes by treatment with ammoniacal silver nitrate. A positive test is signaled by formation of a silver mirror on the walls of the reaction vessel. Walden Sn2 inversion (Section 11.1): the inversion of stereochemistry at a chirality center during an reaction. Nuf + v c— N / \ Williamson ether synthesis (Section 18.2): a primary alkyl halide with an alkoxide Na+ R-0 + R'CH 2 Br ^ method for preparing an ether by treatment of a ion. R-0-CH 2 R' + NaBr 1): a general method of alkene synthesis by treatment of a ketone or aldehyde with an alkylidenetriphenylphosphorane. Wittig reaction (Section 19.1 ° + C ^R' \ C=PPh«3 / R \ / C=C r^ 1 \ + PhoP=0 Name 900 Reactions Wohl degradation (Section 25.6): a multistep reaction sequence for degrading an aldose into the next lower homolog. CHO j CH(OH) | R —— — 1. NH 2 OH - 2. Ac 2 3. NaOEt CHO | R Wolff-Kishner reaction (Section 19.9): a method for converting a ketone or aldehyde into the corresponding hydrocarbon by treatment with hydrazine and strong base. 9 N 2H 4 , KOH C Woodward-Hoffmann R-CH 9 -R' orbital symmetry rules (Section 30.9): a series of rules for predicting the stereochemistry of pericyclic reactions. Even-electron species react thermally through either antarafacial or conrotatory pathways, whereas odd-electron species react thermally through either suprafacial or disrotatory pathways. Abbreviations A symbol for Angstrom unit (10 ADMET acyclic diene metathesis, a Ac- Acetyl group, Ar- aryl 8 cm = 10~ 10 m) method of polymerization ft group at. no. atomic number at. wt. atomic weight [a]o CH 3 C- specific rotation I? Boc (CH 3 ) 3 COC- te?t-butoxycarbonyl group, bp boiling point n-Bu rc-butyl group, sec-Bu CH 3 CH 2 CH2 CH 2sec-butyl group, CH 3 CH 2 CH(CH 3 )- t-Bu terf-butyl group, cm centimeter cm 1 (CH 3 ) 3 C- wavenumber, or reciprocal centimeter D stereochemical designation of carbohydrates and amino acids DCC dicyclohexylcarbodiimide, 6 chemical A symbol for AH dm heat of reaction DMF DMSO DNA DNP dimethylformamide, (E) entgegen, stereochemical designation of double bond geometry E act activation energy El unimolecular elimination reaction ElcB unimolecular elimination that takes place through a carbanion intermediate E2 bimolecular elimination reaction Et ethyl group, g gram shift in -N=C=N-C 6H 1 j ppm downfield from TMS symbol for change heat; also decimeter (0.1 1 m) dimethyl sulfoxide, (CH 3 ) 2 NCHO (CH 3 ) 2 SO deoxyribonucleic acid dinitrophenyl group, as in CH 3 CH2- 2,4-DNP (2,4-dinitrophenylhydrazone) 902 Abbreviations kv symbol for Hz Hertz, or cycles per second (s i- iso IR infrared J Joule J symbol for coupling constant K Ka Kelvin temperature kJ kilojoule L stereochemical designation of carbohydrates and amino acids LAH lithium Me methyl group, mg milligram (0.001 g) MHz mL meg ahertz(10 s milliliter (0.001 L) mm millimeter (0.001 mp melting point Hg microgram (10 m\x millimicron (nanometer, 10 MW molecular weight n- normal, straight-chain alkane or alkyl group ng nanogram (10 light ) acid dissociation constant aluminum hydride, LiAlH 4 CH 3 ) m) -6 g) -9 m) -9 gram) -9 nm nanometer (10 NMR nuclear magnetic resonance -OAc acetate group, -OCCH 3 Ph phenyl group, -C 6 H 5 pH pKa pm measure of acidity of aqueous solution ppm parts per million /i-Pr /i-propyl group, z-Pr isopropyl group, pro-R designation of a prochirality center pro-S designation of a prochirality center R- symbol meter) measure of acid strength (= -log Ka ) -12 picometer(10 m) CH 3CH 2CH 2(CH 3 ) 2 CH- for a generalized alkyl group Abbreviations ( rectus, designation of chirality center R) Re 2 face RNA ROMP a face of a planar, sp -hybridized carbon atom ribonucleic acid ring-opening metathesis polymerization of chirality center (S) sinister, designation sec- secondary Si face a face of a planar, sp -hybridized carbon atom SN 1 unimolecular substitution reaction SN2 bimolecular substitution reaction tert- tertiary THF TMS tetrahydrofuran tetramethylsilane Tos tosylate group, 2 nmr standard, (CH3)4Si —r — CH y 3 UV ultraviolet X- halogen group (-F, -CI, -Br, -I) (Z) zusammen, stereochemical designation of double bond geometry chemical reaction in direction indicated m reversible chemical reaction resonance symbol ^ —>^ curved arrow indicating direction of electron flow is equivalent to > greater than < less than « approximately equal to indicates that the organic fragment m 6+, | 6- shown is a part of a larger molecule single bond coming out of the plane of the paper single bond receding partial bond partial charge into the plane of the paper denoting the transition state 903 Infrared Absorption Frequencies Functional Alcohol Group Frequency (cm -O-H 3300-3600 (s) 1 ) Text Section 17.11 \ -C-O- 1050 (s) / Aldehyde aliphatic -CO-H 2720, 2820 (m) \ 1725 (s) / 1705 (s) C=0 aromatic Alkane 19.14 12.8 \ -C-H \ 2850-2960 (s) / 800-1300 (m) Alkene 12.8 3020-3100 Alkyne (s) H 1650-1670 (m) RCB=CH2 910, 990 (m) R2C=CH2 890 (m) sC-H 3300 -OC- 2100-2260 (m) 12.8 (s) Alkyl bromide 12.8 \ -C-Br 500-600 (s) / Alkyl chloride 12.8 \ — C-Cl / 600-800 (s) Infrared Absorptions Amine, primary 24.10 3400, 3500 (s) H secondary \ r Ammonium -H 3350 (s) 24.10 salt \ + -HST-H 2200-3000 (broad) Ar-H 3030 (m) Ar-R 690-710 (s) 730-770 (s) o-disubstituted 735-770 (s) m-disubstituted 690-710 (s) 810-850 (s) 810-840 (s) Aromatic ring monosubstituted p-disubstituted Carboxylic acid -O-H 2500-3300 (broad) associated \ 1710 (s) free / 1760 (s) G=0 15.8 Acid anhydride 20.8 21.10 \ c=o 1760, 1820 (s) / Acid chloride 21.10 \ 1810 (s) / } 1770 (s) aliphatic \ 1810 (s) aromatic ! 1770 (s) N-substituted 1680 (s) N, N-disubstitu ted 1650 (s) aliphatic aromatic O=0 Amide 21.10 Width: 612 Height: 792 906 Infrared Absorptions 21.10 Ester aliphatic \ 1735 (s) aromatic / 1720 (s) C=0 Ether 18.9 / 1050-1150 Ketone 19.14 aliphatic \ 1715 (s) aromatic f 1690 (s) 6-memb. ring 1715 (s) 5-memb. ring 1750 (s) C=0 20.8 Nitrile aliphatic -C=N 2250 (m) 2230 (m) -O-H 3500 aromatic Phenol (s) = strong; (m) (s) = medium intensity (s) 17.11 NMR Chemical Shifts Proton Chemical Shift Type of Proton (8) Text Section R-CH 3 R-CH2-R R 3 C-H 0.7-1.3 13.9 1 .2-1 .4 13.9 i .4_i .7 13.9 AUyto -C=C-j:~H 1.6-1.9 13.9 atocarbonyl -C-j:-H 2.0-2.3 19.14 Benzylic Ar-j:-H 2.3-3.0 15.8 Acetylenic R-OC-H 2.5-2.7 13.9 Alkyl chloride Cl-C-H 3.0-4.0 13.9 Alkyl bromide Br-^-H 2.5-4.0 13.9 Alkyl iodide I-